TEST BANK FORBSouth-Western Federal Taxation 2025 Corporations Partnerships Estates and Trusts 48th

Page 1


Name:

Class:

Date:

Chapter 01: Understanding and Working with the Federal Tax Law True / False 1. Many states have balanced budgets because laws or constitutional amendments preclude deficit spending. a. True b. False 2. The U.S. Federal government has a provision in the Constitution that precludes deficit spending. a. True b. False 3. Revenue-neutral tax laws reduce deficits. a. True b. False 4. Longer class lives for depreciable property and the required use of the straight-line method of depreciation would likely dampen the tax incentive for purchasing capital assets. a. True b. False 5. The Internal Revenue Code is a compilation of Federal tax legislation that appears in Title 26 of the United States Code. a. True b. False 6. The favorable treatment of research and development expenses (via amortization deductions and tax credits) is one means of controlling the economy. a. True b. False 7. The encouragement of private-sector pension plans can be justified under the encouragement of certain industries. a. True b. False 8. One Internal Revenue Code section enables shareholders in a small business corporation to obtain an ordinary deduction for any loss recognized on a stock investment. a. True b. False 9. One of the justifications for the enactment of the tax law governing corporate reorganizations was the economic benefit it would provide businesses (including making them more efficient). a. True b. False 10. Although a corporation is subject to a Federal income tax, a partnership is not. a. True b. False Powered by Cognero

Page 1


Name:

Class:

Date:

Chapter 01: Understanding and Working with the Federal Tax Law 11. The Federal income tax law allows a taxpayer to claim a deduction for state and local income taxes but limits all state taxes to a maximum of $10,000. a. True b. False 12. Alabama and South Carolina are community property states. a. True b. False 13. A tax bill cannot originate in the Senate Finance Committee. a. True b. False 14. Taxpayers may read Committee Reports to determine the intent of Congress. a. True b. False 15. These Internal Revenue Code citations are incorrect: § 212(1) and § 1221(1). a. True b. False 16. Internal Revenue Code § 6 involves gross income and § 7 outlines itemized deductions. a. True b. False 17. Subchapter P refers to the subchapter in the Internal Revenue Code that deals with partners and partnerships. a. True b. False 18. Regulations are arranged in a different sequence than the Internal Revenue Code. a. True b. False 19. Proposed Regulations have the force and effect of law. a. True b. False 20. Temporary Regulations have the same authoritative value as Final Regulations for four years. a. True b. False 21. Proposed Regulations are published in the Federal Register. a. True b. False 22. Regulations are issued by the Treasury Department. Powered by Cognero

Page 2


Name:

Class:

Date:

Chapter 01: Understanding and Working with the Federal Tax Law a. True b. False 23. Revenue Rulings carry the same legal force and effect as Regulations. a. True b. False 24. A Revenue Ruling is a legislative source of Federal tax law. a. True b. False 25. Revenue Procedures deal with the internal management practices and procedures of the IRS. a. True b. False 26. Treasury Decisions are issued by the Treasury Department to promulgate new Regulations. a. True b. False 27. Determination letters usually involve proposed transactions. a. True b. False 28. Letter rulings are issued by the National Office of the IRS. a. True b. False 29. A taxpayer must pay any tax deficiency assessed by the IRS and sue for a refund to bring suit in the U.S. District Court. a. True b. False 30. In a U.S. District Court, a jury can decide both questions of fact and questions of law. a. True b. False 31. A U.S. District Court must abide by the precedents set by the U.S. Court of Appeals of its jurisdiction. a. True b. False 32. Appeals from the U.S. Court of Federal Claims go to the U.S. Supreme Court. a. True b. False 33. A jury trial is available when a case is heard by a U.S. Court of Appeals. a. True Powered by Cognero

Page 3


Name:

Class:

Date:

Chapter 01: Understanding and Working with the Federal Tax Law b. False 34. Only one judge hears a trial in a U.S. District Court. a. True b. False 35. The Golsen rule no longer applies to the U.S. Tax Court. a. True b. False 36. When there is a direct conflict between an Internal Revenue Code section and a treaty provision, the most recent item takes precedence. a. True b. False 37. “Legislative" regulations carry more weight than “interpretative” regulations. a. True b. False 38. The U.S. national debt is around $20 trillion. a. True b. False 39. A change in the individual tax rate has an almost immediate impact on the economy. a. True b. False 40. The like-kind tax free exchange treatment is an example of the wherewithal to pay concept. a. True b. False 41. Indexation of various income tax components was eliminated by the Tax Cuts and Jobs Act of 2017. a. True b. False 42. When there is a direct conflict between a tax treaty and the Internal Revenue Code, the Internal Revenue Code takes precedence. a. True b. False 43. The Standard Federal Tax Reporter is published by Research Institute of America. a. True b. False 44. The annual gift tax exclusion in 2024 is $16,000. a. True Powered by Cognero

Page 4


Name:

Class:

Date:

Chapter 01: Understanding and Working with the Federal Tax Law b. False 45. Internal Revenue Code Section 318, which deals with the definition of related parties with respect to stock redemptions, includes brothers and sisters in the related-party definition. a. True b. False 46. Complete avoidance of a capital gain tax occurs when the owner of appreciated property transfers it by death. a. True b. False 47. The taxation part of the CPA exam (REG) includes both multiple choice questions and task-based simulations. a. True b. False Multiple Choice 48. Which provision could best be justified as a means of controlling the economy? a. Write-off of research and development expenditures. b. The § 179 immediate expensing of depreciable capital expenditures. c. Amortization of pollution control facilities. d. The rehabilitation tax credit. 49. Which provision could best be justified as encouraging small business? a. Ordinary loss allowed on § 1244 stock. b. Percentage depletion. c. Charitable contributions deduction. d. Interest deduction on home mortgage. 50. Which provision is not justified by social considerations? a. Refundable earned income credit. b. Adoption tax credit. c. Like-kind exchange treatment. d. Disallowance of illegal kickbacks. 51. Which state is not a community property state? a. Arizona. b. Texas. c. New Mexico. d. Virginia. 52. Douglas and Sue, related parties, are landlord and tenant as to certain business property. If the IRS questions the amount of rent Sue is paying to Douglas, this is an illustration of the: a. Arm’s length concept. b. Continuity of interest concept. Powered by Cognero

Page 5


Name:

Class:

Date:

Chapter 01: Understanding and Working with the Federal Tax Law c. Tax benefit rule. d. Substance over form concept. 53. Federal tax legislation generally originates in what committee? a. House Budget Committee. b. Senate Finance Committee. c. House Ways and Means Committee. d. House Taxation Committee. 54. Regulations are first published in: a. Federal Register. b. Cumulative Bulletin. c. Internal Revenue Bulletin. d. I.R.S. Digest. 55. Which citation is considered to be a statutory (legislative) citation? a. Ltr. Rul. 199952058. b. Ann. 94-5, 1994-2 I.R.B. 39. c. Reg. § 1.1014-1(c)(1). d. § 351. 56. A Technical Advice Memorandum is issued by: a. Treasury Department. b. National Office of the IRS. c. Office of Chief Council. d. Area Director. 57. Revenue Procedures are published in the: a. Congressional Record. b. Federal Revenue Bulletin. c. Internal Revenue Bulletin. d. I.R.S. Digest. 58. Determine the incorrect citation: a. TAM 20002704. b. George W. Guill, 112 T.C.__, No. 22 (1999). c. John H. Wong, T.C. Summary Opinion 2009-152. d. Rev. Rul. 98-32, 1998-25 I.R.B. 4. 59. Regarding Technical Advice Memoranda, which statement is incorrect? a. Issued by the National Office of IRS. b. Most often deal with a completed transaction. c. May be cited and used as precedent. d. Issued with multi-digit file numbers. Powered by Cognero

Page 6


Name:

Class:

Date:

Chapter 01: Understanding and Working with the Federal Tax Law 60. Which of the following sources has the highest tax authority? a. Treasury Regulation. b. Revenue Procedure. c. Internal Revenue Code. d. Temporary Regulation. 61. Which of the following is an administrative source of tax law? a. Rev. Rul. 2010-19. b. Joint Conference Committee Report. c. Section 12(a) of the Internal Revenue Code. d. All of these. 62. A decision in which of the following courts carries the lowest tax authority? a. U.S. Court of Appeals for the Federal Circuit. b. U.S. Court of Appeals for the Second Circuit. c. U.S. District Court. d. U.S. Supreme Court. 63. In Forty-Four Cigar Co., 2 B.T.A. 1156, the 1156 stands for: a. The volume number. b. The year of the decision. c. The paragraph number. d. The page number. 64. Which statement is not true about this citation: Bonkowski v. Comm., 29 TCM 1645 (1970), aff’d 458 F.2d 709 (CA-7, 1972), cert. den.? a. The Supreme Court decided not to agree or disagree with the Seventh Court of Appeals. b. The Seventh Court of Appeals disagreed with the Tax Court. c. The Tax Court decision starts on page 1645. d. The Seventh Court of Appeals decision appears in Vol. 458. 65. Which of these notations would appear after a U.S. Tax Court citation if the IRS disagrees with the decision? a. Rev’d 935 F.2d 203 (1991). b. Nonacq. 1979-1 C.B. 1. c. Cert. den. 361 U.S. 875 (1959). d. Acq. 1990-1 C.B. 2. 66. Which of the following refers to a trial court rather than an appellate court decision? a. Forgeus v. Comm., 6 B.T.A. 291 (1927). b. Farris v. Comm., 222 F.2d 320 (CA-10, 1955). c. Danville Plywood Corp., 899 F.2d 3 (Fed Cir. 1990). d. Boehm v. Comm., 326 U.S. 287 (1945). 67. Which citation refers to a Third Circuit Court of Appeals decision? Powered by Cognero

Page 7


Name:

Class:

Date:

Chapter 01: Understanding and Working with the Federal Tax Law a. 40 T.C. 1018. b. 2 TCM 205 (1951). c. 354 F.Supp. 1003 (D. Ct. Ga, 1972). d. 914 F.2d 396 (CA-3, 1990). 68. Which state is located in the jurisdiction of the Fifth Circuit Court of Appeals? a. Louisiana. b. California. c. New York. d. South Carolina. 69. Interpret the following citation: 64-1 USTC 9618, aff’d in 344 F. 2d 966. a. A U.S. Tax Court Small Cases Division decision that was affirmed on appeal. b. A U.S. Tax Court decision that was affirmed on appeal. c. A U.S. District Court decision that was affirmed on appeal. d. A U.S. Court of Appeals decision that was affirmed on appeal. 70. Which citation refers to a Second Circuit Court of Appeals decision? a. 40 T.C. 1018. b. 159 F. 2d 848 (CA-2, 1947). c. 354 F. Supp. 1003 (D. Ct. Ga, 1972). d. 914 F. 2d 396 (CA-3, 1990). 71. Which citation refers to a U.S. Court of Federal Claims decision? a. Apollo Computer, Inc. v. U.S., 95-1 USTC ¶ 50,015 (Fed. Cl., 1994). b. Westreco, Inc., T.C. Memo. 1992-561 (1992). c. Bausch & Lomb, Inc. v. Comm., 933 F. 2d 1084 (CA-2, 1991). d. Portland Manufacturing Co. v. Comm., 35 AFTR 2d 1439 (CA-9, 1975). 72. If these citations appeared after a trial court decision, which one means that the decision was viewed favorably? a. Aff’d 633 F. 2d 512 (CA-7, 1980). b. Rem’d 399 F. 2d 800 (CA-5, 1968). c. Rev’d 914 F. 2d 396 (CA-3, 1990). d. Rev’d 935 F. 2d 203 (CA-5, 1991). 73. Which trial court normally has 16 judges? a. U.S. Tax Court. b. U.S. Court of Federal Claims. c. U.S. Supreme Court. d. U.S. Court of Appeals. 74. Which trial court’s jurisdiction depends on the geographical location of the taxpayer? a. U.S. Tax Court. b. U.S. District Court. Powered by Cognero

Page 8


Name:

Class:

Date:

Chapter 01: Understanding and Working with the Federal Tax Law c. U.S. Court of Federal Claims. d. Small Cases Division of the Tax Court. 75. Which trial court decision is generally less authoritative? a. U.S. District Court. b. U.S. Tax Court. c. U.S. Court of Federal Claims. d. Small Cases Division of the Tax Court. 76. A Memorandum decision of the U.S. Tax Court could be cited as: a. T.C. Memo. 1990-650. b. 68-1 USTC 9200. c. 37 AFTR 2d 456. d. All of these. 77. Which court decision is generally more authoritative? a. A U.S. Tax Court decision. b. A U.S. Court of Federal Claims decision. c. A U.S. District Court decision. d. A U.S. Court of Appeals decision. 78. Which of the following statements about an acquiescence is correct? a. An acquiescence is issued in the Federal Register. b. Acquiescences are published only for certain regular decisions of the U.S. Tax Court. c. An acquiescence is published in the Internal Revenue Bulletin. d. The IRS does not issue acquiescences to adverse decisions that are not appealed. 79. Which is a primary source of tax law? a. J.W. Yarbo v. Comm., 737 F. 2d 479 (CA-5, 1984). b. Article by a Federal judge in Harvard Law Review. c. IRS Determination letter. d. IRS Letter ruling. 80. A landlord leases property upon which the tenant makes improvements. The improvements are significant and are not made in lieu of rent. At the end of the lease, the value of the improvements is not income to the landlord. This rule is an example of: a. The wherewithal to pay concept. b. The tax benefit rule. c. The arm’s length concept. d. A clear reflection of income result. 81. The Internal Revenue Code was codified for the first time in what year? a. 1913. b. 1923. Powered by Cognero

Page 9


Name:

Class:

Date:

Chapter 01: Understanding and Working with the Federal Tax Law c. 1939. d. 1954. 82. What statement is not true with respect to Temporary Regulations? a. They may not be cited as precedent. b. They are issued with Proposed Regulations. c. They automatically expire within three years after the date of issuance. d. They can be found in the Federal Register. 83. What administrative release deals with a proposed transaction rather than a completed transaction? a. Letter Ruling. b. Technical Advice Memorandum. c. Determination Letter. d. Field Service Advice. 84. If a taxpayer decides not to pay a tax deficiency, they must go to which court? a. Appropriate U.S. Circuit Court of Appeals. b. U.S. District Court. c. U.S. Tax Court. d. U.S. Court of Federal Claims. 85. Both economic and social considerations can be used to justify: a. Various tax credits, deductions, and exclusions that are designed to encourage taxpayers to obtain additional education. b. Disallowance of any deduction for expenditures deemed to be contrary to public policy (e.g., fines, penalties, illegal kickbacks, bribes to government officials). c. Favorable tax treatment for accident and health plans provided for employees and financed by employers. d. Allowing a deduction for state and local income taxes paid. 86. Social considerations can be used to justify: a. Allowing a federal income tax deduction for state and local sales tax. b. Allowing excess capital losses to be carried over to other years. c. Allowing accelerated amortization for the cost of installing pollution control facilities. d. Allowance of a credit for child care expenses. 87. Allowing a net operating loss (NOL) carryforward can be justified: a. As mitigating the effect of the annual accounting period concept. b. By economic considerations. c. As promoting administrative feasibility. d. Based on the wherewithal to pay concept. 88. Which, if any, of the following provisions of the tax law cannot be justified as promoting administrative feasibility (simplifying the task of the IRS)? a. Penalties are imposed for failure to file a return or pay a tax on time. b. Prepaid income is taxed in the year received, not in the year earned. Powered by Cognero

Page 10


Name:

Class:

Date:

Chapter 01: Understanding and Working with the Federal Tax Law c. Annual adjustments for indexation increase the amount of the standard deduction allowed. d. A deduction is allowed for charitable contributions. Essay 89. What impact has the community property system had on our Federal tax laws? 90. How does a treaty with a foreign country impact a section in the Internal Revenue Code? 91. Explain the Golsen doctrine. 92. What is the value of Actions on Decisions to a tax researcher? 93. What value is a tax citator to a tax researcher?

Powered by Cognero

Page 11


Name:

Class:

Date:

Chapter 01: Understanding and Working with the Federal Tax Law Answer Key 1. True 2. False 3. False 4. True 5. True 6. False 7. False 8. True 9. True 10. True 11. True 12. False 13. False 14. True 15. False 16. False 17. False 18. False 19. False 20. False 21. True 22. True 23. False 24. False 25. True Powered by Cognero

Page 12


Name:

Class:

Date:

Chapter 01: Understanding and Working with the Federal Tax Law 26. True 27. False 28. True 29. True 30. False 31. True 32. False 33. False 34. True 35. False 36. True 37. True 38. False 39. True 40. True 41. False 42. False 43. False 44. False 45. False 46. True 47. True 48. b 49. a 50. c Powered by Cognero

Page 13


Name:

Class:

Date:

Chapter 01: Understanding and Working with the Federal Tax Law 51. d 52. a 53. c 54. a 55. d 56. b 57. c 58. a 59. c 60. c 61. a 62. c 63. d 64. b 65. b 66. a 67. d 68. a 69. c 70. b 71. a 72. a 73. b 74. b 75. d 76. a Powered by Cognero

Page 14


Name:

Class:

Date:

Chapter 01: Understanding and Working with the Federal Tax Law 77. d 78. c 79. a 80. a 81. c 82. a 83. a 84. c 85. a 86. d 87. a 88. d 89. The position of the residents of community property states was so advantageous that many common law states actually adopted community property systems. The political pressure placed on Congress to correct the disparity in tax treatment was considerable. To a large extent, this correction was accomplished in the Revenue Act of 1948, which extended many of the community property tax advantages to residents of common law jurisdictions. Thus, common law states avoided the trauma of discarding their time-honored legal system familiar to everyone. The impact of community property law on the Federal estate and gift taxes is further explored in Chapters 18 and 19. 90. The United States enters into tax treaties (sometimes called tax conventions) with foreign countries to render mutual assistance in tax enforcement and to avoid double taxation. Neither a tax law nor a tax treaty takes precedence. When there is a conflict, the most recently enacted item will take precedence. 91. Because the Tax Court is a national court, it decides cases from all parts of the country. For many years, the Tax Court followed a policy of deciding cases based on what it thought the result should be, even though its decision might be appealed to a U.S. Circuit Court of Appeals that had previously decided a similar case differently. A number of years ago, this policy was changed in the Golsen decision. After that change, the Tax Court will decide a case as it feels the law should be applied only if the Circuit Court of Appeals of the appropriate jurisdiction has not yet passed on the issue or has previously decided a similar case in accord with the Tax Court’s decision. If the Circuit Court of Appeals of the appropriate jurisdiction has previously held otherwise, the Tax Court will conform under the Golsen rule even though it disagrees with the holding. 92. Actions on Decisions tell a taxpayer the IRS’s reaction to certain court decisions. The IRS follows a practice of either acquiescing (agreeing) or nonacquiescing (not agreeing) with court decisions where guidance may be helpful. This practice does not mean that a particular decision has no value if the IRS has nonacquiesced in the result. It does, however, indicate that the IRS will continue to litigate the issue involved. 93. The use of manual citators or a computer citator search is invaluable to tax research. A citator provides the history of a Powered by Cognero

Page 15


Name:

Class:

Date:

Chapter 01: Understanding and Working with the Federal Tax Law case including the authority relied on (e.g., other judicial decisions) in reaching the result. Reviewing the references listed in the citator discloses whether the decision was appealed and, if so, with what result (e.g., affirmed, reversed, and remanded). It also reveals other cases with the same or similar issues and how they were decided. Thus, a citator reflects on the validity of a case and may lead to other relevant judicial material.

Powered by Cognero

Page 16


Name:

Class:

Date:

Chapter 02: The Deduction for Qualified Business Income for Noncorporate Taxpayes True / False 1. Tomas owns a sole proprietorship, and Lucy is the sole shareholder of a C corporation. In the current year, both businesses make a net profit of $60,000. Neither business distributes any funds to the owners in the year. For the current year, Tomas must report $60,000 of income on his individual tax return, but Lucy is not required to report any income from the corporation on her individual tax return. a. True b. False 2. Carol and Candace are equal partners in Peach Partnership. In the current year, Peach had a net profit of $75,000 ($250,000 gross income – $175,000 operating expenses) and distributed $25,000 to each partner. Peach must pay tax on $75,000 of income. a. True b. False 3. Rajib is the sole shareholder of Cardinal Corporation, a calendar year S corporation. In the current year, Cardinal generated a net profit of $350,000 ($520,000 gross income – $170,000 operating expenses) and distributed $80,000 to Rajib. Rajib must report the Cardinal Corporation profit of $350,000 on his Federal income tax return. a. True b. False 4. Donald owns a 45% interest in a partnership that earned $130,000 in the current year. He also owns 45% of the stock in a C corporation that earned $130,000 during the year. Donald received $20,000 in distributions from each of the two entities during the year. With respect to this information, Donald must report $78,500 of income on his individual income tax return for the year. a. True b. False 5. Quail Corporation is a C corporation that generates net income of $125,000 during the current year. If Quail paid dividends of $25,000 to its shareholders, the corporation must pay tax on $100,000 of net income. Shareholders must report the $25,000 of dividends as income. a. True b. False 6. Eagle Company, a partnership, had a short-term capital loss of $10,000 during the current year. Aaron, who owns 25% of Eagle, will report $2,500 of Eagle’s short-term capital loss on his individual tax return. a. True b. False 7. Matt, the sole shareholder of Pastel Corporation (a C corporation), has the corporation pay him a salary of $600,000 in the current year. The Tax Court has held that $200,000 represents unreasonable compensation. Matt must report a salary of $400,000 and a dividend of $200,000 on his individual tax return. a. True b. False 8. Double taxation of corporate income results because dividend distributions are included in a shareholder’s gross income and are not deductible by the corporation. Powered by Cognero

Page 1


Name:

Class:

Date:

Chapter 02: The Deduction for Qualified Business Income for Noncorporate Taxpayes a. True b. False 9. Jake, the sole shareholder of Peach Corporation (a C corporation) has the corporation pay him $100,000. For income tax purposes, Jake would prefer to have the payment treated as a dividend instead of salary. a. True b. False 10. Thrush Corporation files its Form 1120, which reports taxable income of $200,000 in the current year. The corporation’s tax is $42,000. a. True b. False 11. The corporate marginal income tax rate is lower than the top individual tax rate. a. True b. False 12. Under the Check-the-box Regulations, a two-owner LLC that fails to elect to be to treated as a corporation will be taxed as a sole proprietorship. a. True b. False 13. Employment taxes apply to all entity forms of operating a business. As a result, employment taxes are a neutral factor in selecting the most tax-effective form of operating a business. a. True b. False 14. A C corporation with taxable income of $100,000 in the current year will have a tax liability of $22,250. a. True b. False 15. Katherine, the sole shareholder of Penguin Corporation, has the corporation pay her a salary of $300,000 in the current year. The Tax Court has held that $90,000 represents unreasonable compensation. Katherine has avoided double taxation only to the extent of $210,000 (the portion of the salary that is considered reasonable compensation). a. True b. False 16. One of the purposes of the qualified business income deduction is to reduce the taxes on businesses that are operating in noncorporate business forms (e.g., sole proprietors, partnerships, and S corporations). a. True b. False 17. Instead of providing the qualified business income deduction to owners of noncorporate businesses, Congress could have applied a special tax rate to the business income to achieve a similar result. a. True b. False Powered by Cognero

Page 2


Name:

Class:

Date:

Chapter 02: The Deduction for Qualified Business Income for Noncorporate Taxpayes 18. A qualified trade or business includes any trade or business including providing services as an employee. a. True b. False 19. Unless Congress makes a change, the QBI deduction will expire after 2025. a. True b. False 20. The QBI deduction percentage matches the 21% tax rate applicable to C corporations. a. True b. False 21. Code § 199A permits an individual to deduct 25% of the qualified business income generated through a sole proprietorship, a partnership, or an S corporation. a. True b. False 22. There are three limitations on the qualified business income deduction: an overall limitation (based on modified taxable income), another that applies to high-income taxpayers, and a third that applies to certain types of service businesses. a. True b. False 23. The QBI deduction will reduce both the income tax and self-employment taxes owed by a self-employed individual. a. True b. False 24. Qualified business income (QBI) is defined as the ordinary income less ordinary deductions that a taxpayer

earns from a qualified trade or business (e.g., from a sole proprietorship, S corporation, or partnership) conducted in the United States by the taxpayer. a. True b. False 25. Jane is a self-employed attorney and single. Her annual net earnings from her law practice always exceed $275,000. Jane also has a business selling stained glass windows that she makes. Her earnings from this business are usually about $35,000 per year. Jane claims the standard deduction. Because Jane’s 2024 taxable income exceeds the $241,950 threshold, she may not claim a QBI deduction for either business. a. True b. False 26. Qualified business income includes the reasonable compensation paid to the taxpayer by a qualified trade or business and guaranteed payments made to a partner for services rendered. a. True b. False 27. Ginger is a self-employed driver finding rides via a few different platform companies such as Lyft. In 2024, she is single and claims the $14,600 standard deduction. For 2024, her income from driving is $67,000 and she has no other Powered by Cognero

Page 3


Name:

Class:

Date:

Chapter 02: The Deduction for Qualified Business Income for Noncorporate Taxpayes income. Ginger’s QBI deduction for 2024 is $13,400 ($67,000 x 20%). a. True b. False 28. A partnership will need to report wages paid to its employees as a separate line item on Schedule K-1 to help partners calculate their QBI deduction. a. True b. False 29. Qualified property is used to determine one of the limitations to the qualified business income (QBI)

deduction. Specifically, 2.5% of the unadjusted basis (immediately after acquisition) of qualified property is added to 50% of W-2 wages to determine this limitation. a. True b. False 30. Once a taxpayer reaches certain taxable income thresholds, § 199A limits the qualified business income (QBI) deduction. These thresholds are indexed for inflation every year. a. True b. False 31. For purposes of the qualified business income (QBI) deduction, qualified business income does not include

certain types of investment income [e.g., capital gains or capital losses, dividends, and interest income (unless properly allocable to a trade or business, such as lending)]. a. True b. False 32. Carla is a self-employed online retailer and single. She has no employees. Her annual taxable income is usually around $200,000. Carla could increase her QBI deduction if she incorporated her business, made an S election, and paid herself wages. a. True b. False Multiple Choice 33. Luis is the sole shareholder of a regular C corporation, and Eduardo owns a proprietorship. In the current year, both businesses make a profit of $80,000, and each owner withdraws $50,000 from his business. With respect to this information, which of the following statements is incorrect? a. Eduardo must report $80,000 of income on his return. b. Luis must report $80,000 of income on his return. c. Eduardo’s proprietorship is not required to pay its own income tax, separate from Eduardo, on its $80,000 of income. d. Luis’s corporation must pay income tax on $80,000. 34. Which of the following statements is incorrect about LLCs and the Check-the-box Regulations? a. If an LLC with more than one owner does not make an election, the entity is taxed as a corporation. b. An entity with more than one owner and formed as a corporation cannot elect to be taxed as a partnership. Powered by Cognero

Page 4


Name:

Class:

Date:

Chapter 02: The Deduction for Qualified Business Income for Noncorporate Taxpayes c. If an LLC with one owner does not make an election, the entity is taxed as a sole proprietorship. d. An LLC with one owner can elect to be taxed as a corporation. 35. An individual in a specified service business, such as accounting, with taxable income over the threshold amounts ($241,950 for single or head-of-household taxpayers, or $483,900 if married filing jointly in 2024), will not lose the QBI deduction on such income if: a. Taxable income exceeds the thresholds due to income of a spouse. b. Taxable income did not exceed the thresholds in the prior three years. c. Taxable income exceeds the thresholds because of net capital gain income. d. None of these. 36. In 2024, Sam and Betty, each single, both generate sole proprietor income of $250,000. Sam’s income is generated from a wholesale business while Betty’s is earned from her law practice. Neither has any employees or qualified assets. Both claim the standard deduction and have other income equal to the standard deduction amount. a. Both Sam and Betty will have a QBI deduction of $50,000. b. Sam can obtain a QBI deduction, but Betty cannot because of the taxable income level and law practice is a specified service business. c. Neither Sam nor Betty will be allowed a QBI deduction due to their taxable income levels. d. None of these. 37. In 2024, Layla has $200,000 of QBI from her neighborhood clothing store (a sole proprietorship). Her proprietorship paid $30,000 in W-2 wages and has $20,000 of qualified property. Layla’s spouse earned $51,500 of wages as an employee, and the couple earned $20,000 of interest income during the year. They will be filing jointly and take the standard deduction of $29,200. What is their QBI deduction for 2024? a. $-0-. b. $40,000. c. $48,460. d. $54,000. 38. Jenna owns and manages her single-member LLC, which provides a wide variety of financial services to her clients. She is married and will file a joint tax return with her spouse, Paul. In 2024, her LLC reports $300,000 of net income, W2 wages of $120,000, and assets with an unadjusted basis of $75,000. Their taxable income before the QBI deduction is $285,000 (this is also their modified taxable income). What is their QBI deduction for 2024? a. $0. b. $57,000. c. $60,000. d. $70,000. 39. Ellie (a single taxpayer) is the owner of ABC, LLC. In 2024, the LLC (a sole proprietorship) reports QBI of $900,000 and is not a specified services business. ABC paid total W-2 wages of $300,000, and the total unadjusted basis of property held by ABC is $30,000. Ellie’s taxable income before the QBI deduction is $740,000 (this is also her modified taxable income). What is Ellie’s QBI deduction for 2024? a. $75,750. b. $148,000. c. $150,000. d. $180,000. Powered by Cognero

Page 5


Name:

Class:

Date:

Chapter 02: The Deduction for Qualified Business Income for Noncorporate Taxpayes 40. Danielle is a partner in and sales manager for DG Partners, a domestic business that is not a specified service

trade or business. During the tax year, she receives guaranteed payments of $250,000 from DG Partners for her services to the partnership as its sales manager. In addition, her distributive share of DG Partners’ ordinary income (its only item of income or loss) was $175,000. What is Danielle’s qualified business income? a. $-0-. b. $175,000. c. $250,000. d. $425,000. 41. Aaron is the sole shareholder and CEO of ABC, Inc., an S corporation that is a qualified trade or business.

During the current year, ABC has net income of $325,000 after deducting Aaron’s $100,000 salary. In addition to his compensation, ABC pays Aaron dividends of $250,000. What is Aaron’s qualified business income? a. $-0-. b. $100,000. c. $250,000. d. $325,000. 42. Alicia is the sole shareholder and CEO of ABC, Inc., an S corporation that is a qualified trade or business.

During the current year, ABC has net income of $325,000 after deducting Alicia’s $100,000 salary. In addition to her compensation, ABC pays Alicia dividends of $250,000. After reviewing comparable companies, you determine that reasonable compensation for someone with her experience and responsibilities is $200,000. What is Alicia’s qualified business income? a. $-0-. b. $200,000. c. $225,000. d. $325,000. 43. Taylor, a single taxpayer, has taxable income before the QBI deduction of $221,950 in 2024. Taylor, a CPA,

operates an accounting practice as a single-member LLC (which he reports as a sole proprietorship). During 2024, his proprietorship reports net income of $150,000, W-2 wages of $125,000, and $10,000 of qualified property. What is Taylor’s qualified business income deduction? a. $-0-. b. $12,000. c. $30,000. d. $31,500. 44. Jason and Paula are married. They file a joint return for 2024 on which they report taxable income before the

QBI deduction of $200,000. Jason operates a sole proprietorship, and Paula is a partner in the PQRS Partnership. Both are a qualified trade or business and neither is a specified services business. Jason’s sole proprietorship reports $150,000 of net income, W-2 wages of $45,000, and has qualified property of $50,000. Paula’s partnership reports a loss for the year, and her allocable share of the loss is $40,000. The partnership reports no W-2 wages and Paula’s share of the partnership’s qualified property is $20,000. What is their qualified business income deduction for the year? a. $-0-. Powered by Cognero

Page 6


Name:

Class:

Date:

Chapter 02: The Deduction for Qualified Business Income for Noncorporate Taxpayes b. $11,750. c. $22,000. d. $30,000. e. None of these. 45. Tanuja Singh is a CPA and operates her own accounting firm (Singh CPA, LLC). As a single-member LLC, she reports her accounting firm operations as a sole proprietor. In 2024, Tanuja has QBI from her accounting

firm of $540,000, reports W-2 wages of $156,000, and the unadjusted basis of property used in the LLC is $425,000. Tanuja is married and will file a joint tax return with her spouse. Their taxable income before the QBI deduction is $520,000, and their modified taxable income is $448,000. What is Tanuja’s QBI deduction for 2024? a. $-0-. b. $49,625. c. $78,000. d. $89,600. 46. Which of the following types of income are included in qualified business income (QBI)? a. Income generated from a qualified trade or business. b. Guaranteed payments made in compensation for services performed by a partner to a partnership. c. Wages paid to an employee. d. Income earned from foreign business operations. 47. What happens to the § 199A deduction if a qualified trade or business generates a loss? a. If the net amount of income, gain, deduction, and loss is less than zero, the net amount of the

deduction can be carried back to a previous year or the taxpayer can elect to carry it forward. b. If the net amount of income, gain, deduction, and loss is less than zero, the net amount of the deduction is lost and is not available to carryforward or carryback. c. If the net amount of income, gain, deduction, and loss is less than zero, the net amount is treated as a loss in the succeeding year. d. None of these. 48. Where is the § 199A deduction taken on Form 1040? a. It is a deduction from AGI, much like the standard deduction or itemized deductions, and is the last

deduction taken in determining taxable income. b. It is a business deduction and is taken on Schedule C (Form 1040). c. It is a deduction that reduces self-employment income and is taken on Schedule SE (Form 1040). d. It is an itemized deduction taken on Schedule A (Form 1040). 49. Which of the following is considered qualified property in the calculation of the deduction for qualified

business income (§ 199A)? a. All business property (both tangible and intangible). b. Tangible business property subject to depreciation. c. Tangible property placed in service during the year, but not used in the production of qualified Powered by Cognero

Page 7


Name:

Class:

Date:

Chapter 02: The Deduction for Qualified Business Income for Noncorporate Taxpayes business income. d. Fully depreciated tangible business property. 50. In 2024, Kendra has taxable income before the QBI deduction of $274,000. Kendra is single and has income

from her law firm (a sole proprietorship operating as an LLC) of $200,000. Her law firm paid wages of $82,000 and has qualified property of $20,000. What is Kendra’s QBI deduction? a. $0. b. $40,000. c. $41,000. d. $54,800. Multiple Response 51. Which of the following taxpayers is eligible for a qualified business income deduction regarding the activity noted? (circle all that apply) a. Tom’s Burger Place, a sole proprietorship. b. A driver for Uber or Lyft. c. An employee working for Apple, Inc. d. A partner of a Big 4 firm. 52. Which of the following taxpayers is potentially eligible for a qualified business income deduction based on the noted activity? (circle all that apply) a. A shareholder of General Electric. b. A sole proprietor operating a restaurant. c. A self-employed doctor. d. Jennifer, owner of a winery operated as an S corporation. 53. Which of the following self-employed individuals are in a specified service trade or business? (circle all that apply) a. Dentist. b. Consultant. c. Architect. d. CPA. Subjective Short Answer 54. Rebecca and Elizabeth are married and will file jointly in 2024. Rebecca earns $300,000 from her single-member LLC (a law firm). She reports her business as a sole proprietorship. Wages paid by the law firm amount to $40,000; the law firm has no significant property. Elizabeth is employed as a tax manager by a local CPA firm. Their modified taxable income is $443,900 (this is also their taxable income before the deduction for qualified business income). Determine their QBI deduction for 2024. 55. Ashley (a single taxpayer) is the owner of ABC, LLC. In 2024, the LLC (a sole proprietorship) reports QBI

of $900,000 and is not a specified services business. ABC paid total W-2 wages of $300,000, and the total unadjusted basis of property held by ABC is $30,000. Ashley’s taxable income before the QBI deduction is $740,000 (this is also her modified taxable income). What is Ashley’s QBI deduction for 2024? Powered by Cognero

Page 8


Name:

Class:

Date:

Chapter 02: The Deduction for Qualified Business Income for Noncorporate Taxpayes 56. Susan, a single taxpayer, owns and operates a bakery (as a sole proprietorship). The business is not a

specified services business. In 2024, the business pays $60,000 in W-2 wages, has $150,000 of qualified property, and $200,000 in net income (all of which is qualified business income). Susan also has a part-time job earning wages of $15,200, receives $4,000 of interest income, and will take the standard deduction. What is Susan’s qualified business income deduction? 57. Ben owns and operates a machine repair shop as a sole proprietorship. It generates a profit of about

$180,000 annually. The business pays wages of about $50,000 annually. The building and most of the equipment are leased so there is no qualified property. Ben files as single and claims the standard deduction. He has a large unrealized gain in bitcoin that he acquired in 2019 and is wondering when he should sell it and whether he should sell it all in one year or over a few years. Advise Ben as to how the sale of the bitcoin and its resulting capital gain can affect his QBI deduction in 2024. 58. Sergio owns and manages his single-member LLC which provides a wide variety of accounting services to

his clients. He is married and will file a joint tax return with his spouse, Goretty. His LLC reports $250,000 of net income, W-2 wages of $120,000, and assets with an unadjusted basis of $75,000. Their taxable income before the QBI deduction is $215,000 (this is also their modified taxable income). Determine their QBI deduction for 2024. 59. Taylor owns a wide variety of commercial rental properties held in a single-member LLC. Her LLC reports

rental income of $750,000. The LLC pays no W-2 wages; rather, it pays a management fee to an S corporation that Taylor controls. The management company pays W-2 wages, but reports no income (or loss). Taylor’s total unadjusted basis of the commercial rental property is $5,000,000 and her taxable income before the QBI deduction (and her modified taxable income) is $1,000,000. What is Taylor’s QBI deduction for 2024? 60. Jansen, a single taxpayer, owns and operates a restaurant (as a sole proprietorship). The business is not a

specified services business. In 2024, the business pays $125,000 in W-2 wages, has $187,500 of qualified property, and $440,000 in net income (all of which is qualified business income). Jansen has no other items of income or loss and will take the standard deduction. What is Jansen’s qualified business income deduction? 61. Felicia, who is single, operates three sole proprietorships that generate the following information in 2024

(none are “specified services” businesses): Business A B C

QBI $240,000 $(108,000) $120,000

W-2 Wages $72,000 $48,000 $-0-

Capital Investment $ -0$ -0$ -0-

Felcia chooses not to aggregate the businesses. She also earns $150,000 of wages from an unrelated business and her modified taxable income (before any QBI deduction) is $304,000. a. b.

What is Felicia’s QBI deduction? Assume that Felicia can aggregate these businesses. Determine her QBI deduction if she decides to aggregate the businesses.

Essay Powered by Cognero

Page 9


Name:

Class:

Date:

Chapter 02: The Deduction for Qualified Business Income for Noncorporate Taxpayes 62. Compare the basic tax and nontax factors of doing business as a partnership, an S corporation, and a C

corporation. Circle the correct answers. Tax Questions

Column A Partnership

Column B S Corporation

Who pays tax on the entity’s income?

Partners Partnership

Shareholders S corporation

Column C C Corporation Shareholders C Corporation

Are operating losses passed through to owners?

Yes No

Yes No

Yes No

Yes No

Yes No

Yes No

Are distributions of profits taxable to owners?

Yes No

Yes No

Yes No

Nontax Factors

Partnership

S Corporation

C Corporation

Is the liability of owners limited?

Yes No

Yes No

Yes No

Is there free transferability of ownership interests?

Yes No

Yes No

Yes No

Are capital gains (losses) reported on owners’ tax returns as such?

63. Sofía is the sole shareholder of Thrush Corporation, a C corporation. In the current year, Thrush earned $350,000 and distributed $75,000 to Sofía. Kirk is the sole shareholder of Swallow Corporation, an S corporation. In the current year, Swallow earned $350,000 and distributed $75,000 to Kirk. Contrast the tax treatment of Thrush Corporation and Sofía with the tax treatment of Swallow Corporation and Kirk. 64. What is a limited liability company? What favorable nontax and tax attributes does the LLC entity form offer taxpayers? 65. The qualified business income deduction is severely limited for specified services businesses. What is a specified services trade or business? 66. Describe the limitations on the qualified business income deduction that apply to high income taxpayers. 67. How does property used in a qualified trade or business factor into the QBI deduction calculation? What

types of property are considered for the QBI deduction? Powered by Cognero

Page 10


Name:

Class:

Date:

Chapter 02: The Deduction for Qualified Business Income for Noncorporate Taxpayes Answer Key 1. True 2. False 3. True 4. True 5. False 6. True 7. True 8. True 9. True 10. True 11. True 12. False 13. False 14. False 15. True 16. True 17. True 18. False 19. True 20. False 21. False 22. True 23. False 24. True 25. False Powered by Cognero

Page 11


Name:

Class:

Date:

Chapter 02: The Deduction for Qualified Business Income for Noncorporate Taxpayes 26. False 27. False 28. True 29. False 30. True 31. True 32. True 33. b 34. a 35. d 36. c 37. b 38. b 39. b 40. b 41. d 42. c 43. b 44. c 45. a 46. a 47. c 48. a 49. b 50. a Powered by Cognero

Page 12


Name:

Class:

Date:

Chapter 02: The Deduction for Qualified Business Income for Noncorporate Taxpayes 51. a, b, d 52. b, c, d 53. a, b, d 54. Normally, Rebecca and Elizabeth would be entitled to a QBI deduction of $60,000 ($300,000 x 20%). But

since their taxable income exceeds the threshold for married taxpayers ($383,900), and Rebecca’s QBI is from a specified services business (a law firm), their QBI deduction is limited to $14,400, computed as follows: (1)

Determine Applicable Percentage:

Applicable % = 100%–$60,000 ($443,900 – $383,900) $100,000 (2) Determine QBI deduction: 1. 20% of QBI ($300,000 x 20%)

= 40%

$60,000

x Applicable percentage

2.

x 40% $24,000

But no more than the greater of: •

50% of W-2 wages ($40,000 x 50% x 40%), or

25% of W-2 wages ($40,000 x 25% x 40%) plus

$ 8,000

$4,000 •

2.5% of the unadjusted basis of qualified property 2.5% x 40%)

($-0- x

-0- $ 4,000

Because Rebecca and Elizabeth’s modified taxable income exceeds $383,900, but is less than $483,900 and the W-2 Wages/Capital portion of the computation is the limiting factor, the general 20% QBI amount is used, but reduced as follows:

(1)

Determine difference between the general 20% QBI deduction amount and the W-2 Wages/Capital amount.

General 20% QBI deduction amount Less: The W-2 Wages/Capital Investment limit Excess

$ 24,000

( 8,000) $16,000

(2) Determine the Reduction Ratio: Reduction Ratio = $60,000 ($443,900 – $383,900) = 60% Powered by Cognero

Page 13


Name:

Class:

Date:

Chapter 02: The Deduction for Qualified Business Income for Noncorporate Taxpayes $100,000

(3)

Determine the reduction in the W-2 Wages/Capital Investment limit: Excess ($16,000) x Reduction Ratio (60%) = $9,600

(4) Determine final QBI amount:

General 20% QBI deduction amount Less: Reduction in the W-2 Wages/Capital limit Final QBI amount

$24,000 ( 9,600) $14,400

55. As Ashley’s taxable income before the QBI deduction exceeds the $241,950 threshold, the W-2

Wages/Capital Investment Limit must be considered. Ashley’s QBI deduction is $148,000, computed as follows: 1.

20% of QBI ($900,000 x 20%)

2.

But no more than the greater of: •

50% of W-2 wages ($300,000 x 50%), or

25% of W-2 wages ($300,000 x 25%) plus

2.5% of the unadjusted basis of qualified property ($30,000 x 2.5%)

$180,000

$150,000 $ 75,000 750 $ 75,750

And, no more than: 3.

20% of modified taxable income ($740,000 x 20%)

$148,000

56. Susan’s taxable income before the QBI deduction is $204,600 (her proprietorship net income of

$200,000 plus her wages of $15,200 and her $4,000 of interest income less her $14,600 standard deduction). Because Susan’s taxable income before the QBI deduction exceeds $191,950, the W-2 Powered by Cognero

Page 14


Name:

Class:

Date:

Chapter 02: The Deduction for Qualified Business Income for Noncorporate Taxpayes Wages/Capital Investment limit must be considered: $ 40,000

1. 20% of QBI ($200,000 x 20%) 2.

But no more than the greater of: •

50% of W-2 wages ($60,000 x 50%), or

25% of W-2 wages ($60,000 x 25%) plus

• •

2.5% of the unadjusted basis of qualified property ($150,000 x 2.5%)

$ 30,000 $ 15,000

3,750

$ 18,750

And, no more than:

$ 40,920

3. 20% of modified taxable income ($204,600 x 20%)

So, initially, Susan’s QBI deduction is limited to $30,000. However, as Susan’s taxable income before the QBI deduction exceeds $191,950, but is less than $241,950 and the W-2 Wages/Capital Investment portion of the computation is the limiting factor, the general 20% QBI amount is used, but reduced as follows: (1)

Determine difference between the general 20% QBI deduction amount and the W-2 Wages/Capital amount.

General 20% QBI deduction amount Less: The W-2 Wages/Capital limit Excess

$40,000 (30,000) $10,000

(2) Determine the Reduction Ratio: Reduction Ratio = $12,650 ($204,600 – $191,950) = 25.3% $50,000

(3)

Determine the reduction in the W-2 Wages/Capital Limit: excess ($10,000) x Reduction Ratio (25.3%) = $2,530

(4) Determine Final QBI Amount:

General 20% QBI deduction amount Less: Reduction in the W-2 Wages/Capital limit Powered by Cognero

$40,000 ( 2,530) Page 15


Name:

Class:

Date:

Chapter 02: The Deduction for Qualified Business Income for Noncorporate Taxpayes Final QBI Amount

$37,470

57. The capital gain might increase Ben’s taxable income to the point that it could exceed $191,950. At that

point, his QBI deduction will be limited to 50% of the W-2 wages paid ($25,000). If taxable income remains at $191,950 or less, his QBI deduction will be 20% of his income from the repair business ($36,000; $180,000 x 20%). Ben might want to sell all of the bitcoin in one year rather than over several years if doing so prevents his taxable income from exceeding $191,950 and thereby imposing a limit on his QBI deduction. 58. Even though this is a “specified services” business, Sergio and Goretty’s taxable income before the QBI

deduction is below the $383,900 threshold in 2024. As a result, their QBI deduction is $43,000, computed as follows: 1.

20% of qualified business income ($250,000 x 20%), or $50,000

2.

20% of modified taxable income ($215,000 x 20%)

$43,000

59. Because Taylor’s modified taxable income exceeds the $483,900 threshold in 2024, the W-2 Wages/Capital

Investment Limit comes into play. Taylor’s QBI deduction is $125,000, computed as follows:

1. 20% of qualified business income ($750,000 x 20%)

$150,000

2. But no more than the greater of: $

-50% of W-2 wages ($-0- x 50%), or -25% of W-2 wages ($-0- x 25%) plus 0-2.5% of the unadjusted basis of qualified property ($5,000,000 x 2.5%)

$

-0-

-

125,000 $ 125,000

And, no more than: 20% of modified taxable income ($1,000,000 x 3. 20%)

$ 200,000

60. Jansen’s taxable income before the QBI deduction is $425,400 (his proprietorship net income of Powered by Cognero

Page 16


Name:

Class:

Date:

Chapter 02: The Deduction for Qualified Business Income for Noncorporate Taxpayes $440,000 less the $14,600 single standard deduction); this is also his modified taxable income. Because Jansen’s taxable income before the QBI deduction exceeds the $241,950 threshold, the W-2 Wages/Capital Investment

limit must be considered. Jansen’s QBI deduction is $62,500, computed as follows: 1. 20% of qualified business income ($440,000 x 20%) $ 88,000 2. But no more than the greater of: 50% of W-2 wages ($125,000 x 50%), or 25% of W-2 wages ($125,000 x 25%) plus 2.5% of the unadjusted basis of qualified property ($187,500 x 2.5%)

$ 62,500 $31,250 4,688

$ 35,938

And, no more than:

3. 20% of modified taxable income ($425,400 x 20%)

$ 85,080

61. a. Under Reg. § 1.199A-1(d), Felicia must allocate Business B’s negative QBI

to Business A and Business C in proportion to their positive QBI amounts ($240,000 for Business A; $120,000 for Business C). As a result, the negative QBI from Business B is apportioned 66.66% to Business A and 33.33% to Business C. So $(72,000) is apportioned to Business A and $(36,000) to Business C. Business A

Adjusted QBI $168,000 ($240,000 – $72,000) $-0- [$(108,000) + $108,000] $84,000 ($120,000 $36,000)

B C

W-2 Wages $72,000

Capital Investment $ -0-

$48,000

$ -0-

$-0-

$ -0-

Felicia now applies the “W-2 Wages” limitation by determining the lesser of 20% of QBI and 50% of W-2 wages for each business.

Business

QBI x 20%

A B C

$33,600 ($168,000 x 20%) $ -0$16,800 ($84,000 x 20%)

W-2 Wages x 50% $36,000 $24,000 $ -0-

Lesser $33,600 $ -0$ -0-

Felicia’s “combined qualified business income amount” is $33,600 ($33,600 + $-0- + $-0-). Because this amount is less than 20% of Felicia’s modified taxable income ($60,800; $304,000 x 20%), Powered by Cognero

Page 17


Name:

Class:

Date:

Chapter 02: The Deduction for Qualified Business Income for Noncorporate Taxpayes Felicia’s QBI deduction is $33,600 and her taxable income is $270,400. There is no carryover of any loss into the following taxable year for purposes of § 199A (the Business B negative QBI was completely used).

b. Because Felicia’s taxable income is above the threshold amount, her QBI deduction is subject to the W-2 wages and capital investment limitations. Because the businesses are aggregated, these limitations are applied on an aggregated basis. Business A B C Total

QBI $240,000 $(108,000) $120,000 $252,000

W-2 Wages $72,000 $48,000 $-0$120,000

Capital Investment $ -0$ -0$ -0$ -0-

None of the businesses own “qualified property.” As a result, only the “W-2 Wages” limitation applies. Felicia’s “combined qualified income amount” is $50,400, the lesser of 20% of the QBI from the aggregated businesses ($50,400; $252,000 x 20%), or 50% of W-2 wages from the aggregated businesses ($60,000; $120,000 x 50%). Felicia’s QBI deduction is equal to the lesser of $50,400 or 20% of her modified taxable income ($60,800; $304,000 x 20%). As a result, Felicia’s QBI deduction is $50,400, and her taxable income is $253,600. By aggregating her businesses, Felicia has increased the size of her QBI deduction.

62. The correct answers are shaded. Tax Questions

Column A Partnership

Column B S Corporation

Column C C Corporation

Who pays tax on the entity’s income?

Partners Partnership

Shareholders S corporation

Shareholders C Corporation

Are operating losses passed through to owners?

Yes No

Yes No

Yes No

Are capital gains (losses) reported on owners’ tax returns as such?

Yes No

Yes No

Yes No

Are distributions of profits taxable to owners?

Yes No

Yes No

Yes No

Nontax Factors

Partnership

S Corporation

C Corporation

Powered by Cognero

Page 18


Name:

Class:

Date:

Chapter 02: The Deduction for Qualified Business Income for Noncorporate Taxpayes Is the liability of owners limited?

Yes No

Yes No

Yes No

Is there free transferability of ownership interests?

Yes No

Yes No

Yes No

63. A C corporation is a separate taxable entity; thus, Thrush Corporation is taxed on the $350,000 of earnings. Income of a C corporation has no effect on the shareholders until such time a dividend is paid. When dividends are paid, shareholders must report dividend income on their tax returns. Thus, Sofía is taxed on $75,000 of dividends, and the 0%/15%/20% preferential tax rate applies with respect to the dividends. Generally, an S corporation is not subject to an entity-level Federal income tax. Instead, the corporation’s income, gains, deductions, and losses are passed through to and reported by the shareholders on their tax returns. Thus, Swallow reports the $350,000 of earnings on its tax return (Form 1120S) but pays no income tax. Kirk is taxed on the $350,000 of earnings from Swallow on his individual income tax return (Form 1040). Distributions from S corporations are not taxable to the shareholder (to the extent of stock basis). Thus, Kirk is not taxed on the $75,000 distribution from Swallow. 64. Similar to the corporate entity form, a limited liability company is an entity created under the laws of a specific state (or the District of the Columbia) and, pursuant to such laws, an LLC has the corporate feature of limited liability. This is the primary nontax characteristic that makes LLC status attractive. Other nontax attributes that are available with the LLC entity form include centralized management, continuity of life, and free transferability of ownership interests. Which of these nontax attributes are allowed will be dependent on the laws of the state of LLC organization. The principal tax advantage of the LLC entity form is the ability to avoid double taxation of the entity’s profits. Most LLCs will be taxed as either partnerships (two or more owner LLCs) or sole proprietorships (one-owner LLCs), although the Check-the-box Regulations do provide the opportunity to have an LLC taxed as a corporation (including an S corporation). 65. A specified service trade or business includes those involving: The performance of services in certain fields, including health, law, accounting, actuarial science, performing arts, consulting, athletics, financial services, and brokerage services; Services consisting of investing and investment management, trading or dealing in securities, partnership interests, or commodities; and Any trade or business in which its principal asset is the reputation of one or more of its employees or owners. Architects and engineers are specifically excluded from this definition.

66. The basic application of § 199A becomes considerably more complex once a taxpayer reaches certain taxable income thresholds. These taxable income thresholds – determined without regard to the QBI deduction are $383,900 for married taxpayers filing jointly and $191,950 for single taxpayers in 2024. These amounts will be indexed for inflation annually. Powered by Cognero

Page 19


Name:

Class:

Date:

Chapter 02: The Deduction for Qualified Business Income for Noncorporate Taxpayes Once these thresholds are reached, § 199A imposes two independent limitations:

1.

2.

First, § 199A imposes a cap on the QBI deduction that is determined by reference to a percentage of the W-2 wages paid by the business (i.e., wages paid to its employees) or by references to a smaller percentage of W-2 wages paid and a percentage of the cost of its depreciable property used to produce QBI. Second, the QBI deduction generally is not available for income earned from certain specified service businesses.

67. Qualified property is used to determine one of the limitations to the QBI deduction. Specifically, 2.5 percent

of qualified property is added to 25 percent of W-2 wages to determine this limitation. Qualified property includes depreciable tangible property – real or personal – that is used by the QTB during the year and whose “depreciable period” has not ended before the end of the taxable year. As a result, land and intangible assets are not qualified property. Given the broad-based changes to MACRS – allowing taxpayers to expense (via § 179 and/or bonus depreciation) property other than real estate – the depreciable period for qualified property under § 199A is a minimum of 10 years.

Powered by Cognero

Page 20


Name:

Class:

Date:

Chapter 03: Corporations Introduction and Operating Rules True / False 1. Thrush Corporation, a calendar year C corporation, files its current year Form 1120, which reports taxable income of $200,000 for the year. The corporation’s tax is $42,000. a. True b. False 2. For taxable years beginning in 2024, the corporate income tax is a flat 25%. a. True b. False 3. A personal service corporation must use a calendar year and is not permitted to use a fiscal year. a. True b. False 4. As a general rule, C corporations must use the accrual method of accounting. However, under several exceptions to this rule (e.g., average annual gross receipts of $30,000,000 or less for the most recent 3-year period), a C corporation can use the cash method. a. True b. False 5. On December 31, 2024, Lavender, Inc., (an accrual basis, calendar year C corporation), accrues a $50,000 bonus to Barry, its vice president and a 40% shareholder. Lavender pays the bonus to Barry, who is a cash basis taxpayer, on March 10, 2025. Lavender can deduct the bonus in 2025, the year in which it is included in Barry’s gross income. a. True b. False 6. Azure Corporation, a C corporation, had a long-term capital gain of $50,000 in the current year. The amount of tax applicable to the capital gain is $10,500 ($50,000 × 21%). a. True b. False 7. Albatross, a C corporation, had $140,000 net income from operations and a $25,000 short-term capital loss in the current year. Albatross Corporation’s taxable income is $140,000. a. True b. False 8. If a C corporation uses straight-line depreciation on real estate (§ 1250 property), no portion of a recognized gain on the sale of the property will be recaptured as ordinary income. a. True b. False 9. The passive activity loss rules apply to closely held C corporations and to personal service corporations but not to S corporations. a. True b. False Powered by Cognero

Page 1


Name:

Class:

Date:

Chapter 03: Corporations Introduction and Operating Rules 10. Peach Corporation had $210,000 of net active income, $45,000 of portfolio income, and a $230,000 passive activity loss during the current year. If Peach is a closely held C corporation that is not a PSC, it can deduct $210,000 of the passive activity loss in the year. a. True b. False 11. On December 16, 2024, the directors of Quail Corporation (an accrual basis, calendar year C corporation) authorized a cash donation of $5,000 to the American Cancer Society, a qualified charity. The payment, which is made on April 6, 2025, may be claimed as a deduction for tax year 2024. a. True b. False 12. In the current year, Oriole Corporation donated a painting worth $30,000 to the Texas Art Museum, a qualified public charity. The museum included the painting in its permanent collection. Oriole Corporation purchased the painting five years ago for $10,000. Oriole’s charitable contribution deduction is $30,000 (ignoring the taxable income limitation). a. True b. False 13. In the current year, Crow Corporation, a C corporation, donated scientific property (basis of $30,000, fair market value of $50,000) to State University, a qualified charitable organization, to be used in research. Crow had held the property for four months as inventory. Crow Corporation may deduct $50,000 for the charitable contribution (ignoring the taxable income limitation). a. True b. False 14. The $1,000,000 limitation on the deduction of executive compensation currently applies to compensation paid to a publicly traded corporation's principal executive officer, principal financial officer, and board of directors. a. True b. False 15. Heron Corporation, a calendar year C corporation, had an excess charitable contribution for 2023 of $5,000. In 2024, Heron made a further charitable contribution of $20,000. Heron’s 2024 deduction is limited to $15,000 (10% of taxable income). The 2024 contribution must be applied first against the $15,000 limitation. a. True b. False 16. A corporate net operating loss arising in 2024 for a calendar year C corporation can be carried back 2 years and forward 20 years to offset taxable income for those years. a. True b. False 17. In the current year, Azul Corporation, a calendar year C corporation, received a dividend of $30,000 from Naranja Corporation. Azul owns 25% of the Naranja Corporation stock. Assuming it is not subject to the taxable income limitation, Azul’s dividends received deduction is $19,500. a. True b. False Powered by Cognero

Page 2


Name:

Class:

Date:

Chapter 03: Corporations Introduction and Operating Rules 18. Because of the taxable income limitation, no dividends received deduction is allowed if a corporation has an NOL for the current taxable year. a. True b. False 19. No dividends received deduction is allowed unless the corporation has held the stock for more than 90 days. a. True b. False 20. Hornbill Corporation, a cash basis and calendar year C corporation, was formed and began operations on May 1, 2024. Hornbill incurred the following expenses during its first year of operations (May 1 – December 31, 2024): temporary directors meeting expenses of $10,500, state of incorporation fee of $5,000, stock certificate printing expenses of $1,200, and legal fees for drafting corporate charter and bylaws of $7,500. Wanting to deduct as much as possible in the current year, Hornbill Corporation’s 2024 deduction for organizational expenditures is $5,800. a. True b. False 21. Lilac Corporation incurred $4,700 of legal and accounting fees associated with its incorporation. The $4,700 is deductible as startup expenditures on Lilac’s tax return for the year in which it begins business. a. True b. False 22. A calendar year personal service corporation with taxable income of $100,000 in the current year will have a tax liability of $21,000. a. True b. False 23. The accumulated earnings and personal holding company taxes both can be avoided by distributing sufficient dividends. a. True b. False 24. A calendar year C corporation can receive an automatic 9-month extension to file its corporate return (Form 1120) by timely filing a Form 7004 for the tax year. a. True b. False 25. A corporation must file a Federal income tax return even if it has no taxable income for the year. a. True b. False 26. For purposes of the estimated tax payment rules, a “large corporation” is defined as a corporation that had taxable income of $1,000,000 or more in any of the three preceding years. a. True b. False 27. Schedule M-1 is used to reconcile net income as computed for financial accounting purposes with taxable income Powered by Cognero

Page 3


Name:

Class:

Date:

Chapter 03: Corporations Introduction and Operating Rules reported on the corporation’s income tax return. a. True b. False 28. An expense that is deducted in computing net income per books but not deductible in computing taxable income is a subtraction item on Schedule M-1. a. True b. False 29. On December 31, 2024, Flamingo, Inc., a calendar year, accrual method C corporation, accrues a bonus of $50,000 to its president (a cash basis taxpayer) who owns 75% of the corporation’s outstanding stock. The $50,000 bonus is paid to the president on February 4, 2025. For Flamingo’s 2024 Form 1120, the $50,000 bonus will be a subtraction item on Schedule M-1. a. True b. False 30. Income that is included in net income per books but not included in taxable income is a subtraction item on Schedule M-1. a. True b. False 31. Schedule M-2 is used to reconcile unappropriated retained earnings at the beginning of the year with unappropriated retained earnings at the end of the year. a. True b. False 32. A corporation with $5,000,000 or more in assets must file Schedule M-3 (instead of Schedule M-1). a. True b. False 33. Schedule M-3 is similar to Schedule M-1 in that the form is designed to reconcile net income per books with taxable income. However, an objective of Schedule M-3 is more transparency between financial statements and tax returns than that provided by Schedule M-1. a. True b. False 34. Katherine, the sole shareholder of Penguin Corporation, has the corporation pay her a salary of $300,000 in the current year. The Tax Court has held that $90,000 represents unreasonable compensation. Katherine has avoided double taxation only to the extent of $210,000 (the portion of the salary that is considered reasonable compensation). a. True b. False 35. In general, all corporations that maintain inventory for sale to customers are required to use the accrual method of accounting for determining sales and cost of goods sold. a. True b. False Powered by Cognero

Page 4


Name:

Class:

Date:

Chapter 03: Corporations Introduction and Operating Rules 36. Canary Corporation, a calendar year C corporation, received an $80,000 dividend from Stork Corporation. Canary owns 18% of the Stork Corporation stock. Assuming it is not subject to the taxable income limitation, Canary’s dividends received deduction is $40,000. a. True b. False 37. The limitation on the deduction of business interest does not apply to noncorporate taxpayers. a. True b. False 38. The accumulated earnings and personal holding company taxes are designed to prevent the accumulation of earnings within a corporation. a. True b. False 39. Canary Corporation, which sustained a $5,000 net short-term capital loss during the year, will enter $5,000 as an addition on Schedule M-1 of Form 1120. a. True b. False 40. In tax planning for charitable contributions, a current-year’s contribution might have to be deferred to a later year in order to deduct a contribution carryover amount. a. True b. False 41. An alternative minimum tax applies to a 2024 calendar year C corporation with more than $1 billion average annual adjusted financial statement income for the 3-year period ending December 31, 2023. a. True b. False Multiple Choice 42. Elk, a C corporation, has $370,000 operating income and $290,000 operating expenses during the current year. In addition, Elk has a $10,000 long-term capital gain and a $17,000 short-term capital loss. Elk’s taxable income is: a. $63,000. b. $73,000. c. $80,000. d. $90,000. 43. Patrick, an attorney, is the sole shareholder of Gander Corporation, a C corporation. Gander is a personal service corporation with a fiscal year ending November 30 (pursuant to a § 444 election). The corporation paid Patrick a salary of $180,000 during its fiscal year ending November 30, 2024. How much salary must Gander pay Patrick during the period December 1 through December 31, 2024, to permit the corporation to continue to use its fiscal year without negative tax effects? a. $0 b. $15,000 c. $30,000 Powered by Cognero

Page 5


Name:

Class:

Date:

Chapter 03: Corporations Introduction and Operating Rules d. $180,000 44. Copper Corporation, a C corporation, had gross receipts of $30,000,000 in 2021, $31,000,000 in 2022, and $28,000,000 in 2023. Gold Corporation, a personal service corporation (PSC), had gross receipts of $28,000,000 in 2021, $31,000,000 in 2022, and $32,000,000 in 2023. Which of the corporations will be allowed to use the cash method of accounting in 2024? a. Copper Corporation only. b. Gold Corporation only. c. Both Copper Corporation and Gold Corporation. d. Neither Copper Corporation nor Gold Corporation. 45. Saleh, an accountant, is the sole shareholder of Turquoise Corporation, a C corporation. Turquoise is a personal service corporation with a fiscal year ending September 30 (pursuant to a § 444 election). The corporation paid Saleh a salary of $330,000 during its fiscal year ending September 30, 2024. How much salary must Turquoise pay Saleh during the period October 1 through December 31, 2024, if the corporation is to continue to use its fiscal year without negative tax effects? a. $0 b. $27,500 c. $82,500 d. $247,500 46. On December 31, 2024, Peregrine Corporation, an accrual method, calendar year taxpayer, accrued a performance bonus of $100,000 to Charles, a cash basis, calendar year taxpayer. Charles is president and sole shareholder of the corporation. When can Peregrine deduct the bonus? a. In 2024, if the bonus was authorized by the Board of Directors and payment was made on or before April 15, 2025. b. In 2025, if payment was made at any time during that year. c. In 2024, if payment was made on or before April 15, 2025. d. In 2025, but only if payment was made on or before April 15, 2025. 47. Ivory Corporation, a calendar year, accrual method C corporation, has two cash method, calendar year shareholders who are unrelated to each other. Craig owns 35% of the stock, and Oscar owns the remaining 65%. During 2024, Ivory paid a salary of $100,000 to each shareholder. On December 31, 2024, Ivory accrued a bonus of $25,000 to each shareholder. Assuming that the bonuses are paid to the shareholders on February 1, 2025, compute Ivory Corporation’s 2024 deduction for the above amounts. a. $250,000 b. $225,000 c. $200,000 d. $125,000 48. Carrot Corporation, a C corporation, has a net short-term capital gain of $65,000 and a net long-term capital loss of $250,000 during 2024. Carrot Corporation had taxable income from other sources of $720,000. Prior years’ transactions included the following: 2020 2021 Powered by Cognero

Net long-term capital gain Net short-term capital gain

$150,000 60,000 Page 6


Name:

Class:

Date:

Chapter 03: Corporations Introduction and Operating Rules 2022 2023

Net short-term capital gain Net long-term capital gain

45,000 35,000

Compute the amount of Carrot’s capital loss carryover to 2025. a. $0 b. $32,000 c. $45,000 d. $185,000 49. In 2024, Bluebird Corporation had net income from operations of $100,000. Further, Bluebird recognized a long-term capital gain of $30,000 and a short-term capital loss of $45,000. Which of the following statements is correct? a. Bluebird Corporation will have taxable income in 2024 of $100,000 and will have a net capital loss of $15,000 that can be carried back 3 years and forward 5 years. b. Bluebird Corporation may use the capital loss to offset the capital gain and must carry the net capital loss of $15,000 forward five years as a short-term capital loss. c. Bluebird Corporation may deduct $33,000 of the capital loss in 2024 and may carry forward the remainder of the capital loss indefinitely to offset capital gains. d. Bluebird Corporation will have taxable income in 2024 of $85,000. 50. In the current year, Sunset Corporation (a C corporation) had operating income of $200,000 and operating expenses of $175,000. In addition, Sunset had a $30,000 long-term capital gain, a $52,000 short-term capital loss, and $5,000 taxexempt interest income. What is Sunset Corporation’s taxable income for the year? a. $0 b. $3,000 c. $22,000 d. $25,000 51. Beige Corporation, a C corporation, purchases a warehouse on August 1, 2008, for $1,000,000. Straight-line depreciation is taken in the amount of $411,750 before the property is sold on June 12, 2024, for $1,200,000. What is the amount and character of the gain recognized by Beige on the sale of the realty? a. Ordinary income of $0 and § 1231 gain of $611,750. b. Ordinary income of $411,750 and § 1231 gain of $200,000. c. Ordinary income of $82,350 and § 1231 gain of $529,400. d. Ordinary income of $117,650 and § 1231 gain of $494,100. 52. In the current year, Woodchuck, Inc., a closely held personal service corporation, has $115,000 of net active income, $40,000 of portfolio income, and $135,000 of passive activity loss. What is Woodchuck’s taxable income for the current year? a. $0 b. $20,000 c. $40,000 d. $155,000 53. Grackle Corporation, a personal service corporation, had $230,000 of net active income, $40,000 of portfolio income, and a $250,000 passive activity loss during the current year. How much is Grackle’s taxable income for the year? a. $20,000 Powered by Cognero

Page 7


Name:

Class:

Date:

Chapter 03: Corporations Introduction and Operating Rules b. $40,000 c. $270,000 d. $520,000 54. Grebe Corporation, a closely held corporation that is not a PSC, had $75,000 of net active income, $60,000 of portfolio income, and a $105,000 passive activity loss during the current year. How much of the passive activity loss can Grebe deduct in the current year? a. $0 b. $60,000 c. $75,000 d. $105,000 55. In the current year, Violet, Inc., a closely held corporation (not a PSC), has $55,000 of passive activity loss, $80,000 of net active income, and $20,000 of portfolio income. How much is Violet’s taxable income for the current year? a. $20,000 b. $45,000 c. $80,000 d. $100,000 56. Kingbird Corporation (a calendar year C corporation) reports the following income and expenses in the current year: Income from operations Expenses from operations Dividends received (15% ownership)

$200,000 140,000 15,000

On October 1, Kingbird Corporation made a contribution to a qualified charitable organization of $9,000 in cash (not included in any of the above items). Determine Kingbird’s charitable contribution deduction for the year. a. $9,000 b. $7,500 c. $6,750 d. $6,525 57. Owl Corporation (a C corporation), a retailer of children’s apparel, made the following donations to qualified charitable organizations in the current year. Adjusted Basis Fair Market Value Children’s clothing held as inventory, to Haven for Hope

$10,000

$15,000

Stock in Exxon Corporation acquired two years ago and held as an investment, to City University

5,000

3,000

Land acquired four years ago and held as an investment, to Humane Society

50,000

75,000

How much qualifies for the charitable contribution deduction (ignoring the taxable income limitation)? Powered by Cognero

Page 8


Name:

Class:

Date:

Chapter 03: Corporations Introduction and Operating Rules a. $63,000 b. $65,000 c. $90,500 d. $92,500 58. Plum Corporation (a C corporation and a computer manufacturer) donated 100 laptop computers to a local university (a qualified educational organization) in the current year. The computers were constructed by Plum earlier this year, and the university will use the computers for research and research training. Plum’s basis in the computers is $35,000, and their fair market value is $120,000. What is Plum’s deduction for the contribution of the computers (ignoring the taxable income limitation)? a. $35,000 b. $70,000 c. $77,500 d. $85,000 59. Wanda is the Chief Executive Officer of Pink corporation, a publicly traded, calendar year C corporation. For the current year, Wanda's compensation package consists of: Cash compensation Nontaxable fringe benefits Taxable fringe benefits Bonus tied to company performance

$ 2,500,000 250,000 150,000 2,000,000

How much of Wanda's compensation is deductible by Pink Corporation? a. $1,000,000. b. $1,250,000. c. $3,250,000. d. $4,900,000. 60. In the current year, Crimson, Inc., a calendar year C corporation, has income from operations of $180,000 and operating deductions of $225,000. Crimson also had $30,000 of dividends from a 15% stock ownership in a domestic corporation. Which of the following statements is correct with respect to Crimson for the current year? a. Crimson’s NOL is $15,000. b. A dividends received deduction is not allowed in computing Crimson’s NOL. c. The NOL is carried back 3 years and forward 10 years by Crimson. d. Crimson’s dividends received deduction is $15,000. 61. In the current year, Red Corporation (a calendar year C corporation), which owns stock in Blue Corporation, had net operating income of $200,000 for the year. Blue pays Red a dividend of $40,000. Red takes a dividends received deduction of $20,000. Which of the following statements is correct? a. Red owns 80% of Blue Corporation. b. Red owns 20% or more but less than 80% of Blue Corporation. c. Red owns 80% or more of Blue Corporation. d. Red owns less than 20% of Blue Corporation. Powered by Cognero

Page 9


Name:

Class:

Date:

Chapter 03: Corporations Introduction and Operating Rules 62. Which of the following statements is incorrect with respect to the treatment of a net operating loss arising in 2024? a. The deduction for any carryover year of the NOL is limited to 80% of taxable income (determined without regard to the NOL deduction). b. A corporation may claim a dividends received deduction in computing an NOL. c. An NOL is generally carried back 2 years and forward 20 years. d. Unlike individuals, corporations do not adjust their NOLs for net capital losses or nonbusiness deductions. 63. Eagle Corporation, a calendar year C corporation, owns stock in Hawk Corporation and has taxable income of $100,000 for the year before considering the dividends received deduction. In the current year, Hawk Corporation pays Eagle a dividend of $130,000, which was considered in calculating the $100,000. What amount of dividends received deduction may Eagle claim if it owns 15% of Hawk’s stock? a. $0 b. $50,000 c. $65,000 d. $84,500 64. Copper Corporation, a calendar year C corporation, owns stock in Bronze Corporation and has net operating income of $900,000 for the current year. Bronze Corporation pays Copper a dividend of $150,000. What amount of dividends received deduction may Copper claim if it owns 85% of Bronze stock (and the two corporations are members of the same affiliated group)? (Assume Copper’s dividends received deduction is not limited by its taxable income.) a. $75,000 b. $97,500 c. $120,000 d. $150,000 65. Orange Corporation, a calendar year C corporation, owns stock in White Corporation and has net operating income of $400,000 for the current year. White Corporation pays Orange a dividend of $60,000. What amount of dividends received deduction may Orange claim if it owns 45% of White stock (assuming Orange’s dividends received deduction is not limited by its taxable income)? a. $30,000 b. $39,000 c. $42,000 d. $60,000 66. Which of the following statements is incorrect regarding the dividends received deduction? a. A corporation must hold stock for more than 90 days in order to qualify for a deduction with respect to dividends on such stock. b. The taxable income limitation does not apply with respect to the 100% deduction available to members of an affiliated group. c. If a stock purchase is financed 75% by debt, the deduction for dividends on such stock is reduced by 75%. d. The taxable income limitation does not apply if the normal deduction (i.e., 50% or 65% of dividends) results in a net operating loss for the corporation. 67. Emerald Corporation, a calendar year C corporation, was formed and began operations on April 1, 2024. The following expenses were incurred during the first tax year (April 1 through December 31, 2024) of operations. Powered by Cognero

Page 10


Name:

Class:

Date:

Chapter 03: Corporations Introduction and Operating Rules Expenses of temporary directors and of organizational meetings Fee paid to the state of incorporation Accounting services incident to organization Legal services for drafting the corporate charter and bylaws Expenses incident to the printing and sale of stock certificates

$27,000 1,000 15,500 9,500 6,000

Assuming a § 248 election, what is Emerald’s deduction for organizational expenditures for 2024? a. $0 b. $4,550 c. $5,000 d. $7,400 68. During the current year, Sparrow Corporation, a calendar year C corporation, had operating income of $425,000, operating expenses of $280,000, a short-term capital loss of $10,000, and a long-term capital gain of $25,000. How much is Sparrow’s income tax liability for the year? a. $32,700 b. $33,600 c. $45,650 d. $62,400 69. Nancy Smith is the sole shareholder and employee of White Corporation, a calendar year C corporation that is engaged exclusively in accounting services. During the current year, White has operating income of $320,000 and operating expenses (excluding salary) of $150,000. Further, White Corporation pays Nancy a salary of $100,000. The salary is reasonable in amount and Nancy is in the 32% marginal tax bracket regardless of any income from White. Assuming that White Corporation distributes all after-tax income as dividends, how much total combined income tax do White and Nancy pay in the current year? (Ignore any employment tax considerations.) a. $40,295 b. $54,995 c. $63,325 d. $64,396 70. Which of the following statements is incorrect regarding the taxation of C corporations? a. NOLs may be carried forward indefinitely. b. A corporation with total assets of $1 million or more is required to file Schedule M–3. c. A tax return must be filed whether or not the corporation has taxable income. d. The alternative minimum tax does not apply to most corporations. 71. Which of the following statements is correct regarding the taxation of C corporations? a. Schedule M-1 is used to reconcile net income computed for financial accounting purposes with taxable income reported on the corporation’s tax return. b. The corporate tax return is filed on Form 1120S. c. Corporations can receive an automatic extension of nine months for filing the corporate return by filing Form 7004 by the due date for the return. d. A corporation with total assets of $7,500,000 or more is required to file Schedule M-3. Powered by Cognero

Page 11


Name:

Class:

Date:

Chapter 03: Corporations Introduction and Operating Rules 72. Robin Corporation, a calendar year C corporation, had taxable income of $700,000, $1,200,000, and $1,000,000 for 2022, 2023, and 2024, respectively. Robin has taxable income of $1,800,000 for 2025. The minimum 2025 estimated tax installment payments for Robin are: a. April 15, 2025, $52,500; June 17, 2025, $52,500; September 16, 2025, $52,500; December 16, 2025, $52,500. b. April 15, 2025, $52,500; June 17, 2025, $94,500; September 16, 2025, $94,500; December 16, 2025, $94,500. c. April 15, 2025, $94,500; June 17, 2025, $94,500; September 16, 2025, $94,500; December 16, 2025, $94,500. d. April 15, 2025, $52,500; June 17, 2025, $136,500; September 16, 2025, $94,500; December 16, 2025, $94,500. 73. Schedule M-1 of Form 1120 is used to reconcile financial net income with taxable income reported on the corporation’s income tax return as follows: net income per books + additions – subtractions = taxable income. Which of the following items is an addition on Schedule M-1? a. Tax depreciation in excess of book depreciation. b. Proceeds of life insurance paid on death of key employee. c. Excess of capital losses over capital gains. d. Tax-exempt interest. 74. Schedule M-1 of Form 1120 is used to reconcile financial net income with taxable income reported on the corporation’s income tax return as follows: net income per books + additions – subtractions = taxable income. Which of the following items is a subtraction on Schedule M-1? a. Book depreciation in excess of tax depreciation. b. Excess of capital losses over capital gains. c. Proceeds on key employee life insurance. d. Income subject to tax but not recorded on the books. 75. Luis is the sole shareholder of a regular C corporation, and Eduardo owns a proprietorship. In the current year, both businesses make a profit of $80,000, and each owner withdraws $50,000 from his business. With respect to this information, which of the following statements is incorrect? a. Eduardo must report $80,000 of income on his return. b. Luis must report $80,000 of income on his return. c. Eduardo’s proprietorship is not required to pay income tax on $80,000. d. Luis’s corporation must pay income tax on $80,000. 76. Rodney, the sole shareholder of Orange Corporation, an accrual method, calendar year corporation, loaned the corporation a substantial amount of money on January 1, 2024. Orange Corporation accrued $45,000 of interest expense on the loan on December 31, 2024. Orange pays the interest to Rodney, a cash basis taxpayer, on January 1, 2025. Based on these facts: a. Orange Corporation will be allowed to deduct the interest expense in 2024 and Rodney will be required to report the interest income in 2025. b. Orange Corporation will be allowed to deduct the interest expense in 2025 and Rodney will be required to report the interest income in 2024. c. Orange Corporation will be allowed to deduct the interest expense in 2024 and Rodney will be required to report the interest income in 2024. d. Orange Corporation will be allowed to deduct the interest expense in 2025 and Rodney will be required to report the interest income in 2025. 77. Opal Corporation, an accrual method, calendar year C corporation, was formed and began operations on July 1, 2024. Powered by Cognero

Page 12


Name:

Class:

Date:

Chapter 03: Corporations Introduction and Operating Rules The following expenses were incurred during the first tax year (July 1 through December 31, 2024) of operations. Expenses of temporary directors and of organizational meetings Fee paid to the state of incorporation Accounting services incident to organization Legal services for drafting the corporate charter and bylaws Expenses incident to the printing and sale of stock certificates

$8,000 2,000 3,500 4,300 6,000

Assuming a § 248 election, what is Opal’s deduction for organizational expenditures for 2024? a. $593. b. $460. c. $5,427. d. $5,627. 78. During the current year, Jay Corporation, a calendar year personal service C corporation, had operating income of $300,000, operating expenses of $200,000, a short-term capital gain of $5,000, and a long-term capital loss of $35,000. How much is Jay’s income tax liability for the year? a. $14,700. b. $21,000. c. $22,250. d. $35,000. 79. In working with Schedule M-2 (analysis of unappropriated retained earnings per books) of Form 1120, which of the following is an addition to beginning retained earnings? a. Cash dividends. b. Net loss per books. c. Property dividends. d. Net income per books. 80. In the current year, Tern, Inc., a calendar year C corporation, has $9,000,000 of adjusted taxable income, $300,000 of business interest income, zero floor plan financing interest, and $3,200,000 million of business interest expense. Tern has average gross receipts for the prior three-year period of $45,000,000. Which of the following statements is correct about the treatment of Tern's business interest expense? a. Current year deduction of $3,200,000. b. Current year deduction of $2,790,000, carryforward of $410,000. c. Current year deduction of $2,790,000, carryback of $410,000. d. Current year deduction of $3,000,000, carryforward of $200,000. 81. Plum Corporation, a calendar year C corporation, reports net income of $2.3 billion on its 2024 financial statements, including financial statement depreciation of $300 million. Also, for 2024, Plum reports taxable income of $1.1 billion, including tax depreciation of $500 million. Assume that Plum’s average annual AFSI for the 3-year period ending December 31, 2024, is in excess of $1 billion. Also assume that Plum has no AMT foreign tax credit for 2024. What is Plum 's 2024 corporate AMT? a. $0. b. $84 million. c. $114 million. Powered by Cognero

Page 13


Name:

Class:

Date:

Chapter 03: Corporations Introduction and Operating Rules d. $315 million. Subjective Short Answer 82. During the current year, Skylark Company (a calendar year entity) had operating income of $420,000 and operating expenses of $250,000. In addition, Skylark had a long-term capital loss of $20,000, and a charitable contribution of $5,000. How does Toby, the sole owner of Skylark Company, report this information on his individual income tax return under following assumptions? a.

Skylark is an LLC, and Toby does not withdraw any funds from the company during the year.

b.

Skylark is an S corporation, and Toby does not withdraw any funds from the company during the year.

c.

Skylark is a regular (C) corporation, and Toby does not withdraw any funds from the company during the year.

83. Canary Corporation, an accrual method C corporation, uses the calendar year for tax purposes. Leticia, a cash method taxpayer, is both a shareholder of Canary and the corporation’s CFO. On December 31, 2024, Canary has accrued a $75,000 bonus to Leticia. Describe the tax consequences of the bonus to Canary and to Leticia under the following independent situations. a.

Leticia owns 35% of Canary Corporation’s stock and the corporation pays the bonus to Leticia on February 4, 2025.

b.

Leticia owns 75% of Canary Corporation’s stock and the corporation pays the bonus to Leticia on May 6, 2025.

c.

Leticia owns 75% of Canary Corporation’s stock and the corporation pays the bonus to Leticia on April 7, 2025.

84. Ostrich, a C corporation, has a net short-term capital gain of $20,000 and a net long-term capital loss of $90,000 during 2024. Ostrich also has taxable income from other sources of $1,000,000. Prior years’ transactions included the following: 2020 net short-term capital gains 2021 net long-term capital gains 2022 net short-term capital gains 2023 net long-term capital gains a. b. c. d.

$20,000 15,000 25,000 5,000

How are the capital gains and losses treated on Ostrich’s 2024 tax return? Determine the amount of the 2024 net capital loss that is carried back to each of the previous years. Compute the amount of capital loss carryover, if any, and indicate the years to which the loss may be carried. If Ostrich were a proprietorship, how would Ellen, the owner, report these transactions on her 2024 tax return?

Powered by Cognero

Page 14


Name:

Class:

Date:

Chapter 03: Corporations Introduction and Operating Rules

85. During the current year, Gray Corporation, a C corporation in the financial services business, made charitable contributions to qualified organizations as follows: ∙

Stock (basis of $20,000, fair market value of $45,000) in Drab Corporation, held for six months as an investment, to the Salvation Army. (Salvation Army plans on selling the stock.)

Painting (basis of $90,000, fair market value of $250,000), held for four years as an investment, to the Museum of Fine Arts. (The Museum plans on including the painting in its collection.)

Gray Corporation’s taxable income (before any charitable contribution deduction) is $1,800,000. a.

What is the total amount of Gray’s charitable contributions for the year?

b.

What is the amount of Gray’s charitable contribution deduction in the current year, and what happens to any excess charitable contribution, if any?

86. On December 28, 2024, the board of directors of Taupe Corporation, a calendar year, accrual method C corporation, authorized a contribution of land to a qualified charitable organization. The land (basis of $75,000, fair market value of $125,000) was acquired five years ago and held as an investment. For purposes of the taxable income limitation applicable to charitable deductions, Taupe has taxable income of $800,000 and $950,000 for 2024 and 2025, respectively. Describe the tax consequences to Taupe Corporation under the following independent situations. a.

The donation is made on February 15, 2025.

b.

The donation is made on May 10, 2025.

87. During the current year, Quartz Corporation (a calendar year C corporation) has the following transactions: Income from operations Expenses from operations Dividends received from ABC Corporation

$350,000 370,000 50,000

Quartz owns 25% of ABC Corporation’s stock. How much is Quartz Corporation’s taxable income (loss) for the year? 88. Warbler Corporation, an accrual method regular corporation, was formed and began operations on March 1, 2024. The following expenses were incurred during its first year of operations (March 1 - December 31, 2024): Expenses of temporary directors and organizational meetings Incorporation fee paid to state Expenses incurred in printing and selling stock certificates Accounting services incident to organization

$25,000 2,000 10,000 12,000

a.

Assuming a valid election under § 248 to amortize organizational expenditures, what is the amount of Warbler’s deduction for 2024?

b.

Same as a., except that Warbler also incurred in 2024 legal fees of $15,000 for the drafting of the corporate charter and bylaws. What is the amount of

Powered by Cognero

Page 15


Name:

Class:

Date:

Chapter 03: Corporations Introduction and Operating Rules Warbler’s 2024 deduction for organizational expenditures?

89. During the current year, Coyote Corporation (a calendar year C corporation) has the following transactions: Income from operations Expenses from operations Dividends received from Roadrunner Corporation

$260,000 305,000 115,000

a.

Coyote owns 5% of Roadrunner Corporation’s stock. How much is Coyote Corporation’s taxable income (loss) for the year?

b.

Would your answer change if Coyote owned 25% of Roadrunner Corporation’s stock?

90. In each of the following independent situations, determine the C corporation’s income tax liability. Assume that all corporations use a calendar year 2024.

Violet Corporation Indigo Corporation Orange Corporation Blue Corporation Green Corporation (personal service corporation)

Taxable Income $ 63,000 180,000 510,000 11,100,000 225,000

91. Tonya, an actuary, is the sole shareholder of Shrike Corporation, a professional C corporation. The corporation paid Tonya a salary of $360,000 during its fiscal year ending September 30, 2024. How much salary must Shrike Corporation pay Tonya during the period October 1 through December 31, 2024, to enable the corporation to continue to use its fiscal year without negative tax effects?

92. Almond Corporation, a calendar year C corporation, had taxable income of $900,000, $1,100,000 million, and $1,200,000 million for 2022, 2023, and 2024, respectively. Almond’s taxable income is $2,000,000 for 2025. Compute the minimum estimated tax payments for 2025 for Almond Corporation. 93. Heron Corporation, a calendar year, accrual basis taxpayer, provides the following information for the current year and asks you to prepare Schedule M-1. Net income per books (after-tax) Taxable income Federal income tax liability Interest income from tax-exempt bonds Interest paid on loan incurred to purchase tax-exempt bonds Life insurance proceeds received as a result of death of Heron’s president Powered by Cognero

$258,050 195,000 40,950 5,000 2,000 100,000 Page 16


Name:

Class:

Date:

Chapter 03: Corporations Introduction and Operating Rules Premiums paid on policy on life of Heron’s president Excess of capital losses over capital gains Retained earnings at beginning of year Cash dividends paid Tax depreciation in excess of book depreciation

4,500 2,000 375,000 90,000 7,500

Essay 94. Adrian is the president and sole shareholder of Pigeon Corporation. He also lends money and rents a building to the corporation. Discuss how these business relationships between Adrian and Pigeon Corporation can help avoid double taxation. What limitations are there on the use of such relationships? 95. Nancy is a 40% shareholder and president of Robin Corporation, a calendar year C corporation. The board of directors of Robin has decided to pay Nancy a $75,000 bonus for the current year based on her outstanding performance. The directors want to pay the $75,000 as salary, but Nancy would prefer to have it paid as a dividend. If Nancy is in the 37% marginal tax bracket regardless of the treatment of the bonus, discuss which form of payment would be most beneficial for each party. (Ignore any employment tax considerations.) 96. Dawn is the sole shareholder of Thrush Corporation, a calendar year C corporation. In the current year, Thrush earned $350,000 and distributed $75,000 to Dawn. Kirk is the sole shareholder of Swallow Corporation, an S corporation. In the current year, Swallow earned $350,000 and distributed $75,000 to Kirk. Contrast the tax treatment of Thrush Corporation and Dawn with the tax treatment of Swallow Corporation and Kirk. 97. Explain the rules regarding the accounting periods available to corporate taxpayers. 98. Huan, a cash basis taxpayer, owns 70% of the stock of Black Corporation, a calendar year, accrual basis C corporation. On December 31, 2024, Black accrued a bonus of $80,000 to Huan, and paid the bonus to Huan on January 1, 2025. When does Huan report the bonus, and when does Black Corporation deduct the bonus? Would your answers change if Huan was a 40% shareholder of Black? 99. Briefly describe the accounting methods available for adoption by a C corporation. 100. Contrast the tax treatment of capital gains and losses of C corporations with that of individual taxpayers. 101. Briefly describe the charitable contribution deduction rules applicable to C corporations. 102. Briefly discuss the current-year requirements for the dividends received deduction. 103. In connection with the deduction for startup expenditures, comment on the following: a. b. c.

Qualifying expenditures. Election process. Amount of deduction.

104. What is the annual required estimated tax payment for a C corporation? What are the rules regarding payment of the estimated tax? 105. What is the purpose of Schedule M-3? Which corporations are required to file Schedule M-3? Powered by Cognero

Page 17


Name:

Class:

Date:

Chapter 03: Corporations Introduction and Operating Rules 106. For purposes of the accumulated earnings tax, earnings can be accumulated for reasonable needs of the business. List several examples of what is included and several examples of what is not included in the reasonable needs of the business. 107. In applying the $1,000,000 limit on deducting executive compensation, what corporations are subject to the deduction limit? What executives are covered? What compensation is included in the limit? 108. How is the limitation on the deduction of business interest computed? Does it apply to all taxpayers? What happens to any business interest deduction disallowed by the limitation?

Powered by Cognero

Page 18


Name:

Class:

Date:

Chapter 03: Corporations Introduction and Operating Rules Answer Key 1. True 2. False 3. False 4. True 5. False 6. True 7. True 8. False 9. True 10. True 11. True 12. True 13. False 14. False 15. True 16. False 17. True 18. False 19. False 20. True 21. False 22. True 23. True 24. False 25. True Powered by Cognero

Page 19


Name:

Class:

Date:

Chapter 03: Corporations Introduction and Operating Rules 26. True 27. True 28. False 29. False 30. True 31. True 32. False 33. True 34. True 35. True 36. True 37. False 38. True 39. True 40. True 41. True 42. c 43. b 44. c 45. c 46. b 47. b 48. c 49. a 50. d Powered by Cognero

Page 20


Name:

Class:

Date:

Chapter 03: Corporations Introduction and Operating Rules 51. c 52. d 53. c 54. c 55. b 56. b 57. c 58. b 59. b 60. d 61. d 62. c 63. b 64. d 65. b 66. a 67. b 68. b 69. b 70. b 71. a 72. d 73. c 74. c 75. b 76. d Powered by Cognero

Page 21


Name:

Class:

Date:

Chapter 03: Corporations Introduction and Operating Rules 77. c 78. b 79. d 80. d 81. b 82. a.

b.

c.

A single-member LLC is taxed as a proprietorship. Consequently, Toby reports the $170,000 operating profit, $20,000 long-term capital loss, and $5,000 charitable contribution on his individual return (Form 1040). The LTCL will be subject to the capital loss limitations applicable to individual taxpayers. Toby would report any related deduction for qualified business income on his Form 1040. Income, deductions, gains, and losses of an S corporation flow through to the shareholders. Separately stated items (e.g., LTCL and charitable contribution) retain their character at the shareholder level. Consequently, Toby reports the $170,000 operating profit, $20,000 longterm capital loss, and $5,000 charitable contribution on his individual return (Form 1040). The LTCL will be subject to the capital loss limitations applicable to individual taxpayers. Toby would report any related deduction for qualified business income on his Form 1040. Shareholders of a regular (C) corporation report income from the corporation to the extent of dividends received. Therefore, Toby does not report any of Skylark’s operating profit, longterm capital loss, or charitable contribution on his individual return. [Skylark Company would report taxable income of $165,000 ($170,000 operating profit – $5,000 charitable contribution) on its corporate return (Form 1120). The net capital loss of $20,000 is not deductible in the current year; rather, the loss is carried back three years and forward five years (as STCL).]

83. Under § 267(a)(2), an accrual method taxpayer must defer a deduction for an expenditure attributable to a cash method related party until such time the related party reports the amount as income. For purposes of this limitation, a more-than50% shareholder of the corporation is a related party. a.

Leticia is not a related party for purposes of the § 267(a)(2) limitation; thus, Canary deducts the bonus, under the accrual method, in 2024. Since Leticia is on the cash method, she includes the bonus in her income in 2025.

b.

Since Leticia, a cash method related party, does not include the bonus in her income until its receipt in 2025, Canary’s deduction for the bonus occurs in 2025.

c.

Again, Leticia is a cash method related party who does not include the bonus in her income until its receipt in 2025; thus, Canary’s deduction for the bonus is deferred until 2025. The fact that the payment to Leticia occurs prior to the filing date for Canary’s 2024 tax return is of no consequence.

Powered by Cognero

Page 22


Name:

Class:

Date:

Chapter 03: Corporations Introduction and Operating Rules

84. a. Net short-term capital gain Net long-term capital loss Net capital loss

$ 20,000 (90,000) ($70,000)

The net capital loss of $70,000 is not deductible in 2024 but must be carried back to the three preceding years, applying it to 2021, 2022, and 2023, in that order. Such net capital loss is carried back or forward as a short-term capital loss. b.

c.

d.

2024 net capital loss Offset against— 2021 net long-term capital gains 2022 net short-term capital gains 2023 net long-term capital gains Total carrybacks

($70,000) $15,000 25,000 5,000 $45,000

$25,000 ($70,000 – $45,000) STCL carryover to 2025, 2026, 2027, 2028, and 2029 in that order. Ellen would net these transactions with all other capital transactions for 2024. Assuming these were her only capital transactions in 2024, she would offset $20,000 of capital losses against the capital gains and deduct an additional $3,000 in capital losses on her return. The remaining $67,000 ($90,000 – $20,000 – $3,000) would be carried forward indefinitely as a LTCL.

85. a.

Gray’s total amount of charitable contributions is $270,000 [$20,000 (stock) + $250,000 (painting)], computed as follows: Stock: this is ordinary income property, because a sale of the stock would not result in a long-term capital gain or a § 1231 gain for Gray (i.e., STCG). Thus, the amount of the contribution is the stock’s basis, or $20,000. Painting: this is capital gain property, because a sale of the painting would result in a longterm capital gain for Gray. The painting is tangible personal property and its use is related to the charitable organization's exempt function. Thus, the amount of the contribution is

Powered by Cognero

Page 23


Name:

Class:

Date:

Chapter 03: Corporations Introduction and Operating Rules the painting’s fair market value, or $250,000. b.

Gray’s current-year charitable deduction is limited to $180,000 [10% × $1,800,000 (taxable income before charitable deduction)], and the excess charitable contribution of $90,000 ($270,000 – $180,000) is carried forward to the five succeeding tax years.

86. In general, charitable contributions are deductible in the year made. However, in the case of an accrual method corporation, a deduction can be claimed in the current year for a charitable contribution made in the subsequent year if (1) the contribution is approved by the board of directors of the corporation in the current year, and (2) the contribution is made on or before the fifteenth day of the fourth month of the subsequent year. The land is capital gain property; thus, the amount of the charitable contribution is the land’s fair market value of $125,000. a.

b.

The requirements for an accrual of the charitable deduction are satisfied; thus, the $125,000 contribution is deductible by Taupe in 2024, subject to the taxable income limitation. For 2024, the taxable income limitation for charitable deductions is $80,000 (10% × $800,000). The excess contribution amount of $45,000 carries forward to 2025 (five-year carryover limit). The requirements for an accrual of the charitable deduction are not satisfied; thus, the $125,000 contribution is deductible by Taupe in 2025 (the year the contribution is made), subject to the taxable income limitation. For 2025, the taxable income limitation for charitable deductions is $95,000 (10% × $950,000). The excess contribution amount of $30,000 carries forward to 2026 (five-year carryover limit).

87. Quartz has an NOL, computed as shown here: Gross income: From operations Dividends Less: Expenses from operations Dividends received deduction ($50,000 × 65%) Net operating loss

$350,000 50,000

$400,000

$370,000 32,500

(402,500) ($ 2,500)

The dividends received deduction is not limited to the taxable income limitation because it creates a net operating loss.

88. a.

Warbler has qualifying organizational expenditures of $39,000 [$25,000 (expenses of temporary directors and organizational meetings) + $2,000 (incorporation fee) + $12,000 (accounting fees)]. Expenses related to the printing or selling of stock or other securities do not qualify as organizational expenditures. Warbler’s 2024 deduction for the organizational expenditures is $6,889 {$5,000 (amount immediately expensed) + [($39,000 – $5,000)/180 × 10 months]}.

Powered by Cognero

Page 24


Name:

Class:

Date:

Chapter 03: Corporations Introduction and Operating Rules b.

Warbler now has qualifying organizational expenditures of $54,000 [$39,000 (as computed in part a., above) + $15,000 (legal fees)]. Warbler’s 2024 deduction for the organizational expenditures is $3,944 {$1,000 (amount immediately expensed) + [($54,000 – $1,000)/180 × 10 months]}. The $5,000 immediate expensing amount is reduced to the extent that qualifying organizational expenditures exceed $50,000; thus, only $1,000 of the expenditures is immediately deductible, and the remainder of the expenditures is amortized over 180 months.

89. The key to this question is the relationship between the dividends received a. deduction and the NOL deduction. The dividends received deduction is limited to a percentage of taxable income of the corporation (unless taking the full dividends received deduction would cause or increase an NOL). In this case, the dividends received deduction is limited to 50% of taxable income. Gross income: From operations Dividends Less: Expenses from operations Taxable income before the dividends received deduction Dividends received deduction (50% × $70,000) Taxable income

$260,000 115,000

$375,000 (305,000) $ 70,000 (35,000) $ 35,000

The dividends received deduction is limited to 50% of taxable income because taking 50% of $115,000 ($57,500) would not create an NOL. If Coyote Corporation owns 25% of Roadrunner Corporation’s stock, the b. percentage for calculating the dividends received deduction is 65%. Under these circumstances, taking the full dividends received deduction would create an NOL. Gross income: From operations Dividends Less: Expenses from operations Taxable income before the dividends received deduction Dividends received deduction (65% × $115,000) Net operating loss

Powered by Cognero

$260,000 115,000

$375,000 (305,000) $ 70,000 (74,750) ($ 4,750)

Page 25


Name:

Class:

Date:

Chapter 03: Corporations Introduction and Operating Rules 90. Violet Corporation: Tax on $63,000 × 0.21

$13,230

Indigo Corporation: Tax on $180,000 × 0.21

$37,800

Orange Corporation: Tax on $510,000 × 0.21

$107,100

Blue Corporation: Tax on $11,100,000 × 0.21

$2,331,000

Green Corporation (personal service corporation): Tax on $225,000 × 0.21

$47,250

A flat 21% tax rate applies to all C corporations (including PSCs). 91. The salary for the deferral period (October 1 through December 31) must be at least proportionate to the employee’s salary received for the fiscal year. The amount that Shrike Corporation must pay Tonya during the period October 1 through December 31, 2024, to permit the continued use of its fiscal year without negative tax effects is $90,000 [($360,000 × (3 ÷ 12)].

92. A corporation that had taxable income of $1,000,000 or more in any of the three preceding years is a “large corporation” for purposes of utilizing the prior year’s tax exception for estimated tax payments. As such, Almond Corporation can use the prior year’s tax exception for computing its first 2024 estimated tax payment only, and any shortfall as a result of such use must be paid with the second installment. Payment April 15, 2025 June 16, 2025 September 15, 2025 December 15, 2025 Total

Amount $ 63,000 * 147,000 ** 105,000 105,000 $420,000

*Based on preceding year’s tax, for first installment only: ($1,200,000 taxable income × 21%) = $252,000 ÷ 4 = $63,000. **Based on current year’s tax, for remaining installments: ($2,000,000 taxable income × 21%) = $420,000 ÷ 4 = $105,000. Second installment must include shortfall from first installment: [$105,000 + ($105,000 – $63,000)] = $147,000.

Powered by Cognero

Page 26


Name:

Class:

Date:

Chapter 03: Corporations Introduction and Operating Rules

93. Net income per books is reconciled to taxable income as follows: Net income per books (after tax) Plus: Items that decreased net income per books but did not affect taxable income + Federal income tax liability + Excess of capital losses over capital gains + Interest paid on loan incurred to purchase tax-exempt bonds + Premiums paid on policy on life of president of the corporation Subtotal Minus: Items that increased net income per books but did not affect taxable income – Interest income from tax-exempt bonds – Life insurance proceeds received as a result of the death of the corporate president – Tax depreciation in excess of book depreciation Taxable income

$258,050

40,950 2,000 2,000 4,500 $307,500

(5,000) (100,000) (7,500) $195,000

94. As president of Pigeon Corporation, Adrian can have the corporation pay him a salary. As a creditor, he can have the corporation pay him interest on the loans. As a landlord, he can have the corporation pay him rent. All of these expenses can be deducted by the corporation. In order to avoid disallowance of any of these deductions at the corporate level, the payments to Adrian must be reasonable in amount. Payments deemed to be unreasonable in amount will be treated as corporate dividends to Adrian and nondeductible by Pigeon. However, to the extent that the payments are reasonable in amount and deductible by Pigeon Corporation, the corporate tax is avoided on such amounts. The payments received by Adrian would be income (i.e., salary, interest, and rent) to him and taxed as such, but this would be the only tax incurred on such amounts (i.e., double taxation is avoided to the extent of any payments deductible by Pigeon). 95. Robin Corporation prefers treating the payment as salary, because a $75,000 deduction for such would provide the corporation with a tax savings of $15,750 [$75,000 (salary deduction) × 21%]. If, instead, the payment were treated as a dividend, none of the $75,000 would deductible by Robin. Nancy prefers treating the payment as a dividend, because a preferential tax rate of 20% would apply to the $75,000 and result in only $15,000 of tax. If, instead, the payment were treated as salary, Nancy would incur tax of $27,750 [$75,000 (salary) × 37% (marginal tax rate)]. Thus, Nancy would save $12,750 of tax if the payment were treated as a dividend instead of salary. 96. A C corporation is a separate taxable entity; thus, Thrush Corporation is taxed on the $350,000 of earnings. Income of a C corporation has no effect on the shareholders until such time a dividend is paid. When dividends are paid, shareholders must report dividend income on their tax returns. Thus, Dawn is taxed on $75,000 of dividends and the 0%/15%/20% preferential tax rate applies with respect to the dividends. Generally, an S corporation is not subject to an entity level Federal income tax. Instead, the corporation’s income, gains, deductions, and losses are passed through to and reported by the shareholders on their tax returns. Thus, Swallow reports the $350,000 of earnings on its tax return (Form 1120S), but pays no income tax. Kirk is taxed on the $350,000 of earnings from Swallow on his individual income tax return (Form 1040). Kirk would report any related deduction for Powered by Cognero

Page 27


Name:

Class:

Date:

Chapter 03: Corporations Introduction and Operating Rules qualified business income on his Form 1040. Distributions from S corporations are not taxable to the shareholder (to the extent of stock basis). Thus, Kirk is not taxed on the $75,000 distribution from Swallow. 97. In general, a corporate taxpayer may select a calendar year or a fiscal year for tax return reporting purposes. A newly formed corporation generally can select its initial reporting period without having to obtain IRS consent. However, certain types of corporate taxpayers are subject to restrictions on their reporting period. In general, personal service corporations (PSCs) and S corporations are required to use the calendar year for tax reporting. Exceptions to this rule apply, and a fiscal year can be elected by a PSC (or S corporation) under any of the following conditions:

A business purpose for the year can be demonstrated.

The PSC tax year results in a deferral of not more than three months’ income. An election under § 444 is required, and the PSC will be subject to the deduction limitations of § 280H. The corporation must pay the shareholder/employee’s salary during the portion of the calendar year after the close of the fiscal year. In addition, the salary for that period must be at least proportionate to the employee’s salary for the fiscal year. (For an S corporation electing a § 444 deferral, the required payments provision of § 7519 must be satisfied. See Chapter 11.)

The PSC (or S corporation) retained the same year that was used for its fiscal year ending 1987, provided an election was made under § 444 and subject to the deduction limitations of § 280H (or § 7519, in the case of an S corporation).

98. Huan is a more than 50% shareholder of Black Corporation; thus, the taxpayers are related parties under § 267. Huan, a cash basis taxpayer, reports the salary income in 2025, the year of receipt. Under § 267, Black Corporation, an accrual basis taxpayer, cannot deduct the salary expense when accrued in 2024. Instead, Black’s deduction for the bonus is in 2025, the year Huan reports the salary as income. If Huan were a 40% shareholder, the § 267 related party rules would not apply to the bonus deduction. Instead, Black Corporation, an accrual basis taxpayer, deducts the bonus in 2024, the year of accrual. Huan, a cash basis taxpayer, still reports the bonus as income in 2025, the year of receipt. 99. In general, a C corporation must adopt the accrual method of accounting. However, there are several exceptions to this rule, and the following C corporations can use the cash method of accounting: •

Corporations engaged in the trade or business of farming or timber.

Qualified personal service corporations.

Corporations with average annual gross receipts of $30,000,000 million or less for the previous 3-year period.

There is a limitation on the use of the cash method by otherwise qualifying corporations that maintain inventories. In general, these corporations must use the accrual method in determining sales and cost of goods sold. However, corporations with average annual gross receipts of $30,000,000 or less for the most recent 3-year period are generally not subject to the limitation. Powered by Cognero

Page 28


Name:

Class:

Date:

Chapter 03: Corporations Introduction and Operating Rules Accrual method corporations are subject to a limitation on the deductibility of an accrued expenditure attributable to a cash method related party (e.g., a more-than-50% shareholder). In such cases, the corporation’s deduction for the expenditure is deferred until the recipient includes the amount in income.

100. The definition of capital assets is the same for both corporate and individual taxpayers. Also, both types of taxpayers net short- and long-term gains and losses to arrive at a net capital gain or loss. In the case of a net capital gain, individual taxpayers receive a preferential tax rate (0%/15%/20%) with respect to LTCGs but corporations do not receive any preference as to LTCG tax rates. In the case of a net capital loss, individual taxpayers can deduct up to $3,000 of net capital loss against ordinary income in the current year but corporations cannot deduct any amount of a net capital loss in the current year. Individual taxpayers carry forward capital losses indefinitely, with such losses retaining their character as short term or long term. Corporate taxpayers carry capital losses back three years and forward five years, and such losses are treated as STCL in such years. 101. Tax year of deduction: In general, a charitable contribution is deductible only in the year the gift is made. For an accrual basis corporation, however, a charitable contribution can be deducted in the current year for a contribution that is (1) approved by the corporation’s board of directors by the end of such year and (2) paid on or before the fifteenth day of the fourth month of the next year. Amount of contribution: In addition to cash gifts, property contributions to qualified charitable organizations are also deductible. For property that is depreciated (fair market value less than basis), the amount of the contribution is the property’s fair market value. For property that is appreciated (fair market value greater than basis), the amount of the contribution depends on whether the property is “capital gain property” or “ordinary income property.” Capital gain property is property that, if sold, would result in a long-term capital gain or § 1231 gain. A contribution of capital gain property generally results in a deductible amount equal to the property’s fair market value. If the capital gain property is tangible personal property and the charitable organization’s use of the property is unrelated to its exempt function, the amount of the contribution is equal to the property’s basis. (Contributions of capital gain property to certain private foundations are similarly limited to the property’s basis.) Ordinary income property is property that, if sold, would not result in a long-term capital gain or § 1231 gain. Typically, the deduction for a contribution of ordinary income property is equal to the property’s basis. However, charitable contributions of certain inventory property by corporations can result in an enhanced deduction amount. For such inventory property, the deductible amount is equal to the lesser of (1) the sum of the property’s basis plus 50% of the appreciation on the property or (2) twice the property’s basis. Annual limitation on deduction: A corporate taxpayer’s charitable deduction is limited to 10% (15% for contributions of food inventory) of taxable income (determined without regard to the charitable contribution deduction, any net operating loss carryback or capital loss carryback, and dividends received deduction). Any contributions in excess of the 10% limitation may be carried forward for five years. In any tax year for which there is a charitable contribution carryover, current year’s gifts are applied against the 10% limitation first, with carryover amounts deducted in order of time. 102. The dividends received deduction (DRD) is available to C corporations with respect to dividends received from domestic corporations. The amount of the DRD is generally equal to 50% (for stock ownership of less than 20%) or 65% (for stock ownership of 20% or more but less than 80%) of the dividends received. A limitation applies if the applicable percentage (50% or 65%) of taxable income (computed without regard to the DRD, NOL deduction, and capital loss carryback) is less than the normal DRD amount. However, the taxable income limitation does not apply if the normal DRD amount creates or increases an NOL. (For stock ownership interests of 80% or more, the applicable DRD percentage is 100% and the taxable income limitation does not apply.) Two additional limitations apply to the DRD. First, no DRD is allowed unless the corporation has held the stock for more than 45 days. Second, the amount of the DRD is reduced by the percentage of the investment in the stock that is debt financed. This reduction in the DRD cannot exceed that amount of the interest expense deduction allocable to the dividend. Powered by Cognero

Page 29


Name:

Class:

Date:

Chapter 03: Corporations Introduction and Operating Rules 103. a. “Startup expenditures” are expenses incurred after the organization of a trade or business but before such trade or business has begun operations. Examples of startup expenditures include rent, payroll, accounting, advertising, insurance, utilities, and other operating expenses associated with the preopening of a trade or business. b. A corporation is deemed to have made the election to amortize startup expenditures for the taxable year in which the corporation begins business. The startup expenditures deduction is claimed on the corporation’s return for such taxable year without any separate statement or specific identification of the deduction. If a corporation wants to forgo the deduction of startup expenditures, a separate statement to that effect should accompany the corporation’s tax return for its first taxable year. c. In general, startup expenditures are amortized over a 180-month period beginning with the month the corporation begins business. However, the first $5,000 of startup expenditures is expensed in the first taxable year with the remaining amount of expenditures amortized over the 180-month period. The $5,000 expensing amount is reduced to the extent that startup expenditures exceed $50,000 (i.e., there is no immediate expensing if startup expenditures equal or exceed $55,000). 104. Estimated tax payments are required if the corporation’s tax liability is expected to be $500 or more. The required annual payment is the lesser of (1) 100% of the corporation’s tax for the current year or (2) 100% of the corporation’s tax for the preceding year. Estimated payments are made quarterly, due on or before the 15th day of the 4th, 6th, 9th, and 12th months of the taxable year. Underpayment of estimated tax penalty can be avoided if the quarterly payments are filed timely and equal to the corporation’s tax liability for the prior year (or tax liability computed on an annualized method). A corporation with taxable income of $1,000,000 or more in any of its three preceding years can use the prior year’s tax liability for computing only the first installment payment. In such cases, the corporation’s second installment payment must include any shortfall resulting from using the prior year’s liability (instead of the current-year’s liability) for the first installment. 105. Schedule M-3 was created, in part, in response to financial reporting scandals, such as Enron and WorldCom. Schedule M-3 requires corporations to report much more information regarding the differences between financial net income (loss) and taxable income than is required of Schedule M-1. This greater transparency should allow the IRS to more easily identify corporations that engage in aggressive tax practices, because those transactions generally result in book/tax differences that must be reported on Schedule M-3. Entities with total assets of $10,000,000 or more must file Schedule M-3 (in lieu of Schedule M-1). The financial figures (e.g., amount of total assets, net income or loss) required of the Schedule M-3 are drawn from the corporation’s Form 10-K. If Form 10-K is not filed, then another financial source (e.g., certified financial statements) is used. 106. Reasonable needs of the business include expansion of the business, replacement of plant and equipment, working capital needs, product liability losses, debt retirement, self-insurance, and loans to suppliers and customers. Reasonable needs do not include loans to shareholders, investments in unrelated properties or businesses, and unrealistic hazards and contingencies. 107. The $1,000,000 limit on deducting the compensation of a covered employee applies to publicly traded corporations. Covered employees include the principal (or chief) executive officer, the principal (or chief) financial officer, and the three other most highly compensated officers. The limitation applies to all compensation for services performed by a covered employee, including commissions and performance-based compensation. However, the limitation does not apply to retirement plan contributions or tax-free employer provided benefits. Powered by Cognero

Page 30


Name:

Class:

Date:

Chapter 03: Corporations Introduction and Operating Rules 108. The deduction of business interest for any year is limited to the sum of (1) the taxpayer's business interest income for the year, (2) 30% of the taxpayer's adjusted taxable income for the year, and (3) the taxpayer's floor plan financing interest for the year. In general, the limitation applies to all taxpayers but there is a small business exception (i.e., taxpayers having average gross receipts for the prior three-year period of $30,000,000 or less). Any business interest deduction disallowed by the limit is treated as business interest paid or accrued in the succeeding tax year.

Powered by Cognero

Page 31


Name:

Class:

Date:

Chapter 04: Corporations Organization and Capital Structure True / False 1. Similar to the like-kind exchange provision, § 351 can be partly justified under the wherewithal to pay concept. a. True b. False 2. Similar to like-kind exchanges, the receipt of “boot” under § 351 can cause loss to be recognized. a. True b. False 3. Tina incorporates her sole proprietorship with assets having a fair market value of $100,000 and an adjusted basis of $110,000. Even though § 351 applies, Tina may recognize her realized loss of $10,000. a. True b. False 4. In a § 351 transfer, a shareholder receives boot of $10,000 but ends up with a realized loss of $3,000. Only $7,000 of the boot will be taxed to the shareholder. a. True b. False 5. A taxpayer may never recognize a loss on the transfer of property in a transaction subject to § 351. a. True b. False 6. If a transaction qualifies under § 351, any recognized gain is equal to the value of the boot received. a. True b. False 7. Allen transfers marketable securities with an adjusted basis of $120,000, fair market value of $300,000, for 85% of the stock of Heron Corporation. In addition, he receives cash of $40,000. Allen recognizes a capital gain of $40,000 on the transfer. a. True b. False 8. When consideration is transferred to a corporation in return for stock, the definition of “property” is important because tax deferral treatment of § 351 is available only to taxpayers who transfer property. a. True b. False 9. The transfer of an installment obligation in a transaction qualifying under § 351 is a taxable disposition of the obligation to the transferor. a. True b. False 10. Gabriella and Maria form Luster Corporation with each receiving 50 shares of its stock. Gabriella transfers cash of $50,000, while Maria transfers a proprietary formula (basis of $0; fair market value of $50,000). Neither Gabriella nor Maria will recognize gain on the transfer. Powered by Cognero

Page 1


Name:

Class:

Date:

Chapter 04: Corporations Organization and Capital Structure a. True b. False 11. Because services are not considered property under § 351, a taxpayer must report as income the fair market value of stock received for such services. a. True b. False 12. For § 351 purposes, stock rights and stock warrants are included in the definition of “stock.” a. True b. False 13. In a § 351 transaction, if a transferor receives consideration other than stock, the transaction can result in taxable income. a. True b. False 14. The receipt of nonqualified preferred stock in exchange for the transfer of appreciated property to a controlled corporation results in recognition of gain to the transferor. a. True b. False 15. Ruth transfers property worth $200,000 with a basis of $60,000 to Goldfinch Corporation. In return, she receives 80% of its stock worth $180,000 and a long-term note executed by Goldfinch and made payable to Ruth worth $20,000. Ruth will recognize no gain on the transfer. a. True b. False 16. The control requirement under § 351 requires that the person or persons transferring property to the corporation immediately after the transfer own stock possessing at least 80% of the total combined voting power of all classes of stock entitled to vote and at least 80% of the total number of shares of all other classes of stock of the corporation. a. True b. False 17. To retain the services of Eve, a key employee in Ted’s sole proprietorship, Ted agrees to make Eve a 30% owner. Ted incorporates the business, receiving in return 100% of the stock. Three days later, Ted transfers 30% of the stock to Eve. Under these circumstances, § 351 will apply to the incorporation of Ted’s business. a. True b. False 18. Sofia forms Lark Corporation with a transfer of appreciated property in exchange for all of its shares. Shortly thereafter, she transfers half her shares to her son, Ted. The later transfer to Ted could cause the original transfer to be taxable. a. True b. False 19. A person who performs services for a corporation in exchange for stock cannot be treated as a member of the § Powered by Cognero

Page 2


Name:

Class:

Date:

Chapter 04: Corporations Organization and Capital Structure 351 transferring group even if that person also transfers some property to the corporation. a. True b. False 20. The use of § 351 is not limited to the initial formation of a corporation, and it can apply to later transfers as well. a. True b. False 21. Because boot is generated under § 357(b) (i.e., the liability has no bona fide business purpose), the transferor shareholder will always recognize gain. a. True b. False 22. When incorporating her sole proprietorship, Samantha transfers all of its assets and liabilities to the corporation. Included in the $30,000 of liabilities assumed by the corporation is $500 that relates to a personal expenditure. Under these circumstances, the entire $30,000 will be treated as boot. a. True b. False 23. To determine whether § 357(c) applies, the taxpayer must assess whether the liabilities involved exceed the bases of all assets a shareholder transfers to the corporation. a. True b. False 24. A taxpayer transfers assets and liabilities to a corporation in return for its stock. If the liabilities exceed the basis of the assets transferred, the taxpayer will have a negative basis in the stock. a. True b. False 25. If both §§ 357(b) and (c) apply to the same transfer (i.e., the liability lacks a bona fide business purpose and also exceeds the basis of the properties transferred), only § 357(c) applies. a. True b. False 26. When a taxpayer transfers property subject to a mortgage to a controlled corporation in an exchange qualifying under § 351, the transferor shareholder’s basis in stock received in the transferee corporation is increased by the amount of the mortgage on the property. a. True b. False 27. Basis of appreciated property transferred minus boot received (including liabilities transferred) plus gain recognized equals basis of stock received in a § 351 transfer. a. True b. False 28. Carl and Ben form Eagle Corporation. Carl transfers cash of $50,000 for 50 shares of stock of Eagle. Ben transfers proprietary information with a tax basis of zero and a fair market value of $50,000 for the remaining 50 shares in Eagle. Powered by Cognero

Page 3


Name:

Class:

Date:

Chapter 04: Corporations Organization and Capital Structure Carl will have a tax basis of $50,000 in his stock in Eagle Corporation and Ben’s basis in his stock will be zero. a. True b. False 29. In general, the basis of property to a corporation in a transfer that qualifies as a nontaxable exchange under § 351 is the basis in the hands of the transferor shareholder decreased by the amount of any gain recognized on the transfer. a. True b. False 30. In return for legal services worth $60,000 rendered incident to its formation, Crimson Corporation issues stock to Greta, an attorney. Crimson cannot immediately deduct the value of any of this stock but instead must capitalize it as an organizational expenditure. a. True b. False 31. Yuna, a real estate dealer, and others form Eagle Corporation under § 351. Yuna contributes inventory (land held for resale) in return for Eagle stock. The holding period for the stock includes the holding period of the inventory. a. True b. False 32. A shareholder transfers a capital asset to Red Corporation for its stock. If the transfer qualifies under § 351, Red’s holding period for the asset begins on the day of the exchange. a. True b. False 33. A shareholder’s holding period for stock received under § 351 can include the holding period of the property transferred to the corporation. a. True b. False 34. When depreciable property is transferred to a controlled corporation under § 351, any recapture potential disappears and does not carry over to the corporation. a. True b. False 35. To encourage the development of an industrial park, a county donates land to Ecru Corporation. The donation results in gross income to Ecru. a. True b. False 36. A city contributes $500,000 to a corporation as an inducement to locate in the city. Within the next 12 months, the corporation uses the money to purchase property worth $500,000. The corporation has income of $500,000 and must reduce its tax basis in the property by the same amount. a. True b. False 37. To ease a cash shortage, all of the shareholders of Osprey Corporation contribute additional cash to its capital. Osprey Powered by Cognero

Page 4


Name:

Class:

Date:

Chapter 04: Corporations Organization and Capital Structure has no tax consequences from the contribution. a. True b. False 38. A shareholder contributes land to his wholly owned corporation but receives no stock in return. The corporation has a zero basis in the land. a. True b. False 39. In structuring the capitalization of a corporation, the tax law is neutral for the investor as to debt versus equity financing. a. True b. False 40. To help avoid the thin capitalization problem, it is advisable to make the repayment of the debt contingent upon the corporation’s earnings. a. True b. False 41. Ira, a calendar year taxpayer, purchases as an investment stock in Redbird Corporation on November 3, 2023. On February 2, 2024, Redbird Corporation is declared bankrupt, and Ira’s stock becomes worthless. Presuming § 1244 (stock in a small business corporation) does not apply, Ira has a short-term capital loss for 2024. a. True b. False 42. Amy owns 20% of the stock of Wren Corporation, which she acquired several years ago at a cost of $10,000. Amy is vice president of Wren and earns a salary of $80,000 annually. Last year, Wren Corporation was experiencing financial problems, and Amy loaned the corporation $25,000. In the current year, Wren becomes bankrupt, and both Amy's stock investment and the loan become worthless. Amy has a nonbusiness bad debt deduction this year of $25,000. a. True b. False 43. If a shareholder owns stock received as a gift from her mother, it cannot be § 1244 or § 1202 stock. a. True b. False 44. A transferor who receives stock for both property and services may not be included in the control group in determining whether an exchange meets the requirements of § 351. a. True b. False 45. One month after Sally incorporates her sole proprietorship, she gives 25% of the stock to her children. Section 351 cannot apply to Sally because she has not satisfied the 80% control requirement. a. True b. False 46. A long-term note is treated as “boot.” Thus, the note received could trigger gain for Eve. Powered by Cognero

Page 5


Name:

Class:

Date:

Chapter 04: Corporations Organization and Capital Structure a. True b. False 47. When a taxpayer incorporates her business, she transfers several liabilities to the corporation. If one of the liabilities is personal in origin, the release of only that liability is treated as boot. a. True b. False 48. A corporation’s holding period for property received under § 351 includes the holding period of the transferor shareholder. a. True b. False 49. Alan, an Owl Corporation shareholder, makes a contribution to capital of equipment to Owl, basis of $40,000 and fair market value of $50,000. Owl’s basis of the equipment that Alan contributes is equal to $50,000, the property’s fair market value. a. True b. False 50. Silver Corporation receives $1 million in cash from Madison County as an inducement to expand its operations there. Within one year, Silver spends $1.5 million to enlarge its existing plant. Silver Corporation’s basis in the expansion is $500,000. a. True b. False 51. Under Federal tax law, a bias for corporate issuers exists in favor of debt as compared to equity when financing the operations of a corporation. a. True b. False 52. If a corporation is thinly capitalized, all debt is reclassified as equity. a. True b. False 53. A shareholder lends money to his corporation in his capacity as an investor. If the loans become worthless, a business bad debt results. a. True b. False Multiple Choice 54. Mitchell and Powell form Green Corporation. Mitchell transfers property with a basis of $105,000 and worth $90,000 while Powell transfers land with a basis of $8,000 and worth $75,000 plus $15,000 of cash. Each receives 50% of Green Corporation’s stock (total value of $180,000). As a result of these transfers: a. Mitchell has a recognized loss of $15,000, and Powell has a recognized gain of $67,000. b. Neither Mitchell nor Powell has any recognized gain or loss. c. Mitchell has no recognized loss, but Powell has a recognized gain of $15,000. Powered by Cognero

Page 6


Name:

Class:

Date:

Chapter 04: Corporations Organization and Capital Structure d. Green Corporation will have a basis in the land of $23,000. 55. Jane transfers property with a $180,000 basis worth $500,000 to Green Corporation for 80% of its stock and a longterm note payable to Jane worth $75,000. a. Jane recognizes no gain. b. Jane recognizes a gain of $75,000. c. Jane recognizes a gain of $270,000. d. Jane recognizes a gain of $320,000. 56. Eileen transfers property worth $200,000 with a basis of $190,000 to Goldfinch Corporation. In return, she receives 80% of the stock in Goldfinch Corporation worth $180,000 and $20,000 cash. Eileen recognizes gain on the transfer of: a. $0. b. $10,000. c. $20,000. d. $190,000. 57. Gabriella and Juanita form Luster Corporation. Gabriella transfers cash of $50,000 for 50 shares of stock, and Juanita transfers information concerning a proprietary process with a basis of zero and worth $50,000 for 50 shares of stock. a. The transfers to Luster are fully taxable to both Gabriella and Juanita. b. Juanita must recognize gain of $50,000. c. Because Juanita is required to recognize gain on the transfer, Gabriella also must recognize gain. d. Neither Gabriella nor Juanita will recognize gain on the transfer. 58. Three individuals form Skylark Corporation with the following contributions: Cliff, cash of $50,000 for 50 shares; Brad, land worth $20,000 (basis of $11,000) for 20 shares; and Ron, cattle worth $9,000 (basis of $6,000) for 9 shares and services worth $21,000 for 21 shares. a. These transfers are fully taxable and not subject to § 351. b. Ron’s basis in his stock is $27,000. c. Ron’s basis in his stock is $6,000. d. Brad’s basis in his stock is $20,000. 59. Seoyun and Nicole form Indigo Corporation with the following transfers: inventory from Seoyun (basis of $360,000 and fair market value of $400,000) and improved real estate from Nicole (basis of $320,000 and fair market value of $375,000). Nicole, an accountant, also agrees to contribute her services (worth $25,000) in organizing Indigo. The corporation’s stock is distributed equally to Seoyun and Nicole. As a result of these transfers: a. Indigo can deduct $25,000 immediately as a business expense. b. Nicole has a recognized gain of $55,000 on the transfer of the real estate. c. Indigo has a basis of $360,000 in the inventory. d. Indigo has a basis of $375,000 in the real estate. 60. Ann transferred land worth $200,000 with a tax basis of $40,000 to Brown Corporation, an existing entity, for 100 shares of its stock. Brown Corporation has two other shareholders, Bill and Bob, each of whom holds 100 shares. With respect to the transfer: a. Ann has no recognized gain. b. Brown Corporation has a basis of $160,000 in the land. c. Ann has a basis of $200,000 in her 100 shares in Brown Corporation. Powered by Cognero

Page 7


Name:

Class:

Date:

Chapter 04: Corporations Organization and Capital Structure d. Ann has a basis of $40,000 in her 100 shares in Brown Corporation. 61. Roberto, a cash basis taxpayer, incorporates his sole proprietorship. He transfers the following items to newly created Orange Corporation. Adjusted Basis Cash $ 10,000 Building 120,000 Mortgage payable (secured by the building and held 135,000 for 15 years)

Fair Market Value $ 10,000 175,000 135,000

With respect to this transaction: a. Orange Corporation’s basis in the building is $120,000. b. Roberto has no recognized gain. c. Roberto has a recognized gain of $5,000. d. Roberto has a recognized gain of $10,000. 62. Albert transfers land with a basis of $140,000 and worth $320,000 to Gold Corporation for 80% of its stock plus a note payable in the amount of $80,000. Gold assumes Albert’s mortgage on the land of $200,000. a. Albert's recognized gain on the transfer is $140,000. b. Albert's total recognized gain on the transfer is $80,000. c. Albert's total recognized gain on the transfer is $60,000. d. Gold Corporation has a basis in the land of $220,000. 63. Rachel owns 100% of the stock of Cardinal Corporation. In the current year, Rachel transfers an installment obligation that has a tax basis of $180,000 and fair market value of $350,000 for additional stock in Cardinal worth $350,000. a. Rachel has a taxable gain of $180,000. b. Rachel has a taxable gain of $170,000. c. Rachel recognizes no gain on the transfer. d. Rachel has a basis of $350,000 in the additional stock she received in Cardinal Corporation. 64. Rob and Yi form Bluebird Corporation with the following investments. Adjusted Basis

Fair Market Value

From Rob— Cash

$400,000

$400,000

From Yi— Land

500,000

440,000

Each receives 50% of Bluebird’s stock. In addition, Yi receives cash of $40,000. One result of these transfers is that Yi has a: a. Recognized loss of $60,000. b. Recognized loss of $20,000. c. Basis of $460,000 in the Bluebird stock (assuming Bluebird reduces its basis in the land to $440,000). Powered by Cognero

Page 8


Name:

Class:

Date:

Chapter 04: Corporations Organization and Capital Structure d. Basis of $400,000 in the Bluebird stock (assuming Bluebird reduces its basis in the land to $440,000). 65. Dwayne and Paul form Swan Corporation with the following investments. Dwayne transfers machinery (basis of $40,000 and fair market value of $100,000) and Paul transfers land (basis of $20,000 and fair market value of $90,000) and services rendered (worth $10,000) in organizing the corporation. Each is issued 25 shares in Swan Corporation. With respect to the transfers: a. Dwayne has no recognized gain; Paul recognizes income/gain of $80,000. b. Neither Dwayne nor Paul has recognized gain or income on the transfers. c. Swan Corporation has a basis of $30,000 in the land transferred by Paul. d. Paul has a basis of $30,000 in the 25 shares he acquires in Swan Corporation. 66. Rick transferred the following assets and liabilities to Warbler Corporation.

Building Equipment Trucks Mortgage (held for four years) on building

Adjusted Basis

Fair Market Value

$210,000 45,000 15,000 30,000

$225,000 75,000 30,000 30,000

In return, Rick received $75,000 in cash plus 90% of Warbler Corporation’s only class of stock outstanding (fair market value of $225,000). a. Rick has a recognized gain of $60,000. b. Rick has a recognized gain of $75,000. c. Rick’s basis in the stock of Warbler Corporation is $270,000. d. Warbler Corporation has the same basis in the assets received as Rick does in the stock. 67. Sarah and Tony (mother and son) form Dove Corporation with the following investments: cash by Sarah of $65,000; land by Tony (basis of $25,000 and fair market value of $35,000). Dove Corporation issues 400 shares of stock, 200 each to Sarah and Tony. Thus, each receives stock in Dove worth $50,000. a. Section 351 cannot apply as Sarah should have received 260 shares instead of only 200. b. Section 351 may apply because stock need not be issued to Sarah and Tony in proportion to the value of the property transferred. c. Tony’s basis in the stock of Dove Corporation is $50,000. d. As a result of the transfer, Tony recognizes a gain of $10,000. 68. Rhonda and Marta form Blue Corporation. Rhonda transfers land (basis of $55,000 and fair market value of $180,000) for 50 shares plus $20,000 cash. Marta transfers $160,000 cash for 50 shares in Blue Corporation. a. Rhonda’s basis in the Blue Corporation stock is $55,000. b. Blue Corporation’s basis in the land is $55,000. c. Blue Corporation’s basis in the land is $180,000. d. Rhonda recognizes a gain on the transfer of $125,000. 69. Erica transfers land worth $500,000 with a basis of $100,000 to newly formed Robin Corporation, for all of it's stock worth $300,000 and a $200,000 10-year note payable to Erica. As a result of the transfer: a. Erica does not recognize gain. Powered by Cognero

Page 9


Name:

Class:

Date:

Chapter 04: Corporations Organization and Capital Structure b. Erica recognizes gain of $400,000. c. Robin Corporation has a basis of $100,000 in the land. d. Robin Corporation has a basis of $300,000 in the land. 70. Donghai transferred the following assets to Starling Corporation.

Cash Machinery Land

Adjusted Basis $120,000 48,000 108,000

Fair Market Value $120,000 36,000 144,000

In exchange, Donghai received 50% of Starling Corporation’s only class of stock outstanding. The stock has no established value. However, all parties believe that the value of the stock Donghai received is the equivalent of the value of the assets she transferred. The only other shareholder, Rick, formed Starling Corporation five years ago. a. Donghai has no gain or loss on the transfer. b. Starling Corporation has a basis of $48,000 in the machinery and $108,000 in the land. c. Starling Corporation has a basis of $36,000 in the machinery and $144,000 in the land. d. Donghai has a basis of $276,000 in the stock of Starling Corporation. 71. Danielle, a sole proprietor, was engaged in a service business and reported her income on a cash basis. Later, she incorporates her business and transfers the assets of the business to the corporation in return for all the stock in the corporation plus the corporation’s assumption of the liabilities of her proprietorship. All the receivables and the unpaid trade payables are transferred to the newly formed corporation. The assets of the proprietorship had a basis of $105,000 and fair market value of $300,000. The trade accounts payable totaled $25,000. The corporation also assumed a note payable to the bank for $95,000 that financed the purchase of computers and other business equipment. a. Danielle has a gain on the transfer of $15,000. b. The basis of the assets to the corporation is $300,000. c. Danielle has a basis of $10,000 in the stock she receives. d. Danielle has a zero basis in the stock she receives. 72. Carl transfers land to Cardinal Corporation for 90% of Cardinal stock worth $20,000 plus a $40,000 note payable to Carl. Cardinal also assumed a $100,000 mortgage on the land. The land, which has a basis to Carl of $70,000, is worth $160,000. a. Carl will have a recognized gain on the transfer of $90,000. b. Carl will have a recognized gain on the transfer of $70,000. c. Cardinal Corporation will have a basis of $70,000 in the land transferred by Carl. d. Cardinal Corporation will have a basis of $160,000 in the land transferred by Carl. 73. Kirby and Helen form Red Corporation. Kirby transfers property with a basis of $20,000 and worth $300,000 for 100 shares in Red Corporation. Helen transfers property with a basis of $40,000 and worth $280,000 and provides $20,000 of legal services in organizing the corporation. In return, Helen receives 100 shares in Red Corporation. Regarding these transfers: a. Kirby will recognize gain. b. Helen will not recognize any gain or income. c. Red Corporation will have a basis of $280,000 in the property it acquired from Helen. d. Red must capitalize Helen's services. Powered by Cognero

Page 10


Name:

Class:

Date:

Chapter 04: Corporations Organization and Capital Structure 74. Chang and Kay form Gull Corporation. Chang transfers cash of $250,000 for 200 shares in Gull Corporation. Kay transfers property with a basis of $50,000 and fair market value of $240,000. She agrees to accept 200 shares in Gull Corporation for the property and for providing bookkeeping services to the corporation in its first year of operation. The value of Kay’s services is $10,000. With respect to the transfer: a. Gull Corporation has a basis of $240,000 in the property transferred by Kay. b. Neither Chang nor Kay recognizes gain or income on the exchanges. c. Gull Corporation has a compensation deduction of $10,000. d. Gull capitalizes $10,000 as organizational costs. 75. Earl and Mary form Crow Corporation. Earl transfers property with a basis of $200,000 and worth $1,600,000 for 50 shares in Crow Corporation. Mary transfers property with a basis of $80,000 and worth $1,480,000 and agrees to serve as manager of Crow for one year. In return, Mary receives 50 shares of Crow. The value of Mary’s services is $120,000. With respect to the transfers: a. Mary will not recognize gain or income. b. Earl will recognize a gain of $1,400,000. c. Crow Corporation has a basis of $1,480,000 in the property it received from Mary. d. Crow will have a business deduction of $120,000 for the value of the services Mary will render. 76. Four individuals form Chickadee Corporation under § 351. Two of these individuals, Shanice and Walt, made the following contributions: Adjusted Basis

Fair Market Value

From Shanice— Cash Patent

$360,000 –0–

$360,000 40,000

From Walt— Equipment (depreciation claimed of $100,000)

240,000

370,000

Both Shanice and Walt receive stock in Chickadee Corporation equal to the value of their investments. a. Shanice must recognize income of $40,000; Walt has no income. b. Neither Shanice nor Walt recognize income. c. Walt must recognize income of $130,000; Shanice has no income. d. Walt must recognize income of $100,000; Shanice has no income. 77. Leah transfers equipment with a basis of $400,000 and worth $500,000 for additional stock in Crow Corporation. After the transfer, Leah owns 80% of Crow’s stock. Associated with the equipment is § 1245 depreciation recapture potential of $70,000. As a result of the transfer: a. Leah recognizes ordinary income of $70,000. b. The § 1245 depreciation recapture potential carries over to Crow Corporation. c. The § 1245 depreciation recapture potential disappears. d. Leah recognizes ordinary income of $70,000 and § 1231 gain of $30,000. 78. To induce Yellow Corporation to build a new manufacturing facility in Knoxville, Tennessee, the city donates land (fair market value of $400,000) and cash of $100,000 to the corporation. Several months after the donation, Yellow Corporation spends $450,000 (which includes the $100,000 received from Knoxville) on the construction of a new plant located on the donated land. Powered by Cognero

Page 11


Name:

Class:

Date:

Chapter 04: Corporations Organization and Capital Structure a. Yellow recognizes income of $100,000 as to the donation. b. Yellow has a zero basis in the land and a basis of $450,000 in the plant. c. Yellow recognizes income of $500,000 as to the donation. d. Yellow has a zero basis in the land and a basis of $350,000 in the plant. 79. George transfers cash of $150,000 to Finch Corporation, a newly formed corporation, for 100% of the stock in Finch worth $80,000 and debt in the amount of $70,000, payable in equal annual installments of $7,000 plus interest at the rate of 9% per annum. In the first year of operation, Finch has net taxable income of $40,000. If Finch pays George interest of $6,300 and $7,000 principal payment on the note: a. George has dividend income of $13,300. b. Finch Corporation does not have a tax deduction with respect to the payment. c. George has dividend income of $7,000. d. Finch Corporation has an interest expense deduction of $6,300. 80. Angel transfers cash of $300,000 and land worth $200,000 to Camel Corporation for 100% of the stock in Camel. In the first year of operation, Camel has net taxable income of $70,000. If Camel distributes $50,000 to Angel: a. Angel has a taxable dividend of $50,000. b. Camel Corporation has a tax deduction of $50,000. c. Angel has no taxable income from the distribution. d. Camel Corporation reduces its basis in the land to $150,000. 81. Wren Corporation (a minority shareholder in Lark Corporation) has made loans to Lark Corporation that become worthless in the current year. a. Wren Corporation is not permitted a deduction for the loans. b. The loans result in a nonbusiness bad debt deduction to Wren Corporation. c. The loans provide Wren Corporation with a business bad debt deduction. d. Wren claims a capital loss due to the uncollectible loans. 82. George (an 80% shareholder) has made loans to Mountainview Corporation that become worthless in the current year. George is not employed by Mountainview. a. George is not permitted a deduction for the worthless loans. b. The loans provide a nonbusiness bad debt deduction to George in the current year. c. The loans provide George with a business bad debt deduction. d. George may claim an ordinary loss as to the worthless loans. 83. When Pheasant Corporation was formed under § 351, Kristen transferred property (basis of $26,000 and fair market value of $22,500) for § 1244 stock. Kristen’s basis in the Pheasant stock is $26,000. Three years later, Pheasant Corporation goes bankrupt and its stock becomes worthless. Kristen, who is single, owned the stock as an investment. Kristen’s loss is: a. $26,000 capital. b. $22,500 ordinary and $3,500 capital. c. $3,500 ordinary and $22,500 capital. d. $26,000 ordinary. 84. Art, an unmarried individual, transfers property with a basis of $130,000 and worth $120,000 to Condor Corporation in exchange for § 1244 stock. The transfer qualifies as a nontaxable exchange under § 351. Because the property is loss Powered by Cognero

Page 12


Name:

Class:

Date:

Chapter 04: Corporations Organization and Capital Structure property, Condor takes a basis of $120,000 in the property. Five years later, Art sells the Condor stock for $50,000. With respect to the sale, Art has: a. An ordinary loss of $80,000. b. An ordinary loss of $70,000 and a capital loss of $10,000. c. A capital loss of $80,000. d. A capital loss of $30,000 and an ordinary loss of $50,000. 85. Lindsay and Malcolm form Yellow Corporation. Lindsay transfers equipment worth $950,000 with a basis of $200,000 plus cash of $50,000 to Yellow Corporation for 50% of its stock. Malcolm transfers a building and land worth $1,050,000 with a basis of $400,000 for 50% of Yellow’s stock and $50,000 in cash. a. Lindsay recognizes no gain; Malcolm recognizes gain of $50,000. b. Lindsay recognizes a gain of $50,000; Malcolm has no gain. c. Neither Lindsay nor Malcolm recognizes gain. d. Lindsay recognizes a gain of $750,000; Malcolm recognizes gain of $650,000. 86. Eve transfers property with a basis of $120,000 and worth $400,000 to Green Corporation for 80% of its stock worth $350,000 and a long-term note worth $50,000 executed by Green Corporation and made payable to Eve. As a result of the transfer: a. Eve recognizes no gain. b. Eve recognizes a gain of $230,000. c. Eve recognizes a gain of $280,000. d. Eve recognizes a gain of $50,000. 87. Mary transfers a building with an adjusted basis of $15,000 and worth $90,000 to White Corporation. In return, Mary receives 80% of White Corporation’s stock worth $65,000 and an automobile worth $5,000. In addition, there is an outstanding mortgage of $20,000 (taken out 15 years ago) on the building, that White Corporation assumes. For this transaction: a. Mary’s recognized gain is $10,000. b. Mary’s recognized gain is $5,000. c. Mary has no recognized gain. d. White Corporation’s basis in the building is $15,000. 88. Hunter and Warren form Tan Corporation. Hunter transfers equipment with a basis of $210,000 and worth $180,000 while Warren transfers land with a basis of $15,000 and worth $150,000 plus $30,000 of cash. Each receives 50% of Tan’s stock. As a result of these transfers: a. Hunter has a recognized loss of $30,000, and Warren has a recognized gain of $135,000. b. Neither Hunter nor Warren has any recognized gain or loss. c. Hunter has no recognized loss, but Warren has a recognized gain of $30,000. d. Tan Corporation will have a basis in the land of $45,000. 89. Blue Corporation (a seller of goods to Cedar Corporation) has made loans to Cedar Corporation, which become worthless in the current year. a. Blue Corporation cannot claim a deduction for the worthless loans. b. The loans provide a nonbusiness bad debt deduction to Blue Corporation. c. The loans provide Blue Corporation with a business bad debt deduction. d. Blue must recognize income. Powered by Cognero

Page 13


Name:

Class:

Date:

Chapter 04: Corporations Organization and Capital Structure 90. Lynn transfers property with a basis of $225,000 and worth $300,000 to Condor Corporation in exchange for § 1244 stock. The transfer qualifies as a nontaxable exchange under § 351. In the current year, Lynn sells the Condor stock for $100,000. Assume Lynn files a joint return with her husband, Ricky. For this sale, Lynn has: a. An ordinary loss of $125,000. b. An ordinary loss of $100,000 and a capital loss of $25,000. c. A capital loss of $125,000. d. An ordinary loss of $100,000 and a capital loss of $100,000. Subjective Short Answer 91. Penny, Miesha, and Sabrina transfer property to Owl Corporation for 75% of its stock. Nancy, their attorney, receives 25% of the stock in Owl for legal services rendered in incorporating the business. What are the tax consequences of these transactions? How could they have structured this transaction to defer tax? 92. Nick exchanges property with a basis of $100,000 and worth $3,000,000 for 65% of the stock of Yellow Corporation. The other 35% of the stock is owned by Gloria who acquired it several years ago. What are the tax consequences to Nick? 93. Perry organized Cardinal Corporation 10 years ago by contributing property worth $2 million with a basis of $450,000 for 2,500 shares of stock in Cardinal, representing 100% of the stock in the corporation. Five years ago, Perry later gave each of his children, Brittany and Julie, 750 shares of stock in Cardinal Corporation. This year, Perry transfers property worth $600,000 with a basis of $150,000 to Cardinal for 1,000 shares in the corporation. What gain, if any, will Perry recognize on the transfer? 94. Ashley, a 70% shareholder of Wren Corporation, transfers property with a basis of $250,000 and a fair market value of $900,000 to Wren Corporation for additional stock. Ashley owns 78% of Wren after the transfer. Two other shareholders in Wren transfer a nominal amount of property to Wren along with Ashley’s transfer so that Ashley and the two shareholders own 90% of the Wren stock after the transfer. Does Ashley have taxable gain on the transfer? 95. Rita forms Finch Corporation by transferring land with a basis of $125,000 and worth $750,000, which is subject to a mortgage of $375,000. Two weeks prior to incorporating Finch, Rita borrows $125,000 for personal purposes and gives the lender a second mortgage on the land. Finch Corporation issues stock worth $250,000 to Rita and assumes the two mortgages on the land. What are the tax consequences to Rita and to Finch Corporation? 96. Nancy, Guy, and Rod form Goldfinch Corporation with the following consideration. Adjusted Basis

Fair Market Value

From Nancy— Cash Inventory

$120,000 90,000

$120,000 130,000

From Guy— Land and building

120,000

250,000

–0–

50,000

From Rod— Legal and accounting services to incorporate

Goldfinch issues its 500 shares of stock as follows: 250 to Nancy, 200 to Guy, and 50 to Rod. In addition, Guy gets $50,000 in cash. Powered by Cognero

Page 14


Name:

Class:

Date:

Chapter 04: Corporations Organization and Capital Structure a.

Does Nancy, Guy, or Rod recognize gain (or income)?

b.

What basis does Guy have in the Goldfinch stock?

c.

What basis does Goldfinch Corporation have in the inventory? In the land and building?

d.

What basis does Rod have in the Goldfinch stock?

97. Sean, a sole proprietor, is engaged in a service business and uses the cash basis of accounting. In the current year, Sean incorporates his business by forming Aqua Corporation. In exchange for all of its stock, Aqua receives: assets with a basis of $400,000 and worth $2,000,000, trade accounts payable of $110,000, and loan of $390,000 due to a bank. The proceeds from the bank loan were used by Sean to provide operating funds for the business. Aqua Corporation assumes all of the liabilities transferred to it. a.

Does Sean recognize any gain on the incorporation? Explain.

b.

What basis does Sean have in the Aqua stock?

c.

What basis does Aqua Corporation have in the assets it receives?

98. Trish and Tyrone form Pine Corporation. Trish transfers inventory with a basis of $60,000 and worth $110,000 for 50% of the stock in Pine. Tyrone transfers machinery with a basis of $20,000 and worth $60,000 and agrees to serve as manager of Pine Corporation for one year for 50% of the stock. What are the tax consequences to Trish, Tyrone, and Pine Corporation? 99. Tan Corporation desires to set up a manufacturing facility in the western part of the United States. After considerable negotiations with Butte, Montana, Tan accepts the following offer: land worth $4,500,000 and cash of $1,500,000. a.

How much income, if any, must Tan recognize?

b.

What basis will Tan Corporation have in the land?

c.

Within one year of the contribution, Tan purchases equipment for $1,600,000. What basis will Tan have in the equipment?

100. Lark City donates land worth $300,000 and cash of $100,000 to Orange Corporation as an inducement to locate in the city. Ann, the sole shareholder, contributes equipment with a basis of $70,000 and worth $200,000 to help Orange in its new operations. What are the tax consequences of these transfers to Orange Corporation? 101. Stock in Merlin Corporation is held equally by Jane, Eve, and Fred. Merlin seeks additional capital to buy a valuable tract of land that will cost $6,000,000. Jane, Eve, and Fred propose to loan Merlin $2,000,000 each, taking from Merlin a $2,000,000 10-year note with interest payable annually at five points above the prime rate. Merlin Corporation has current taxable income of $7,000,000. How are the payments on the notes treated for tax purposes? 102. Karen formed Grebe Corporation with an investment of $100,000 cash for which she received $10,000 in stock and $90,000 in 7% interest-bearing bonds maturing in 10 years. A few years later, Karen loaned Grebe an additional $60,000 on open account. Grebe becomes insolvent in the current year and is adjudged bankrupt. Karen was the president of Grebe Corporation and was paid an annual salary of $50,000 for the past three years. Karen has no other employment. How will Karen treat her losses for tax purposes? 103. Four years ago, Don, a single taxpayer, acquired stock in a corporation that qualified as a small business corporation Powered by Cognero

Page 15


Name:

Class:

Date:

Chapter 04: Corporations Organization and Capital Structure under § 1244, at a cost of $60,000. Don wants to give his son, Ron, $20,000 to help finance Ron’s college education. The stock is currently worth $20,000. Don is considering selling the stock in the current year for $20,000 and giving the cash to Ron. As an alternative, Don could give the stock to Ron and let Ron sell it for $20,000. Which alternative should Don choose? 104. Joyce, a single taxpayer, transfers property with a basis of $120,000 and worth $60,000 to Wren Corporation in exchange for shares of § 1244 stock. Because the transfer qualifies under § 351, Joyce takes a $120,000 basis in the Wren stock. In the current year, Joyce sells the Wren Corporation stock for $40,000. What are the consequences of the sale to Joyce? Essay 105. What is the rationale underlying the tax deferral treatment available under § 351? 106. How is the transfer of liabilities in a property transaction generally treated for tax purposes? How is a transfer of liabilities generally treated in a § 351 transaction? What exceptions could arise to this usual treatment in a § 351 setting? 107. For transfers falling under § 351, what are the holding period rules for stock received by the shareholder and for the assets transferred to the corporation? 108. When forming a corporation, a transferor-shareholder may choose to receive some corporate debt along with stock. Identify some of the issues the transferor must consider when deciding whether debt should be a part of the transaction. 109. What are the tax consequences if an individual investor incurs a loss because of worthlessness on the following:

a.

Stock that is not § 1244 stock.

b.

Stock that is § 1244 stock.

c.

A corporate bond.

d.

An uncollectible loan made to a corporation.

110. Tom sold all of his corporate stock in Blazer Corporation to an unrelated party for $28,000,000. Tom invested $1,500,000 plus his proprietary know-how in January 2013 and his stock basis had not changed since that investment. Tom is the sole shareholder of Blazer. What is Tom's taxable gain and income tax liability from the sale, assuming that he is in the 37% tax bracket and has no other property sales during the year?

Powered by Cognero

Page 16


Name:

Class:

Date:

Chapter 04: Corporations Organization and Capital Structure Answer Key 1. True 2. False 3. False 4. False 5. True 6. False 7. True 8. True 9. False 10. True 11. True 12. False 13. True 14. True 15. False 16. True 17. False 18. True 19. False 20. True 21. False 22. True 23. True 24. False 25. False Powered by Cognero

Page 17


Name:

Class:

Date:

Chapter 04: Corporations Organization and Capital Structure 26. False 27. True 28. True 29. False 30. True 31. False 32. False 33. True 34. False 35. True 36. False 37. True 38. False 39. False 40. False 41. False 42. False 43. True 44. False 45. False 46. True 47. False 48. True 49. False 50. False Powered by Cognero

Page 18


Name:

Class:

Date:

Chapter 04: Corporations Organization and Capital Structure 51. True 52. False 53. False 54. b 55. b 56. b 57. d 58. b 59. c 60. c 61. c 62. a 63. c 64. c 65. d 66. a 67. b 68. a 69. d 70. c 71. c 72. b 73. d 74. c 75. d 76. b Powered by Cognero

Page 19


Name:

Class:

Date:

Chapter 04: Corporations Organization and Capital Structure 77. b 78. c 79. d 80. a 81. c 82. b 83. b 84. d 85. a 86. d 87. a 88. b 89. c 90. b 91. Based on the facts provided, the transaction will be taxable to all persons involved. Section 351 treatment will be lost if stock is transferred to persons who did not contribute property, causing those who did to lack control immediately after the exchange. However, if a person performs services for the corporation in exchange for stock and also transfers some property, they may be treated as a member of the transferring group although the value of the stock issued for services is taxed. 92. Nick has a taxable gain of $2,900,000. Section 351 does not apply because Nick failed to receive at least 80% control of Yellow Corporation. Therefore, the transaction is a taxable exchange. Nick has a $3 million basis in his stock and Yellow Corporation has a basis of $3,000,000 in the property. 93. Perry recognizes a gain of $450,000 on the transfer [$600,000 (value of the stock received) – $150,000 (basis in the property)]. The transfer does not qualify under § 351. Although Perry originally owned 100% of Cardinal Corporation, he only owns 57% of Cardinal Corporation after the transfer [2,500 (shares originally owned) – 1,500 (shares transferred to Brittany and Julie) + 1,000 (shares acquired in the transfer), or 2,000 shares out of a total of 3,500 shares]. The ownership of the shares held by Brittany and Julie cannot be counted because the attribution rules of § 318 do not apply to a § 351 transfer. 94. Ashley would have a taxable gain of $650,000 on the transfer. She does not have the requisite 80% control. The transfer by the two shareholders will not qualify the transfer for § 351 treatment because the primary purpose of the transfer was to qualify under this section. If the transfer of property by the two shareholders has a value equal to or in excess of 10% of the fair market value of the stock owned by them after the transfer, the transfer would qualify. Powered by Cognero

Page 20


Name:

Class:

Date:

Chapter 04: Corporations Organization and Capital Structure 95. Both §§ 357(b) and (c) are applicable. Because the land is subject to two mortgages that are in excess of basis, under § 357(c) Rita would have a recognized gain of $375,000 on the transfer. But § 357(b) also is applicable because Rita borrowed the $125,000 shortly before incorporating and used the money for personal purposes. As § 357(b) causes all the liabilities to be tainted, Rita has boot of $500,000. Of Rita’s realized gain of $625,000 [$750,000 (value of the stock received and release of mortgages) – $125,000 (basis in the land)], $500,000 gain (the amount of boot) is recognized. When §§ 357(b) and (c) both apply to the same transfer, § 357(b) applies. Finch Corporation has a basis of $625,000 in the land, computed as follows: $125,000 (carryover basis from Rita) + $500,000 (gain recognized by Rita). Rita has a basis of $125,000 in her stock, computed as follows: $125,000 (basis in the land) + $500,000 (gain recognized) – $500,000 (liabilities assumed by Finch Corporation). 96. a. Nancy recognizes no gain. Due to the boot he receives, Guy recognizes $50,000 of gain. Rod has ordinary income of $50,000 for the services he performs. b.

Guy’s basis in the Goldfinch stock is $120,000 [$120,000 (basis in the land and building) + $50,000 (gain recognized) – $50,000 (boot received)].

c.

Goldfinch Corporation’s basis in the inventory is $90,000. Its basis in the land and building is $170,000 [$120,000 (Guy’s basis) + $50,000 (gain recognized by Guy)].

d.

Rod’s basis in the Goldfinch stock is $50,000.

97. a. Initially, it seems as if the liabilities of $500,000 [$110,000 (trade accounts payable) + $390,000 (bank loan)] exceed the basis of the assets so as to make § 357(c) apply. However, for this purpose the trade accounts payable are not counted since they originate from a cash basis taxpayer and would give rise to a deduction. Thus, Sean has no recognized gain. b.

$10,000 [$400,000 (basis in the assets) – $390,000 (bank loan assumed by Aqua Corporation)].

c.

$400,000 (Sean’s basis in the assets).

98. Tyrone’s stock in Pine Corporation is counted in determining control for purposes of § 351. All of Tyrone’s stock, not just the shares received for the machinery, is included because the property he transfers has more than a nominal value in comparison to the value of the services rendered. (The property transferred has a value of at least 10% of the value of the services provided.) Trish recognizes no gain on the transfer and has a basis of $60,000 in the stock received. Pine Corporation has a basis of $60,000 in the inventory it receives. Tyrone recognizes ordinary income of $50,000 on the transaction. Even though the transfer of the machinery qualifies under § 351, his transfer of services for stock does not. Tyrone has a basis of $70,000 in his stock computed as follows: $20,000 (his basis in the machinery) + $50,000 (value of his services). [His services are valued at $50,000 because this is the difference between the value of the property transferred ($60,000) and the value of the stock received ($110,000).] Pine Corporation has a basis of $20,000 in the machinery and can claim a deduction of $50,000 for the services Tyrone will render. 99. a. Tan Corporation recognizes $6 million ($4,500,000 land + $1,500,000 cash) of income from the receipt of land and cash because it does not qualify as a nontaxable capital contribution under § 118. b.

Tan has a $4,500,000 basis in the land.

Powered by Cognero

Page 21


Name:

Class:

Date:

Chapter 04: Corporations Organization and Capital Structure c.

Tan Corporation takes a $1,600,000 basis in the equipment (i.e., the cost of the equipment).

100. Orange Corporation does not have income on the transfers from Ann (a shareholder), but it does recognize $400,000 of income on the transfers from Lark City (a nonshareholder). Orange will have a basis of $70,000 in the equipment it receives from Ann and a $300,000 basis in the land it receives from Lark City. Finally, the transfer, which is a capital contribution by Ann, increases her stock basis in Orange by $70,000. 101. Payments on the notes will probably be treated as dividends for tax purposes. The debt instruments have too many features of stock. The debt does not bear a legitimate rate of interest, and the debt is proportionate to the stock holdings of Jane, Eve, and Fred. Merlin Corporation has substantial current taxable income indicating an attempt to withdraw earnings in the form of principal and interest payments on debt obligations rather than as dividends. 102. If the stock is § 1244 stock, Karen has an ordinary loss on the worthless stock. Otherwise, her $10,000 stock investment is a capital loss. The IRS could argue thin capitalization to make the long-term debt equity, and thus result in a capital loss. Also, the IRS could contend that both the long-term debt (regardless of whether it can be deemed hybrid stock) and the $60,000 open account debt are nonbusiness bad debts and, therefore, short-term capital losses. Karen would counter with the argument that the $60,000 open account debt is a business bad debt because the primary motive in loaning money to the corporation was to protect her employment. Although the loan is more than her annual salary, she is paid the salary continually. Thus, in that context, the salary is more than the investment. Further, she only works for the corporation. 103. Don should sell the stock. He will have a $40,000 ordinary loss deduction in the current year. Only the original holder of the stock (Don) qualifies under § 1244 for ordinary loss treatment. If Don gives the stock to Ron, Ron will have a basis of $20,000 in the stock and, thus, will have no loss deduction. A carryover basis for gifts applies unless the fair market value of the property is less on the date of the gift and the property is sold at a loss. 104. Joyce recognizes a loss of $80,000 [$40,000 (selling price of the stock) – $120,000 (basis)]. She has an ordinary loss under § 1244 of $20,000 [$40,000 (selling price) – $60,000 (basis under § 1244)] and a capital loss of $60,000. 105. Realized gain or loss is not recognized in a § 351 transaction when a taxpayer’s economic status has not changed. This provision reflects the principle that gain should not be recognized when a taxpayer’s investment has not substantively changed. When a business is incorporated, the owner’s economic status remains the same; only the form of the investment has changed. Gain deferral is also justified under the wherewithal to pay concept discussed in Chapter 1. This concept recognizes that if the shareholder receives solely stock in the exchange, they are hardly in a position to pay a tax on any realized gain. Finally, § 351 was enacted because Congress believed that incorporating a business should not trigger a tax cost. Otherwise, recognizing gain and paying tax could discourage corporate formations when otherwise optimal. 106. Generally, when another party assumes a liability in a property transaction, the party no longer responsible for the debt is treated as having received cash or boot. This is consistent with the form of benefit doctrine and with like-kind exchange treatment under § 1031. However, when the acquiring corporation assumes a liability in a § 351 transaction, § 357(a) provides that the transfer does not result in boot to the transferor-shareholder for gain recognition purposes. To do so could trigger gain to the property transferor if the corporation assumed a mortgage on the transfer of encumbered property, which could, in turn, discourage the use of the corporate form of business. The general rule of § 357(a) has two exceptions: (1) § 357(b) provides that if the principal purpose of the assumption of Powered by Cognero

Page 22


Name:

Class:

Date:

Chapter 04: Corporations Organization and Capital Structure the liabilities is to avoid tax or if there is no bona fide business purpose behind the exchange, the liabilities are treated as boot, and (2) § 357(c) provides that if the sum of the liabilities exceeds the adjusted basis of the properties transferred, the excess is taxable gain. 107. In a § 351 transaction, the shareholder’s holding period for stock received in exchange for a capital asset or § 1231 property includes the holding period of the property transferred to the corporation. That is, the holding period of the property is “tacked on” to the holding period of the stock. The holding period for stock received for any other property begins on the day after the exchange. The corporation’s holding period for property acquired in a § 351 transfer is the holding period of the transferor-shareholder, regardless of the character of the property in the transferor’s hands. 108. Significant tax differences exist between debt and equity in the capital structure. ∙

Interest payments on debt are deductible by the corporation while dividend payments on stock are not.

Loan repayments of debt are not taxable to investors unless the repayments exceed basis; however, a shareholder’s nonliquidating receipt of property from a corporation cannot be taxfree as long as the corporation has earnings and profits.

Dividend income on equity holdings is taxed to individual investors at the preferential capital gains rates while interest income on debt is taxed at the higher ordinary income tax rates.

109. a. Stock that is not § 1244 stock: If stock is a capital asset, losses from its worthlessness results in capital loss treatment as of the last day of the taxable year in which the stock becomes worthless. No deduction is allowed for a mere decline in value. A loss on partial worthlessness may be recognized if the stock is sold in a taxable transaction. b.

Stock that is § 1244 stock: Section 1244 provides ordinary (rather than capital) loss treatment on the sale or worthlessness. The amount of ordinary loss treatment is limited in any one year to $50,000 ($100,000 on a joint return). If the loss exceeds the amount allowed as ordinary, the excess is a capital loss.

c.

Corporate bond: If a bond is a capital asset, losses result in capital loss treatment as of the last day of the taxable year in which the bond becomes worthless. No deduction is allowed for a mere decline in value. A loss on partial worthlessness may be recognized if the bond is sold in a taxable transaction.

d.

An uncollectible loan made to a corporation: An uncollectible loan is treated either as a business bad debt or a nonbusiness bad debt. Business bad debts are ordinary losses while nonbusiness bad debts are short-term capital losses. For individuals lending money to a corporation in their capacity as investors, bad debts are classified as nonbusiness. If a loan is made in a capacity that qualifies as a trade or business, business bad debt treatment results.

110. If the stock qualifies as qualified small business stock, section 1202 will apply. If section 1202 applies, Tom's gain equals $11,500,000, which is the gain ($28,000,000 - $1,500,000 basis) less the § 1202 exclusion of $15,000,000 ($1,500,000 * 10, which exceeds $10,000,000). His tax liability at the top capital gain rate of 23.8% is $2,737,000.

Powered by Cognero

Page 23


Name:

Class:

Date:

Chapter 05: Corporations Earnings and Profits and Dividend Distributions True / False 1. Distributions by a corporation to its shareholders are presumed to be a dividend unless the parties can prove otherwise. a. True b. False 2. A distribution from a corporation will be taxable to the recipient shareholders only to the extent of the corporation’s E & P. a. True b. False 3. All distributions that are not dividends are a return of capital and decrease the shareholder’s basis. a. True b. False 4. All cash distributions received from a corporation with a positive balance in accumulated E & P at the beginning of the year will be taxed as dividend income. a. True b. False 5. A distribution in excess of E & P is treated as capital gain by shareholders. a. True b. False 6. The terms “earnings and profits” and “retained earnings” are identical in meaning. a. True b. False 7. To determine E & P, some (but not all) previously excluded income items are added back to taxable income. a. True b. False 8. When computing E & P, taxable income is not adjusted for § 179 expense. a. True b. False 9. When computing current E & P, taxable income must be adjusted for the deferred gain in a § 1031 like-kind exchange. a. True b. False 10. An increase in the LIFO recapture amount must be added to taxable income to determine E & P. a. True b. False 11. Use of MACRS cost recovery when computing taxable income does not require an E & P adjustment. a. True Powered by Cognero

Page 1


Name:

Class:

Date:

Chapter 05: Corporations Earnings and Profits and Dividend Distributions b. False 12. No E & P adjustment is required for regular tax gains under the installment method. a. True b. False 13. A corporation borrows money to purchase State of Texas bonds. The interest on the loan has no impact on either taxable income or current E & P. a. True b. False 14. Federal income tax paid in the current year must be subtracted from taxable income to determine E & P. a. True b. False 15. To determine current E & P, taxable income must be increased for any dividends received deduction. a. True b. False 16. Nondeductible meal expense must be subtracted from taxable income to determine current E & P. a. True b. False 17. The dividends received deduction has no impact on E & P. a. True b. False 18. A realized gain from an involuntary conversion under § 1033 that is not recognized for income tax purposes has no effect on E & P. a. True b. False 19. In the current year, Carnation Corporation has a § 179 expense of $20,000. As a result, in the current year, taxable income must be increased by $16,000 to determine current E & P. a. True b. False 20. A deficit in current E & P is treated as occurring ratably during the year unless the taxpayer can show otherwise. a. True b. False 21. When current E & P has a deficit and accumulated E & P is positive, the two accounts are netted at the date of the distribution. If a positive balance results, the distribution is a dividend to the extent of the balance. a. True b. False Powered by Cognero

Page 2


Name:

Class:

Date:

Chapter 05: Corporations Earnings and Profits and Dividend Distributions 22. When current E & P is positive and accumulated E & P has a deficit balance, the two accounts are netted for dividend determination purposes. a. True b. False 23. Regardless of any deficit in current E & P, distributions during the year are taxed as dividends to the extent of accumulated E & P. a. True b. False 24. Corporate distributions are presumed to be paid out of E & P and are treated as dividends unless the parties to the transaction can show otherwise. a. True b. False 25. Dividends paid to shareholders who hold both long and short positions do not qualify for the reduced tax rate available to individuals in certain years. a. True b. False 26. Dividends taxed as ordinary income are considered investment income for purposes of the investment interest expense limitation. a. True b. False 27. Certain dividends from foreign corporations can be qualified dividends for purposes of the preferential rate available to individuals. a. True b. False 28. During the year, Blue Corporation distributes land to its sole shareholder. If the fair market value of the land is less than its adjusted basis, Blue will not be able to recognize a loss on the distribution. a. True b. False 29. In a property distribution, the amount of dividend income recognized by a shareholder is always reduced by the amount of liability assumed by a shareholder. a. True b. False 30. Property distributed by a corporation as a dividend is subject to a liability in excess of its basis. For purposes of determining gain on the distribution, the basis of the property is treated as being not less than the amount of liability. a. True b. False 31. A corporation that distributes a property dividend must reduce its E & P by the adjusted basis of the property less any liability on the property. Powered by Cognero

Page 3


Name:

Class:

Date:

Chapter 05: Corporations Earnings and Profits and Dividend Distributions a. True b. False 32. Under certain circumstances, a distribution can generate (or add to) a deficit in E & P. a. True b. False 33. Constructive dividends do not need to satisfy the legal requirements for a dividend as set forth by applicable state law. a. True b. False 34. Constructive dividends have no effect on a distributing corporation’s E & P. a. True b. False 35. If a stock dividend is taxable, the shareholder’s basis in the newly received shares is equal to the fair market value of the shares received in the distribution. a. True b. False 36. A corporate shareholder who receives a constructive dividend cannot apply a dividends received deduction to the distribution. a. True b. False 37. If a distribution of stock rights is taxable and their fair market value is less than 15 percent of the value of the old stock, then either a zero basis or a portion of the old stock basis may be assigned to the rights at the shareholder’s option. a. True b. False 38. If there is sufficient E & P, a distribution of nonconvertible preferred stock to common shareholders is taxable. a. True b. False 39. The rules used to determine the taxability of stock dividends also apply to distributions of stock rights. a. True b. False 40. If stock rights are taxable, the recipient has income to the extent of the fair market value of the rights. a. True b. False Multiple Choice 41. The tax treatment of corporate distributions at the shareholder level does not depend on: a. The character of the property being distributed. Powered by Cognero

Page 4


Name:

Class:

Date:

Chapter 05: Corporations Earnings and Profits and Dividend Distributions b. The earnings and profits of the corporation. c. The basis of stock in the hands of the shareholder. d. Whether the distributed property is received by an individual or a corporation. 42. Rose Corporation (a calendar year taxpayer) has taxable income of $300,000, and its financial records reflect the following for the year. Federal income taxes paid Net operating loss carryforward deducted currently Gain recognized this year on an installment sale from a prior year Depreciation deducted on tax return (ADS depreciation would have been $10,000) Interest income on Iowa state bonds

$110,000 70,000 44,000 40,000 8,000

Rose Corporation’s current E & P is: a. $254,000. b. $214,000. c. $194,000. d. $104,000. 43. Tern Corporation, a cash basis taxpayer, has taxable income of $500,000 for the current year. Tern elected $25,000 of § 179 expense. It also had a related-party loss of $20,000 and a realized (not recognized) gain from an involuntary conversion of $75,000. It paid Federal income tax of $150,000 and paid a nondeductible fine of $10,000. Tern’s current E & P is: a. $415,000. b. $350,000. c. $340,000. d. $320,000. 44. Silver Corporation, a calendar year taxpayer, has taxable income of $550,000. Among its transactions for the year are the following: Collection of proceeds from insurance policy on life of corporate officer (in excess of cash surrender value) Realized gain (not recognized) on an involuntary conversion Nondeductible fines and penalties

$82,500 11,000 44,000

Disregarding any provision for Federal income taxes, Silver Corporation’s current E & P is: a. $500,500. b. $588,500. c. $599,500. d. $687,500. 45. Which of the following statements is incorrect with respect to determining current E & P? a. All tax-exempt income should be added back to taxable income. b. Dividends received deductions should be subtracted from taxable income. c. Current-year charitable contributions in excess of the 10% of taxable income limit should be subtracted from taxable income. d. Federal income tax refunds should be added back to taxable income. Powered by Cognero

Page 5


Name:

Class:

Date:

Chapter 05: Corporations Earnings and Profits and Dividend Distributions 46. Aaron and Michele, equal shareholders in Cavalier Corporation, receive $25,000 each in distributions on December 31 of the current year. During the current year, Cavalier sold an appreciated asset for $60,000 (basis of $15,000). Payment for the sale of the asset will be made as follows: 50% next year and 50% in the following year with interest payable at a rate of 6 percent. Before considering the effect of the asset sale, Cavalier’s current-year E & P is $40,000 and it has no accumulated E & P. How much of Aaron’s distribution will be taxed as a dividend? a. $0 b. $20,000 c. $25,000 d. $42,500 47. Tracy and Jerome, equal shareholders in Macaw Corporation, receive $600,000 each in distributions on December 31 of the current year. Macaw’s current-year taxable income is $1,000,000 and it has no accumulated E & P. Last year, Macaw sold an appreciated asset for $1,200,000 (basis of $400,000). Payment for one-half of the sale of the asset was made this year. How much of Tracy’s distribution will be taxed as a dividend? a. $0 b. $300,000 c. $500,000 d. $600,000 48. Falcon Corporation ended its first year of operations with taxable income of $250,000. At the time of Falcon’s formation, it incurred $50,000 of organizational expenses. In calculating its taxable income for the year, Falcon claimed an $8,000 deduction for the organizational expenses. What is Falcon’s current E & P? a. $200,000 b. $208,000 c. $250,000 d. $258,000 49. During the current year, Hawk Corporation sold equipment for $600,000 (adjusted basis of $360,000). The equipment was purchased a few years ago for $760,000 and $400,000 in MACRS deductions have been claimed. ADS depreciation would have been $300,000. As a result of the sale, the adjustment to taxable income needed to determine current E & P is: a. No adjustment is required. b. Subtract $100,000. c. Add $100,000. d. Add $80,000. 50. On January 2, 2024, Orange Corporation purchased equipment for $300,000 with an ADS recovery period of 10 years and a MACRS useful life of 7 years. Section 179 was not elected. MACRS depreciation properly claimed on the asset, including depreciation in the year of sale, totaled $79,605. The equipment was sold on July 1, 2025, for $290,000. As a result of the sale, the adjustment to taxable income needed to arrive at current E & P is: a. No adjustment is required. b. Decrease $49,605. c. Increase $49,605. d. Decrease $79,605. 51. Tungsten Corporation, a calendar year cash basis taxpayer, made estimated tax payments of $800 each quarter in Powered by Cognero

Page 6


Name:

Class:

Date:

Chapter 05: Corporations Earnings and Profits and Dividend Distributions 2024, for a total of $3,200. Tungsten filed its 2024 tax return in 2025 and the return showed a tax liability $4,200. When it filed its tax return in 2025, Tungsten paid an additional $1,000 in Federal income taxes. How does the additional payment of $1,000 impact Tungsten’s E & P? a. Increase by $1,000 in 2024. b. Increase by $1,000 in 2025. c. Decrease by $1,000 in 2024. d. Decrease by $1,000 in 2025. 52. Inka and Eva each own one-half of the stock in Parakeet Corporation, a calendar year taxpayer. Cash distributions from Parakeet are $350,000 to Inka on April 1 and $150,000 to Eva on May 1. If Parakeet’s current E & P is $60,000, how much is allocated to Eva’s distribution? a. $5,000 b. $10,000 c. $18,000 d. $30,000 53. Cedar Corporation is a calendar year taxpayer formed in 2020. Cedar’s E & P before distributions for each of the past five years is listed below. 2024 2023 2022 2021 2020

$28,000 $40,000 $39,000 $68,000 $16,000

Cedar Corporation made the following distributions in the previous five years. 2023 2020

Land (basis of $70,000, fair market value of $80,000) $20,000 cash

Cedar’s accumulated E & P as of January 1, 2025 is: a. $91,000. b. $95,000. c. $101,000. d. $105,000. 54. Maria and Christopher each own 50% of Cockatoo Corporation, a calendar year taxpayer. Distributions from Cockatoo are $750,000 to Maria on April 1 and $250,000 to Christopher on May 1. Cockatoo’s current E & P is $300,000 and its accumulated E & P is $600,000. How much of the accumulated E & P is allocated to Christopher’s distribution? a. $0 b. $75,000 c. $150,000 d. $300,000 55. Robin Corporation, a calendar year taxpayer, has a deficit in current E & P of $200,000 and a $580,000 positive balance in accumulated E & P. If Robin determines that a $700,000 distribution to its shareholders is appropriate at some point during the year, what is the maximum amount of the distribution that could potentially be treated as a dividend? Powered by Cognero

Page 7


Name:

Class:

Date:

Chapter 05: Corporations Earnings and Profits and Dividend Distributions a. $0 b. $380,000 c. $480,000 d. $580,000 56. Pheasant Corporation, a calendar year taxpayer, has $400,000 of current E & P and a deficit in accumulated E & P of $180,000. If Pheasant pays a $600,000 distribution to its shareholders on July 1, how much dividend income do the shareholders report? a. $0 b. $20,000 c. $220,000 d. $400,000 57. Jasmine is the sole shareholder of Condor Corporation. She sold her stock to Melissa on October 31 for $150,000. Jasmine’s basis in Condor stock was $50,000 at the start of the year. Condor distributed land to Jasmine immediately before the sale. Condor’s basis in the land was $20,000 (fair market value of $25,000). On December 31, Melissa received a $75,000 cash distribution from Condor. During the year, Condor has $20,000 of current E & P and its accumulated E & P balance on January 1 is $10,000. Which of the following statements is true? a. Jasmine recognizes a $110,000 gain on the sale of her stock. b. Jasmine recognizes a $100,000 gain on the sale of her stock. c. Melissa receives $5,000 of dividend income. d. Jasmine receives $20,000 of dividend income. 58. Blue Corporation has a deficit in accumulated E & P of $300,000 and has current E & P of $225,000. On July 1, Blue distributes $250,000 to its sole shareholder, Sam, who has a basis in his stock of $52,500. As a result of the distribution, Sam has: a. Dividend income of $225,000 and reduces his stock basis to $27,500. b. Dividend income of $52,500 and reduces his stock basis to zero. c. Dividend income of $225,000 and no adjustment to stock basis. d. No dividend income, reduces his stock basis to zero, and has a capital gain of $250,000. 59. Renee, the sole shareholder of Indigo Corporation, sold her stock to Chad on July 1 for $180,000. Renee’s stock basis at the beginning of the year was $120,000. Indigo made a $60,000 cash distribution to Renee immediately before the sale and Chad received a $120,000 cash distribution from Indigo on November 1. As of the beginning of the current year, Indigo had $26,000 in accumulated E & P and current E & P (before distributions) was $90,000. Which of the following statements is correct? a. Renee recognizes a $60,000 gain on the sale of the stock. b. Renee recognizes a $64,000 gain on the sale of the stock. c. Chad recognizes dividend income of $120,000. d. Chad recognizes dividend income of $30,000. 60. Tangelo Corporation has an August 31 year-end. Tangelo had $50,000 in accumulated E & P at the beginning of its 2025 fiscal year (September 1, 2024) and during the year, it incurred a $75,000 operating loss. It also distributed $65,000 to its sole shareholder, Cass, on November 30, 2024. If Cass is a calendar year taxpayer, how should she treat the distribution when she files her 2024 income tax return (assuming the return is filed by April 15, 2025)? a. $65,000 of dividend income. Powered by Cognero

Page 8


Name:

Class:

Date:

Chapter 05: Corporations Earnings and Profits and Dividend Distributions b. $60,000 of dividend income and $5,000 recovery of capital. c. $50,000 of dividend income and $15,000 recovery of capital. d. The distribution has no effect on Cass in the current year. 61. As of January 1, Cassowary Corporation has a deficit in accumulated E & P of $100,000. For the tax year, current E & P (accrued ratably) is $240,000 (prior to any distributions). On July 1, Cassowary Corporation distributes $275,000 to its sole shareholder. The amount of the distribution that is a dividend is: a. $20,000. b. $140,000. c. $240,000. d. $275,000. 62. At the beginning of the current year, both Doug and Amelia each own 50% of Amaryllis Corporation (a calendar year taxpayer). In July, Doug sold his stock to Kevin for $140,000. At the beginning of the year, Amaryllis Corporation had accumulated E & P of $240,000 and its current E & P is $280,000 (prior to any distributions). Amaryllis distributed $300,000 on February 15 ($150,000 to Doug and $150,000 to Amelia) and distributed another $300,000 on November 1 ($150,000 to Kevin and $150,000 to Amelia). Kevin has dividend income of: a. $150,000. b. $140,000. c. $110,000. d. $70,000. 63. On January 1, Eagle Corporation (a calendar year taxpayer) has accumulated E & P of $300,000. During the year, Eagle incurs a net loss of $420,000 from operations that accrues ratably. On June 30, Eagle distributes $180,000 to Libby, its sole shareholder, who has a basis in her stock of $112,500. How much of the $180,000 is a dividend to Libby? a. $0 b. $90,000 c. $112,500 d. $180,000 64. Which of the following is not a consequence of the double tax on dividends? a. Corporations have an incentive to retain earnings and structure distributions to avoid dividend treatment. b. Corporations have an incentive to invest in noncorporate rather than corporate businesses. c. The cost of capital for corporate investments is decreased. d. Corporations have an incentive to finance operations with debt rather than equity. 65. Which one of the following statements is false? a. Most countries that trade with the United States impose a double tax on dividends. b. Tax proposals that include corporate integration would eliminate the double tax on dividends. c. The double tax on dividends may make corporations more financially vulnerable during economic downturns. d. Many of the arguments in support of the double tax on dividends relate to fairness. 66. In June of the current year, Marigold Corporation declares a $4 dividend out of E & P on each share of common stock to shareholders of record on August 1. Ellen and Tim each purchase 100 shares of Marigold stock on July 1. On July 15, Ellen also purchases a short position in Marigold. Tim sells 50 of his shares on August 10 and continues to hold the remaining 50 shares through the end of the year. Ellen closes her short position in Marigold on October 15. With respect Powered by Cognero

Page 9


Name:

Class:

Date:

Chapter 05: Corporations Earnings and Profits and Dividend Distributions to the dividends, which of the following is correct? a. Ellen will have $400 of qualifying dividends subject to reduced tax rates and $400 of ordinary income (from dividends paid on the short position of Marigold stock). b. Tim will have $200 of qualifying dividends subject to reduced tax rates and $200 of ordinary income. c. All $800 of Ellen’s dividends will qualify for reduced tax rates. d. All $400 of Tim’s dividends will qualify for reduced tax rates. 67. In the current year, Warbler Corporation (E & P of $250,000) made the following property distributions to its shareholders (all corporations):

Pink Corporation stock (held for investment) Non-LIFO inventory

Adjusted Basis $150,000 80,000

Fair Market Value $120,000 110,000

Warbler Corporation is not a member of a controlled group. As a result of the distribution: a. The shareholders have dividend income of $200,000. b. The shareholders have dividend income of $230,000. c. Warbler has a recognized gain of $30,000 and a recognized loss of $30,000. d. Warbler has no recognized gain or loss. 68. Purple Corporation makes a property distribution to its sole shareholder, Kyung. The property distributed is a house (fair market value of $189,000; basis of $154,000) that is subject to a $245,000 mortgage that Kyung assumes. Before considering the consequences of the distribution, Purple’s current E & P is $35,000 and its accumulated E & P is $140,000. Purple makes no other distributions during the current year. What is Purple’s taxable gain on the distribution of the house? a. $0 b. $21,000 c. $35,000 d. $91,000 69. Puffin Corporation makes a property distribution to its sole shareholder, Bonnie. The property distributed is a car (basis of $30,000; fair market value of $20,000) that is subject to a $6,000 liability, which Bonnie assumes. Puffin has no accumulated E & P and $30,000 of current E & P from other sources during the year. What is Puffin’s E & P after taking into account the distribution of the car? a. $4,000 b. $6,000 c. $10,000 d. $14,000 70. Navy Corporation has E & P of $240,000. It distributes land with a fair market value of $70,000 (adjusted basis of $25,000) to its sole shareholder, Troy. The land is subject to a liability of $55,000 that Troy assumes. Troy has: a. A taxable dividend of $15,000. b. A taxable dividend of $25,000. c. A taxable dividend of $70,000. d. A basis in the machinery of $55,000. Powered by Cognero

Page 10


Name:

Class:

Date:

Chapter 05: Corporations Earnings and Profits and Dividend Distributions 71. Which one of the following statements about property distributions is false? a. When the basis of distributed property is greater than its fair market value, a deficit may be created in E & P. b. When the basis of distributed property is less than its fair market value, the distributing corporation recognizes gain. c. When the basis of distributed property is greater than its fair market value, the distributing corporation does not recognize loss. d. The amount of a distribution received by a shareholder is measured by using the property’s fair market value. 72. Brett owns stock in Oriole Corporation (basis of $100,000) as an investment. Oriole distributes property (fair market value of $375,000; basis of $187,500) to him during the year. Oriole has current E & P of $25,000 (which includes the E & P gain on the property distribution), accumulated E & P of $100,000, and makes no other distributions during the year. What is Brett’s capital gain on the distribution? a. $0 b. $100,000 c. $150,000 d. $187,500 73. Rust Corporation distributes property to its sole shareholder, Andre. The property has a fair market value of $350,000, an adjusted basis of $205,000, and is subject to a liability of $220,000. Current E & P is $500,000. With respect to the distribution, which of the following statements is correct? a. Rust has a gain of $15,000 and Andre has dividend income of $350,000. b. Rust has a gain of $145,000 and Andre’s basis in the distributed property is $130,000. c. Rust has a gain of $130,000 and Andre’s basis in the distributed property is $350,000. d. Rust has a gain of $145,000 and Andre has dividend income of $130,000. 74. Purple Corporation has accumulated E & P of $100,000 on January 1, 2024. In 2024, Purple has current E & P of $130,000 (before any distribution). On December 31, 2024, the corporation distributes $250,000 to its sole shareholder, Cara (an individual). Purple Corporation’s E & P as of January 1, 2025 is: a. $0. b. ($20,000). c. $100,000. d. $130,000. 75. Starling Corporation has accumulated E & P of $60,000 on January 1, 2024. In 2024, Starling Corporation had an operating loss of $80,000. It distributed cash of $40,000 to Zoe, its sole shareholder, on December 31, 2024. Starling Corporation’s balance in its E & P account as of January 1, 2025, is: a. $60,000 deficit. b. $20,000 deficit. c. $0. d. $60,000. 76. Robin Corporation distributes furniture (basis of $40,000; fair market value of $50,000) as a property dividend to its shareholders. The furniture is subject to a liability of $55,000. Robin Corporation recognizes gain of: a. $55,000. b. $15,000. c. $10,000. Powered by Cognero

Page 11


Name:

Class:

Date:

Chapter 05: Corporations Earnings and Profits and Dividend Distributions d. $0. 77. Ten years ago, Carrie purchased 2,000 shares of common stock in Osprey Corporation for $20,000. In the current year, Carrie receives a nontaxable stock dividend of 20 shares of Osprey preferred. Values at the time of the dividend are $8,000 for the preferred stock and $72,000 for the common. Based on this information, Carrie’s basis in the stock is: a. $20,000 in the common and $8,000 in the preferred. b. $2,000 in the common and $18,000 in the preferred. c. $18,000 in the common and $2,000 in the preferred. d. $19,802 in the common and $198 in the preferred. 78. Which of the following statements regarding constructive dividends is not correct? a. Constructive dividends do not need to be formally declared or designated as a dividend. b. Constructive dividends need not be paid pro rata to the shareholders. c. Corporations that receive constructive dividends may not use the dividends received deduction. d. Constructive dividends are taxable as dividends only to the extent of earnings and profits. 79. Pink Corporation declares a nontaxable dividend payable in rights to subscribe to common stock. Each right entitles the holder to purchase one share of stock for $25. One right is issued for every two shares of stock owned. Jocelyn owns 100 shares of stock in Pink, which she purchased three years ago for $3,000. At the time of the distribution, the value of the stock is $45 per share and the value of the rights is $2 per share. Jocelyn receives 50 rights. She exercises 25 rights and sells the remaining 25 rights three months later for $2.50 per right. a. Jocelyn must allocate a part of the basis of her original stock in Pink to the rights. b. If Jocelyn does not allocate a part of the basis of her original stock to the rights, her basis in the new stock is zero. c. Sale of the rights produces ordinary income to Jocelyn of $62.50. d. If Jocelyn does not allocate a part of the basis of her original stock to the rights, her basis in the new stock is $625. 80. On January 30, Juan receives a nontaxable distribution of stock rights from Platinum Corporation. Each right entitles the holder to purchase one share of stock for $40. One right is issued for every share of stock owned. Juan owns 100 shares of stock purchased two years ago for $4,000. At the date of distribution, the rights are worth $1,000 (100 rights at $10 per right) and Juan’s stock in Platinum is worth $5,000 (or $50 per share). On December 1, Juan sells all 100 stock rights for $12 per right. How much gain does Juan recognize on the sale? a. $1,200 b. $533 c. $400 d. $0 Matching Using the legend provided, classify each statement accordingly. In all cases, assume that taxable income is being adjusted to arrive at current E & P for 2024. a. Increase b. Decrease c. No effect 81. Gain on installment sale in 2024 deferred until 2025. Powered by Cognero

Page 12


Name:

Class:

Date:

Chapter 05: Corporations Earnings and Profits and Dividend Distributions 82. Interest received from municipal bonds in 2024. 83. Federal income tax refunds from tax paid in prior years. 84. Loss on sale between related parties in 2024. 85. Meal expense not deducted in 2024 because of the 50% limitation. 86. Cash dividends distributed to shareholders in 2024. 87. Gain realized but not recognized in a like-kind exchange transaction in 2024. 88. Additional first-year (bonus) depreciation deduction claimed in 2024. 89. Premiums paid on key employee life insurance policy (assume no increase in cash surrender value of policy) in 2024. 90. Section 179 expense in second year following election. Using the legend provided, classify each statement accordingly. In All cases, assume that taxable income is being adjusted to arrive at current E & P for 2024. a. Increase b. Decrease c. No effect 91. Penalties paid to state government for failure to comply with state law. 92. Dividends received deduction. 93. Charitable contribution carryforward deducted in the current year. 94. Intangible drilling costs deducted currently. 95. Gain realized (but not recognized) on a like-kind exchange. 96. A decrease in the LIFO recapture amount during the year. 97. Excess capital loss in year incurred. 98. State income tax paid in the current year. 99. Proceeds of life insurance received upon the death of a key employee (policy had no cash surrender value). Subjective Short Answer 100. On January 1, Gold Corporation (a calendar year taxpayer) has E & P of $30,000 and generates no additional E & P during the year. On March 31, the corporation distributes $40,000 to its sole shareholder, Ava (basis in stock of $8,000). Determine the effect of the distribution on Ava’s taxable income and stock basis. Powered by Cognero

Page 13


Name:

Class:

Date:

Chapter 05: Corporations Earnings and Profits and Dividend Distributions 101. On January 1, Tulip Corporation (a calendar year taxpayer) has accumulated E & P of $300,000. Its current E & P for the year is $90,000 (before considering dividend distributions). During the year, Tulip distributes $600,000 ($300,000 each) to its equal shareholders, Anne and Tom. Anne has a basis in her stock of $65,000, and Tom’s basis is $120,000. What is the effect of the distribution by Tulip Corporation on Anne and Tom? 102. Daisy Corporation is the sole shareholder of Ostrich Corporation, which it hopes to sell within the next three years. The Ostrich stock (basis of $25,000,000) is currently worth $30,000,000, but Daisy believes that it would be easier to find a buyer if it was worth less. To lower the value of its stock, Ostrich distributes $4,000,000 cash to Daisy (sufficient E & P exists to cover the distribution). At a later date, Daisy sells Ostrich for $26,000,000. a.

What are the tax consequences to Daisy on the sale?

b.

What would be the tax consequences if Ostrich had not first distributed the $4,000,000 in cash and Daisy sold the Ostrich stock for $30,000,000?

103. Ashley, the sole shareholder of Hawk Corporation, has a stock basis of $200,000 at the beginning of the year. On July 1, she sells all of her stock to Francisco for $1,000,000. On January 1, Hawk has accumulated E & P of $90,000 and during the year, current E & P of $160,000. Hawk makes the following cash distributions: $270,000 to Ashley on March 31 and $90,000 to Francisco on December 1. How are the distributions to Ashley and Francisco taxed? What is Ashley’s recognized gain on the sale to Francisco? 104. Brown Corporation, an accrual basis corporation, has taxable income of $150,000 in the current year. Included in its determination of taxable income are the following transactions. ∙ ∙ ∙ ∙ ∙

Brown incurred a $65,000 capital loss from the sale of stock. Because Brown had no capital gains this year, none of the loss is deductible. The corporation’s Federal income tax liability is $31,500. Brown incurred $18,000 in nondeductible meal expenses. Brown uses the LIFO method when accounting for inventory. This year, the company’s LIFO recapture amount increased by $3,000. Brown claimed a dividends received deduction of $1,500.

What is Brown’s current E & P for the year?

105. Finch Corporation (E & P of $400,000) distributed machinery ($10,000 adjusted basis, $150,000 fair market value) to its sole shareholder, Kathleen. The property is subject to a $50,000 mortgage, which Kathleen assumed. How much dividend income does Kathleen recognize as a result of the distribution and what is her basis in the machinery? 106. Sylvia owns 25% of Cormorant Corporation, which sells diamonds to retail jewelry businesses. While Cormorant has a deficit in accumulated E & P of $56,000 at the beginning of the year, its current E & P is $500,000. Since the company had a successful year, Cormorant pays a $36,000 distribution to each of the company’s four shareholders on December 15. Three shareholders receive cash, but Cormorant distributes a diamond (adjusted basis of $40,000 and a fair market value of $36,000) to Sylvia in lieu of cash. Determine the effect of distributing the diamond on Cormorant’s and on Sylvia’s taxable income. What is Sylvia’s basis in the diamond? Was the distribution good tax planning on the part of Cormorant? Why or why not? 107. Thrush, Inc., is a calendar year, accrual basis corporation with Haruki as its sole shareholder (basis in his stock is $90,000). On January 1 of the current year, Thrush has accumulated E & P of $200,000. Before considering the effect of the distribution described below, the corporation’s current E & P is $50,000. On November 1, Thrush distributes an office building to Haruki. The office building has an adjusted basis of $80,000 (fair market value of $100,000) and is subject to a mortgage of $110,000. Assume that the building has been depreciated using the ADS method for both income tax and Powered by Cognero

Page 14


Name:

Class:

Date:

Chapter 05: Corporations Earnings and Profits and Dividend Distributions E & P purposes. What are the tax consequences of the distribution to Thrush and to Haruki? (In your answer, be sure to describe the effects on taxable income for both Thrush and Haruki, the impact of the distribution on Thrush’s E & P, and Haruki’s basis in the building.) 108. Scarlet Corporation is an accrual basis, calendar year corporation. Scarlet distributes inventory (basis of $20,000; fair market value of $40,000) to Frank, its shareholder. Assuming that Scarlet has $500,000 of current E & P, what is the impact of the distribution on Scarlet Corporation and on Frank? 109. Pebble Corporation, an accrual basis taxpayer, has struggled to survive since its formation six years ago. As a result, it has a deficit in accumulated E & P of $340,000 at the beginning of the year. This year, however, Pebble earned a significant profit; taxable income was $240,000. Consequently, Pebble made two cash distributions to Martha, its sole shareholder: $150,000 on July 1 and $200,000 December 31. The following information might be relevant to determining the tax treatment of the distributions. ∙

This year’s taxable income included a net operating loss carryover of $50,000.

The corporation’s Federal income tax liability is $50,400 for the year.

Pebble paid nondeductible fines and kickbacks of $10,000. The company also paid nondeductible life insurance premiums of $22,000.

The cash surrender value of the corporate-owned life insurance policies increased by $11,000 during the year. The company sold a piece of equipment during the year and reported a § 1231 gain of $105,000 and recapture income under § 1245 of $35,000. There were no other § 1231 transactions during the year, but the corporation did have a capital loss carryforward of $30,000.

MACRS depreciation exceeds E & P depreciation by $14,000. In addition, an election under § 179 was made this year for $18,000 of assets.

a.

Compute Pebble’s E & P for the year.

b.

What are the tax consequences of the two distributions made during the year to Martha (her stock basis is $64,000)?

110. Stephanie is the sole shareholder and president of Hawk Corporation. She feels that she can justify at least a $220,000 bonus this year because of her performance. However, rather than a bonus in the form of a salary, she plans to have Hawk pay her a $220,000 dividend. Because Stephanie’s marginal tax rate is 32%, she prefers to receive a dividend taxed at 15%. Her accountant, however, suggests a $275,000 bonus in lieu of the $220,000 dividend since Hawk Corporation is in the 21% tax bracket. Should Stephanie take the $220,000 dividend or the $275,000 bonus? Support your answer by computing the after-tax cost of the two alternatives to Hawk and to Stephanie. 111. Thistle Corporation declares a nontaxable dividend payable in rights to subscribe to common stock. One right and $25 entitle the holder to subscribe to one share of stock. One right is issued for each share of stock held. Annette, a shareholder, owns 200 shares of stock that she purchased five years ago for $3,000. At the date of distribution of the rights, the market values were $50 per share for the stock and $25 for a right. Annette received 200 rights. She exercises Powered by Cognero

Page 15


Name:

Class:

Date:

Chapter 05: Corporations Earnings and Profits and Dividend Distributions 160 rights and purchases 160 additional shares of stock. She sells the remaining 40 rights for $1,080. What are the tax consequences to Annette? 112. Albatross Corporation acquired land for investment purposes in 2009 at a cost of $100,000. Albatross sold the land to Monty on December 30, 2024, and did not elect out of the installment method of accounting. The selling price of the property was $400,000. Monty made a cash down payment of $50,000 on the date of sale and executed a $350,000 note, payable in seven annual installments of $50,000 each plus interest at the rate of 6% per annum. The first installment of $50,000 was due in 2025 which Monty paid, plus interest of $21,000. Discuss the effect of this sale on Albatross’s taxable income and its E & P account in 2024 and 2025. 113. Kite Corporation, a calendar year taxpayer, has taxable income of $360,000 for 2025. Among its transactions for the year are the following: Collection of proceeds from insurance policy on life of corporate officer (in excess of cash surrender value) $ 9,000 Realized gain (not recognized) on an involuntary conversion 10,000 Nondeductible fines and penalties 21,000 Disregarding any provision for Federal income taxes, determine Kite Corporation’s current E & P for 2025. 114. Maria owns 75% and Christopher owns 25% of Cockatoo Corporation, a calendar year taxpayer. Cockatoo makes a $600,000 distribution to Maria on April 1 and a $200,000 distribution to Christopher on May 1. Cockatoo’s current E & P is $120,000 and its accumulated E & P is $500,000. What are the tax implications of the distributions to Maria and Christopher? 115. Jen, the sole shareholder of Mahogany Corporation, sold her stock to Jason on July 1 for $90,000. Jen’s stock basis at the beginning of the year was $60,000. Mahogany made a $30,000 cash distribution to Jen immediately before the sale, and Jason received a $60,000 cash distribution from Mahogany on November 1. As of the beginning of the current year, Mahogany had $16,000 in accumulated E & P, and current E & P (before distributions) is $30,000. What are the tax consequences of these transactions to Jen and Jason? 116. At the beginning of the current year, both Paul and John own 50% of Apple Corporation. In July, Paul sold his stock to Sarah for $110,000. At the beginning of the year, Apple Corporation had accumulated E & P of $200,000 and its current E & P is $250,000 (prior to any distributions). Apple distributed $260,000 on March 1 ($130,000 to Paul and $130,000 to John) and distributed another $260,000 on October 1 ($130,000 to Sarah and $130,000 to John). What are the tax implications of the $130,000 distribution to Sarah? Essay 117. Gold Corporation has accumulated E & P of $2,000,000 as of January 1 of the current year. During the year, it expects to have earnings from operations of $1,680,000 and to distribute $900,000 in cash to shareholders. Gold Corporation also expects to sell an asset for a loss of $2,000,000. Thus, it anticipates incurring a deficit of $320,000 for the year. What can Gold do to minimize the amount of dividend income to its shareholders? 118. Antonio owns 100% of Forsythia Corporation’s stock. Corporate employees and annual salaries include Antonio ($300,000); Richard, Antonio’s son ($80,000); Rita, Antonio’s daughter ($100,000); and Sandy ($120,000). The operation of Forsythia Corporation is shared about equally between Antonio and Sandy (an unrelated party). Richard and Rita are full-time college students at a university about 150 miles away. Forsythia Corporation has substantial E & P but has not distributed a dividend for the past five years. Discuss problems related to the salary arrangement for Forsythia Corporation. Powered by Cognero

Page 16


Name:

Class:

Date:

Chapter 05: Corporations Earnings and Profits and Dividend Distributions 119. Briefly describe the reason a corporation might distribute a property dividend to a shareholder in lieu of a cash distribution. Describe the tax effects of the property distribution on the shareholder and on the corporation. 120. How does the definition of accumulated E & P differ from the definition of current E & P? 121. How does the payment of a property dividend affect E & P? 122. Goldfinch Corporation distributes stock rights to its shareholders. How is the basis of the stock rights received by Goldfinch’s shareholders determined? 123. Briefly describe the rationale for the reduced tax rate on dividends for individual taxpayers. 124. Christian, the president and sole shareholder of Venture Corporation, is paid an annual salary of $150,000. Christian would like to draw additional funds from the corporation but is concerned that an increased salary might cause the IRS to contend that his salary is unreasonable. Further, Christian does not want the corporation to pay any dividends. He would like to contribute $40,000 to his alma mater to establish scholarships for needy students. If Christian makes a pledge to the university to provide $40,000 for scholarships, would there be a problem if Venture Corporation paid the pledge on his behalf? Explain. 125. Briefly define the term “earnings and profits.” 126. Provide a brief outline on computing current E & P. 127. In general, how are current and accumulated earnings and profits allocated to corporate distributions? 128. Briefly discuss the rules related to distributions of noncash property. 129. What is a constructive dividend? Provide several examples of the term.

Powered by Cognero

Page 17


Name:

Class:

Date:

Chapter 05: Corporations Earnings and Profits and Dividend Distributions Answer Key 1. True 2. False 3. False 4. False 5. False 6. False 7. False 8. False 9. False 10. True 11. False 12. False 13. False 14. True 15. True 16. True 17. False 18. True 19. True 20. True 21. True 22. False 23. False 24. True 25. True Powered by Cognero

Page 18


Name:

Class:

Date:

Chapter 05: Corporations Earnings and Profits and Dividend Distributions 26. True 27. True 28. True 29. True 30. False 31. False 32. False 33. True 34. False 35. True 36. False 37. False 38. False 39. True 40. True 41. a 42. a 43. c 44. b 45. b 46. c 47. b 48. d 49. b 50. b Powered by Cognero

Page 19


Name:

Class:

Date:

Chapter 05: Corporations Earnings and Profits and Dividend Distributions 51. d 52. c 53. d 54. b 55. d 56. d 57. a 58. a 59. b 60. a 61. c 62. c 63. b 64. c 65. a 66. b 67. b 68. d 69. b 70. a 71. a 72. c 73. d 74. a 75. b 76. b Powered by Cognero

Page 20


Name:

Class:

Date:

Chapter 05: Corporations Earnings and Profits and Dividend Distributions 77. c 78. c 79. d 80. b 81. a 82. a 83. a 84. b 85. b 86. b 87. c 88. a 89. b 90. b 91. b 92. a 93. a 94. a 95. c 96. b 97. b 98. c 99. a 100. Ava recognizes dividend income of $30,000 (the amount of E & P distributed). In addition, she reduces her stock basis from $8,000 to zero and recognizes a taxable capital gain of $2,000 (the excess of the distribution over the stock basis). Powered by Cognero

Page 21


Name:

Class:

Date:

Chapter 05: Corporations Earnings and Profits and Dividend Distributions 101. Anne and Tom each have dividend income of $195,000 {[$300,000 (Tulip’s accumulated E & P) + $90,000 (Tulip’s current E & P)] ÷ 2}. The remaining $210,000 distributed reduces the basis in Tulip stock with the excess treated as capital gain. Thus, Anne reduces her stock basis to zero and has a capital gain of $40,000 [($210,000 distribution in excess of E & P ÷ 2) – $65,000 basis]. Tom reduces his stock basis to $15,000 [$120,000 basis – ($210,000 distribution in excess of E & P ÷ 2)]. 102. a.

Because Daisy is the sole shareholder of Ostrich, it has a 100% dividends received deduction on the $4,000,000 cash distribution. Thus, Daisy Corporation is not taxed on the $4,000,000 distribution, and it has a gain on the sale of its stock in Ostrich of $1,000,000 [$26,000,000 (sales price) – $25,000,000 (stock basis)].

b.

If Daisy had sold the stock for $30,000,000, Daisy would have a taxable gain on the sale of $5,000,000 [$30,000,000 (sales price) – $25,000,000 (stock basis)].

103. The $160,000 in current E & P is allocated pro rata to the two distributions made during the year; thus, $120,000 is allocated to Ashley and $40,000 is allocated to Francisco. Because accumulated E & P is applied in chronological order, it is allocated entirely to Ashley. Consequently, of the $270,000 distribution to Ashley on March 31, $210,000 is taxed as dividend income [$90,000 (accumulated E & P) + $120,000 (current E & P)] and the remaining $60,000 reduces her stock basis to $140,000. She then recognizes a capital gain of $860,000 on the sale of her stock [$1,000,000 (sales price) – $140,000 (remaining stock basis)]. As to the $90,000 distribution to Francisco, $40,000 is taxed as a dividend (from current E & P) and the remaining $50,000 reduces his basis to $950,000 [$1,000,000 (original basis) – $50,000 (return of capital)]. 104. Taxable income Current-year capital loss Federal income tax Nondeductible meal expenses LIFO recapture adjustment Dividends received deduction Current E & P

$ 150,000 (65,000) (31,500) (18,000) 3,000 1,500 $ 40,000

105. As a result of the distribution, Kathleen has a taxable dividend of $100,000 [$150,000 (fair market value) – $50,000 (liability)].The basis of the property to Kathleen is its fair market value, or $150,000. 106. Losses on distributed property are not recognized at the corporate level, so there is no impact on Cormorant’s taxable income. Because there is sufficient current E & P to cover the distribution, Sylvia has a taxable dividend of $36,000 and her basis in the diamond is also $36,000. The distribution reflects poor tax planning by Cormorant because the built-in $4,000 loss on the diamond ($36,000 fair market value – $40,000 adjusted basis) has been wasted. If Cormorant had sold the diamond for its $36,000 fair market value, it could have recognized the loss. The $36,000 cash received from the sale would be distributed to Sylvia instead. 107. Thrush recognizes gain of $30,000 [$110,000 (liability) – $80,000 (adjusted basis)]. The $30,000 gain increases the corporation’s current E & P from $50,000 to $80,000. Because the liability exceeds the fair market value of the property, the distribution itself will not impact E & P. Haruki has no taxable income because the liability exceeds the fair market value of the property received. Further, Haruki’s basis in the office building is its deemed fair market value, or $110,000 (the amount of the liability assumed). Powered by Cognero

Page 22


Name:

Class:

Date:

Chapter 05: Corporations Earnings and Profits and Dividend Distributions 108. Scarlet’s E & P is increased by the $20,000 gain [$40,000 (fair market value) – $20,000 (adjusted basis)] and decreased by the $40,000 fair market value of the distribution. Frank has dividend income of $40,000. 109. a. Taxable income Net operating loss carryover Federal income tax Fines and kickbacks Life insurance premiums Cash surrender value of life insurance Capital loss carryforward Excess of MACRS depreciation over E & P depreciation Section 179 expense (80% × $18,000) Current E & P b.

$ 240,000 50,000 (50,400) (10,000) (22,000) 11,000 30,000 14,000 14,400 $ 277,000

Martha has a dividend of $277,000 (the amount of the current E & P). The distributions during the year exceed current E & P by $73,000 ($350,000 – $277,000). Consequently, Martha’s stock basis is reduced to $0 and she has a capital gain equal to the extent to which the $73,000 exceeds her stock basis ($64,000), or $9,000.

110. Stephanie should choose the $275,000 bonus instead of the $220,000 dividend because the after-tax benefit to her is the same and the after-tax cost for Hawk is less. Stephanie’s after-tax benefit for the bonus is $187,000 [$275,000 × (1 – 0.32)], while her after-tax benefit for the dividend is $187,000 [$220,000 × (1 – 0.15)]. Hawk Corporation’s after-tax cost for the bonus is $217,250 [$275,000 bonus – ($275,000 × 0.21) taxes saved], and its after-tax cost for the dividend is $220,000 (the dividend is not deductible). 111. Because the fair market value of the rights is 15% or more of the value of the old stock, Annette allocates her basis in the stock between the stock and the stock rights. Annette allocates basis as follows. Fair market value of stock: 200 shares × $50 Fair market value of rights: 200 rights × $25

$10,000 5,000 $15,000

Basis of stock: 10/15 × $3,000 = $2,000 Basis of rights: 5/15 × $3,000 = $1,000 = $5 per right There is a capital gain on the sale of the rights of $880, computed as follows. Sales price of 40 rights Less: basis of 40 rights Long-term capital gain

$ 1,080 (200) $ 880

Basis of the new stock is $4,800, computed as follows. 160 rights × $5 Additional consideration (160 × $25)

$ 800 4,000 $4,800

The holding period of the 160 new shares begins on the date of purchase.

Powered by Cognero

Page 23


Name:

Class:

Date:

Chapter 05: Corporations Earnings and Profits and Dividend Distributions 112. The gross profit percentage on the sale is 75%, computed as follows: [$300,000 (gross profit) ÷ $400,000 (selling price)]. In 2024, Albatross includes a long-term capital gain of $37,500 in its taxable income (75% of the $50,000 cash down payment). However, the entire gain of $300,000 increases E & P in 2024. Thus, to compute E & P, taxable income will be increased by the $262,500 gain not already recognized ($300,000 total gain less $37,500 gain recognized in 2024). In 2025, Albatross Corporation again includes a long-term capital gain of $37,500 in taxable income (75% of the $50,000 installment), plus ordinary interest income of $21,000. In determining its 2025 E & P, it reduces taxable income by $37,500. 113. Taxable Income Plus: Life insurance proceeds in excess of CSV Less: Fines and penalties Current Earning & Profits

$ 360,000 9,000 (21,000) $ 348,000

The realized gain (not recognized) on the involuntary conversion has no effect on E & P. 114. Current E & P is allocated on a pro rata basis to each distribution made during the year. Cockatoo Corporation made $800,000 of distributions during the year. Christopher’s distribution represents 25% ($200,000/$800,000) of that amount. Consequently, 25% of Cockatoo’s current E & P, or $30,000 ($120,000 × 25%), is allocated to Christopher’s distribution. Maria’s distribution represents 75% ($600,000/$800,000) of total distributions. Consequently, 75% of Cockatoo’s current E & P, or $90,000 ($120,000 × 75%), is allocated to Maria’s distribution. Accumulated E & P is applied in chronological order beginning with the earliest distribution. When Maria’s distribution is made, Cockatoo has $590,000 of dividend-paying capacity ($500,000 of accumulated E & P plus $90,000 of current E & P). Therefore, $590,000 of Maria’s distribution is treated as a dividend with the balance ($10,000) being a return of capital (to the extent of her stock basis) and then a capital gain. After this distribution, Cockatoo has no accumulated E & P remaining. When Christopher’s distribution is made, Cockatoo has $30,000 remaining in current E & P. Therefore, $30,000 of Christopher’s distribution is treated as a dividend with the balance ($170,000) being a return of capital (to the extent of his stock basis) and then a capital gain. After the distribution to Christopher, Cockatoo has no remaining current or accumulated E & P.

115. The $30,000 in current E & P is allocated on a pro rata basis to the two distributions made during the year; thus, $10,000 of current E & P is allocated to Jen ($30,000 × $30,000/$90,000) and $20,000 is allocated to Jason ($30,000 × $60,000/$90,000). Because accumulated E & P is allocated in chronological order, all of Mahogany’s $16,000 of accumulated E & P is allocated to Jen’s distribution. Therefore, the distribution to Jen is treated as a $26,000 dividend and a $4,000 reduction in stock basis. Jason’s distribution consists of a $20,000 dividend and a $40,000 reduction in stock basis. Because Jen sells her stock for $90,000 and her basis immediately after the distribution is $56,000 ($60,000 original basis – $4,000 recovery of capital), she has a $34,000 gain on the sale. 116. Because current E & P is allocated on a pro rata basis to distributions made during the year, one-half, or $125,000 ($250,000 × $260,000/$520,000), is allocated to the March 1 distribution and one-half ($125,000) is allocated to the October 1 distribution. The $200,000 of accumulated E & P is allocated chronologically. As a result, on March 1, Apple has $325,000 of Powered by Cognero

Page 24


Name:

Class:

Date:

Chapter 05: Corporations Earnings and Profits and Dividend Distributions dividend-paying capacity ($125,000 of current E & P and $200,000 of accumulated E & P). Therefore, the March 1 distribution is entirely treated as a dividend and Apple has $65,000 of accumulated E & P remaining after the distribution. On October 1, Apple has $190,000 of dividend-paying capacity ($125,000 of current E & P and $65,000 of accumulated E & P). So, of the $260,000 distribution, $190,000 is treated as a dividend and, as a 50% shareholder, Sarah’s share of this is $95,000. Thus, of the $130,000 received by Sarah, $95,000 is a dividend distributed from E & P ($62,500 current E & P + $32,500 accumulated E & P), and the remaining $35,000 is a nontaxable recovery of capital. Consequently, her stock basis is reduced to $75,000 ($110,000 – $35,000). 117. Gold should recognize the loss as soon as possible and immediately thereafter make the cash distribution. For example, assume these two steps took place on January 2. Because current E & P is a deficit, accumulated E & P is brought up to date. At the time of the distribution, the combined E & P balance is zero [$2,000,000 (beginning balance in E & P) – $2,000,000 (existing deficit in current E & P)], and the entire $900,000 is a return of capital. Current deficits are allocated pro rata throughout the year unless the parties can provide otherwise. Here they can. 118. The salaries paid to Richard and Rita are vulnerable to constructive dividend treatment. Neither appears to earn their salary. Although they are not shareholders, their relationship to Antonio is enough of a tie-in to raise the unreasonable compensation issue. There is also a problem regarding the $300,000 salary payment to Antonio. Why is he receiving $180,000 more than Sandy when it appears they share equally in the operation of the corporation? Forsythia Corporation has not distributed a dividend for the past five years although it has substantial E & P. The IRS might be successful in contending that the entire salaries paid to Richard and Rita are unreasonable compensation and that $180,000 of the salary paid to Antonio is unreasonable. 119. A corporation could distribute property to a shareholder because a shareholder may want a particular piece of property held by the corporation. Another reason might be that the corporation has low cash reserves but still wants to make a distribution to its shareholders. The amount distributed to the shareholder is measured by the fair market value of the property on the date of distribution. Like cash, the portion of a property distribution covered by existing E & P is a dividend, and any excess is treated as a return of capital. If the market value of the property distributed exceeds the corporation’s E & P and the shareholder’s basis in the stock investment, a capital gain usually results. The amount distributed is reduced by any liabilities to which the distributed property is subject immediately before and immediately after the distribution and by any liabilities of the corporation assumed by the shareholder. The basis of the distributed property for the shareholder is the fair market value of the property on the date of the distribution. All distributions of appreciated property generate gain to the distributing corporation. In effect, a corporation that distributes gain property is treated as if it had sold the property to the shareholder for its fair market value. However, the distributing corporation does not recognize loss on the distributions of property. If the distributed property is subject to a liability in excess of basis or the shareholder assumes such a liability, a special rule applies. For purposes of determining gain on the distribution, the fair market value of the property is treated as not being less than the amount of the liability (and this deemed fair market value will also be the basis of the property in the shareholder's hands). Corporate distributions reduce E & P by the greater of the fair market value or the adjusted basis of property distributed, less the amount of any liability on the property. E & P is increased by gain recognized on appreciated property distributed as a property dividend. A property distribution cannot generate a deficit in E & P or add to a deficit in E & P. 120. Accumulated E & P is the total of all previous years’ current E & P (since February 28, 1913) reduced by distributions made from E & P in previous years. Current E & P is determined by making a series of adjustments to the corporation’s taxable income. Current E & P is determined at year-end and is not reduced by current year distributions. Powered by Cognero

Page 25


Name:

Class:

Date:

Chapter 05: Corporations Earnings and Profits and Dividend Distributions 121. Corporate distributions reduce E & P by the greater of the fair market value or the adjusted basis of property distributed less the amount of any liability on the property. E & P is also increased by the gain recognized on appreciated property distributed as a property dividend. However, reductions to E & P due to distributions cannot generate or add to an E & P deficit. 122. The determination of the basis differs, depending on whether the distribution of the stock rights is a taxable event. If the distribution of stock rights is taxable, then shareholders’ basis in the rights is equal to their fair market value. If the distribution of stock rights is not taxable and if their value is less than 15% of the value of the stock on which they are distributed, then the basis of the rights is zero. However, shareholders in this case can elect to have some of the basis in their stock allocated to the stock rights. If the fair market value of the stock rights is 15% or more of the value of the stock on which they are received, and the rights are exercised or sold, then shareholders are required to allocate some of the basis in their stock to the rights. When basis is allocated to stock rights, it is allocated based on the relative proportion of the value of the rights to the overall value of the stock and rights. 123. The double tax on dividends creates a number of distortions in the economy, including (1) an incentive to invest in noncorporate taxpayers rather than corporate taxpayers, (2) a preference by corporations to finance operations with debt rather than equity, and (3) a motivation for corporations to retain earnings and to structure distributions to avoid the double tax. Taken together, these distortions raise the cost of capital for corporate investments by causing reliance on debt financing. This increases the vulnerability of corporations during economic downturns. By taxing dividends at a lower rate, policy makers argue that the negative impacts of the double tax are mitigated. As a result, capital formation in the corporate sector should increase, stimulating the economy. Reducing the double tax through a lower tax rate on dividends should also make the United States more competitive internationally. 124. There would be a problem. Venture Corporation will have satisfied Christian’s obligation. Thus, the payment to the university may be treated as indirect compensation. In determining whether the corporation has paid Christian “unreasonable” compensation, both the direct payment (his salary) of $150,000 and the indirect payment to the university of $40,000 would be considered. If Christian had not made a pledge, the corporation could have established the scholarships on his behalf. 125. In general, earnings and profits (E & P) represents the dividend-paying capacity of a corporation (i.e., the upper limit on the amount of dividend income a shareholder must recognize on corporate distributions). It represents a corporation’s economic income (i.e., its “economic ability” to pay dividends) without impairing its capital. E & P is computed on an annual basis at the end of the tax year (without reduction for any distributions made during the year). The term “earning and profits” is not defined in the Internal Revenue Code. It is roughly analogous to—but different than—the financial accounting concept of retained earnings. 126. In general, the following formula can be used to compute current E & P:

+ – +/– +/– =

Taxable income (computed at end of tax year) Additions to taxable income Subtractions from taxable income Timing adjustments Accounting method adjustments Current E & P

127. (1) Current E & P is applied first to distributions on a pro rata basis; then accumulated E & P is applied (as necessary) in chronological order beginning with the earliest distribution. (2) When a deficit exists in accumulated E & P and a positive balance exists in current E & P, distributions are regarded as dividends to the extent of the current E & P balance. Current and accumulated E & P are not netted. Powered by Cognero

Page 26


Name:

Class:

Date:

Chapter 05: Corporations Earnings and Profits and Dividend Distributions (3) When a deficit exists in current E & P and a positive balance exists in accumulated E & P, the two accounts are netted at the date of distribution. If the resulting balance is zero or a deficit, the distribution is treated as a return of capital, first reducing the basis of the stock to zero and generating taxable gain. If a positive balance results, the distribution is a dividend to the extent of the balance. Any loss in current E & P is deemed to accrue ratably throughout the year unless the corporation can show otherwise. (4) When a deficit exists in both current E & P and accumulated E & P, the distribution is treated as a return of capital, first reducing the basis of the stock to zero and then generating taxable gain. 128. Amounts distributed as dividends in the form of property rather than cash are measured by the fair market value of the property on the date of distribution. This amount is reduced by any liabilities associated with the property that are assumed by the shareholder. A shareholder’s basis in the distributed property is its fair market value on the distribution date. Under § 311(b), gain (but not loss) is recognized to a corporation that distributes property as a dividend. The distribution of appreciated property is treated as if the property were sold to the shareholder at its fair market value. If the property distributed is subject to a liability, or if the shareholder assumes a liability that exceeds the basis of the distributed property, the fair market value of the property will not be less than the amount of the liability (and this deemed fair market value will also be the basis of the property in the shareholder's hands). The distributing corporation’s E & P is increased by any gain recognized on the appreciated property distributed. The distributing corporation’s E & P is reduced by the greater of the fair market value or the adjusted basis of the property distributed less the amount of any liability on the property. 129. Constructive dividends generally occur in closely held corporations where dealings with shareholders are often informal. They result in an economic benefit to the shareholder from the corporation that are not labeled as a dividend. Constructive dividends have the same general Federal tax consequences as regular dividends. That is, the dividend is income to the shareholder but not deductible by the corporation. A number of different scenarios may lead to the determination that a constructive dividend has occurred. Amounts paid to a shareholder in excess of what the IRS considers reasonable may give rise to a constructive dividend. Personal shareholder expenses paid by the corporation without expectation of repayment can also be classified as constructive dividends to the shareholder in an amount equal to the fair market value of the benefit received. Depending upon the facts and circumstances of the transaction, the IRS may attempt to treat a shareholder advance that is not a bona fide loan (e.g., poor or nonexistent documentation) as a constructive dividend. Interest on shareholder loans with below-market interest rates can also constitute a constructive dividend. Likewise, if a corporation, without adequate consideration, assumes a debt or other legal obligation of a shareholder or makes payments on the debt, a constructive dividend may result. Use of corporate property by shareholders can also result in a constructive dividend. Typical situations include the use of corporate-owned autos, boats, airplanes, vacation homes, and other property if the shareholder does not repay the corporation for the use of this property at a fair rental value. In addition, the value of improvements made by the corporation to property leased from a shareholder that were in excess of normal lessee improvements (based on the type and value of the property and the term of the lease) can be a constructive dividend. Finally, bargain purchases of corporate property by a shareholder can also result in a constructive dividend to the extent the FMV of the property exceeds the purchase price.

Powered by Cognero

Page 27


Name:

Class:

Date:

Chapter 06: Corporations Redemptions and Liquidations True / False 1. The Code treats corporate distributions that are a return of a shareholder’s investment as sales or exchanges and corporate distributions that are a return from a shareholder’s investment as dividends. a. True b. False 2. In general, if a shareholder’s ownership interest is not diminished as a result of a stock redemption, the Code will treat the transaction as a sale or exchange. a. True b. False 3. Corporate shareholders generally receive less favorable tax treatment from a qualifying stock redemption than from a dividend distribution. a. True b. False 4. Yolanda owns 60% of the outstanding stock of Amber Corporation. In a qualifying stock redemption, Amber distributes $20,000 to Yolanda in exchange for one-half of her shares (basis of $35,000). As a result of the redemption, Yolanda has a recognized capital loss of $15,000. a. True b. False 5. A shareholder’s basis in property acquired in a stock redemption is the property’s fair market value as of the date of redemption. a. True b. False 6. Vireo Corporation redeemed shares from its sole shareholder pursuant to a written agreement between the parties that clearly identified the transaction as a stock redemption (not a dividend distribution). As the agreement is binding under state law, the shareholder will receive sale or exchange treatment with respect to the redemption. a. True b. False 7. In applying the § 318 stock attribution rules to a stock redemption, a shareholder is treated as owning the stock of her spouse, children, grandchildren, parents, and siblings. a. True b. False 8. A redemption will qualify as a not essentially equivalent redemption only if the shareholder’s interest in the redeeming corporation has been meaningfully reduced. a. True b. False 9. As a result of a redemption, a shareholder’s interest (direct and indirect) in the corporation decreased from 80% to 55%. The redemption qualifies for sale or exchange treatment as a disproportionate redemption. a. True Powered by Cognero

Page 1


Name:

Class:

Date:

Chapter 06: Corporations Redemptions and Liquidations b. False 10. Puffin Corporation’s 2,000 shares outstanding are owned as follows: Paul, 800 shares; Sandra (Paul’s sister), 800 shares; and Greta (Paul’s granddaughter), 400 shares. During the current year, Puffin (E & P of $1,000,000) redeemed 600 shares of Paul’s stock for $100,000. If Paul acquired the 600 shares five years ago for $30,000, he will have a long-term capital gain of $70,000 from the redemption. a. True b. False 11. For purposes of the waiver of the family attribution rules in a complete termination redemption, the former shareholder must notify the IRS within 30 days of acquiring a prohibited interest in the corporation during the 10-year period following the redemption. a. True b. False 12. Reginald and Roland (Reginald’s son) each own 50% of the stock of Robin Corporation. Reginald’s stock interest is entirely redeemed by Robin. Two years later, Reginald loans Robin $250,000. The loan to Robin Corporation is not a prohibited interest for purposes of the family attribution waiver. a. True b. False 13. Six years ago, both Ronald and his mom owned 50% of the stock of Bronze Corporation. At such time, Bronze redeemed all of Ronald’s stock. For the redemption year, Ronald filed the agreement required of the family attribution waiver and reported the transaction as a complete termination redemption (i.e., sale or exchange). In the current year, the mom passed away and willed her entire stock interest in Bronze to Ronald. The inheritance of Bronze stock by Ronald is a prohibited interest for purposes of the family attribution waiver. a. True b. False 14. For purposes of a partial liquidation, a distribution is not essentially equivalent to a dividend if it results in a genuine contraction of the business of the corporation. a. True b. False 15. For purposes of a partial liquidation, the termination of a business test is a subjective test that should be relied upon only after obtaining a favorable ruling from the IRS. a. True b. False 16. To qualify for a partial liquidation under the termination of a business test, the distribution must consist of the assets of a qualified trade or business. a. True b. False 17. In determining whether a distribution qualifies as a § 303 redemption to pay death taxes, the stock attribution rules must be applied. a. True Powered by Cognero

Page 2


Name:

Class:

Date:

Chapter 06: Corporations Redemptions and Liquidations b. False 18. Betty’s adjusted gross estate is $18,000,000. The death taxes and funeral and administration expenses of her estate total $2,400,000. Included in Betty’s gross estate is stock in Heron Corporation valued at $6,600,0000 as of the date of her death. Betty had acquired the stock six years ago at a cost of $1,620,000. If Heron Corporation redeems $2,400,000 of Heron stock from the estate, the transaction will qualify under § 303 as a redemption to pay death taxes and receive sale or exchange treatment. a. True b. False 19. Grackle Corporation (E & P of $600,000) distributes cash of $200,000 and land (fair market value of $400,000; basis of $250,000) to a shareholder in a qualifying stock redemption. The land distributed is subject to a mortgage of $460,000. Grackle will recognize a gain of $210,000 as a result of the distribution. a. True b. False 20. At a time when Blackbird Corporation had E & P of $700,000 and 1,000 shares of stock outstanding, the corporation distributed $300,000 to redeem 400 shares of its stock. The transaction qualified as a disproportionate redemption for the shareholder. Blackbird’s E & P is reduced by $300,000 as a result of the distribution. a. True b. False 21. Tan Corporation paid interest expense on a debt incurred in financing a redemption of its stock. The interest expense is not deductible as it was incurred in connection with a stock redemption. a. True b. False 22. In a redemption of § 306 stock, the redemption proceeds constitute dividend income to the extent of the corporation’s E & P on the date of the redemption. a. True b. False 23. Tammy forms White Corporation in a transaction qualifying under § 351. In that transaction, Tammy transferred cash and equipment in exchange for White Corporation common (1,000 shares) and preferred (200 shares) stock. The preferred stock is not § 306 stock for Tammy. a. True b. False 24. Three years ago, Darlene received preferred (§ 306) stock pursuant to a nontaxable stock dividend from Grackle Corporation. In the current year, Darlene gives the Grackle preferred stock to her sister, Nancy. The Grackle preferred stock is § 306 stock with regard to Nancy. a. True b. False 25. Legal dissolution under state law is required for a liquidation to be complete for tax purposes. a. True b. False Powered by Cognero

Page 3


Name:

Class:

Date:

Chapter 06: Corporations Redemptions and Liquidations 26. One similarity between the tax treatment accorded liquidating and nonliquidating distributions is with respect to a shareholder’s basis in property received in such distributions. For each type of distribution, the shareholder’s basis is the property’s fair market value on the date of distribution. a. True b. False 27. As a general rule, a liquidating corporation recognizes gains but not losses on the distribution of property in complete liquidation. a. True b. False 28. Liquidation expenses incurred by a corporation are generally deductible as § 162 trade or business expenses. a. True b. False 29. The related-party loss limitation applies to distributions to related parties and either the distribution is pro rata or the property distributed is disqualified property. a. True b. False 30. The built-in loss limitation in a complete liquidation does not apply to losses attributable to a decline in a property’s fair market value after its transfer to the corporation. a. True b. False 31. The related-party loss limitation in a complete liquidation applies only to distributions of property, while the built-in loss limitation can apply to a distribution or sale of property. a. True b. False 32. Pursuant to a liquidation, Coral Corporation distributes to Lucinda, a shareholder, land (basis of $90,000, fair market value of $200,000). The land is subject to a $75,000 liability. Lucinda will have a basis of $125,000 in the land. a. True b. False 33. Section 332 does not apply to a parent-subsidiary liquidation if the subsidiary corporation is insolvent on the date of the liquidation. a. True b. False 34. If a liquidation qualifies under § 332, any minority shareholder will recognize gain or loss equal to the difference between the fair market value of assets received and the basis of the shareholder’s stock. a. True b. False 35. A subsidiary corporation is liquidated at a time when it is indebted to its parent corporation. The subsidiary Powered by Cognero

Page 4


Name:

Class:

Date:

Chapter 06: Corporations Redemptions and Liquidations corporation distributes property to the parent corporation in satisfaction of the indebtedness. If the liquidation is governed by § 332, neither the subsidiary nor the parent recognizes gain or loss on the transfer of property in satisfaction of indebtedness. a. True b. False 36. Brown Corporation purchased 85% of the stock of Green Corporation five years ago for $850,000. In the current year, Brown Corporation liquidates Green Corporation and acquires assets with a basis to Green Corporation of $700,000 (fair market value of $1,100,000). Brown Corporation will have a basis in the assets of $850,000, the same as Brown’s basis in its Green stock. a. True b. False 37. Sparrow Corporation purchased 90% of the stock of Warbler Corporation eight years ago for $1,000,000. In the current year, Sparrow liquidates Warbler and acquires assets with a basis to Warbler of $850,000 (fair market value of $1,200,000). Sparrow will have a basis in the assets of $850,000 (Warbler’s basis in the assets), and no recognized gain or loss. a. True b. False 38. A subsidiary is liquidated pursuant to § 332. The parent has held 100% of the stock in the subsidiary for the past 10 years. The subsidiary has a net operating loss carryover of $400,000. The net operating loss does not carry over to the parent. a. True b. False 39. If a parent corporation makes a § 338 election, the subsidiary corporation is treated as a new corporation as of the day following the qualified stock purchase date. a. True b. False 40. If a parent corporation makes a § 338 election, the subsidiary corporation recognizes gain but not loss on the deemed sale of its assets on the qualified stock purchase date. a. True b. False 41. If a parent corporation makes a § 338 election, the subsidiary corporation must be liquidated. a. True b. False 42. One advantage of acquiring a corporation via an asset purchase instead of a stock purchase is that an asset purchase avoids the transfer of the acquired corporation’s liabilities. a. True b. False 43. In applying the § 318 stock attribution rules to a stock redemption, the stock of a partnership is deemed to be owned proportionately by its partners. Powered by Cognero

Page 5


Name:

Class:

Date:

Chapter 06: Corporations Redemptions and Liquidations a. True b. False 44. Penguin Corporation (E & P of $500,000) distributes land (fair market value of $150,000; basis of $160,000) to an estate in a redemption to pay death taxes under § 303. Penguin Corporation recognizes a $10,000 loss as a result of the distribution. a. True b. False 45. The sale of § 306 stock generally results in dividend income to the shareholder and a reduction in the corporation’s E & P. a. True b. False 46. A liquidation can occur for tax purposes even though the corporation has retained some assets to pay remaining debts and preserve legal status. a. True b. False 47. Gains and losses are recognized by the liquidating corporation on distributions to a minority shareholder in a § 332 liquidation. a. True b. False 48. Ruby Corporation has announced plans to liquidate. Bronze Corporation owns 85% of Ruby’s stock. If Bronze wants to avoid the nontaxable treatment associated with a § 332 liquidation (e.g., nonrecognition of loss), it could reduce its stock ownership in Ruby to below 80%. a. True b. False Multiple Choice 49. Seven years ago, Eleanor transferred property she had used in her sole proprietorship to Blue Corporation for 2,000 shares of Blue Corporation in a transaction that qualified under § 351. The assets had a tax basis to her of $400,000 and a fair market value of $700,000 on the date of the transfer. In the current year, Blue Corporation (E & P of $1,000,000) redeems 600 shares from Eleanor for $260,000 in a transaction that does not qualify for sale or exchange treatment. With respect to the redemption, Eleanor will have a: a. $140,000 dividend. b. $260,000 dividend. c. $140,000 capital gain. d. $260,000 capital gain. 50. Seven years ago, Eleanor transferred property she had used in her sole proprietorship to Blue Corporation for 2,000 shares of Blue Corporation in a transaction that qualified under § 351. The assets had a tax basis to her of $400,000 and a fair market value of $700,000 on the date of the transfer. In the current year, Blue Corporation (E & P of $1,000,000) redeems 600 shares from Eleanor for $260,000 in a transaction that qualifies for sale or exchange treatment. With respect to the redemption, Eleanor will have a: Powered by Cognero

Page 6


Name:

Class:

Date:

Chapter 06: Corporations Redemptions and Liquidations a. $140,000 dividend. b. $260,000 dividend. c. $140,000 capital gain. d. $260,000 capital gain. 51. Finch Corporation distributes property (basis of $225,000, fair market value of $300,000) to a shareholder in a distribution that is a qualifying stock redemption. The property is subject to a liability of $160,000, which the shareholder assumes. The basis of the property to the shareholder is: a. $0. b. $140,000. c. $225,000. d. $300,000. 52. Coffee Corporation has 2,000 shares of common stock outstanding. John owns 700 of the shares, John’s grandfather owns 100 shares, John’s father owns 100 shares, John’s former spouse owns 700 shares, and Redbird Partnership owns 400 shares. John is a 50% partner in Redbird Partnership. How many shares is John deemed to own in Coffee Corporation under the § 318 attribution rules? a. 700 b. 1,000 c. 1,100 d. 1,700 53. Kite Corporation has 1,000 shares of stock outstanding. Kent owns 300 shares, Kent’s father owns 200 shares, Kent’s daughter owns 100 shares, and Kent’s aunt owns 200 shares. Plover Corporation owns the other 200 shares in Kite Corporation. Kent owns 75% of the stock in Plover Corporation. Applying the § 318 stock attribution rules, how many shares does Kent own in Kite Corporation? a. 500 b. 600 c. 750 d. 950 54. Keshia owns 200 shares in Parakeet Corporation. Keshia has a 30% beneficiary interest in her deceased grandmother’s estate. The estate owns 400 shares in Parakeet Corporation. None of the other beneficiaries of the estate owns stock in Parakeet. In applying the § 318 attribution rules,: a. The estate owns 400 shares. b. Keshia owns 320 shares. c. Keshia owns 600 shares. d. The estate owns 460 shares. 55. Which of the following is an incorrect statement regarding the application of the § 318 stock attribution rules? a. An individual is not deemed to own the shares owned by their siblings. b. Stock owned by an estate is deemed to be owned in full by a beneficiary. c. Stock owned by any shareholder owning 50% or more of a corporation’s stock is deemed to be owned in full by the corporation. d. Stock owned by a partnership is deemed to be owned proportionately by a partner. Powered by Cognero

Page 7


Name:

Class:

Date:

Chapter 06: Corporations Redemptions and Liquidations 56. Bristlebird Corporation (E & P of $700,000) has 3,000 shares of common stock outstanding. Juan owns 1,500 shares and his spouse, Roberta, owns 1,500 shares. Both Juan and Roberta have a basis of $90,000 in their Bristlebird stock. In the current year, Bristlebird Corporation redeems 1,000 shares from Juan for $250,000. With respect to the distribution in redemption of the Bristlebird stock: a. Juan has dividend income of $250,000. b. Juan has dividend income of $190,000. c. Juan has a capital gain of $250,000. d. Juan has a capital gain of $190,000. 57. Hazel, Emily, and Frank, unrelated individuals, own all of the stock in Wren Corporation (E & P of $1,200,000) as follows: Hazel, 1,500 shares; Emily, 300 shares; and Frank, 200 shares. Wren redeems 900 of Hazel’s shares (basis of $210,000) for $625,000. With respect to the distribution in redemption of the stock: a. Hazel has a capital gain of $415,000. b. Hazel has a capital gain of $625,000. c. Hazel has dividend income of $415,000. d. Hazel has dividend income of $625,000. 58. Lucinda owns 1,100 shares of Blackbird Corporation stock at a time when Blackbird has 2,000 shares of stock outstanding. The remaining shareholders are unrelated to Lucinda. What is the minimum number of shares Blackbird must redeem from Lucinda so that the transaction will qualify as a disproportionate redemption? a. 220 b. 393 c. 484 d. 880 59. Hannah, Greta, and Winston own the stock in Redpoll Corporation (E & P of $900,000) as follows: Hannah, 600 shares; Greta, 400 shares; and Winston, 1,000 shares. Greta is Hannah’s daughter, and Winston is Hannah’s brother. Redpoll Corporation redeems 400 of Hannah’s shares (basis of $55,000) for $240,000. Hannah purchased the stock three years ago as an investment. With respect to the stock redemption, Hannah has: a. Long-term capital gain of $185,000. b. Long-term capital gain of $240,000. c. Dividend income of $185,000. d. Dividend income of $240,000. 60. Kiyoshi, Hannah, and Samuel, unrelated individuals, own the stock in Broadbill Corporation (E & P of $700,000) as follows: Kiyoshi, 300 shares; Hannah, 300 shares; and Samuel, 400 shares. Broadbill redeems 200 of Samuel’s shares (basis of $175,000) for $250,000. If Samuel’s stock is a capital asset and has been held for over three years, Samuel has: a. A long-term capital gain of $75,000. b. A short-term capital gain of $75,000. c. Ordinary income of $250,000. d. Ordinary income of $75,000. 61. Julian, Berta, and Maria own 400 shares, 400 shares, and 200 shares, respectively, in Caramel Corporation (E & P of $750,000). Berta is Julian’s sister, and Maria is Julian’s aunt. Caramel Corporation redeems all of Julian’s stock for $420,000. Julian paid $200 a share for the stock five years ago. Julian continued to serve on Caramel’s board of directors after the redemption. With respect to the redemption: Powered by Cognero

Page 8


Name:

Class:

Date:

Chapter 06: Corporations Redemptions and Liquidations a. Dividend income is $340,000. b. Dividend income is $420,000. c. Long-term capital gain is $340,000. d. Long-term capital gain is $420,000. 62. Both Lupe and Rodrigo, father and son, own 50% of the stock outstanding of Heron Corporation (E & P of $400,000). During the current year, Heron redeems all of Lupe’s shares for $250,000. The transaction cannot qualify as a complete termination redemption if: a. Lupe received a $250,000 note receivable from Heron in the stock redemption. b. Lupe loaned Heron Corporation $50,000 two years following the redemption. c. Lupe continued to serve on Heron Corporation’s board of directors for two years following the redemption. d. Three years after the redemption, Lupe inherited Rodrigo’s shares in Heron as a result of his son’s death. 63. Leon owns 750 shares of the 2,000 outstanding shares of Crane Corporation (E & P of $900,000). None of the other shareholders of Crane is related to Leon. Leon acquired his Crane shares 10 years ago for $80,000. Crane has operated several trades or businesses for more than 5 years. In the current year, Crane sells the assets of one of those trades or businesses and distributes the proceeds from the asset sale to the shareholders in a pro rata stock redemption. In this transaction, Leon receives $250,000 in redemption of 300 shares of Crane. As a result of this transaction, Leon will recognize: a. $218,000 dividend income. b. $250,000 dividend income. c. $218,000 long-term capital gain. d. $250,000 long-term capital gain. 64. Which of the following statements is correct with respect to a partial liquidation? a. The genuine contraction of a corporate business requirement is an objective test that taxpayers can rely upon with certainty. b. The distribution of proceeds from the sale of excess inventory to shareholders in exchange for part of their stock will not satisfy the not essentially equivalent to a dividend test. c. A stock redemption pursuant to a partial liquidation cannot be pro rata with respect to the shareholders. d. The termination of a business test requires that the distributing corporation actively conducted at least three trades or businesses for at least five years. 65. The adjusted gross estate of Keith, decedent, is $24,000,000. Included in the gross estate is stock in Gold Corporation (E & P of $2,600,000), a closely held corporation, valued at $9,200,000 as of the date of Keith’s death. Keith had acquired the stock 12 years ago at a cost of $1,800,000. Death taxes and funeral and administration expenses for Keith’s estate are $4,600,000. Gold Corporation redeems one-half of the stock from Keith’s estate in a § 303 redemption to pay death taxes using property with a fair market value of $4,600,000 (adjusted basis of $3,800,000). Which of the following is a correct statement regarding the tax consequences of this redemption? a. The estate will have a basis of $4,600,000 in the property received from Gold Corporation in redemption of the estate’s stock. b. Gold Corporation will not reduce its E & P as a result of the distribution of the property to Keith’s estate. c. The estate will recognize a $2,800,000 long-term capital gain on the redemption. d. Gold Corporation recognizes no gain (or loss) on the distribution of the property to Keith’s estate. 66. The adjusted gross estate of Debra, decedent, is $16,000,000. Debra’s estate will incur death taxes and funeral and administration expenses of $2,000,000. Debra’s gross estate includes stock in Silver Corporation that she had purchased Powered by Cognero

Page 9


Name:

Class:

Date:

Chapter 06: Corporations Redemptions and Liquidations 12 years ago for $1,200,000 (date of death fair market value of $6,000,000). At the time of her death, Debra owned 80% of the stock in Silver Corporation. Silver Corporation (E & P of $8,000,000) redeems all of the estate’s stock in the corporation for $6,000,000 million. Debra’s will names her daughter, Dena, who owns the remaining 20% interest in Silver Corporation, as the sole heir of the estate. With respect to this redemption, Debra’s estate has the following income: a. $0. b. $4,000,000 long-term capital gain. c. $4,000,000 dividend. d. $6,000,000 dividend. 67. Which of the following is a correct statement regarding a redemption to pay death taxes under § 303? a. An estate recognizes gain on the redemption equal to the excess of the distribution proceeds over the decedent’s basis in the stock. b. The § 318 stock attribution rules do not apply to the redemption. c. The value of the stock in the decedent’s gross estate must exceed 40% of the value of the adjusted gross estate. d. A corporation recognizes gains and losses on the distribution of property in the redemption. 68. Copper Corporation (E & P of $1,200,000) distributes land (basis of $410,000, fair market value of $650,000) to Lauren, a shareholder, to carry out a qualifying stock redemption. Lauren had a basis of $90,000 in the shares redeemed. Which of the following is an incorrect statement regarding the redemption? a. If the land is distributed subject to a $500,000 liability, Copper Corporation will recognize a gain of $90,000. b. If the land is distributed subject to a $500,000 liability, Lauren will have a basis in the land of $650,000. c. If the land is distributed subject to a $500,000 liability, Lauren will recognize a gain of $60,000. d. If the land is distributed subject to a $700,000 liability, Copper Corporation will recognize a gain of $290,000. 69. To carry out a qualifying stock redemption, Turaco Corporation (E & P of $800,000) transfers land held for investment purposes to Aida, a shareholder. The land had a basis of $250,000, a fair market value of $400,000, and is subject to a $300,000 liability. Aida has a basis of $70,000 in the shares redeemed. Which of the following is a correct statement regarding the tax consequences of this redemption? a. Aida will recognize a gain of $30,000. b. Aida will have a $100,000 basis in the land. c. Turaco Corporation will recognize a gain of $50,000. d. Aida will have $400,000 of dividend income. 70. Canary Corporation has 5,000 shares of stock outstanding. It redeems in a qualifying stock redemption 1,200 shares for $475,000 at a time when it has paid-in capital of $300,000 and E & P of $1,500,000. What would be the charge to Canary’s E & P as a result of the redemption? a. $72,000 b. $300,000 c. $360,000 d. $475,000 71. In the current year, Quail Corporation distributed installment notes payable in redemption of some of its shares. Quail incurred the following expenditures in connection with the redemption: accounting fees of $7,000 and legal fees of $8,000. In addition, Quail paid $10,000 of interest expense on the installment notes payable. The distribution was a qualifying stock redemption. How much of the $25,000 is deductible in the current year? a. $0 Powered by Cognero

Page 10


Name:

Class:

Date:

Chapter 06: Corporations Redemptions and Liquidations b. $7,000 c. $10,000 d. $25,000 72. Which of the following is an incorrect statement regarding the tax consequences of a § 306 stock disposition? a. In a sale of § 306 stock, the shareholder generally recognizes ordinary income equal to the fair market value of the preferred stock on the date it was acquired in the stock dividend. b. No loss is recognized on a sale of § 306 stock. c. The issuing corporation’s E & P is reduced by a sale of § 306 stock. d. In a redemption of § 306 stock, the shareholder generally recognizes dividend income equal to the amount of the redemption proceeds. 73. Connie sold 400 shares of § 306 stock (basis of $20,000) in Blackbird Corporation to Larry (an unrelated individual) for $50,000. When the § 306 stock was issued to Connie, the stock had a value of $50,000, and Blackbird had E & P of $500,000. At the time the § 306 stock is sold, Blackbird’s E & P is $550,000. At the time of the sale, Connie owned 900 shares of common stock (basis of $210,000) in Blackbird. With respect to the sale of the § 306 stock by Connie: a. Connie has $50,000 of ordinary income. b. Blackbird Corporation reduces its E & P by $50,000. c. Connie has a $30,000 capital gain. d. After the sale, Connie has a $210,000 basis in the common stock. 74. Rodolfo makes a gift of § 306 stock (basis of $75,000, fair market value of $100,000) in Kiwi Corporation to his daughter, Josie. When the stock was issued to Rodolfo, his share of Kiwi Corporation’s E & P was $80,000. When its E & P is $200,000, Kiwi Corporation redeems all of Josie’s stock for $100,000. With respect to the stock redemption: a. Josie will recognize a capital gain of $25,000. b. The redemption does not reduce Kiwi’s E & P. c. Josie will recognize dividend income of $80,000. d. Josie will recognize dividend income of $100,000. 75. In comparing a qualifying stock redemption with a complete liquidation, which of the following statements is incorrect? a. Liquidations and qualifying stock redemptions parallel each other in terms of the effect that E & P has on the nature of the gain or loss recognized by the shareholder. b. The basis of property acquired is its fair market value on the date of distribution for both a qualifying stock redemption and a liquidation. c. Both a qualifying stock redemption and a complete liquidation produce sale or exchange treatment to the shareholder. d. Section 267 disallows recognition of losses between related parties in a complete liquidation but not in a qualifying stock redemption. 76. Pursuant to a complete liquidation, Lilac Corporation distributes the following assets to its unrelated shareholders: land held for three years as an investment (basis of $300,000, fair market value of $600,000), inventory (basis of $100,000, fair market value of $80,000), and marketable securities held for four years as an investment (basis of $200,000, fair market value of $240,000). What are the tax consequences to Lilac Corporation as a result of the liquidation? a. Lilac Corporation would recognize no gain or loss on the liquidation. Powered by Cognero

Page 11


Name:

Class:

Date:

Chapter 06: Corporations Redemptions and Liquidations b. Lilac Corporation would recognize a net capital gain of $320,000. c. Lilac Corporation would recognize a net capital gain of $340,000 and an ordinary loss of $20,000. d. Lilac Corporation would recognize a net capital gain of $340,000. 77. Pursuant to a complete liquidation, Oriole Corporation distributes to its shareholders land with a basis of $350,000 and a fair market value of $800,000. The land is subject to a liability of $920,000. What is Oriole’s recognized gain or loss on the distribution? a. $0 b. $120,000 loss c. $450,000 gain d. $570,000 gain 78. The stock in Rhea Corporation is owned by Jennifer (80%) and Lucy (20%), mother and daughter. In a liquidation of the corporation in the current year, Rhea distributes land that it purchased two years ago for $675,000 to Lucy. The property has a fair market value on the date of distribution of $450,000. One year later, Lucy sells the land for $400,000. What loss, if any, will Rhea Corporation recognize with respect to the distribution of land? a. $0 b. $45,000 c. $225,000 d. $275,000 79. The stock in Toucan Corporation is held equally by two brothers. Four years ago, the shareholders transferred property (basis of $200,000, fair market value of $220,000) to Toucan Corporation as a contribution to capital. In the current year and pursuant to a complete liquidation of Toucan, the property is distributed proportionately to the brothers. At the time of the distribution, the property had a fair market value of $40,000. What amount of loss will Toucan Corporation recognize on the distribution of the property? a. $0 b. $20,000 c. $160,000 d. $180,000 80. Magenta Corporation acquired land in a § 351 exchange one year ago. The land had a basis of $320,000 and a fair market value of $350,000 on the date of the transfer. Magenta Corporation has two shareholders, Mark (70%) and Megan (30%), who are brother and sister. Magenta Corporation adopts a plan of liquidation in the current year. On this date, the land has decreased in value to $250,000. Magenta Corporation sells the land for $250,000 and distributes the proceeds pro rata to Mark and Megan. What amount of loss may Magenta Corporation recognize on the sale of the land? a. $0 b. $21,000 c. $30,000 d. $70,000 81. Purple Corporation has two equal shareholders, Joshua and Ellie, who are father and daughter. One year ago, the two shareholders transferred properties to Purple in a § 351 exchange. Joshua transferred land (basis of $600,000, fair market value of $450,000) and securities (basis of $70,000, fair market value of $250,000), and Ellie transferred equipment (basis of $420,000, fair market value of $700,000). In the current year, Purple Corporation adopts a plan of liquidation, sells all of its assets, and distributes the proceeds pro rata to Joshua and Ellie. The only loss realized upon disposition of the properties was with respect to the land that had decreased in value to $310,000 and was sold for this amount. Purple never Powered by Cognero

Page 12


Name:

Class:

Date:

Chapter 06: Corporations Redemptions and Liquidations used the land for any business purpose during the time it was owned by the corporation. What amount of loss can Purple Corporation recognize on the sale of the land? a. $0 b. $140,000 c. $150,000 d. $290,000 82. Last year, Crow Corporation acquired land in a transaction that qualified under § 351. The land had a basis of $400,000 to the contributing shareholder and a fair market value of $310,000. Assume that the shareholder also transferred equipment (basis of $100,000, fair market value of $200,000) in the same § 351 exchange. In the current year, Crow Corporation adopted a plan of liquidation and distributed the land to Ali, a shareholder who owns 20% of the stock in Crow Corporation. The land’s fair market value was $230,000 on the date of the distribution to Ali. Crow Corporation acquired the land to use as security for a loan it had hoped to obtain from a local bank. In negotiating with the bank for a loan, the bank required the additional capital investment by Crow as a condition of its making a loan to Crow Corporation. How much loss can Crow Corporation recognize on the distribution of the land? a. $0 b. $80,000 c. $90,000 d. $170,000 83. During the current year, Ecru Corporation is liquidated and distributes its only asset, land, to Kena, the sole shareholder. On the date of distribution, the land has a basis of $250,000, a fair market value of $650,000, and is subject to a liability of $500,000. Kena, who takes the land subject to the liability, has a basis of $120,000 in the Ecru stock. With respect to the distribution of the land, which of the following statements is correct? a. Kena recognizes a gain of $530,000. b. Ecru Corporation recognizes a gain of $250,000. c. Kena recognizes a gain of $30,000. d. Kena has a basis of $250,000 in the land. 84. In the current year, Dove Corporation (E & P of $1,000,000) distributes all of its property in a complete liquidation. Alexandra, a shareholder, receives land having a fair market value of $200,000. Dove Corporation had purchased the land as an investment three years ago for $125,000, and the land was distributed subject to a $100,000 liability. Alexandra took the land subject to the $100,000 liability. What is her basis in the land? a. $0 b. $100,000 c. $125,000 d. $200,000 85. Indigo corporation has a basis of $1,000,000 in the stock of Owl Corporation, a subsidiary in which it owns 100% of all classes of stock. Indigo purchased the stock in Owl 10 years ago. In the current year, Indigo liquidates Owl and acquires assets worth $1,200,000. At the time of its liquidation, Owl Corporation had a basis of $800,000 in the assets and E & P of $500,000. Which of the following statements is correct with respect to the liquidation? a. Owl recognizes a gain of $400,000. b. Indigo has an $800,000 basis in the assets. c. Owl’s E & P of $500,000 is eliminated. d. Indigo recognizes a gain of $200,000. Powered by Cognero

Page 13


Name:

Class:

Date:

Chapter 06: Corporations Redemptions and Liquidations 86. The stock of Lavender Corporation is held as follows: 80% by Jade Corporation (basis of $400,000) and 20% by Tiffany (basis of $100,000). Lavender Corporation is liquidated in December of the current year, pursuant to a plan adopted earlier in the year. Pursuant to the liquidation, Lavender Corporation distributed Asset A (basis of $600,000, fair market value of $900,000) to Jade, and Asset B (basis of $250,000, fair market value of $225,000) to Tiffany. No election is made under § 338. With respect to the liquidation of Lavender: a. Lavender recognizes a loss of $25,000 on the distribution of Asset B. b. Jade has a basis in Asset A of $900,000. c. Tiffany has a basis in Asset B of $225,000. d. Jade recognizes a gain of $500,000. 87. Penguin Corporation purchased bonds (basis of $190,000) of its 100% owned subsidiary, Finch Corporation, at a discount. Pursuant to a § 332 liquidation and in satisfaction of the indebtedness, Finch distributes land worth $200,000 (basis of $160,000) to Penguin. Which of the following statements is correct with respect to the distribution of land? a. Neither Finch nor Penguin recognize gain (or loss). b. Finch recognizes no gain and Penguin recognizes a gain of $10,000. c. Finch recognizes a gain of $40,000 and Penguin recognizes no gain. d. Finch recognizes a gain of $40,000 and Penguin recognizes a gain of $10,000. 88. Scarlet Corporation, the parent corporation, has a basis of $600,000 in the stock of Brown Corporation, a subsidiary in which Scarlet owns 90% of all classes of stock. Scarlet purchased the stock in Brown Corporation 10 years ago. In the current year, Scarlet Corporation liquidates Brown Corporation and acquires assets worth $800,000 and with a tax basis to Brown Corporation of $950,000. What basis will Scarlet Corporation have in the assets acquired from Brown Corporation? a. $0 b. $600,000 c. $800,000 d. $950,000 89. The stock of Loon Corporation is held as follows: 85% by Duck Corporation and 15% by Gerald, an individual. Loon Corporation is liquidated in December of the current year pursuant to a plan adopted earlier in the year. Loon Corporation distributes land with a basis of $350,000 and fair market value of $390,000 to Gerald in liquidation of his stock interest. Gerald had a basis of $200,000 in his Loon stock. How much gain will Loon Corporation recognize in this liquidating distribution? a. $0 b. $40,000 c. $190,000 d. $390,000 90. During the current year, Goldfinch Corporation purchased 100% of the stock of Dove Corporation and made a qualified election under § 338. Which of the following statements is incorrect with respect to the § 338 election? a. Dove is treated as a new corporation as of the day following the qualified stock purchase date. b. Dove must be liquidated pursuant to the § 338 election. c. Dove is treated as having sold its assets on the qualified stock purchase date. d. Dove can recognize gain or loss as a result of the § 338 election. 91. Which of the following statements is correct with respect to the § 338 election? Powered by Cognero

Page 14


Name:

Class:

Date:

Chapter 06: Corporations Redemptions and Liquidations a. The subsidiary corporation makes the § 338 election. b. A qualified stock purchase occurs when a corporation acquires in a taxable transaction at least 80% of the stock (voting power and value) of another corporation within an 18-month period. c. The parent recognizes no gain (loss) as a result of the election. d. Gain but not loss is recognized by the subsidiary as a result of a deemed sale of its assets. 92. Rosa owns all 100 shares of the outstanding stock of Stork Corporation (E & P of $500,000). Rosa had acquired the stock 12 years ago for $100,000. Stork, which has several trades or businesses that it has developed and operated for more than 5 years, sells one of those trades or businesses and, in a transaction qualifying as a partial liquidation [under § 302(b)(4)], redeems 20 shares of Rosa’s stock for $300,000. As a result of this transaction, Rosa must recognize: a. $280,000 dividend income. b. $280,000 long-term capital gain. c. $300,000 dividend income. d. $300,000 long-term capital gain. 93. Veronica and Tracy, unrelated individuals, own all the stock in Beige Corporation as equal shareholders. Each has a basis of $20,000 in her 20 shares. Beige Corporation has accumulated E & P of $900,000. Veronica wishes to retire in the current year and wants to sell her stock for $500,000, the fair market value. Tracy would like to purchase Veronica’s shares and, thus, become the sole shareholder in Beige Corporation. However, because Tracy is short of funds, Beige Corporation redeems all of Veronica’s shares for $500,000. Which of the following is a correct statement regarding the tax consequences of this redemption? a. Tracy will have dividend income of $500,000. b. Veronica will have a capital gain of $480,000. c. Veronica will have dividend income of $500,000. d. Beige Corporation will not reduce its E & P as a result of the redemption. 94. Hubert receives a nontaxable stock dividend of 100 shares of preferred stock (fair market value of $200,000) from Owl Corporation, at a time when Owl’s E & P is $800,000. As a result of the stock dividend, Hubert properly allocates $30,000 of his common stock basis to the preferred stock. Two years after the stock dividend, Hubert sells the preferred stock to Tomas, an unrelated party, for $250,000. Owl’s E & P at the time of the sale is $900,000. With respect to the sale of the preferred stock, Hubert has: a. Ordinary income of $250,000. b. Ordinary income of $200,000 and a long-term capital gain of $20,000. c. Long-term capital gain of $220,000. d. Dividend income of $200,000 and a long-term capital gain of $50,000. 95. Pursuant to a complete liquidation, Rust Corporation distributes to its shareholders land with a basis of $150,000 and a fair market value of $400,000. The land is subject to a liability of $300,000. What is Rust’s recognized gain on the distribution? a. $0. b. $100,000. c. $150,000. d. $250,000. 96. Skylark Corporation owned 100% of the outstanding stock of Quail Corporation having purchased the stock six years ago for $200,000. Pursuant to a plan of liquidation adopted by Quail Corporation earlier in the current year, Quail distributed all its property to its shareholder. Quail Corporation had never been insolvent and had E & P of $700,000 on Powered by Cognero

Page 15


Name:

Class:

Date:

Chapter 06: Corporations Redemptions and Liquidations the date of liquidation. Pursuant to the liquidation, Quail distributes property worth $650,000 (basis of $340,000) to Skylark Corporation. How much gain must the parties recognize on the transfer of this property to Skylark? a. $0 as to both Skylark and Quail. b. $140,000 as to Skylark. c. $310,000 as to Quail. d. $450,000 as to Skylark. 97. Which of the following statements is incorrect regarding tax planning opportunities for qualifying stock redemptions? a. A corporation that uses installment obligations to finance a redemption can deduct the related interest expense. b. With a “bootstrap acquisition,” a third party first acquires a small amount of a corporation’s stock, and then the corporation redeems the remaining stock of the other shareholders. c. For the shareholders of a family-owned corporation, the disproportionate redemption represents the best opportunity for a qualifying stock redemption. d. The not essentially equivalent redemption is of limited utility and should be considered only as a last resort. Subjective Short Answer 98. Tanya is in the 32% tax bracket. She acquired 1,000 shares of stock in Swan Corporation seven years ago for $100 a share. In the current year, Swan Corporation (E & P of $1,200,000) redeems all of Tanya's shares for $160,000. What are the income tax consequences to Tanya if: a.

The redemption qualifies for sale or exchange treatment and Tanya has no other transactions in the current year involving capital assets?

b.

The redemption does not qualify for sale or exchange treatment?

99. Steve has a capital loss carryover of $30,000 in the current year. He owns 3,000 shares of stock in Carmine Corporation, which he purchased six years ago for $20 per share. In the current year, Carmine Corporation (E & P of $750,000) redeems all of his shares for $140,000. Steve is in the 32% tax bracket. What is his income tax liability with respect to the corporate distribution if: a.

The redemption qualifies for sale or exchange treatment, and Steve has no other transactions in the current year involving capital assets?

b.

The redemption does not qualify for sale or exchange treatment, and Steve has no other transactions in the current year involving capital assets?

100. Hawk Corporation has 2,000 shares of stock outstanding: Marina owns 800 shares, Russell owns 500 shares, Velvet Partnership owns 400 shares, and Yellow Corporation owns 300 shares. Marina and Russell, unrelated individuals, are equal partners of Velvet. Marina owns 35% of the stock in Yellow.

a.

Applying the § 318 stock attribution rules, determine how many shares in Hawk Corporation each shareholder owns, directly and indirectly: Marina Russell Velvet Partnership

Powered by Cognero

Page 16


Name:

Class:

Date:

Chapter 06: Corporations Redemptions and Liquidations Yellow Corporation b.

Assume, instead, that Marina owns 60% of Yellow Corporation. How many shares does Marina own, directly and indirectly, in Hawk Corporation?

101. Starling Corporation was organized fifteen years ago to construct office furniture. Eight years ago, Starling began a fast-food business. In the current year, Starling discontinues its fast-food business and sells all of the assets used in that business for $2,000,000. Further, Starling distributes the entire sales proceeds in a pro rata redemption of 250 shares of stock from each of its two equal shareholders—Morgan, an individual, and Magpie Corporation. Morgan has a basis of $100,000 in her redeemed stock, Magpie Corporation has a basis of $125,000 in its redeemed stock, and both shareholders have held their stock interest in Starling for several years. Starling Corporation has E & P of $4,000,000 and 2,000 shares outstanding at the time of the distribution. What are the tax consequences of the stock redemption to Morgan, to Magpie, and to Starling? 102. Raul’s gross estate includes 1,500 shares of stock of Orange Corporation (basis to Raul of $1,200,000, fair market value on date of death of $8,200,000). The estate will incur $4,400,000 of death taxes and funeral and administration expenses, and the adjusted gross estate is $22,000,000. Denise, Raul’s daughter and sole heir of his estate, owns the remaining 500 shares of Orange Corporation’s shares outstanding. In the current year, Orange (E & P of $10,000,000) redeems all of the estate’s 1,500 shares for $8,200,000. What are the tax consequences of the redemption to Raul’s estate? 103. Ivory Corporation (E & P of $1,000,000) has 2,000 shares of common stock outstanding owned by unrelated parties as follows: Veronica, 1,000 shares, and Tommie, 1,000 shares. Both Veronica and Tommie paid $150 per share for the Ivory stock 12 years ago. In May of the current year, Ivory distributes land held as an investment (basis of $180,000, fair market value of $390,000) to Veronica in redemption of 350 of her shares. a.

What are the tax results to Veronica on the redemption of her Ivory stock?

b.

What are the tax results to Ivory Corporation on the distribution of the land?

104. As of January 1 of the current year, Grouse Corporation has E & P of $600,000. Fumiko owns 320 shares of Grouse’s common stock (basis of $45,000). On that date, Grouse Corporation declares and distributes a nontaxable preferred stock dividend of which Fumiko receives 100 shares. Immediately after the stock dividend, the fair market value of one share of Grouse common stock is $500, and the fair market value of one share of Grouse preferred stock is $200. Two months later, Fumiko sells the 100 shares of preferred stock to an unrelated individual for $20,000.

a.

Assuming Fumiko is in the 32% tax bracket, what are his income tax consequences resulting from the sale of the preferred stock?

b.

What is the effect on Grouse Corporation’s E & P as a result of the sale of the preferred stock?

105. The stock in Crimson Corporation is owned by Angel and Melawi, who are unrelated. Angel owns 60% and Melawi owns 40% of the stock. All of Crimson Corporation’s assets were acquired by purchase. The following assets are to be distributed in complete liquidation of Crimson Corporation:

Cash Inventory Equipment Land Powered by Cognero

Adjusted Basis $300,000 110,000 180,000 460,000

Fair Market Value $300,000 100,000 200,000 400,000 Page 17


Name:

Class:

Date:

Chapter 06: Corporations Redemptions and Liquidations a.

What gain or loss, if any, would Crimson Corporation recognize if it distributes the cash, inventory, and equipment to Angel and the land to Melawi?

b.

What gain or loss, if any, would Crimson Corporation recognize if it distributes the equipment and land to Angel and the cash and inventory to Melawi?

106. Mary and Jane, unrelated taxpayers, own Gray Corporation’s stock equally. One year before the complete liquidation of Gray, Mary transfers land (basis of $200,000, fair market value of $130,000) to Gray Corporation as a contribution to capital. Assume that Mary also contributed other property in the same transaction having a basis of $20,000 and fair market value of $100,000. In liquidation, Gray distributes the land to Jane. At the time of the liquidation, the land is worth $110,000.

a. b.

How much loss, if any, may Gray Corporation recognize on the distribution of the land to Jane? Assume that the transfer of land to Gray Corporation was made so that the corporation could subdivide the land and build residential housing. However, a subsequent deterioration of the housing market forced Gray Corporation to abandon its plans. What amount of loss may Gray Corporation recognize on the distribution of the land to Jane?

107. The stock of Tan Corporation (E & P of $1,500,000) is owned as follows: 90% by Egret Corporation (basis of $900,000), and 10% by Zoe (basis of $70,000). Both shareholders acquired their shares in Tan more than six years ago. In the current year, Tan Corporation liquidates and distributes land (fair market value of $1,100,000, basis of $1,300,000) and equipment (fair market value of $700,000, basis of $410,000) to Egret Corporation, and securities (fair market value of $200,000, basis of $260,000) to Zoe. What are the tax consequences of these distributions to Egret, to Tan, and to Zoe? 108. On March 17, 2023, Blue Corporation purchased 10% of the Gold Corporation stock outstanding. Blue Corporation purchased an additional 40% of the stock in Gold on October 21, 2023, and an additional 25% on April 1, 2024. On July 22, 2024, Blue Corporation purchased the remaining 25% of Gold Corporation stock outstanding.

a.

For purposes of the § 338 election, on what date does a qualified stock purchase occur?

b.

What is the due date for making the § 338 election?

Essay 109. Do noncorporate and corporate shareholders typically have the same preference for the tax treatment of a stock redemption? Explain. 110. Explain the stock attribution rules that apply in the case of stock redemptions. 111. When does a redemption qualify as a not essentially equivalent redemption under § 302(b)(1)? 112. What are the requirements that must be satisfied for a distribution to qualify under § 302(b)(2) as a disproportionate redemption? 113. Explain the requirements for the termination of a business test for purposes of a partial liquidation. Why is this test generally preferable over the genuine contraction of a corporate business test for qualifying a distribution as a partial Powered by Cognero

Page 18


Name:

Class:

Date:

Chapter 06: Corporations Redemptions and Liquidations liquidation? 114. When is a redemption to pay death taxes under § 303 most advantageous? 115. Under what circumstances will preferred stock received in a nontaxable stock dividend not generate ordinary income, under § 306, upon its disposition? 116. Explain why the antistuffing rules were enacted to limit the deductibility of losses realized by a corporation upon liquidation. 117. What are the tax consequences of a § 332 liquidation to the parent corporation, subsidiary corporation, and minority shareholder? 118. Compare the sale of a corporation’s assets with a sale of its stock from the perspective of the seller.

Powered by Cognero

Page 19


Name:

Class:

Date:

Chapter 06: Corporations Redemptions and Liquidations Answer Key 1. True 2. False 3. True 4. False 5. True 6. False 7. False 8. True 9. False 10. True 11. True 12. True 13. False 14. True 15. False 16. False 17. False 18. True 19. True 20. False 21. False 22. True 23. True 24. True 25. False Powered by Cognero

Page 20


Name:

Class:

Date:

Chapter 06: Corporations Redemptions and Liquidations 26. True 27. False 28. True 29. False 30. True 31. True 32. False 33. True 34. True 35. False 36. False 37. True 38. False 39. True 40. False 41. False 42. True 43. True 44. False 45. False 46. True 47. False 48. True 49. b 50. c Powered by Cognero

Page 21


Name:

Class:

Date:

Chapter 06: Corporations Redemptions and Liquidations 51. d 52. b 53. c 54. b 55. b 56. a 57. d 58. b 59. a 60. a 61. c 62. c 63. c 64. b 65. a 66. c 67. b 68. a 69. a 70. c 71. c 72. c 73. a 74. d 75. d 76. c Powered by Cognero

Page 22


Name:

Class:

Date:

Chapter 06: Corporations Redemptions and Liquidations 77. d 78. a 79. a 80. d 81. b 82. d 83. c 84. d 85. b 86. c 87. b 88. d 89. b 90. b 91. c 92. b 93. b 94. b 95. d 96. a 97. c 98. a. If the redemption qualifies for sale or exchange treatment, Tanya will have a long-term capital gain of $60,000 [$160,000 (amount realized) – $100,000 (stock basis)]. Her income tax liability on the $60,000 gain will be $9,000 ($60,000 × 15%). b.

If the redemption distribution of $160,000 does not qualify as a sale or exchange, it will be treated as dividend income and Tanya's tax liability will be $24,000 ($160,000 × 15%). (The entire $160,000 will be subject to tax at the 15% rate; Tanya will have no basis offset.)

Powered by Cognero

Page 23


Name:

Class:

Date:

Chapter 06: Corporations Redemptions and Liquidations 99. a. Steve will have a long-term capital gain of $80,000 on the redemption [$140,000 (amount realized) – $60,000 (stock basis)]. Steve can offset the $30,000 capital loss carryover against the $80,000 of capital gain. His income tax liability on the remaining $50,000 gain will be $7,500 ($50,000 × 15%). b.

If the redemption distribution does not qualify for sale or exchange treatment, the entire $140,000 will be taxed as a dividend at 15%, producing a tax of $21,000. With no other capital gain transactions in the current year, Steve can deduct only $3,000 of the $30,000 capital loss carryover to offset his other (ordinary) income.

100. a. Marina owns 1,000 shares [800 shares directly and 200 shares indirectly from Velvet Partnership (400 shares × 50% partnership interest)]. The stock attribution rules do not apply to stock held by a corporation if the shareholder owns less than 50% of the stock in that corporation. Russell owns 700 shares [500 shares directly and 200 shares indirectly from Velvet Partnership (400 shares × 50% partnership interest)]. Velvet Partnership owns 1,700 shares (400 shares directly plus 800 shares indirectly from Marina plus 500 shares indirectly from Russell). Stock owned by a partner is deemed to be owned in full by a partnership. Yellow Corporation owns 300 shares in Hawk Corporation. There is no attribution to Yellow Corporation from Marina, a less than 50% shareholder. b.

Marina owns 1,180 shares [800 shares directly plus 200 shares indirectly from Velvet Partnership (400 shares × 50% partnership interest) plus 180 shares indirectly from Yellow Corporation (300 shares × 60% shareholder interest)]. Stock owned by a corporation is deemed to be owned proportionately by any shareholder owning 50% or more of the corporation’s stock.

101. The redemption satisfies the termination of a business test under the partial liquidation rules. As a result, the redemption is a partial liquidation as to Morgan but not as to Magpie Corporation. Section 302(b)(4) permits sale or exchange treatment only to noncorporate shareholders. Morgan has a long-term capital gain of $900,000 [$1,000,000 (one-half of the sales proceeds) – $100,000 (basis in shares redeemed)]. Magpie Corporation has dividend income of $1,000,000 (one-half of the sales proceeds), reduced by a dividends received deduction under § 243 of $650,000 (65% × $1,000,000) or a net increase in taxable income of $350,000. Magpie’s $125,000 basis in the redeemed shares is added to the basis in its remaining shares of Starling stock. Starling Corporation’s E & P will be decreased $1,500,000 {$500,000 [12.5% (percentage of shares outstanding redeemed from Morgan) × $4,000,000 (E & P as of the redemption date)], plus $1,000,000 (dividend distribution to Magpie Corporation)}. 102. A portion of the redemption qualifies under § 303 as a sale or exchange. The value of the stock in Orange Corporation exceeds 35% of the value of the adjusted gross estate [($8,200,000 ÷ $22,000,000) = 37.3%]. A redemption to pay death taxes applies to the extent of the sum of the death taxes and funeral and administration expenses, or $4,400,000. The estate’s basis in the shares redeemed under § 303 is $4,400,000 (stepped-up basis); thus, this portion of the redemption results in no gain or loss to the estate. The remainder of the distribution ($3,800,000) must be tested under the qualifying stock redemption provisions of § 302 for sale or exchange treatment. For purposes of § 302, the stock Powered by Cognero

Page 24


Name:

Class:

Date:

Chapter 06: Corporations Redemptions and Liquidations attribution rules of § 318 apply and the shares owned by Denise, the estate’s sole beneficiary, are deemed to be owned by the estate. As such, the estate owns (directly and indirectly) 100% of the Orange shares outstanding after the redemption and none of the § 302 provisions are satisfied. The $3,800,000 therefore is treated as a dividend distribution to the estate. The estate’s basis in the shares not qualifying for sale or exchange treatment ($3,800,000 stepped-up basis) attaches to the basis of Denise’s shares in Orange Corporation. 103. a. Veronica has a long-term capital gain of $337,500 [$390,000 (amount realized) – $52,500 (stock basis)]. The distribution qualifies as a disproportionate redemption under § 302(b)(2). Veronica has a 50% (1,000 shares ÷ 2,000 shares) ownership interest in Ivory Corporation before the redemption and a 39.4% (650 shares ÷ 1,650 postredemption shares) ownership interest after the redemption. Both the 50% and the 80% [i.e., 39.4% < 40% (80% × 50%)] tests are met. Veronica will have a basis of $390,000 in the land. b.

Ivory Corporation has a recognized capital gain of $210,000 [$390,000 (fair market value) – $180,000 (adjusted basis)] on the distribution of the land. Gains (but not losses) are recognized in nonliquidating distributions. In a qualifying stock redemption, E & P is reduced by no more than the ratable share of the E & P attributable to the stock redeemed; thus, Ivory reduces its E & P by $175,000 [$1,000,000 E & P × 17.5% (percentage of stock redeemed) (less than the redemption price of $390,000)].

104. a. The preferred stock is § 306 stock and that provision governs the treatment of the sale. After the distribution and before the sale, the preferred stock has a basis to Fumiko of $5,000 [($20,000 value of preferred ÷ $180,000 value of preferred and common) × $45,000 (the original basis of the common stock)]. The sale of the § 306 stock results in $20,000 of ordinary income to Fumiko. This is the amount of dividend income Fumiko would have recognized if cash had been distributed instead of the preferred stock (the § 306 taint). The $20,000 is treated as dividend income for purposes of the 15% preferential tax rate on such income; thus, the preferred stock sale results in a tax of $3,000 ($20,000 × 15%) for Fumiko. The $5,000 basis in the preferred stock is added back to the basis of the common stock, giving such stock a $45,000 basis again. b.

The ordinary income that results from a sale of § 306 stock is not dividend income; thus, the stock sale does not affect Grouse Corporation’s E & P.

105. a. With respect to the distributions to Angel, Crimson Corporation will recognize a gain of $20,000 on the distribution of the equipment but not the loss of $10,000 on the distribution of the inventory. This is a distribution of loss property to a related party and the distribution is not pro rata; thus, the related-party loss limitation applies. With respect to the distribution of the land to Melawi, Crimson Corporation will recognize a loss of $60,000. Melawi is not a related party and the built-in loss limitation does not apply.

b.

With respect to the distributions to Angel, Crimson Corporation will recognize a gain of $20,000 on the distribution of the equipment but not the loss of $60,000 on the distribution of the land. Again, this is a distribution of loss property to a related party and the distribution is not pro rata. With respect to the distribution of the inventory to Melawi, Crimson Corporation will recognize a loss of $10,000. Melawi is not a related party and the built-in loss limitation

Powered by Cognero

Page 25


Name:

Class:

Date:

Chapter 06: Corporations Redemptions and Liquidations does not apply.

106. a. Since the land was acquired by Gray Corporation as a contribution to capital within two years of liquidation, a tax avoidance purpose is assumed and the built-in loss limitation applies. As a result, Gray will recognize only $20,000 of the loss. This represents the amount of decline in value occurring after Gray acquired the property. The built-in loss of $70,000 [$130,000 (fair market value) – $200,000 (land basis)] existing at the time of transfer to Gray is disallowed. Since Jane does not own more than 50% of the Gray stock, the related-party loss limitation does not apply.

b.

Because there was a business purpose for the transfer of the property to Gray, all of the $90,000 loss realized [$110,000 (fair market value) – $200,000 (land basis)] on the distribution is recognized. Since Jane does not own more than 50% of the Gray stock, the related-party loss limitation does not apply.

Note that the § 362(e)(2) basis step-down rules for loss properties acquired in carryover basis transactions does not apply to the undeveloped land, as there was no net built-in loss on the two properties transferred by Mary. Section § 362(e)(2) is discussed in Chapter 4. 107. The liquidating distribution to Egret is governed by § 332, resulting in no recognized gain or loss to either Egret or Tan. Egret has a carryover basis in both the land ($1,300,000) and the equipment ($410,000), and the basis in its Tan stock disappears. Tan’s E & P and other tax attributes carry over to Egret as provided under § 381. In distributions from a subsidiary corporation to a minority shareholder pursuant to a § 332 parent-subsidiary liquidation, gains but not losses are recognized [§ 336(d)(3)]. Thus, in the case of Tan, the $60,000 loss realized [$200,000 (fair market value) – $260,000 (basis)] on the distribution of the securities to Zoe is not recognized. Zoe recognizes a long-term capital gain of $130,000 [$200,000 (fair market value of securities) – $70,000 (basis in Tan stock)], and Zoe has a basis in the securities equal to their fair market value, or $200,000. 108. a. A qualified stock purchase occurs when one corporation acquires in a taxable transaction stock representing at least 80% of the total voting power and at least 80% of the value of another corporation within a 12-month period. For purposes of Blue Corporation, a qualified stock purchase occurs with the July 22, 2024 purchase [90% = 40% (October 21, 2023) + 25% (April 1, 2024) + 25% (July 22, 2024)]. b.

The § 338 election must be made by the fifteenth day of the ninth month beginning after the month in which a qualified stock purchase occurs. Since Blue’s qualified stock purchase date is July 22, 2024, the election must be filed by April 15, 2025.

109. No, noncorporate and corporate shareholders typically do not have the same preference for the tax treatment of a stock redemption. Noncorporate taxpayers generally prefer sale or exchange treatment (a qualifying stock redemption) Powered by Cognero

Page 26


Name:

Class:

Date:

Chapter 06: Corporations Redemptions and Liquidations over that of a dividend distribution. In a qualifying stock redemption, the shareholder is allowed to recover their redeemed stock basis tax-free. Also, the excess of the redemption distribution over the stock basis is (typically) a capital gain. If the shareholder has capital losses from other transactions, the capital gain resulting from a qualifying stock redemption can increase the shareholder’s deductibility of such capital losses. In a nonqualified stock redemption, the entire distribution is taxable as a dividend (assuming adequate E & P) that cannot be used to increase the utilization of capital losses. Corporate taxpayers, however, generally prefer dividend treatment (a nonqualified stock redemption) for a stock redemption. This preference stems from the availability of the dividends received deduction for such taxpayers. As a result of the dividends received deduction, only a nominal amount of any dividend resulting from a nonqualified stock redemption would be subject to tax. 110. In general, the § 318 stock attribution rules apply in determining a taxpayer’s ownership interest (direct and indirect) in a corporation before and after a stock redemption. The stock attribution rules do not apply in the case of partial liquidations or redemptions to pay death taxes. Further, the family attribution rules can be waived in the case of some complete termination redemptions (e.g., taxpayer has no prohibited interest for 10 years after redemption). Family attribution: An individual is deemed to own the stock owned by their spouse, children, grandchildren, and parents (not siblings or grandparents). Partnership attribution: A partner is deemed to own the stock owned by a partnership to the extent of the partner’s proportionate interest in the partnership. Stock owned by a partner is deemed to be owned in full by a partnership. Estate or trust attribution: A beneficiary or heir is deemed to own the stock owned by an estate or trust to the extent of the beneficiary or heir’s proportionate interest in the estate or trust. Stock owned by a beneficiary or heir is deemed to be owned in full by an estate or trust. Corporation attribution: Stock owned by a corporation is deemed to be owned proportionately by any shareholder owning 50% or more of the corporation’s stock. Stock owned by a shareholder who owns 50% or more of a corporation is deemed to be owned in full by the corporation. 111. To qualify as a not essentially equivalent redemption, the distribution must result in a meaningful reduction in the shareholder’s interest in the redeeming corporation. In determining whether a shareholder’s interest has been meaningfully reduced, the courts place greatest emphasis on the decrease in the shareholder’s voting control that results from the redemption. Other factors considered in the application of the meaningful reduction test include reductions in the shareholder’s rights to current and liquidating distributions. The stock attribution rules of § 318 apply for purposes of the meaningful reduction test. 112. To qualify as a disproportionate redemption, the stock redemption must satisfy the following requirements: (1) The shareholder’s ownership interest in the corporation after the redemption must be less than 80% of their ownership interest in the corporation before the redemption. (2) After the redemption, the shareholder must own less than 50% of the total combined voting power of all classes of stock entitled to vote. The stock attribution rules apply in determining a shareholder’s ownership interest before and after the redemption.

113. To qualify for the termination of a business test, a corporation must satisfy the following requirements: Powered by Cognero

Page 27


Name:

Class:

Date:

Chapter 06: Corporations Redemptions and Liquidations •

The corporation has two or more qualified trades or businesses (i.e., businesses that have been conducted for the five-year period ending on the date of the distribution and the businesses were not acquired in a taxable transaction during such five-year period).

The distribution consists of the assets of a qualified trade or business or the proceeds from the sale of such assets.

The corporation is actively engaged in the conduct of a qualified trade or business immediately after the distribution.

The termination of a business test provides an objective safe harbor rule for meeting the not essentially equivalent to a dividend requirement. The genuine contraction of a corporate business test also can be applied to satisfy the not essentially equivalent to a dividend requirement. However, the genuine contraction of a corporate business test is subjective and should be relied upon only after receiving a favorable IRS ruling. Because of the greater certainty provided by the termination of a business test, it is the preferred avenue for qualifying a distribution as a partial liquidation. 114. The principal advantage of a § 303 redemption to pay death taxes is that the stock attribution rules do not apply to such redemptions. Thus, a redemption to pay death taxes is most fortuitous when a redemption would not satisfy any of the other qualifying stock redemption provisions due to the attribution rules. Such would be the case when a corporation’s stock is owned entirely by the decedent’s estate and the beneficiaries of the estate. In such cases, a redemption of stock from the estate would not satisfy any of the § 302 qualifying stock redemptions because under that provision the estate is deemed to own the stock owned by the beneficiaries. However, since the stock attribution rules do not apply to a redemption to pay death taxes, sale or exchange treatment is available under § 303. While sale or exchange treatment under § 303 is limited to the sum of the death taxes and funeral and administration expenses, there is no requirement that the redemption proceeds be used for such expenditures or that the estate even require additional funds to pay such expenditures. An estate might have sufficient liquidity to meet the death-related expenses and still use a § 303 redemption to cash out some of its stock holdings at little or no tax consequence. 115. Section 306 does not result in ordinary income on the disposition of preferred stock received in a nontaxable dividend in a number of instances. For example, preferred stock is not “§ 306 stock” if, at the time of the stock dividend distribution, the corporation had no E & P. Thus, the disposition of such preferred stock will not generate ordinary income under § 306. Also, § 306 does not apply to a sale or redemption of preferred stock that results in the complete termination of the shareholder’s entire stock interest (§ 318 attribution rules apply). Further, a disposition in a partial liquidation or complete liquidation of the corporation will not result in ordinary income under § 306. Finally, § 306 ordinary income treatment upon disposition is limited to the corporation’s E & P at the time of the stock dividend distribution, in the case of a sale of the preferred stock or at the time of the redemption of the preferred stock. A disposition for more than this ordinary income taint will result in return of capital or capital gain treatment. 116. The loss limitation (“antistuffing”) rules were enacted primarily to address the artificial losses that could otherwise result with respect to properties that were acquired by a corporation in carryover basis transactions (i.e., § 351 and contribution to capital transactions). When loss property (i.e., fair market value less than adjusted basis) is transferred to a corporation and a carryover basis applies, there exists the possibility that loss could be recognized twice: once upon disposition of the property by the corporation and again upon disposition of the transferring shareholder’s stock. The antistuffing rules are not limited to carryover basis scenarios (e.g., the related-party loss limitation can apply with respect to property that is acquired in a noncarryover basis transaction), but the overriding reason for the enactment of the rules was to limit the opportunities for taxpayers to use carryover basis transactions to create artificial losses. The enactment of the basis step-down rules for loss properties acquired in carryover basis scenarios reduces the impact of the antistuffing rules upon liquidation, particularly with respect to the built-in loss limitation. Powered by Cognero

Page 28


Name:

Class:

Date:

Chapter 06: Corporations Redemptions and Liquidations 117. A parent corporation recognizes no gain or loss pursuant to a § 332 liquidation, and the parent’s basis in property received is equal the subsidiary’s basis in such property. In addition, a carryover holding period applies for the property. The parent also acquires the subsidiary’s other tax attributes (e.g., E & P, net operating loss carryover, business tax credit carryover, capital loss carryover) subject to § 381. The parent’s basis in the subsidiary stock disappears. (If property is received from the subsidiary in satisfaction of indebtedness, the parent recognizes gain or loss equal to the difference between the fair market value of the property received and the parent’s basis in the indebtedness.) A subsidiary corporation recognizes no gain or loss on distributions of property to a parent corporation in a § 332 liquidation, including property distributed in satisfaction of indebtedness. However, a subsidiary recognizes gain (but not loss) on the distribution of property to any minority shareholder. A minority shareholder recognizes gain or loss equal to the difference between the fair market value of the property received and the shareholder’s basis in the subsidiary stock. The minority shareholder’s basis in property received equals the property’s fair market value on the date of distribution. 118. A sale of a corporation’s assets presents more problems than a sale of its stock. When a corporation’s assets are sold, the sale proceeds are distributed to the shareholders in liquidation of the corporation. Both the corporation and the shareholders must treat the transaction as a sale for tax purposes. The transfer of assets requires that title be changed and that creditors be notified. Further, if valuable nontransferable trademarks, contracts, or licenses are involved, an asset sale may not be feasible. The sale of stock avoids these nontax problems, and only the shareholders report the transaction for tax purposes.

Powered by Cognero

Page 29


Name:

Class:

Date:

Chapter 07: Corporations Reorganizations True / False 1. Obtaining a favorable letter ruling from the IRS can ensure the desired tax treatment for parties contemplating a corporate reorganization. a. True b. False 2. For corporate restructurings, meeting the § 368 reorganization “Type” requirements is all that needs to be considered when planning the structure of the transaction. a. True b. False 3. For a corporate restructuring to qualify as a tax-free reorganization, the step transaction doctrine must apply. a. True b. False 4. The tax treatment of reorganizations almost parallels the Federal income tax treatment for like-kind exchanges. a. True b. False 5. In corporate reorganizations, if an acquiring corporation is using property other than stock or debt as consideration, it may recognize gains but not losses on the transaction. a. True b. False 6. The gain recognized by a shareholder in a corporate reorganization is based on the shareholder’s proportionate share of E & P. a. True b. False 7. The gains that shareholders recognize as a part of a corporate reorganization may be treated as a dividend to the extent of the corporation’s E & P. a. True b. False 8. Target shareholders recognize gain or loss when they receive assets (boot) as well as stock in the acquiring corporation in a transaction meeting the § 368 requirements. a. True b. False 9. As debt holders do not own stock, they do not fall under the corporate reorganization rules. a. True b. False 10. The basis for the acquiring corporation in the target’s assets is increased by any gain recognized by the target. a. True Powered by Cognero

Page 1


Name:

Class:

Date:

Chapter 07: Corporations Reorganizations b. False 11. In a “Type A” merger or consolidation, the acquiring corporation may select liabilities of the target it assumes. a. True b. False 12. While a “Type B” reorganization requires that voting stock be used by the acquiring corporation, in a “Type A,” the acquiring corporation can use common or preferred stock and still have the restructuring meet the qualifications of § 368. a. True b. False 13. To qualify as a “Type A” reorganization, mergers must comply with the requirements of pertinent foreign, state, and Federal statutes. a. True b. False 14. In a “Type B” reorganization, the acquiring corporation obtains control by exchanging common and preferred stock in the same percentages as the target’s outstanding common and preferred stock. a. True b. False 15. In a "Type B" reorganization, the acquiring corporation must obtain at least 40% of the target corporation’s stock through the reorganization. a. True b. False 16. When substantially all the assets of the target corporation are received in exchange for voting stock and selected liabilities, the restructuring can qualify as a “Type C” reorganization. a. True b. False 17. If the acquiring corporation purchased 25% of target stock for cash 10 years ago, the acquiring corporation still can meet the “Type C” reorganization requirement that 80% of the target’s assets be acquired with stock. a. True b. False 18. In a divisive “Type D” reorganization, the distributing corporation obtains control of the new target by exchanging some of its assets for at least 80% of the new target’s outstanding stock. a. True b. False 19. For a capital restructuring to qualify as a “Type E,” there must be at least a 50% change in common stock ownership. a. True b. False 20. An exchange of common stock for preferred stock or bonds for preferred stock can qualify as a “Type E” reorganization. Powered by Cognero

Page 2


Name:

Class:

Date:

Chapter 07: Corporations Reorganizations a. True b. False 21. The acquiring corporation in a “Type G” reorganization reduces the tax attributes carried over from the bankrupt corporation by the percentage change in ownership. a. True b. False 22. A tax avoidance motive is an example of establishing a sound business purpose for a reorganization. a. True b. False 23. The continuity of business enterprise doctrine requires that at least 40% of the target’s assets are acquired with stock. a. True b. False 24. The continuity of interest doctrine requires that all target shareholders receive some acquiring stock. a. True b. False 25. Without evidence to the contrary, the IRS views transactions occurring within one year of a reorganization as part of the restructuring, under the step transaction doctrine. a. True b. False 26. Liabilities generally are not considered boot in corporate reorganizations except in a “Type C” when cash or other property is received by the target in the transaction. a. True b. False 27. When the § 382 limitation is evoked, the acquiring corporation is limited in its use of the tax loss carryover of the loss corporation. The limitation is based on the net present value of the loss corporation using the Federal long-term discount rate. a. True b. False 28. One advantage of acquiring a corporation with net operating losses is that after a tax-free reorganization, the remaining corporation may combine the negative earnings and profits (E & P) of the target corporation with positive E & P of the acquiring corporation. a. True b. False 29. In a year in which an ownership change occurs for a corporation, the NOL carryforward is limited not only by the § 382 annual limitation but also by the percentage of the year remaining. a. True b. False Powered by Cognero

Page 3


Name:

Class:

Date:

Chapter 07: Corporations Reorganizations 30. When applying the § 382 limitation to deductible losses, the § 382 limit first is applied to capital loss carryovers and then to NOLs. a. True b. False 31. Jambo Corporation transfers 20% of its voting stock, with a FMV of $20 million, to Diamond Corporation in exchange for all of Diamond’s assets. Both companies are publicly traded corporations. After the exchange, Diamond distributes the Jambo stock (its only remaining asset) to its shareholders and liquidates. If Jambo repurchased the shares used in the exchange, the excise tax on corporate stock buybacks will apply. a. True b. False Multiple Choice 32. Which of the following statements is true? a. The dollar amounts involved in reorganizations are generally substantial; thus, it is important that the financial and tax treatment of the reorganization be consistent. b. A letter ruling indicates the income tax treatment the IRS likely applies to the proposed corporate restructuring transaction. c. Careful planning can ensure that all gains recognized by individual shareholders receive beneficial dividend treatment. d. If boot is transferred in a reorganization qualifying under § 368, losses can be recognized. 33. All the following statements are true about corporate reorganization except: a. Taxable amounts for shareholders are classified as a dividend or capital gain. b. Reorganizations receive treatment similar to corporate formations under § 351. c. The transfers of stock to and from shareholders qualify for like-kind exchange treatment. d. The value of the stock received by the shareholder less the gain not recognized (postponed) will equal the shareholder’s basis in the stock received. 34. A shareholder bought 10,000 shares of Coral Corporation for $50,000 several years ago. When the stock is valued at $90,000, Coral redeems the shares in exchange for 5,000 shares of Blush Corporation stock and a $10,000 Blush bond. This transaction meets the requirements of § 368. Which of the following statements is false regarding this transaction? a. The shareholder has a realized gain of $40,000. b. The shareholder has a postponed gain of $30,000. c. The shareholder has a basis in the Blush stock of $60,000. d. The shareholder has a recognized gain of $10,000. 35. Bobcat Corporation redeems all of Zed’s 4,000 shares and distributes to him 2,000 shares of Van Corporation stock plus $50,000 cash. Zed’s basis in his 20% interest in Bobcat is $100,000 and the stock’s value is $250,000. At the time Bobcat is acquired by Van, the accumulated earnings and profits of Bobcat are $200,000 and of Van are $75,000. How does Zed treat this transaction for tax purposes? a. Zed recognizes no gain in this reorganization. b. Zed reports a $50,000 recognized dividend. c. Zed reports a $50,000 recognized capital gain. d. Zed reports a $40,000 recognized dividend and a $10,000 capital gain. Powered by Cognero

Page 4


Name:

Class:

Date:

Chapter 07: Corporations Reorganizations 36. Mars Corporation merges into Jupiter Corporation by exchanging all of its assets for 300,000 shares of Jupiter stock valued at $2 per share and $100,000 cash. Wanda, the sole shareholder of Mars, surrenders her Mars stock (basis of $900,000) and receives all of the Jupiter stock transferred to Mars plus the $100,000. How does Wanda treat this transaction on her Federal income tax return? a. Wanda recognizes a $100,000 gain. Her Jupiter stock basis is $900,000. b. Wanda recognizes a loss of $100,000. Her Jupiter stock basis is $800,000. c. Wanda recognizes a $100,000 gain. Her Jupiter stock basis is $700,000. d. Wanda recognizes a loss of $0. Her Jupiter stock basis is $800,000. 37. Racket Corporation and Laocoon Corporation create Raccoon Corporation. Racket transfers $600,000 in assets for all of Raccoon’s common stock. Racket distributes its remaining assets ($300,000) and the Raccoon common stock to its shareholder, Mia, for all her stock in Racket (basis $950,000) and then liquidates. Laocoon receives all the Raccoon preferred stock for its $400,000 of assets. Laocoon distributes its remaining assets ($300,000) and the Raccoon preferred stock to its shareholder, Carlos, for all his stock in Laocoon (basis $200,000) and then liquidates. How will this transaction be treated for tax purposes? a. This qualifies as a “Type A” reorganization. Mia recognizes no gain or loss, but Carlos recognizes $300,000 gain. b. This qualifies as a “Type C” reorganization. Mia and Carlos recognize $300,000 gain, to the extent of the boot. c. This qualifies as a “Type D” reorganization. Neither Mia nor Carlos recognizes a gain or loss. d. This is a taxable transaction. Mia recognizes $50,000 loss and Carlos recognizes $500,000 gain. 38. Why is the shareholder’s basis in the new stock received in a corporate reorganization equal to the value of the stock received less any postponed gain? a. This ensures that the basis is the value of the stock given up in the reorganization. b. The realized gain is the amount that would be recognized if the stock was sold outright. This gain may not be recognized, however, unless there is boot involved. c. This ensures that the gain postponed will be recognized in the future when the stock is sold. d. A carryover basis or a substituted basis will not include the postponed gain that is necessary in a tax-deferred transaction such as a reorganization. 39. Which of the following conclusions regarding bonds in tax-favored corporate reorganizations is false? a. Bonds exchanged must have the same face value to ensure that the holder will receive equal value when the bonds are repaid. b. The interest rates on the bonds should be the same percentage because if the bondholder receives a security with a higher interest rate, the bondholder is receiving an asset with a greater value. c. Debt instruments with lives longer than 10 years are treated as securities because they have more risk associated with the likelihood that they will be repaid; this is similar to the risk with owning stock long-term. d. Bonds exchanged for stock do not receive tax-favored treatment because this exchange is essentially the purchase of stock by changing a debt holder into a shareholder. 40. Manx Corporation transfers 40% of its stock and $50,000 in cash to Somali Corporation for $500,000 of assets and all $200,000 of its liabilities. Somali exchanges the Manx stock, cash, and its remaining $100,000 of assets with its shareholders for all their Somali stock. After the exchange, Somali liquidates. The exchange qualifies as what type of transaction? a. “Type A” reorganization. b. “Type B” reorganization. c. “Type C” reorganization. Powered by Cognero

Page 5


Name:

Class:

Date:

Chapter 07: Corporations Reorganizations d. A taxable exchange. 41. Which of the following statements is true regarding “Type A” reorganizations? a. At least 80% of the acquiring corporation’s consideration must be voting stock, but the other 20% can be cash or preferred stock. b. The target shareholders must receive a proprietary interest in the acquiring corporation. This means that target shareholders must receive at least 40% of all the acquiring corporation’s stock. c. Substantially all the target’s assets must be transferred to the acquiring corporation, equaling at least 90% of the net asset value. d. Assumption of all liabilities for a “Type A” reorganization includes unknown and contingent liabilities. 42. Ocelot Corporation is merging into Tiger Corporation, meeting all state law requirements. Ocelot transfers assets worth $300,000 to Tiger. Ocelot receives 30,000 shares of Tiger stock and $200,000 cash. Ocelot transfers the Tiger stock, $200,000 cash, and all of its liabilities ($50,000) to its shareholder, Van, in exchange for all his Ocelot stock (basis $100,000). Ocelot then liquidates. How is this transaction treated for Federal income tax purposes? a. As this qualifies as a “Type A” reorganization, Van recognizes no gain. b. As this qualifies as a “Type C” reorganization, Van recognizes a $200,000 gain. c. As this qualifies as a “Type A” reorganization, Van recognizes a $150,000 gain. d. As this does not qualify as a reorganization, Van recognizes a $150,000 gain. 43. Xian Corporation and Win Corporation would like to combine into one entity. Win redeems 90% of its common stock and all of its nonvoting preferred stock in exchange for 40% of Xian’s common and 20% of its nonvoting preferred stock. Win then distributes the Xian stock to its shareholders. Win is now a subsidiary of Xian. a. This is a taxable transaction. b. This restructuring qualifies as a divisive “Type D” reorganization. c. This restructuring qualifies as a “Type B” reorganization. d. This restructuring qualifies as a “Type E” reorganization. e. This restructuring qualifies as a “Type C” reorganization. 44. Against the will of Rally Corporation’s management, Buoy Corporation offers Rally’s shareholders 2 shares of Buoy common stock for each share of Rally common and 50 shares of Buoy common for each share of Rally preferred. The results of a hostile takeover yield Buoy 85% of Rally common stock and 100% of the preferred. The only stock it did not obtain was that owned by management. This transaction qualifies as a(n): a. “Type A” consolidation. b. “Type B” reorganization. c. “Type D” split-up reorganization. d. Taxable event. 45. Humming Inc. is interested in acquiring BirdCo, a supplier of materials for Humming’s products, and feels that it could improve the management of BirdCo. Current management has been lax in monitoring product quality, which could lead to recalls or lawsuits. Management of BirdCo is not supportive of a merger because they could lose their positions, whereas most of the shareholders support the acquisition as a method of obtaining new management. There is a very small minority of shareholders who do not want to be shareholders of Humming. BirdCo holds assets of $5 million with a basis of $6 million. Its liabilities are $2 million. Which of the following would be the best solution for Humming in its acquisition of BirdCo? a. “Type A” reorganization. b. “Type B” reorganization. Powered by Cognero

Page 6


Name:

Class:

Date:

Chapter 07: Corporations Reorganizations c. “Type C” reorganization. d. Humming buys BirdCo’s assets for cash, and BirdCo distributes the cash to its shareholders and liquidates. 46. Which of the following statements regarding “Type B” reorganizations is true? a. As a parent-subsidiary relationship is created, the tax attribute carryover limitations are problematic. b. The acquisition of liabilities can cause problems when the liabilities of the target are greater than 20% of the total consideration and the acquiring owned target stock prior to the “Type B” reorganization. c. The acquisition of common and preferred target stock by the acquiring entity can be directly from the shareholders or from the target corporation. d. The acquiring corporation must distribute the target stock it obtains to its shareholders. The acquiring shareholders do not always have to turn in acquiring stock in exchange for the target stock. 47. Angus Corporation purchased 15% of Hereford Corporation four years ago for $150,000. Angus acquires 75% more of Hereford’s stock directly from Hereford shareholders in an exchange for 25% of Angus common stock currently outstanding. There is still 10% of the Hereford stock held by its original shareholders because they are not interested in being common shareholders of Angus. This transaction qualifies as what type of reorganization? a. “Type A” reorganization. b. “Type B” reorganization. c. “Type C” reorganization. d. None. This is a taxable exchange. 48. Acquiring Corporation transfers $1 million of its voting common stock and $100,000 cash to Target Corporation in exchange for 90% of Target's assets. The assets retained by Target are used to settle its liabilities. Target then distributes the Acquiring stock and cash received to its shareholders in exchange for all their Target shares, after which it liquidates. This restructuring is a: a. “Type A” reorganization. b. “Type B” reorganization. c. “Type C” reorganization. d. Taxable exchange. 49. Ula purchased stock in Purple, Inc., six years ago for $150,000. Purple has assets with a value of $225,000 ($175,000 basis) and liabilities of $60,000. Purple transfers $200,000 of assets and all its liabilities to White Corporation in exchange for White common stock. Purple distributes the White stock and its $25,000 remaining asset (cash) to Ula in exchange for all her Purple stock. Purple then liquidates. How is this transaction treated for Federal income tax purposes? a. Ula recognizes a $15,000 gain on the exchange. b. Ula recognizes a $25,000 gain on the exchange. c. Ula recognizes a $25,000 gain and Purple recognizes a $25,000 gain on the exchange. d. Purple recognizes a $50,000 gain on the exchange. 50. Long Corporation has $500,000 of assets with a basis of $350,000 and liabilities of $125,000. ShortCo acquires Long’s assets and $100,000 of liabilities by exchanging $400,000 of its voting stock. Long distributes the ShortCo stock and remaining liabilities to its shareholder in exchange for the shareholder's Long stock with a basis of $275,000; then it liquidates. Which, if any, of the following statements is correct? a. This restructuring qualifies as a “Type A” reorganization with no recognized gains or losses. b. This restructuring qualifies as a “Type C” reorganization with no recognized gains or losses. c. This qualifies as either a “Type A” or “Type C” and the shareholder has a $25,000 recognized gain. Powered by Cognero

Page 7


Name:

Class:

Date:

Chapter 07: Corporations Reorganizations d. The restructuring is taxable this year; liabilities cannot be distributed to shareholders in a tax-free reorganization. 51. Saucer Corporation has a value of $800,000, basis in its assets of $670,000, and liabilities of $200,000. Cup Corporation acquires 90% of Saucer’s assets by exchanging $550,000 of its voting stock, $20,000 cash, and assuming $150,000 of Saucer’s liabilities. The remaining 10% of Saucer’s assets not acquired is $80,000 cash. Saucer distributes the Cup stock, $100,000 in cash and associated $50,000 in liabilities to its shareholder, Sam, in exchange for his Saucer stock (basis $560,000). Saucer then liquidates. How will this transaction be treated for Federal income tax purposes? a. As a “Type A” reorganization, Sam recognizes $50,000 gain and Saucer recognizes $20,000 gain. b. As a “Type A” reorganization, Sam recognizes $100,000 gain and Saucer recognizes $120,000 gain. c. As a “Type C” reorganization, Sam recognizes $50,000 gain and Saucer recognizes $20,000 gain. d. As a “Type C” reorganization, Sam recognizes $40,000 gain and Saucer recognizes no gain. e. As a taxable transaction. 52. Grebe Corporation is a car dealership that has existed for 10 years. It also has been in the car leasing business for 6 years. Both businesses produce substantial amounts of cash, and Grebe has been investing this cash in mutual funds for the past 10 years. Grebe is interested in separating its businesses. It wants to create (a) new corporation(s) to receive assets in exchange for stock. Which of the following is correct regarding this transaction? a. Grebe must distribute at least 80% of the new corporation(s) stock to its shareholders in exchange for a proportionate amount of Grebe’s stock. If the shareholders do not exchange stock, the transaction receives dividend treatment. b. Grebe may create up to three new corporations because it has three lines of business. If three new corporations are created, Grebe ceases to exist because it will have no assets. c. The new corporations created will carry over no tax attributes or earnings and profits from Grebe. d. Using a split-off “Type D” reorganization, Grebe can transfer the car leasing business to the new corporation and exchange the new corporation stock for some of the Grebe stock held by its shareholders. 53. Spoonbill Corporation has assets with a FMV of $800,000 and adjusted basis of $600,000. It has been manufacturing engineering equipment and laboratory tools for the last eight years. Spoonbill forms a new corporation, Roseate Corporation, by acquiring all of its stock in exchange for the laboratory tool division of Spoonbill. Each of the Spoonbill shareholders receives 1 share of Roseate stock for each 50 shares they own in Spoonbill. How will this transaction be treated for Federal income tax purposes? a. As a split-off “Type D” reorganization. b. As a spin-off “Type D” reorganization. c. As a split-up “Type D” reorganization. d. This transaction is treated as a stock dividend. 54. In which type of corporate reorganization do shareholders receive stock in at least two other corporations in exchange for all the stock in the original corporation? a. “Type A” consolidation reorganization. b. “Type D” spin-off reorganization. c. “Type D” split-off reorganization. d. Some other type of reorganization. 55. Crested Serpent Eagle (CSE) Corporation is owned by Lin Yuan and Yu Chi. It has been in the manufacturing and lumber businesses for 20 years. For liability protection, the manufacturing assets of CSE are transferred to Serpent Corporation for all its stock. This stock is distributed to Lin Yuan in exchange for her CSE stock. The lumber assets are Powered by Cognero

Page 8


Name:

Class:

Date:

Chapter 07: Corporations Reorganizations transferred to Eagle Corporation for all of its stock. Yu Chi receives the Eagle stock in exchange for his CSE shares. CSE then terminates. How will this transaction be treated for Federal income tax purposes? a. The transaction qualifies as a spin-off “Type D” reorganization. b. The transaction qualifies as a split-off “Type D” reorganization. c. The transaction qualifies as a split-up “Type D” reorganization. d. The transaction is taxable. 56. WhydahCo is owned by Gilda and her four nieces and nephews. Gilda owns all the WhydahCo voting stock and its $50,000 bond. She wants to relinquish control of the entity; accordingly, WhydahCo redeems all of Gilda’s voting common stock and issues its preferred stock to her. She also exchanges her bond for preferred stock. The nonvoting preferred shares owned by the nieces and nephews are exchanged for voting common stock. Which of the following statements is correct? a. The exchange of a bond for preferred stock is taxable. b. The exchange of common for preferred is not taxable but the exchange of preferred stock for common stock is taxable. c. All of these transactions are taxable. d. The transaction is not currently taxable; this is a “Type E” reorganization. 57. Peony owns all of the Garden Corporation common stock with a basis of $400,000 and a value of $900,000. Her grandchildren own nonvoting preferred stock with a basis and value of $540,000 that pays a 6% annual dividend. Peony would like to transfer her ownership of Garden to her grandchildren but retain a guaranteed income from Garden. What would be the most income tax effective method of making this transfer? a. Peony sells her common stock to her grandchildren. They pay for the stock on the installment method over 20 years with a 6% interest on the unpaid balance. b. Garden redeems all of Peony’s common stock and issues her a 20-year bond for $900,000 that pays 6% interest. c. Garden redeems Peony’s common stock and issues her preferred stock with a 6% yearly dividend rate. Garden exchanges the grandchildren’s preferred stock for common stock. d. Peony exchanges 60% of her common stock with her grandchildren for all of their preferred stock. The grandchildren then have control and Peony retains 40% of the common stock. 58. Dahlia owns $100,000 in Crimson Topaz preferred stock. The annual dividend rate on the preferred is 4%. She exchanges this preferred stock for $60,000 in Crimson Topaz bonds paying 4% annual interest and $40,000 in common stock. How is this transaction treated for income tax purposes? a. It is taxable in the current year. b. The transaction is not currently taxable because it qualifies as a “Type E” reorganization. c. Only the exchange of the preferred stock for the common stock is taxable, because of the reduction in shareholder rights. d. Only the exchange of the preferred stock for the bond is taxable. 59. Pallid Swift, Inc. is an S corporation located in Colorado. In the past year, Pallid Swift has become profitable but, due to its rapid growth, it holds no excess cash for distributions. Therefore, Pallid Swift decides that it should become a C corporation. a. This transaction qualifies as a “Type F” reorganization. b. This transaction qualifies as a “Type E” reorganization. c. This transaction qualifies as a "Type D" shift reorganization. d. This incorporation is a taxable event. Powered by Cognero

Page 9


Name:

Class:

Date:

Chapter 07: Corporations Reorganizations 60. Cuckoo Corporation has just lost a $500,000 product liability suit. Before the lawsuit, its assets were valued at $600,000 (basis of $400,000), and it had general liabilities of $300,000 and $100,000 of bonds outstanding. It also has a $50,000 capital loss carryover and a $150,000 NOL carryforward. Cuckoo is solely owed by Emmy Lou. A restructuring creates Turaco as the successor company to Cuckoo. Which of the following statements is false? a. This transaction qualifies as a “Type G” reorganization. b. Emmy Lou may not receive any stock in Turaco in the restructuring. c. When Turaco reduces Cuckoo’s tax attributes for the cancellation of debt income relief, it first reduces the capital loss, then the NOL, and last the basis in the assets. d. The bondholders of Cuckoo become shareholders of Turaco. 61. Burmese Corporation is interested in acquiring Javanese Corporation by transferring 30% of its stock for all of Javanese’s assets valued at $500,000 (basis of $150,000) and its $200,000 of liabilities. Javanese has $50,000 of capital loss carryforwards that it has been unable to use. Javanese concentrates on pharmaceutical research, whereas Burmese manufactures sun glasses. Burmese uses a discount factor of 8%, and the Federal applicable rate is 4%. Javanese will terminate after the restructuring. How will this transaction be treated for Federal income tax purposes? a. As Javanese has liabilities in excess of its basis, this excess will be taxable to Javanese. b. The most of the capital loss carryforward that Burmese can use in any year is $3,000. c. This transaction could qualify as a “Type A” or a “Type C” reorganization. d. This transaction does not qualify as a tax-free reorganization. 62. Which of the following statements is false? a. A “Type B” reorganization is most likely to run afoul of the continuity of interest doctrine because the target remains a separate corporation. b. Liabilities of the target can create problems in crafting “Type A” and “Type C” reorganizations. c. The step transaction doctrine is especially pertinent in crafting “Type B” and “Type C” reorganizations. d. “Type E” and “Type F” are not likely to be subject to the § 382 limitation. 63. Target sells assets not desired by Acquirer before entering into a reorganization transaction with Acquirer. In which reorganization will the step transaction doctrine not apply to the sale by Target? a. “Type A” reorganization. b. “Type B” reorganization. c. “Type C” reorganization. d. Only “Type A” and “Type C”. 64. Asity Corporation is interested in acquiring the majority of Pitta Corporation’s assets. Pitta’s assets are currently valued at $950,000, and its liabilities are $250,000. However, Asity is not interested in one operating division of Pitta. Since Pitta desires to be taken over by Asity, Pitta first sells the unwanted division for its net fair market value of $250,000 ($350,000 FMV assets – $100,000 liabilities). Pitta then transfers its remaining assets and liabilities to Asity for $450,000 in common voting stock. Which of the following statements is correct? a. Continuity of interest does not exist for the Pitta shareholders. b. The continuity of business enterprise test is failed. c. There is no sound business purpose for this restructuring. d. The step transaction can be applied to this transaction. 65. Besides the statutory requirements for tax-free treatment for corporate reorganizations, there are several judicial Powered by Cognero

Page 10


Name:

Class:

Date:

Chapter 07: Corporations Reorganizations requirements. Which of the following statements is not true? a. The ownership change doctrine should be met. b. The continuity of business enterprise test must be met. c. There must be a sound business purpose for the restructuring. d. The step transaction doctrine should not apply. 66. Which of the following statements is correct with regard to liabilities in corporate reorganizations? a. While in a “Type A” merger, all the liabilities of the target must be acquired; in a consolidation, only specified liabilities are transferred. b. In a “Type G” reorganization, the target’s liabilities rarely are liquidated. c. Liabilities are problematic for a “Type C” only when the acquiring corporation transfers other property in addition to common stock. d. Long-term liabilities (bonds) can be exchanged tax-free in a “Type E” reorganization as long as the terms of the bonds are greater than 10 years and the interest rates are identical. 67. Weaver Corporation has net assets valued at $800,000 and an NOL of $250,000. On September 30 of the current year, Weaver is acquired by Loom Corporation, a calendar year taxpayer, in a restructuring qualifying as a tax-free reorganization. Weaver shareholders receive 30% of Loom’s shares in exchange for all of their Weaver stock. Assuming that the Federal long-term tax-exempt rate is 3%, what is the maximum amount of Weaver’s NOL available to Loom in a non-leap year? a. $250,000 b. $62,500 c. $24,000 d. $6,049 68. Heart Corporation holds net assets valued at $1 million and an NOL of $250,000. On December 31 of last year, Heart is acquired by Brain Corporation, a calendar year taxpayer, in a restructuring qualifying as a tax-free reorganization. Heart shareholders receive 45% of Brain’s shares in exchange for all of the Heart stock. Assuming that the Federal long-term tax-exempt rate is 3% and Brain’s discount factor is 7%, what is the maximum amount that Brain can use of Heart’s NOL this year? a. $12,500 b. $30,000 c. $100,000 d. $250,000 69. Monal Corporation merged with Bobwhite Corporation two years ago. At the time of the merger, Monal held an earnings and profits (E & P) deficit of $250,000, and Bobwhite held a positive E & P of $200,000. Last year’s current E & P was $100,000 for the successor company. Despite having only $10,000 E & P for the current year, Monal makes a distribution to its shareholders of $270,000. How much of the distribution is taxed to the shareholders? a. $270,000 b. $210,000 c. $110,000 d. $60,000 70. Flower Corporation has two divisions that it has operated for the last 10 years: Gerbera with a value of $806,000 (basis $400,000) and Daisy with a value of $744,000 (basis $800,000). While Gerbera is profitable, Daisy continues to incur losses and created a $350,000 NOL for Flower two years ago. Flower has decided it would like to become two Powered by Cognero

Page 11


Name:

Class:

Date:

Chapter 07: Corporations Reorganizations corporations: Gerbera and Daisy. Which of the following is the best method for Flower to become two corporations? a. Under a § 351 creation of a corporation, Flower transfers all of the Daisy division assets and the NOL to the new corporation (Daisy) for all of its stock. Flower retains the stock and Daisy becomes a subsidiary without limitations on its NOL use. b. Using a split-off, Flower transfers the Gerbera division assets to the new corporation (Gerbera) for all of its stock. Flower distributes all of the Gerbera stock to its shareholders in exchange for 52% of their Flower stock. Flower retains the Daisy division and NOL without limitations on its use. c. Using a spin-off, Flower transfers the Daisy division assets to the new corporation (Daisy) for all of its stock. Flower distributes all of the Daisy stock to its shareholders. Flower retains the Gerbera division and NOL without limitations on its use. d. Using a split-up, Flower transfers the Daisy division assets to the new corporation (Daisy) for all of its stock and transfers the Gerbera division assets to the new corporation (Gerbera) for all of its stock. Flower distributes all of the Daisy and Gerbera stock to its shareholders in exchange for 100% of their Flower stock, and Flower then terminates. The Daisy division retains the NOL without limitations on its use. Completion 71. The tax treatment of the parties involved in a tax-free reorganization almost exactly parallels the treatment under the like-kind exchange provisions. When ____________________ is received, gain may be ____________________, and losses are ____________________. 72. ____________________ is other property received along with stock in a restructuring falling under § 368. If the shareholders receive the other property, it can be taxed as ____________________ and/or ____________________. 73. In a ____________________ reorganization, each corporation must obtain the approval of the majority of its shareholders. The acquiring corporation must assume at least ____________________ percent of the target’s liabilities. 74. Since the “Type B” reorganization precludes the use of ____________________, gain or loss is not recognized. In this type of reorganization, a ____________________ relationship between the acquiring and target corporations is created. 75. For a “Type C” reorganization, "substantially all" of the assets means that at least ____________________ percent of the net asset value and ____________________ percent of the gross asset value. 76. In a ____________________ reorganization, shareholders may exchange preferred stock for common or common for preferred stock. 77. Changing from an S corporation to a C corporation is a Type ____________________ reorganization. 78. Federal bankruptcy legislation created the Type ____________________ reorganization. To qualify for this type of reorganization, the corporation must be ____________________ before the reorganization. 79. The acquiring corporation in a “Type G” reorganization must reduce the tax attributes carried over to it to the extent of the ____________________ relief. 80. The ____________________ doctrine ensures that the acquiring corporation cannot immediately sell the target corporation's assets it receives in the reorganization. The ____________________ doctrine also prevents transactions that appear to be sales from qualifying as nontaxable reorganizations. 81. In a ____________________ reorganization, the original corporation must receive at least 80% percent of new corporations’ stock. The assets transferred to the new corporations must have been owned and conducted as a trade or Powered by Cognero

Page 12


Name:

Class:

Date:

Chapter 07: Corporations Reorganizations business for at least ____________________ years. 82. The ____________________ doctrine treats several transactions as if they were one transaction when they are all integrated. The ____________________ doctrine ensures that the restructuring has a purpose beyond tax avoidance or evasion. 83. Liabilities present income tax problems only in the ____________________ reorganization when the ____________________ corporation transfers other property (boot) as well as stock to the target. 84. The ____________________ provides a restriction on the amount of tax attributes that can be carried over from the target to the acquiring corporation after an ownership change occurs. 85. The yearly § 382 limitation is computed by multiplying the ____________________ of the loss corporation’s stock on the change date by the ____________________. For the ____________________, loss deductions are limited to the § 382 limitation times the number of days remaining in the year divided by the number of days in the year. 86. Loss Corporation carries over an NOL, built-in ordinary losses, and capital losses. The § 382 limitation is applied to these carryover attributes in the following order: ____________________, ____________________, ____________________. Matching Match the following items with the statements that follow. Terms may be used more than once. a. Boot b. Business credits c. Capital gain d. Continuity of business enterprise e. Continuity of interest f. Dividend g. Discount rate h. Earnings and profits i. Federal long-term tax-exempt rate j. Liability assumption k. Ordinary gain l. Ownership change m. Section 382 limitation n. Sound business purpose o. Step transaction 87. Carries over to new corporations in a split-up reorganization. 88. A 50 percentage-point change in ownership that occurs because of tax-free reorganization. 89. Tax avoidance is not enough; transaction must have a corporate economic consequence. 90. Requires the computation of a deduction equivalent when determining its limitation. Powered by Cognero

Page 13


Name:

Class:

Date:

Chapter 07: Corporations Reorganizations 91. Rate used to determine the § 382 limitation. 92. When the transactions are so interdependent that the accomplishment of one would be fruitless without the completion of the series. 93. Shareholders recognize gain to the extent the restructuring qualifies as a redemption. 94. Any asset other than stock or securities received by the target shareholders. 95. Requires at least a 40% carryover ownership by target shareholders. 96. Can be treated as boot if cash as well as voting stock is the consideration used by the acquiring corporation in a “Type C” reorganization. What type of reorganization is effected in each of the following independent transactions? a. “A” reorganization, merger b. “A” reorganization, consolidation c. “B” reorganization d. “C” reorganization e. “D” reorganization, spin-off f. “D” reorganization, split-off g. “D” reorganization, split-up h. “E” reorganization i. “F” reorganization j. “G” reorganization k. Taxable transaction 97. Flower Corporation is owned by Carry and Kim. All of Carry’s stock (value of $200,000) is exchanged for $200,000 in bonds. Kim exchanges no stock in the transaction. 98. Green Corporation transfers 20% of its voting stock to Brown Corporation’s shareholders in exchange for 60% of Brown’s common stock and 90% of its preferred stock. Green acquired 30% of Brown’s common stock three years ago in a taxable transaction. Brown becomes a subsidiary of Green. 99. Solar currently operates nine lines of business that have been in existence for 10 years. To protect its assets from possible lawsuits, Solar transfers each division to newly formed corporations in exchange for all of the stock of the new corporations. Solar distributes the stock in the nine new corporations to its shareholders in exchange for 45% of their stock in Solar. 100. BarkCo and WoodCo contribute all of their assets to Tree Corporation in exchange for all of Tree’s stock. BarkCo and WoodCo distribute the Tree stock to their shareholders in exchange for their stock in BarkCo and WoodCo. The exchange completes the liquidation of BarkCo and WoodCo and each ceases to exist. 101. Pear Corporation wishes to merge with Plum Corporation. Plum has better name recognition with consumers, so Pear would like Plum to be the surviving corporation. Pear transfers all of its assets and only liabilities associated with real estate to Plum for 45% of Plum’s shares. Pear distributes the Plum stock to its shareholders in exchange for their Pear stock. Pear then liquidates. 102. Jones Corporation is insolvent and under state law protection from its creditors. Its assets are transferred to Smith Powered by Cognero

Page 14


Name:

Class:

Date:

Chapter 07: Corporations Reorganizations Corporation in exchange for all of its stock. The stock is distributed to the creditors of Jones, and the shareholders receive only stock valued at 10% of their stock. 103. Acquiring Corporation receives all the assets of Target Corporation in exchange for 1,000 preferred shares and 6,000 common shares of Acquiring, $25,000 cash, and assumption of all the liabilities of Target. After distributing the Acquiring stock and cash to its shareholders, Target liquidates. 104. Snow Corporation, located in Washington state, establishes Rain Corporation, located in Arizona. Snow then transfers all of its assets to Rain in exchange for all of Snow’s stock. After distributing the Rain stock to its shareholders, Snow liquidates. 105. Black Corporation has been engaged in manufacturing toys and investing in high technology corporate stock for eight years. Black creates Blue and White Corporations. It transfers the toy division to Blue and the investments to White in exchange for all of each corporation’s common voting stock. The stock of Blue and White is distributed to Black shareholders in return for all of their Black stock. Black then liquidates. 106. Apple Corporation transfers voting stock to Orange Corporation in exchange for substantially all of its assets and its liabilities associated with the plant and equipment. Its general liabilities are not acquired by Apple. Orange distributes the Apple stock to its shareholders in exchange for their Orange stock. Orange then liquidates. Subjective Short Answer 107. Cotinga Corporation is acquiring Petrel Corporation through a “Type C” reorganization by exchanging 20% of its voting stock and $50,000 for all of Petrel’s assets (value of $800,000 and basis of $600,000) and liabilities ($100,000). Jerrika owns 48% of Petrel (basis $270,000), and Allen owns the remaining 52% (basis $380,000). They exchange their stock in Petrel for their proportionate shares of the Cotinga stock and cash. What is the value of the Cotinga stock received by Jerrika and Allen? What are the amounts of gains/losses each recognizes due to the reorganization? What is Jerrika’s and Allen’s basis in the Cotinga stock? 108. Acquiring Corporation transfers $500,000 stock and land with a value of $400,000 (basis of $250,000) to Target for most of its assets. The assets not acquired in the “Type A” reorganization are distributed to Target’s shareholder, Tia. They are valued at $100,000 (basis of $120,000). Acquiring stock and the land also are distributed to Tia in exchange for her stock in Target. Tia’s basis in her stock is $650,000. What is the gain or loss recognized by Acquiring, Target, and Tia on this restructuring? What is Tia’s basis in the Acquiring stock? 109. Dipper Corporation is acquiring Bulbul Corporation by exchanging 220,000 shares of Dipper stock and $80,000 cash for all of Bulbul’s assets (valued at $500,000), liabilities ($200,000), and accumulated earnings and profits ($120,000). Betty purchased 40% of Bulbul five years ago for $60,000, and Keith purchased the remaining 60% for $90,000. What is the amount of the gain or loss (if any) that Betty and Keith recognize, assuming that the exchange qualifies as a § 368 reorganization? What is the basis in their new Dipper stock? 110. Pipe Corporation is interested in acquiring all of Ore Corporation. It currently owns 30% of the outstanding Ore stock, which it purchased six years ago for $250,000. Pipe is a manufacturer of plumbing pipes with assets valued at $3 million and liabilities of $1 million. Ore supplies Pipe with copper from its mines that are valued at $4 million with $3 million in mortgages. Pipe negotiates the restructuring with Ore’s management. Pipe is concerned about potential environmental issues from the strip mining used by Ore and feels it needs liability protection. a.

Given these facts, what type of reorganization would you suggest for Pipe and Ore?

b.

Provide a diagram of the reorganization you suggest.

Powered by Cognero

Page 15


Name:

Class:

Date:

Chapter 07: Corporations Reorganizations 111. Present Value Tables needed for this question. Tony is the sole shareholder of Create Corporation. He is a chemical engineer and has been working hard to create a unique product but has been unsuccessful. Thus, Create has accumulated an NOL of $420,000. This year he finally finds the right combination for a new cleaning product. Predicting that Create will be very profitable next year, the corporation borrows $250,000 to pay Tony the salary he rightly deserves. Next year, Create does become profitable, earning $100,000 before application of carryovers. Mega Corporation, a huge ($50 million value, 25% combined state and Federal tax bracket) competitor, offers to purchase the patent from Create for $1,050,000. Knowing that Create’s NOL should be useful to Mega, Tony suggests a restructuring where he receives $800,000 in Mega stock, Mega assumes all of Create’s liabilities ($250,000) plus $75,000 cash for the NOL. Mega counter offers with cash for the NOL (to be determined), and $1,050,000 of stock, but it will not assume any liabilities. How much would be the maximum cash offered by Mega for the NOL, assuming that Mega uses a 12% discount factor and the Federal long-term tax-exempt rate is 4%? If Tony accepts Mega’s offer, what type of reorganization, if any, is this restructuring? 112. Tin Corporation was created 10 years ago. It currently is valued at $1.5 million as follows: Tacks division ($420,000), Safety Pins division ($580,000), Paper Clips division ($450,000), and investment assets ($50,000). Tin currently has three shareholders: Antonio, who was the initial shareholder and now owns 40% of the stock (basis in stock $350,000), and Beth and Chang, each of whom purchased 30% of Tin two years ago for $435,000. Tin is having management problems: the shareholders cannot agree on the future of the company. They have determined that it would be best to divide up the company and go their separate ways. Each shareholder feels that the others do not deserve to continue using the Tin Corporation name. a.

Determine what would be the best method to divide the corporation among the shareholders with the least amount of taxes.

b.

Draft a detailed diagram of the solution you suggested in part a.

c.

Any investments that should be received by the new entities will be distributed to the shareholders in exchange for their Tin stock. Beth is the only shareholder who has indicated that she prefers to receive some investment assets. Determine the gain or loss each entity and shareholder will have upon the division.

113. Gera owns 25,000 shares of Flow Corporation’s common stock, for which she paid $250,000. The other 5,000 shares belong to Gera’s brother, Earl, which he purchased for $50,000. Wanting to expand a few years ago, Flow sold $200,000 in bonds to Earl. The expansion has paid off, and Flow now can afford to redeem 50% of Earl’s bonds. Rather than have the bond redeemed, Earl would prefer to receive 100 shares of preferred stock for the $100,000 in bonds. At the same time, Gera would like to own preferred stock, so she turns in 5,000 shares of her common stock in exchange for 100 shares of preferred stock and 5,000 shares of her common for a $100,000 bond. Gera and Earl then each will hold 100 shares of preferred and $100,000 in bonds. Gera still owns 15,000 shares of common, and Earl owns 5,000 shares of common. The common stock is valued at $20 per share the day before the preferred is issued. The preferred shares are valued at $1,000 each. State the type of reorganization, if any, for which these transactions qualify. What is the amount of gain or loss that Gera and Earl recognize on these transactions? 114. Present Value Tables needed for this question. Sugar Corporation would like to acquire Salt Corporation on Powered by Cognero

Page 16


Name:

Class:

Date:

Chapter 07: Corporations Reorganizations January 1 of the current year in a tax-free reorganization. Salt is particularly appealing to Sugar because Salt has a $350,000 capital loss that can carry over for five years. Sugar expects large capital gains for the next several years in addition to its expected $2.5 million net income. At the time of the restructuring, Salt has assets valued at $2 million (basis of $1.4 million). Sugar is proposing exchanging 45% of its stock for all of Salt’s assets. The Federal long-term tax-exempt rate is 2.5% and Sugar’s discount rate for investment decisions is currently 7%. What is the maximum present value of the capital loss to Sugar assuming it has a combined state and Federal tax rate of 25%? 115. On March 1, Cream Corporation transfers all of its assets to Coffee Corporation in exchange for 10% of its common voting stock. At the time of the reorganization, Cream has assets valued at $4 million (basis of $3 million), and its earnings and profits account shows a deficit of $650,000. Coffee’s earnings and profits as of March 1 were $420,000. Due to the reorganization, Coffee incurs an NOL for the current year of $150,000. Coffee still declares its usual dividends of $100,000 paid on April 30, August 31, and December 31 ($300,000 of total dividends). How are the Coffee shareholders taxed on the distribution? 116. Sauce Corporation is very interested in acquiring a controlling interest in Pear Corporation to obtain operating efficiencies. Sauce currently owns 30% of Pear, which it bought six years ago for $600,000. Sauce is a fruit processor with assets valued at $3 million and liabilities of $1 million. Pear supplies Sauce with fruit from its orchards that are valued at $4 million with $3 million in mortgages. Sauce has negotiated a restructuring with most of Pear’s shareholders. It will exchange one share of its stock for two shares of Pear. Pear’s founder, who own 10% of the outstanding common stock, is not willing to relinquish her stock; thus, Sauce cannot own 100% of Pear. What type of reorganization is being used here?

Essay 117. Explain whether shareholders are exempted from gain/loss recognition in nontaxable corporate reorganization or the gain/loss recognition is merely postponed. If it is postponed, how does the Federal income tax law ensure that the postponed gain/loss will be recognized in the future? 118. What will cause the corporations involved in a § 368 reorganization to recognize gain or loss? What will cause shareholders of the companies involved in the corporate reorganization to recognize gain or loss? If gain is recognized by shareholders, how might it be taxed? 119. In each of the following reorganizations, there is an exchange of stock for assets or stock for stock. Indicate for each reorganization the type of stock used for the exchanges, and the minimum percentage of stock that may be used for the restructurings to meet the § 368 requirements. Types: A; B; C; divisive D. 120. Compare the consideration that can be used in “Type A,” “Type B,” and “Type C” reorganizations. 121. Discuss the treatment of accumulated earnings and profits (E & P) in a corporate reorganization when both corporations have positive E & P. When the target corporation holds a negative E & P. 122. Briefly describe three of the Federal judicial doctrines that may apply to tax-free corporate reorganizations. Powered by Cognero

Page 17


Name:

Class:

Date:

Chapter 07: Corporations Reorganizations 123. Provide the formula for the § 382 limitation and demonstrate how the formula is used in the year of the takeover. What is the purpose of the § 382 limitation? 124. Both § 382 and “Type G” reorganizations allow beneficial tax attributes to carry over from the loss corporations to the successor. Compare the list of attributes are carried over for “Type G” reorganizations and for § 382, and the order of reduction/use of these attributes.

Powered by Cognero

Page 18


Name:

Class:

Date:

Chapter 07: Corporations Reorganizations Answer Key 1. True 2. False 3. False 4. True 5. True 6. False 7. True 8. False 9. False 10. True 11. False 12. True 13. True 14. False 15. False 16. True 17. True 18. True 19. False 20. True 21. False 22. False 23. False 24. False 25. True Powered by Cognero

Page 19


Name:

Class:

Date:

Chapter 07: Corporations Reorganizations 26. True 27. False 28. False 29. True 30. True 31. False 32. b 33. c 34. c 35. d 36. d 37. a 38. c 39. b 40. a 41. d 42. d 43. a 44. b 45. b 46. c 47. b 48. c 49. a 50. b Powered by Cognero

Page 20


Name:

Class:

Date:

Chapter 07: Corporations Reorganizations 51. e 52. d 53. b 54. d 55. c 56. d 57. c 58. d 59. a 60. c 61. d 62. a 63. a 64. d 65. a 66. c 67. d 68. b 69. b 70. c 71. boot, recognized, not recognized 72. Boot, dividends, capital gain 73. Type A, 100 74. boot, parent-subsidiary 75. 90, 70 76. Type E Powered by Cognero

Page 21


Name:

Class:

Date:

Chapter 07: Corporations Reorganizations 77. F 78. G, insolvent 79. cancellation-of-debt-income COD income 80. continuity of business enterprise, continuity of interest 81. divisive "D", five 82. step transaction, sound business purpose 83. Type C, acquiring 84. Section 382 limitation § 382 limitation 85. fair market value, Federal long-term tax-exempt rate, initial (first) year 86. capital loss, built-in ordinary loss, NOL 87. h 88. l 89. n 90. b 91. i 92. o 93. c 94. a 95. e 96. j 97. k 98. c 99. f 100. b Powered by Cognero

Page 22


Name:

Class:

Date:

Chapter 07: Corporations Reorganizations 101. d 102. j 103. a 104. i 105. k 106. d 107. Value of Cotinga stock received: Jerrika $312,000; Allen $338,000. Jerrika has $24,000 recognized gain; Allen cannot recognize his loss. Basis in Cotinga stock: Jerrika $270,000; Allen $354,000. Total Cotinga stock exchanged $650,000 ($800,000 assets – $100,000 liabilities – $50,000). Jerrika receives $312,000 in Cotinga stock ($650,000 × 48%) and $24,000 cash for a total of $336,000. Allen receives $338,000 in Cotinga stock ($650,000 × 52%) and $26,000 cash for a total of $364,000. The gain recognized for Jerrika and Allen on the exchange and their basis in the Cotinga stock is computed as follows.

Jerrika

Allen

Realized Gain/Loss $ 336,000 (270,000) $ 66,000

Recognized Gain/Loss $24,000

Postponed Gain/Loss $ 66,000 (24,000) $ 42,000

Adjusted Basis in Cotinga Stock $312,000 (42,000) $270,000

$ 364,000 (380,000) ($ 16,000)

$

–0–

($16,000) (–0–) ($16,000)

$338,000 + 16,000 $354,000

108. Acquiring recognizes $150,000 gain on land; Target cannot recognize its loss on the assets; and Tia recognizes a $350,000 gain. Tia’s basis in her stock $500,000. Acquiring’s gain on the land is $400,000 value – $250,000 basis. While Target realizes a $20,000 loss on the assets ($100,000 value – $120,000 basis), loss recognition is disallowed. Tia receives $1 million for her Target stock ($500,000 Acquiring stock + $400,000 land + $100,000 assets), and her basis in the stock is $650,000. The gain recognized by Tia is caused by the boot received ($400,000 land + $100,000 assets). Since Tia’s recognized gain cannot be greater than her realized gain, her gain is limited to $350,000. Thus, Tia’s stock basis is $500,000. Gain and basis amounts are computed as follows.

Tia

Realized

Recognized

Postponed

Gain $1,000,000

Gain $350,000

Gain $350,000

Powered by Cognero

Adjusted Basis in Acquiring Stock $500,000 Page 23


Name:

Class:

Date:

Chapter 07: Corporations Reorganizations (650,000) $ 350,000

(350,000) $ –0–

(–0–) $500,000

109. Bulbul is worth $300,000 ($500,000 – $200,000). Betty receives $120,000 (40%) of this amount;

Keith receives $180,000.

Betty’s and Keith’s gains and stock basis are computed as follows.

Betty

Keith

Realized Gain $120,000 (60,000) $ 60,000

Recognized Gain $32,000

Postponed Gain $60,000 (32,000) $28,000

Adjusted Basis in Dipper Stock $ 88,000 (28,000) $ 60,000

$180,000 (90,000) $ 90,000

$48,000

$90,000 (48,000) $42,000

$132,000 (42,000) $ 90,000

110. a. The acquisition could be set up as a “Type B” reorganization. Pipe can acquire the 70% of the outstanding common stock by exchanging Pipe voting stock for Ore stock. Since Ore will remain a separate corporation, Pipe can protect its assets from potential environmental lawsuits. With the previously acquired 30% of Ore stock, Pipe will now own 100% of the stock and therefore meets the control requirement for a “Type B” reorganization. The fact that the previously owned 30% was acquired with cash does not violate the solely voting stock consideration requirement. b.

Powered by Cognero

Page 24


Name:

Class:

Date:

Chapter 07: Corporations Reorganizations

111. Mega should not pay more than $59,327 for the NOL. Use a “Type C” reorganization. Since Create would experience a more than 50 percentage-point change in ownership, the § 382 limitation would apply. Mega would acquire assets from Create with a value of $1,050,000. With a 4% Federal long-term tax-exempt rate, the § 382 limitation is $42,000 ($1,050,000 × 4%) and the tax benefit of this deduction would be $10,500 ($42,000 × 25%). It would take Mega 10 years to fully utilize the NOL ($420,000 ÷ $42,000). Using an 12% discount rate for 10 years (5.6502), the $10,500 tax savings for 10 years is valued at $59,327 ($10,500 × 5.6502). The merger proposed by Mega would meet the requirement of a “Type C” reorganization. With a “Type C,” Mega can acquire none of Create’s liabilities. Since the only cash is the $59,327, 95% of the assets are acquired with Mega stock. With a “Type A” merger, all liabilities would need to be assumed by Mega. If the acquisition proceeded as Tony suggested, the transaction would not meet the “Type C” requirements of having 80% of the consideration consist of Mega stock. The $75,000 cash will cause the $250,000 liabilities to be treated as boot. Therefore, $325,000 ($75,000 cash + $250,000 liabilities) of the $1,125,000 total consideration ($800,000 stock + $75,000 cash + $250,000 liabilities) is not stock. This is 29% of the total amount. Thus, stock used would be less than 80% and therefore not meet the “Type C” requirements. Tony should accept Mega’s offer of $1,050,000 in stock and $59,327 as this would meet the requirements of a "Type C" reorganization. 112. a. A divisive “Type D” split-up reorganization will meet the needs of the shareholders with the least amount of taxes. The split-up will ensure that none of the shareholders are able to use the Tin Corporation name in the future, because Tin is required to cease to exist after the reorganization. b.

Powered by Cognero

Page 25


Name:

Class:

Date:

Chapter 07: Corporations Reorganizations

c.

Since Antonio owns 40% of the Tin stock, he should receive 40% of the value of Tin, or $600,000 ($1.5 million × 40%). The Safety Pins division assets ($580,000) should be transferred to the Safety Pin Corporation in exchange for its stock. This Safety Pin stock plus $20,000 of investments will be exchanged for Antonio’s Tin stock. The $20,000 of investments will cause Antonio to recognize a $20,000 gain and his basis in the Safety Pin stock is $350,000 ($600,000 – $350,000 = $250,000 realized gain; recognized $20,000; postponed $230,000; and $580,000 – $230,000 = $350,000 basis in Safety Pin stock). Beth owns 30% of the Tin stock and should receive 30% of the value of Tin, or $450,000 ($1.5 million × 30%). The Tacks division assets ($420,000) should be transferred to the Tacks Corporation in exchange for its stock. The Tacks stock plus $30,000 of investments should be exchanged for Beth’s Tin stock. The $30,000 will cause Beth to recognize a $15,000 gain, and her basis in the Tacks stock is $420,000 ($450,000 – $435,000 = $15,000 realized gain; recognized $15,000; postponed $0; and $420,000 basis in Tacks stock) Chang also owns 30% of the Tin stock and thus also should receive 30% of the value of Tin, or $450,000 ($1.5 million × 30%). The Paper Clips division assets ($450,000) should be transferred to the Paper Clips Corporation in exchange for its stock. The Paper Clips stock will be exchanged for Chang’s Tin stock. Since Chang received no boot, he recognizes no gain and therefore has a carryover basis of $435,000 in the Paper Clip stock.

113. The exchange of Earl’s bond for preferred stock qualifies as a “Type E” reorganization. The exchange of Gera’s common stock for preferred stock is also a “Type E” reorganization, but her exchange of common stock for a bond is a taxable transaction. A “Type E” reorganization does not include an exchange of stock for bonds in its nontaxable treatment. Since the preferred stock received by Earl is valued at $100,000 and the bond he is exchanging is $100,000, he has no realized or recognized gain. His basis in his preferred stock is $100,000. Gera receives a $100,000 bond for 5,000 shares having a basis of $50,000 [($250,000 basis/25,000 shares) × 5,000] and recognizes a gain of $50,000 ($100,000 – $50,000). Gera recognizes no gain on the exchange of her 5,000 common shares (value of $100,000) for 100 shares preferred (value of $100,000).

114. Since there is more than a 50 percentage-point ownership change in Salt, the capital loss carryforward will be subject Powered by Cognero

Page 26


Name:

Class:

Date:

Chapter 07: Corporations Reorganizations to the § 382 limitation. The maximum period for a capital loss carryforward is five years. The amount of capital loss that Sugar can use to offset its future income is $50,000 per year ($2 million × 2.5%), and the tax benefit of this is $12,500 ($50,000 × 25%). . Therefore, Sugar can use only $250,000 of the capital losses in the next five years ($50,000 × 5). Using a 7% discount factor for five years, the present value of the usable capital loss carryforward is $51,253 (12,500 × 4.1002). 115. The $300,000 distributions are taxed to the shareholders as dividends to the extent of $270,000; $30,000 is a return of capital. As of the date of the merger, Coffee maintains two separate earnings and profits accounts: its own original with a positive balance of $420,000 and Cream’s with a deficit of $650,000. The NOL for the year reduces Coffee’s original earnings and profits balance to $270,000 ($420,000 – $150,000). Since Coffee holds an earnings and profits account with a positive balance, the distribution is treated as a dividend to the Coffee shareholders until the earnings and profits are exhausted; then $30,000 of the distribution in December is a return of capital.

116. The acquisition could be set up as a “Type B” reorganization. Sauce can acquire the 60% of the outstanding Pear common stock it does not own either directly from the Pear shareholders or through Pear by exchanging Sauce voting stock for Pear stock. With the 30% previously acquired, Sauce will now own 90% of the Pear stock and therefore meet the 80% control requirement for a “Type B” reorganization. The fact that the previously owned 30% was acquired with cash does not violate the solely voting stock consideration requirement.

117. In reorganizations, neither gain nor loss is recognized in the year of the deal by the shareholders as long as the corporations meet the legislative and judicial requirements for nontaxable treatment. However, this does not mean that the shareholders are exempted from taxation. Rather, the recognition of the gain/loss is merely postponed until there is a taxable transaction. The vehicle for postponing gain/loss that is not recognized is the basis in the new stock received. If no gain or loss is recognized, the basis in the new stock is carried over from the basis in the old stock. If gain is recognized because the shareholder received “boot,” the basis in the new stock is adjusted to account for the remaining gain realized but not recognized (postponed gain). The four-column template of Concept Summary 7.1 is useful for determining the consequences of a corporate reorganization. 118. Corporations involved in § 368 reorganizations are not permitted to recognize losses. The acquiring corporation can recognize gain if it transfers appreciated property (boot) along with its stock to the target. The target will recognize gain if it fails to distribute the boot to its shareholders. The target also can recognize gain if it distributes its own appreciated property to its shareholders. Shareholders recognize gains when they receive boot (nonstock property) in exchange for their stock in their corporation. Shareholders can recognize losses if they only receive boot and no stock. Shareholder recognized gain in a corporate reorganization may have the following tax characteristics. ∙ Dividend to the extent of the shareholder’s proportionate share of corporate earnings and profits Powered by Cognero

Page 27


Name:

Class:

Date:

Chapter 07: Corporations Reorganizations

(E & P). The remaining gain is capital gain. If the requirements of § 302(b) can be met, the transaction will qualify for stock redemption treatment, usually as a capital gain.

119. The stock requirements for § 368 reorganizations are as follows. Type A: Acquiring may use common or preferred, voting or nonvoting in the exchange for target’s assets. However, the continuity of interest doctrine requires at least 40% be common stock. Type B: Acquiring must exchange voting stock for common or preferred, voting or nonvoting stock of the target. After the reorganization, the acquiring must own at least 80% of the target’s stock. The acquiring corporation’s target stock owned before the reorganization can be counted toward the 80% control requirement. Type C: Acquiring must use voting stock in the exchange for target’s assets. At least 80% of the assets must be acquired with voting stock. Type D Divisive: The original corporation must receive at least 80% of the target’s stock (80% of the total combined voting power and 80% of all other classes of stock).

120. With a “Type A” reorganization, the consideration can be stock and other property. There is no requirement that the stock be voting. However, to meet the continuity of interest doctrine, at least 40% of the consideration should be stock. A “Type B” reorganization requires that all of the consideration used by the acquiring corporation be voting stock. If other than voting stock is used, the “Type B” will not be tax-free. Voting stock must be 80% of the consideration given by the acquiring corporation in a “Type C” reorganization. If cash or other property is also given, the liabilities will be considered boot. This can cause the “Type C” to lose its tax-free status. If only stock is used, the liabilities are not considered boot. 121. The accumulated earnings and profits (E & P) of corporations that are parties to a reorganization are added together when both corporations have positive E & P. Thus, the acquiring corporation’s E & P is increased by the target’s E & P. If the target corporation, however, has a negative E & P, the accounts are not commingled. The negative E & P is treated as received by the acquiring corporation as of the reorganization date. The negative E & P may offset only the future earnings of the successor corporation. Thus, the successor corporation must maintain two separate E & P accounts; one for prior E & P and another for the deficit E & P transferred to the corporation and subsequent earnings. 122. Even if the statutory reorganization requirements are literally followed, restructurings will not be treated as tax-free unless they also meet the following judicial doctrines. Step transaction. This doctrine prevents taxpayers from engaging in a series of transactions for the purpose of obtaining tax benefits that would not be allowed if the transaction was accomplished in a single step. When the steps are so interdependent that the accomplishment of one step would be fruitless without the completion of the series of steps, the transactions may be collapsed into a single step. Sound Business Purpose. Tax-free treatment of a restructuring is only available to those transactions that are motivated by some valid business needs of the corporations. The economic benefits of the reorganization must go beyond mere tax avoidance. Thus, a reorganization to acquire tax attributes of another corporation, such as an NOL or business credits, does not have a sound business purpose and will not receive tax-free treatment. Continuity of Business Enterprise. To ensure that the tax-free treatment is limited to transactions that are mere changes in the form and not the substance of the business, this test requires that there be a continuation of the target’s business. Specifically, this test requires the acquiring corporation to either (1) continue the target corporation’s historic business (historic business test) or (2) use a significant portion of the target corporation’s assets in its business (asset use test). Powered by Cognero

Page 28


Name:

Class:

Date:

Chapter 07: Corporations Reorganizations Continuity of Interest. To distinguish between a sale of assets and a reorganization, the continuity of interest doctrine requires that the shareholders of the target corporation have a continuing investment in the succeeding entity after the restructuring. To qualify for tax-free treatment, the target corporation shareholders must receive acquiring corporation stock equal to at least 40% of their prior stock ownership in the target. 123. The formula for determining the § 382 limitation is as follows. Loss Corporation’s FMV on the change dated × Federal long-term tax-exempt rate = Yearly § 382 limitation × Number of days remaining in change year/number of days in year (365 or 366) = Change year § 382 limitation The purpose of the § 382 limitation is to discourage the acquisition of a corporation merely for its loss carryovers or unused tax credits. The § 382 limitation restricts the use of loss tax attributes to a hypothetical future income stream that would have been received if the stock were sold and the proceeds invested in long-term tax-exempt securities. 124. The order of reduction in tax attributes in a “Type G” reorganization is as follows. 1. NOLs 2. Capital losses 3. Basis in property The order for application of the § 382 limitation is as follows. 1. Built-in capital losses 2. Capital losses 3. Built-in ordinary losses 4. NOLs Thus, not only is the order different, but also the tax attributes themselves are different. For the “Type G,” NOLs are reduced before capital losses, whereas with § 382, capital losses are before NOLs. The “Type G” includes a reduction in basis that, at the election of the acquiring corporation, can be reduced before the other attributes. Having a basis reduction does not make sense for the § 382 limitation.

Powered by Cognero

Page 29


Name:

Class:

Date:

Chapter 08: Consolidated Tax Returns True / False 1. A consolidated Federal income tax return may be the product of a merger of the affiliates or another corporate combination. a. True b. False 2. Business reasons, and not tax incentives, constitute the primary motivation for most corporations to form a conglomerate and file tax and financial accounting reports on a consolidated basis. a. True b. False 3. Most of the Federal consolidated income tax return rules are found in detailed sections of the tax Regulations. a. True b. False 4. The rules for computing Federal consolidated taxable income are some of the most complex in the tax law. a. True b. False 5. For consolidated tax return purposes, purchased goodwill is amortized as a deduction to taxable income over 15 years. Under financial accounting rules, 40-year amortization is allowed. a. True b. False 6. A limited partnership can join the parent’s consolidated group for book and for tax purposes. a. True b. False 7. After a takeover, the parent’s balance sheet shows a fair market value basis in the subsidiary for both book and tax purposes. a. True b. False 8. When the parent acquires 51% of a subsidiary U.S. corporation, the subsidiary can join the consolidated financial statements and the consolidated tax return of the parent. a. True b. False 9. The right to file on a consolidated basis is available to a group of corporations when they constitute a “parent-subsidiary affiliated group.” a. True b. False 10. A Federal consolidated group can claim a dividends received deduction for payments that the parent receives from other affiliates. a. True Powered by Cognero

Page 1


Name:

Class:

Date:

Chapter 08: Consolidated Tax Returns b. False 11. Giant Ltd. owns 100% of the stock of Middle Corporation. Bottom Corp is owned 60% by Giant and 40% by Middle. Giant’s Federal consolidated income tax return includes both Middle and Bottom. a. True b. False 12. A subsidiary corporation must leave the consolidated group if it is restructured as an LLC. a. True b. False 13. A corporation organized in Germany and wholly owned by the U.S. parent can be included in a Federal consolidated return. a. True b. False 14. A tax-exempt charitable trust created by a U.S. C corporation can join in a Federal consolidated return. a. True b. False 15. A joint venture subject to income tax laws like a partnership can join in a consolidated Federal income tax return. a. True b. False 16. A calendar year parent corporation wants to file its tax returns on a consolidated basis with its affiliates. The group’s election to file consolidated Federal corporate income tax returns must be made by the extended due date of the first return on which the consolidation is applied (i.e., October 15). a. True b. False 17. Campbell Corporation left the Crane consolidated tax return group after the calendar 2020 tax year. Generally, Crane can add Campbell back to the consolidated group, but no earlier than for the 2026 tax year. a. True b. False 18. The calendar year Sterling Group files its Federal corporate income tax return on a consolidated basis. The group’s Form 1120 is due on April 15, or October 15 if the group files an extension request with the IRS. a. True b. False 19. All affiliates joining in a newly formed consolidated return must consent to the election on Form 1122, as attached to the Form 1120 for the group. a. True b. False 20. Consolidated group members each are jointly and severally liable for the entire consolidated Federal income tax Powered by Cognero

Page 2


Name:

Class:

Date:

Chapter 08: Consolidated Tax Returns liability. a. True b. False 21. With the filing of its first consolidated return, the parent corporation of a Federal consolidated group makes an irrevocable election as to how the group will allocate a tax year’s income tax liability among the group members. a. True b. False 22. A Federal consolidated tax return group can apply the “relative taxable income” method as a means to apportion the tax liabilities of the members among the affiliates. a. True b. False 23. Each member of a consolidated group must use the same tax year-end, and all of the members must use the same tax accounting methods (e.g., LIFO or FIFO). a. True b. False 24. When the net accumulated taxable losses of a subsidiary exceed the parent’s acquisition price, the parent’s basis in the subsidiary’s stock becomes negative. a. True b. False 25. If subsidiary stock is redeemed or sold outside the group when an excess loss account exists, the selling parent corporation recognizes additional gross income equal to the account balance. a. True b. False 26. In computing consolidated E & P, dividends paid to the parent by group members are subtracted. a. True b. False 27. In computing consolidated taxable income, compensation amounts are removed from the taxable incomes of the group members and determined on a group basis. a. True b. False 28. In computing consolidated taxable income, capital gains and losses are removed from the taxable incomes of the group members and determined on a group basis. a. True b. False 29. The starting point in computing consolidated taxable income are the separate Federal taxable income amounts of the affiliated group members. a. True Powered by Cognero

Page 3


Name:

Class:

Date:

Chapter 08: Consolidated Tax Returns b. False 30. An example of an intercompany transaction is the use of the trademarks of the parent corporation by a subsidiary, compensated by an arm’s length licensing fee. a. True b. False 31. When a subsidiary sells to the parent some business-use property that has appreciated from its $20,000 basis to a $50,000 fair market value, the subsidiary immediately recognizes $30,000 ordinary income on the consolidated return. a. True b. False 32. When a consolidated NOL is generated, each affiliate is allocated a share of the loss. a. True b. False 33. Achebe Corporation joined an affiliated group by merger in 2017. The group generated a 2024 consolidated NOL, and Achebe’s share of the loss was $50,000. Achebe’s share of the loss is included in the group’s NOL carryforward to 2025. a. True b. False 34. When a member departs from a consolidated group, it leaves behind any NOLs that it generated while in the group. The parent corporation and remaining affiliates apply those NOLs against future consolidated taxable income. a. True b. False 35. Cooper Corporation joined the Duck consolidated Federal income tax return group when Cooper held a $1 million NOL carryforward. In its first year as a part of the Duck group, Cooper generated a $150,000 operating profit. For that year, Duck can deduct only $150,000 of Cooper’s NOL in computing consolidated taxable income. a. True b. False 36. Cooper Corporation joined the Duck consolidated Federal income tax return group when Cooper held a $1 million NOL carryforward. In its first year as a part of the Duck group, Cooper generated a $150,000 taxable loss. For that year, Duck cannot deduct any of Cooper’s NOL in computing consolidated taxable income. a. True b. False 37. A Federal consolidated filing group aggregates its separate charitable contributions for the tax year, deductions for which are then limited to 10% of consolidated taxable income. a. True b. False Multiple Choice 38. Which of the following potentially is a disadvantage of electing to file a Federal consolidated corporate income tax return? Powered by Cognero

Page 4


Name:

Class:

Date:

Chapter 08: Consolidated Tax Returns a. The § 1231 loss of one member is not offset against the § 1231 gain of another member of the group. b. Recognition of losses from certain intercompany transactions is deferred. c. The tax basis of investments in the stock of subsidiaries is unaffected by members contributing to consolidated taxable income. d. All of the choices are disadvantages of a consolidation election. 39. Which of the following is not generally a disadvantage of filing Federal corporate income tax returns on a consolidated basis? a. Net capital losses from one affiliate can offset the capital gains from another. This can reduce the tax liabilities of the group as a whole. b. Realized losses from transactions between affiliates are not recognized immediately. c. Compliance costs usually are higher when a consolidation election is in effect. d. The election generally is binding for future tax years. 40. Which of the following potentially is a disadvantage of electing to file a Federal corporate income tax consolidated return? a. Increased deduction amounts when computations are made on a group basis. b. Deferral of gains realized in transactions between group members. c. Increased basis in the stock of a subsidiary that generates annual taxable income. d. Additional administrative costs in complying with the election. 41. Which of the following is not a requirement that must be met before a group files a consolidated return? a. None of the corporations can be ineligible under the Code to file on a consolidated basis with the others. b. All of the corporations must be members of an affiliated group. c. The group members must share the same inventory accounting method (e.g., they all must use FIFO). d. The group members must share a common tax year end. 42. Under the consolidated return rules, the realized gain from an intercompany transaction is deferred. For most taxpayers, this produces: a. An advantage in terms of the time value of money. b. A disadvantage in terms of the time value of money. c. A compliance issue that cannot be resolved. d. A 20% penalty on the consolidated group. 43. Which of the following entities is eligible to file Federal income tax returns on a consolidated basis? a. A U.S. C corporation that files on a separate basis for its state income tax returns. b. The charitable foundation of a U.S. C corporation. c. The liquidating trust of a U.S. C corporation. d. A wholly owned French subsidiary of a U.S. C corporation. 44. Which of the following entities is eligible to join in a Federal consolidated return? a. A sole proprietor with annual sales of more than $50 million. b. A U.S. corporation’s § 401(k) plan. c. A partnership organized in Germany. d. A corporation that operates in seven different U.S. states. Powered by Cognero

Page 5


Name:

Class:

Date:

Chapter 08: Consolidated Tax Returns 45. Which of the following entities is eligible to file Federal income tax returns on a consolidated basis? a. Professional sports team operating as a limited partnership. b. Japanese corporation engaged in multinational operations, including two-thirds of its activities in the United States. c. Japanese corporation engaged in multinational operations, including one-third of its activities in the United States. d. U.S. corporation engaged in the marijuana industry. 46. The Rack, Spill, and Ton corporations file Federal income tax returns on a consolidated basis. The group’s tax return currently is under audit. Under a valid tax-sharing agreement, each corporation is liable for one-third of the group’s consolidated tax liability. The affiliates have agreed with the auditor that the group’s unpaid tax liability for the year is $90,000. Because of an incorrect tax return position, another $3,000 in interest and an $18,000 penalty is attributable solely to Ton. At present, only Rack is solvent and has the cash with which to make such a tax payment. What is the maximum amount for which the government could be successful in forcing Rack to satisfy the outstanding liabilities of the consolidated group? a. $0 b. $90,000 c. $93,000 d. $108,000 e. $111,000 47. Conformity among the members of a consolidated group must be implemented for which of the following tax items? a. Use of foreign tax payments (i.e., as a credit or deduction). b. Tax accounting method (i.e., cash or accrual). c. Inventory accounting method (e.g., FIFO or dollar-cost averaging). d. Tax year-end. 48. How are the members of a Federal consolidated group affected by computations related to E & P? a. Each member keeps its own E & P account. b. E & P is computed solely on a consolidated basis. c. Members’ separate E & P balances are frozen as long as the consolidation election is in place. d. Consolidated E & P is computed as the sum of the E & P balances of each of the group members, computed on the last day of the tax year. 49. Azure Corporation joins the Colorful Corporation Federal consolidated return group. As a result: a. Azure continues to file its own Form 1120. b. The existing Colorful consolidation election is terminated. c. Azure’s tax results immediately are added to the Colorful group’s consolidated Form 1120. d. Azure’s tax results first are added to the Colorful group’s consolidated Form 1120 for the next tax year. 50. Azure Corporation leaves the Colorful Corporation Federal consolidated return group on the last day of year 1. As a result: a. Azure’s tax results are included in the group’s consolidated Form 1120 for the final time for year 2. Powered by Cognero

Page 6


Name:

Class:

Date:

Chapter 08: Consolidated Tax Returns b. The existing Colorful consolidation election is terminated. c. Azure’s leaving the group is effective only when IRS permission to do so is granted. d. Azure files its own Form 1120 beginning with year 2. 51. Calendar year ParentCo acquired all of the stock of SubCo on January 1, year 1, for $1,000,000. The parties immediately elected to file consolidated income tax returns. SubCo generated taxable income of $250,000 for year 1 and paid a dividend of $100,000 to ParentCo. In year 2, SubCo generated an operating loss of $350,000, and in year 3, it produced taxable income of $750,000. As of the last day of year 3, what was ParentCo’s basis in the stock of SubCo? a. $1,650,000 b. $1,550,000 c. $1,000,000 d. $0 52. Calendar year ParentCo purchased all of the stock of SubCo on January 1, year 1, for $500,000. In year 1, SubCo produced a loss of $150,000 and distributed cash of $25,000 to ParentCo. In year 2, SubCo generated a loss of $450,000; in year 3, it recognized net income of $90,000. What is ParentCo’s capital gain or (loss) if it sells all of its SubCo stock to a nongroup member on year 4 for $150,000? a. $185,000 b. $150,000 c. ($35,000) d. ($535,000) e. All gain/loss is ordinary when subsidiary stock is sold. 53. In computing a group’s consolidated taxable income, a subsidiary’s net capital gain is: a. Removed from group taxable income. b. Used to compute the group’s net capital gain/loss. c. Both a. and b. occur. d. Neither a. nor b. occurs. 54. In computing a group’s consolidated taxable income, the first step is to: a. Compute the taxable income amounts for each affiliate on a standalone basis. b. Obtain IRS permission to file on a consolidated basis for another tax year. c. Eliminate the results of all intercompany transactions for the tax year. d. Compute the allowable group NOL carryforward. 55. ParentCo owned 100% of SubCo for the entire year, and both companies use the accrual method of tax accounting. During the year, SubCo provided internal audit services to ParentCo, which were worth $40,000. Including these transactions, ParentCo’s separate taxable income was $75,000, and SubCo’s separate taxable income was $100,000. What is the group’s consolidated taxable income for the year? a. $215,000 b. $195,000 c. $175,000 d. $155,000 56. ParentCo owned 100% of SubCo for the entire year. ParentCo uses the accrual method of tax accounting; SubCo uses the cash method. During the year, SubCo sold raw materials to ParentCo for $35,000 under a contract that requires no Powered by Cognero

Page 7


Name:

Class:

Date:

Chapter 08: Consolidated Tax Returns payment to SubCo until the following year. Exclusive of this transaction, ParentCo had income for the year of $30,000, and SubCo had income of $50,000. The group’s consolidated taxable income for the year was: a. $165,000. b. $150,000. c. $115,000. d. $80,000. 57. ParentCo and SubCo have filed consolidated returns as both entities were incorporated in year 1. Taxable income computations for the members include the following. Neither group member incurred any capital gain or loss transactions during these years, nor did they make any charitable contributions. All tax years occur after 2020. ParentCo’s SubCo’s Taxable Consolidated Tax Year Taxable Income Income Taxable Income Year 1 $100,000 $ 35,000 $135,000 Year 2 $100,000 ($ 20,000) $ 80,000 Year 3 $100,000 ($109,000) ? Year 4 $100,000 $190,000 ? The year 3 consolidated loss: a. Is carried forward to year 4. b. Is carried back to year 1 and produces a tax refund. c. Can be used only to offset SubCo’s future income. d. Offsets ParentCo’s year 1 income. 58. The consolidated net operating loss of the Parent Group includes all of the following except the: a. Parent’s operating income/loss. b. Parent’s charitable contributions. c. Parent’s dividends received deduction. d. Subsidiary’s operating income/loss. 59. A Federal consolidated group reported the following taxable income amounts. Parent owns all of the stock of both Junior and Minor. Determine the net operating loss (NOL) that is apportioned to Parent. Parent Junior Minor

($400,000) ($600,000) $100,000

a. $360,000. b. $400,000. c. $500,000. d. $900,000. All NOLs of a consolidated group are apportioned to the parent. 60. Which of the following items is not computed on a group basis for a Federal consolidated income tax return? a. Dividends received deduction. b. Cost recovery deduction. c. Charitable contributions. d. Net capital losses. Powered by Cognero

Page 8


Name:

Class:

Date:

Chapter 08: Consolidated Tax Returns 61. Which of the following statements is true with regard to intercompany transactions? a. An intercompany transaction is eliminated from consolidated taxable income. b. All intercompany gains are recognized, but losses must be deferred. c. A cash sale of a business asset by the purchasing member to an acquirer outside of the group triggers immediate recognition of the gain or loss. d. The gain or loss on an intercompany transaction is deferred for up to 10 years after which it is recognized. 62. Subsidiary holds an allocated net operating loss (NOL) when it leaves the Parent consolidated group. As a result: a. The group keeps Subsidiary’s allocated loss. b. Subsidiary takes its allocated NOL and uses it on subsequent separate tax returns. c. The loss is suspended for five years in case Subsidiary rejoins the group; then Parent can use it. d. The loss is suspended for five years in case Subsidiary rejoins the group. At that time, Subsidiary uses the loss on its separate return. 63. ParentCo purchased all of the stock of ChildCo on January 2, year 2, and the two companies filed consolidated returns for year 2 and thereafter. Both entities were incorporated in year 1. Taxable income computations for the members include the following. Neither group member incurred any capital gain or loss transactions during these years, nor did they make any charitable contributions. No § 382 limit applies. All tax years occur after 2020.

Tax Year Year 1 Year 2 Year 3 Year 4

ParentCo’s Taxable Income $100,000 $100,000 $100,000 $100,000

ChildCo’s Taxable Income ($ 75,000) ($ 40,000) $ 10,000 $125,000

Consolidated Taxable Income N/A $60,000 ? ?

To what extent can ChildCo’s year 1 losses be used by the group in year 4? a. $135,000 b. $125,000 c. $75,000 d. $10,000 e. $0 64. ParentCo purchased all of the stock of ChildCo on January 2, year 2, and the two companies filed consolidated returns for that year and thereafter. Both entities were incorporated in year 1. Taxable income computations for the members include the following. Neither group member incurred any capital gain or loss transactions during these years, nor did either member make any charitable contributions. No § 382 limit applies. All tax years occur after 2020.

Tax Year Year 1 Year 2 Year 3 Year 4

ParentCo’s Taxable Income $10,000 $10,000 ($25,000) $10,000

ChildCo’s Taxable Income ($95,000) $50,000 $40,000 $10,000

Consolidated Taxable Income N/A ? ? ?

To what extent are ChildCo’s year 1 losses used by the group in year 2–year 4? Powered by Cognero

Page 9


Name:

Class:

Date:

Chapter 08: Consolidated Tax Returns a. $100,000 b. $95,000 c. $75,000 d. $0 65. ParentCo and SubCo report the following items of income and deduction for the current year. Item Income (loss) from operations § 1231 loss Capital gain Charitable contribution

ParentCo’s SubCo’s Taxable Taxable Income Income $100,000 ($10,000) (5,000) 15,000 12,000

Compute ParentCo and SubCo’s 2023 taxable income or loss computed on a separate basis. ParentCo SubCo a. $85,000 $5,000 b. $85,000 $3,000 c. $85,500 $5,000 d. $85,500 $3,000 66. ParentCo and SubCo report the following items of income and deduction for the current year. Item Income (loss) from operations § 1231 loss Capital gain Charitable contribution

ParentCo’s Taxable Income $100,000 (5,000)

SubCo’s Taxable Income ($10,000) 15,000

12,000

Compute ParentCo and SubCo’s current year consolidated taxable income or loss. a. $81,000 b. $88,000 c. $90,000 d. $90,500 67. ParentCo and SubCo report the following items of income and deduction for the current year. Item Income (loss) from operations § 1231 loss Capital gain Charitable contribution

ParentCo’s Taxable Income $100,000 (10,000) 20,000

SubCo’s Taxable Income $10,000 12,000 1,000

Compute ParentCo and SubCo’s current year taxable income or loss computed on a separate basis. ParentCo SubCo Powered by Cognero

Page 10


Name:

Class:

Date:

Chapter 08: Consolidated Tax Returns a. $81,000 b. $81,000 c. $70,000 d. $70,000

$21,000 $22,000 $22,000 $21,000

68. ParentCo and SubCo report the following items of income and deduction for the current year. Item Income (loss) from operations § 1231 loss Capital gain Charitable contribution

ParentCo’s Taxable Income $100,000 (10,000)

SubCo’s Taxable Income $10,000

20,000

12,000 1,000

Compute ParentCo and SubCo’s current year consolidated taxable income or loss. a. $91,000 b. $100,800 c. $112,000 d. $122,000 69. ParentCo and SubCo recorded the following items of income and deduction for the current tax year.

Item Income (loss) from operations § 1231 loss Capital gain (loss) Charitable contribution

ParentCo’s Taxable Income $100,000 (8,000) (10,000) 20,000

SubCo’s Taxable Income $20,000 14,000 1,000

Compute ParentCo and SubCo’s current year taxable income or loss computed on a separate basis. ParentCo SubCo a. $78,300 $30,600 b. $80,000 $33,000 c. $81,000 $33,000 d. $82,800 $30,600 e. $82,800 $33,000 70. ParentCo and SubCo recorded the following items of income and deduction for the current tax year. Item Income (loss) from operations § 1231 loss Capital gain (loss) Charitable contribution

ParentCo’s Taxable Income $100,000 (8,000) (10,000) 20,000

SubCo’s Taxable Income $20,000 14,000 1,000

Compute ParentCo and SubCo’s current year consolidated taxable income or loss. a. $95,000 Powered by Cognero

Page 11


Name:

Class:

Date:

Chapter 08: Consolidated Tax Returns b. $99,000 c. $104,400 d. $116,000 e. $120,000 71. ParentCo’s separate taxable income was $200,000, and SubCo’s was $50,000. Consolidated taxable income before contributions was $200,000. Charitable contributions made by the affiliated group included $60,000 by ParentCo and $10,000 by SubCo. Compute the group’s maximum 2023 charitable contribution deduction. a. $70,000 b. $60,000 c. $25,000 d. $20,000 72. ParentCo’s separate taxable income was $200,000, and JuniorCo’s was $50,000. Consolidated taxable income before contributions was $200,000. Charitable contributions made by the affiliated group included $5,000 by ParentCo and $1,000 by JuniorCo. Compute the group’s charitable contribution deduction. a. $0 b. $600 c. $6,000 d. $20,000 e. $25,000 73. JuniorCo sells an asset to SeniorCo at a realized loss. That loss is not recognized by the group in the year of the sale, because of the: a. Wash sale rule. b. Matching rule. c. Acceleration rule. d. None of these choices are correct. The group deducts the loss. 74. Concerning intercompany transactions within an affiliated group, the matching rule: a. Is attractive when gain or income amounts are involved. b. Applies to sales of assets but not services. c. Applies to sales of services but not assets. d. Applies to sales of both assets and services. Completion 75. One of the motivations for the consolidated return rules is to discourage conglomerates from trafficking in the deductible ____________________ of other entities. 76. Most of the rules governing the use of consolidated returns are found in tax ____________________ (Code/Regulations), and not the ____________________ (Code/Regulations). 77. Deferring recognition of an intercompany gain is one ____________________ (advantage/disadvantage) of electing to file consolidated returns. Powered by Cognero

Page 12


Name:

Class:

Date:

Chapter 08: Consolidated Tax Returns 78. The calendar year parent and affiliates must elect to file on a consolidated basis by ____________________ (month) 15 after the first consolidated tax year. 79. A parent-subsidiary controlled group exists where there is ____________________ percent ownership of all of the affiliates within the group, and there is an identifiable ____________________ corporation. 80. In terms of the consolidated return rules, the Flores Charitable Foundation is a(n) ____________________ entity. 81. When an affiliated group elects to file Federal consolidated income tax returns, it gives up any____________________ deduction for distributions of profits paid to other members. 82. Generally, when a subsidiary leaves an ongoing consolidated group, it must wait ____________________ years before it again can reenter the parent’s consolidated group. 83. Consolidated estimated tax payments must begin for the ____________________ (first, second, etc.) tax year after the group’s election. 84. All members of an affiliated group have ____________________ and ____________________ liability for each other’s Federal income tax liabilities. 85. Consolidated return members determine which affiliates will pay how much of the annual Federal income tax liability by making a(n) ____________________ election. 86. If, on joining an affiliated group, SubCo has a different tax year than that of ParentCo, SubCo must switch its year-end to ParentCo’s by the end of the ____________________ (first, second, etc.) tax year after the election to consolidate. 87. Within a Federal consolidated income tax group, SubOne ____________________ (can/cannot) use the LIFO inventory method at the same time that SubTwo uses the FIFO method. 88. Dividends paid out of a subsidiary’s E & P to the parent cause a ____________________ (positive/negative) adjustment to the parent’s stock basis of the subsidiary. 89. When negative adjustments are made to the stock basis of the subsidiary that exceed the basis as of the beginning of the tax year, a(n) ____________________ account is created, rather than assigning the parent a negative tax basis in the stock. 90. If there is a balance in the excess loss account when a subsidiary’s stock is sold, the balance generally is recognized as ____________________ gain/income. 91. The consolidated return rules combine the members’ transactions involving ____________________ items (e.g., § 1231 gain/loss) when computing consolidated taxable income. 92. In computing consolidated taxable income, a net capital gain/loss is an example of a(n) ____________________ item. 93. In computing consolidated taxable income, the profit/loss from a sale between Subsidiary and Parent is an example of a(n) ____________________ item. 94. In computing consolidated taxable income, the purchase at a realized gain of a depreciable asset by Subsidiary from Parent is an example of a(n) ____________________ transaction. 95. The ____________________ rules can limit the net operating loss deduction claimed on a Federal consolidated return. Powered by Cognero

Page 13


Name:

Class:

Date:

Chapter 08: Consolidated Tax Returns 96. If one member of an affiliated group sells an asset with a realized gain to another member, any gain on the transaction is not recognized at that time; such a deferral applies the ____________________ rule. 97. Generally, the matching rule is attractive to the affiliated group when a ____________________ (gain/loss) is involved. Matching Match each of the following terms with the appropriate description in the context of a consolidated Federal income tax return. a. Advantage of consolidating b. Disadvantage of consolidating c. Neither an advantage nor a disadvantage 98. Tax compliance deadlines and recordkeeping. 99. Offsetting gains against other members’ losses. 100. Loss deferral on intercompany transactions. 101. Gain deferral on intercompany transactions. 102. A new affiliate uses the LIFO method for inventories. 103. Binding nature of election over multiple tax years. 104. Tax treatment of operating losses when the basis of the payor’s stock is zero. 105. Joint and several liability for Federal income tax. 106. Choice of tax year-ends by affiliates. Match each of the following items with the appropriate description, indicating whether the item increases or decreases the parent’s basis in the stock of a subsidiary. a. Increases stock basis of subsidiary b. Decreases stock basis of subsidiary c. No effect on stock basis of subsidiary 107. Member’s operating loss. 108. Member’s operating loss, when stock basis = $0. 109. Member’s operating gains/profits. 110. Increase in affiliate’s E & P. 111. Dividend paid to parent out of affiliate’s E & P. 112. Member’s capital gain. Powered by Cognero

Page 14


Name:

Class:

Date:

Chapter 08: Consolidated Tax Returns Match each of the following items with the appropriate description, indicating whether the item’s treatment for financial accounting and Federal income tax purposes is the same or not. a. Tax and book treatment is the same b. Tax and book treatment differ 113. Ownership level of parent at which a subsidiary can join the consolidated group. 114. Parent owns 100% of a U.S. partnership and wants the entity to join the consolidated group. 115. Parent owns 100% of a Brazil corporation and wants the entity to join the consolidated group. 116. Rules for joining the consolidated group is mandatory if ownership requirements are met by the entities. 117. Sales amounts (after returns and allowances) are combined in the consolidation process. Match each of the following items with the appropriate description, indicating whether the item is computed on a group basis on a consolidated tax return. a. Group item b. Not a group item 118. Compensation deductions. 119. Interest income from Detroit School District bonds. 120. Net operating loss. 121. Charitable contributions. 122. Dividends received deduction. Subjective Short Answer 123. ParentCo’s controlled group includes the following members. ParentCo owns all of the stock in each of the listed entities. Which entities can join ParentCo in a consolidated return? ∙

SubCoA, a manufacturer incorporated in India.

SubCoB, a U.S. manufacturer.

SubCoC, a U.S. S corporation.

SubCoD, a U.S. partnership.

SubCoE, a U.S. limited liability company (LLC).

SubCoF, a U.S. manufacturer incorporated in Puerto Rico.

SubCoG, a U.S. manufacturer that claims a deduction for its research activities.

SubCoH, a U.S. tax-exempt museum. Powered by Cognero

Page 15


Name:

Class:

Date:

Chapter 08: Consolidated Tax Returns 124. The Parent consolidated group reports the following results for the tax year. Determine each member’s share of the consolidated tax liability, assuming that all members have consented to use the relative taxable income tax-sharing method. Dollar amounts are listed in millions, and a 21% income tax rate applies to all of the entities. Ordinary income Capital gain/loss §1231 gain/loss

Parent $600 –0– 130

SubOne $200 –0– –0–

SubTwo $ 60 20 (55)

$730

$200

$ 25

Separate taxable incomes

SubThree ($30) (5) –0– ($30), with a $5 capital loss carryover

Consolidated taxable income Consolidated tax liability Energy tax credit, from SubOne Net tax due

Consolidated $830 15 75

$920 $193 (10) $183

125. The Parent consolidated group reports the following results for the tax year. Determine each member’s share of the consolidated tax liability, assuming that the members all have consented to use the relative tax liability tax-sharing method. Dollar amounts are listed in millions, and a 21% income tax rate applies to all of the entities. Ordinary income Capital gain/loss § 1231 gain/loss

Parent $600 –0– 130

SubOne $200 –0– –0–

SubTwo $60 20 (55)

$730

$200

$25

Separate taxable incomes Consolidated taxable income Consolidated tax liability Energy tax credit, from SubOne Net tax due

SubThree ($30) (5) –0– ($30), with a $5 capital loss carryover

Consolidated $830 15 75

$920 $193 (10) $ 183

126. Calendar year Parent Corporation acquired all of the stock of SubCo on January 1, year 1, for $500,000.

The subsidiary’s operating gains and losses are shown below. In addition, a $50,000 dividend is paid early in year 2. Complete the following chart, indicating the appropriate stock basis and excess loss account amounts. Tax Year 1

Operating Gain/Loss

Stock Basis

($100,000)

?

Powered by Cognero

Excess Loss Account ? Page 16


Name:

Class:

Date:

Chapter 08: Consolidated Tax Returns 2 3

($150,000) $300,000

? ?

? ?

127. LargeCo files on a consolidated basis with LittleCo. The subsidiary was acquired for $400,000 on January 1, year 1, and it paid a $75,000 dividend to LargeCo at the end of both year 2 and year 3. a. Given the following information about the subsidiary’s operating results, derive the requested amounts as of December 31 of each year. The group files using a calendar year.

b.

LittleCo’s

LargeCo’s Investment in LittleCo

Tax Year

Operating Gain/(Loss)

Stock Basis

Excess Loss Account

1 2 3

($125,000) ($300,000) $50,000

? ? ?

? ? ?

LargeCo sold LittleCo to an unrelated competitor for $600,000 on December 31, year 3. How will LargeCo account for this sale?

128. In the current year, Parent Corporation provided advertising services to its 100% owned subsidiary, SubCo, under a contract that requires no payments to Parent until next year. Both parties use the accrual method of tax accounting and a calendar tax year. The services that Parent rendered were valued at $250,000. In addition, Parent received $20,000 of interest payments from SubCo. relative to an arm’s length note between them. Including these transactions, Parent’s taxable income for the year amounted to $400,000. SubCo reported $200,000 separate taxable income. Derive the group’s consolidated taxable income using the format of Exhibit 8.3. Separate Taxable Income

Adjustments

PostAdjustment Amounts

ParentCo information SubCo information Group-basis transactions Intercompany events Consolidated taxable income NOTES

129. For each of the indicated tax years, compute consolidated taxable income for the calendar year Holloway Group, which elected consolidated status immediately upon creation of the two member corporations in January year 1. All recognized income related to the data processing services of the firms. No intercompany transactions were completed Powered by Cognero

Page 17


Name:

Class:

Date:

Chapter 08: Consolidated Tax Returns during the indicated years. All tax years occur after 2020.

Tax Year Year 1 Year 2 Year 3 Year 4

Holloway Corporation $250,000 250,000 250,000 250,000

Olson Corporation ($ 70,000) (120,000) (180,000) 110,000

130. For each of the indicated tax years. compute consolidated taxable income for the calendar year Whitman Group, which elected consolidated status immediately upon creation of the two member corporations in January year 1. All recognized income related to the data processing services of the firms. No intercompany transactions were completed during the indicated years. All tax years occur after 2020.

Tax Year Year 1 Year 2 Year 3 Year 4

Whitman Corporation $250,000 250,000 250,000 250,000

Draper Corporation $ 90,000 (170,000) (560,000) 145,000

131. Parent Corporation’s current-year taxable income included $100,000 net profit from operations and a $30,000 net long-term capital gain. Parent also made a $22,000 charitable contribution to State University. SubCo produced $85,000 of income from operations and incurred a $25,000 net short-term capital loss. Use the computational worksheet of Exhibit 8.3 to derive the group members’ separate taxable incomes and the group’s consolidated taxable income. Separate Taxable Income

Adjustments

PostAdjustment Amounts

ParentCo information SubCo information Group-basis transactions Intercompany events Consolidated taxable income NOTES

Essay 132. How many consolidated tax returns are filed annually? What types of taxpayer do they represent? 133. List some of the nontax reasons that groups of corporations form conglomerates and may be eligible also to file consolidated Federal income tax returns. Powered by Cognero

Page 18


Name:

Class:

Date:

Chapter 08: Consolidated Tax Returns 134. Where are the controlling Federal income tax rules regarding consolidated tax returns to be found? What is the general philosophy of those rules? 135. The consolidated income tax return rules apply only for Federal tax purposes. Financial accounting rules can be quite different from the corresponding tax rules, but the tax professional should be familiar with both sets of requirements. Describe the major differences between the book and tax treatment for the conglomerate’s reporting of: a.

The ownership levels required to consolidate.

b.

Goodwill.

c.

The entities included on the report.

136. Outline the major advantages and disadvantages of filing Federal corporate income tax returns on a consolidated basis. Limit your comments to the income tax effects of the election. 137. Certain business entities are ineligible to join an affiliated group and file a consolidated Federal income tax return. List three or more entities that cannot be part of a Federal consolidated tax return group. 138. When a corporate group elects to file Federal income tax returns on a consolidated basis, it is subject to several tax return filing requirements for its first and subsequent tax years. List the most important of those requirements. 139. Gold and Bronze elect to form a Federal consolidated group. Gold, the parent entity, uses a calendar tax year while Bronze’s tax year ends on March 31. Which tax year(s) does the new consolidated group use? 140. Gold, Silver, and Bronze constitute a Federal consolidated tax return group. Which of the members is responsible to pay the tax liability—the parent, the subsidiaries, or both? How are these tax-payable amounts determined? Hint: Use the term tax-sharing agreement in your answer. 141. Discuss how a parent corporation computes its stock basis for a subsidiary that joins in a Federal consolidated income tax return. 142. Parent’s basis in the stock of Child, its subsidiary, is $1 million at the beginning of the year. Child’s share of consolidated taxable income this year is a $1.25 million operating loss. Parent’s basis in the Child stock now is zero. Elaborate. 143. Describe the general computational method used by a Federal consolidated group in computing taxable income. 144. ParentCo owns all of the stock of both SubOne and SubTwo. List three “intercompany transactions” that might occur among the three members of the consolidated filing group. 145. A Federal consolidated group reports a net operating loss for the year. How is this amount allocated to the various group members? Why is this allocation important? 146. The consolidated tax return regulations use “SRLY” limitations with respect to losses of a subsidiary that can be deducted on the consolidated return. Describe the various SRLY rules that might apply to a consolidated group member’s losses. 147. The “SRLY” and § 382 limitation rules for Federal consolidated tax returns are designed to keep corporations from “trafficking” their net operating losses. These rules are restrictive and somewhat complex. Explain why these rules exist Powered by Cognero

Page 19


Name:

Class:

Date:

Chapter 08: Consolidated Tax Returns and how they interact if both apply in the same tax year. 148. Certain tax return items are computed on a group basis when a Federal consolidated return election is in place. List five or more of these group basis items. 149. Members of the ABCD Federal consolidated group conduct various transactions with each other during the tax year. These include the purchase/sale of land and depreciable assets, the licensing of intangible assets, and the conduct of service arrangements. How does the tax law account for these intercompany items? In your answer, be sure to use the terms matching rule and acceleration rule at least once each. 150. Forming a Federal consolidated tax return group is a discretionary action by eligible affiliates. List several tax attributes and situations that might make a subsidiary an attractive partner for a parent corporation on a consolidated return.

Powered by Cognero

Page 20


Name:

Class:

Date:

Chapter 08: Consolidated Tax Returns Answer Key 1. True 2. True 3. True 4. True 5. False 6. False 7. False 8. False 9. True 10. False 11. True 12. True 13. False 14. False 15. False 16. True 17. True 18. True 19. True 20. True 21. False 22. True 23. False 24. False 25. True Powered by Cognero

Page 21


Name:

Class:

Date:

Chapter 08: Consolidated Tax Returns 26. False 27. False 28. True 29. True 30. True 31. False 32. False 33. True 34. False 35. True 36. True 37. True 38. b 39. a 40. d 41. c 42. a 43. a 44. d 45. d 46. e 47. d 48. a 49. c 50. d Powered by Cognero

Page 22


Name:

Class:

Date:

Chapter 08: Consolidated Tax Returns 51. b 52. a 53. c 54. a 55. c 56. d 57. a 58. b 59. a 60. b 61. c 62. b 63. c 64. c 65. c 66. c 67. a 68. b 69. e 70. c 71. d 72. c 73. b 74. d 75. losses 76. regulations, Internal Revenue Code Powered by Cognero

Page 23


Name:

Class:

Date:

Chapter 08: Consolidated Tax Returns 77. advantage 78. April October (the extended Form 1120 due date for a calendar year corporation) 79. 80, parent eighty, parent 80. ineligible 81. dividends received 82. five 5 83. third 3rd 84. joint, several 85. tax-sharing 86. first 1st 87. can 88. negative 89. excess loss 90. capital 91. group 92. group 93. elimination 94. intercompany 95.

separate return limitation year SRLY § 382 Section 382 96. matching Powered by Cognero

Page 24


Name:

Class:

Date:

Chapter 08: Consolidated Tax Returns 97. gain 98. b 99. a 100. b 101. a 102. c 103. b 104. b 105. b 106. b 107. b 108. c 109. a 110. c 111. b 112. a 113. b 114. b 115. b 116. b 117. a 118. b 119. b 120. a 121. a Powered by Cognero

Page 25


Name:

Class:

Date:

Chapter 08: Consolidated Tax Returns 122. a 123. ParentCo can include only its subsidiaries B and G on a consolidated return. The other entities are not eligible to file on a consolidated basis. 124. Consolidated tax liabilities are shared in the following manner. Separate Taxable Allocation Ratio Income $730 730/955 200 200/955 25 25/955 –0– 0 $955

Parent SubOne SubTwo SubThree Totals

Allocated Tax Due $140 38 5 –0– $183

125. Consolidated tax liabilities are shared in the following manner.

Parent

Separate Taxable Income $730

SubOne

200

SubTwo SubThree Totals

25 –0– $955

Separate Tax Liability $153.3 32, after subtracting energy tax credit 5.25 –0– $190.55

Allocation Ratio 153.3/190.55

Allocated Tax Due $147

32/190.55

31

5.25/190.55 0

5 –0– $183

126. Tax Year 1

Operating Gain/(Loss) ($100,000)

2

($150,000)

3

$300,000

127. a. Year 1 Year 2 Year 3

Stock basis $275,000;excess loss account $0 Stock basis $0; excess loss account $100,000, reflecting $75,000 dividend paid Stock basis $0; excess loss account $125,000, reflecting $75,000 dividend paid

b.

Stock Basis $400,000 $200,000, reflecting $50,000 dividend paid $500,000

Excess Loss Account $0 $0 $0

Capital gain income to the extent of the excess loss account plus the amount realized on the sale = $725,000.

Powered by Cognero

Page 26


Name:

Class:

Date:

Chapter 08: Consolidated Tax Returns 128. No eliminating adjustments are required of the group. The members’ deductions incurred offset the income included by the other party to the intercompany transaction (e.g., consolidated taxable income includes both ParentCo’s gross interest income and SubCo’s deduction therefor), so a financial accounting-style elimination results from the use of the consolidated taxable income computation itself. The services transaction is reported next year by both parties, when the services are rendered. Separate Taxable Income ParentCo information SubCo information Group-basis

Adjustments

Post-Adjustment Amounts

$400,000

$400,000

$200,000

$200,000

transactions Intercompany events Consolidated taxable income

$600,000

129. Consolidated Taxable Income Year 1 Year 2 Year 3 Year 4

$180,000 $130,000 $ 70,000 $360,000

The Olson losses offset the Holloway income dollar for dollar, but they do not become large enough to produce a consolidated loss. Because both corporations produce only ordinary income, there are no adjustments to make using the format of Exhibit 8.3. There are no consolidated NOL carryovers in any of the specified years. 130. Consolidated taxable income Year 1

$340,000

Year 2

$80,000

Year 3

$ –0– ($310,000) NOL carryforward.

Year 4

$85,000 after applying NOL carryforward, which does not reduce year 4 taxable income by more than 80%

131. ParentCo Income from operations Capital gain income Less: Charitable contribution (10% TI limit) ParentCo's separate taxable income Powered by Cognero

$100,000 30,000 (13,000) $117,000 Page 27


Name:

Class:

Date:

Chapter 08: Consolidated Tax Returns SubCo’s separate taxable income was $85,000 because the net capital loss is not deductible. Thus, the aggregate separate taxable incomes for the group amounted to $202,000. Upon consolidation, a greater amount of ParentCo’s charitable contribution becomes deductible. The group consolidated taxable income before contributions is $190,000, so its 10% contribution limit is $19,000. In addition, its capital gain is sheltered from current-year tax by SubCo’s net capital loss. The group holds a $3,000 charitable contribution carryforward. Aggregate taxable income is reduced by $31,000 because a consolidated return was filed.

Separate Taxable Income ParentCo information

$117,000

SubCo information

$ 85,000

Group-basis transactions

Adjustments – $30,000 capital gain income** + $13,000 charitable contribution deduction** $25,000 short-term capital loss** $5,000 net long-term capital gain – $22,000 charitable contribution (group’s 10% maximum deduction)**

Intercompany events Consolidated taxable income NOTES

PostAdjustment Amounts $100,000 $ 85,000 $ 5,000 ($ 19,000)

$171,000

* Permanent Eliminations ** Group-Basis Transaction † Matching Rule

132. About 35,000 affiliated groups file consolidated Federal income tax returns. These entities represent the vast majority of the country’s assets and sales revenues. Consolidated returns claim virtually all of the allowed foreign tax credits. 133. Conglomerates are formed to reduce overall tax liabilities and to: ∙

Isolate the assets of some affiliates from the liabilities of others.

Disguise the true owners of certain assets and income.

Preserve intangible assets such as trade names.

Accomplish estate planning objectives.

Expand into underserved markets.

Powered by Cognero

Page 28


Name:

Class:

Date:

Chapter 08: Consolidated Tax Returns

134. Most of the rules controlling the use of consolidated Federal tax returns are found in the extensive Regulations under IRC §§ 1501-1504. The rules are designed to tax the group as if it was one corporation. In addition, the rules limit the tax advantages that otherwise might be available to affiliated groups of corporations. The Treasury seems to want to discourage profitable corporations from “trafficking” in the shares of businesses that generate deductible net losses and tax credits. 135. a.

Generally, GAAP consolidation is mandatory when greater than 50% ownership is attained. Tax consolidation is elective when 80% or greater ownership is attained.

b.

For financial accounting purposes, goodwill is not amortized, but if the value of the goodwill changes during the year, the impairment (a decrease in the value of the goodwill, an expense item) or the reversal of a prior impairment (a revenue item) is recorded on the income statement. Under the tax rules, purchased goodwill is amortized over 15 years.

c.

A Federal consolidated tax return includes only U.S. corporations. Non-U.S. and noncorporate entities can be included in the conglomerate’s financial statements.

136. Assessing only the Federal income tax aspects of the consolidation election, the major advantages include the following. ∙

Application of operating and capital losses of one member against the gains/income of others.

Elimination of all tax liabilities on intercompany dividends.

Deferral of gains from certain intercompany transactions.

Optimization of other deductions, losses, and credits computed on a group basis.

Optimization of the group’s estimated tax payments.

Increase in basis in subsidiary stock when net gains are recognized by the group.

Shareholdings of all group members can be used in meeting other statutory requirements .

Major Federal income tax disadvantages relative to the consolidation election include the following. ∙

The binding status of the election and the five-year cooling-off period after termination thereof.

Deferral of losses from certain intercompany transactions.

Reduction of basis in subsidiary stock when net losses are recognized by the group.

Standardization of the tax year, perhaps resulting in income bunching and loss of flexibility.

Additional costs incurred to meet compliance and administrative requirements, which can be Powered by Cognero

Page 29


Name:

Class:

Date:

Chapter 08: Consolidated Tax Returns burdensome.

137. These are some of the entities ineligible for membership in a consolidated group. ∙

Non-U.S. entities.

Tax-exempt entities.

Partnerships, trusts, estate, and limited liability entities.

Other noncorporate entities.

138. The major filing requirements for a Federal consolidated group include the following. ∙

File the Form 1120 using consolidated taxable income.

File Form 1122 listing the group members consenting to the election to consolidate with the first consolidated return.

File Form 851, another less-detailed listing of the affiliates included in the return, with all subsequent consolidated 1120s.

139. Affiliates must switch to the parent’s tax year (calendar year in this case), but they can continue to use their existing tax accounting methods. A short tax year is created for a new affiliate that joins the parent’s group and needs to change its tax year. 140. All affiliates are responsible for the total Federal corporate income tax liability of the group on a joint and several basis. This rule applies to income tax penalties and interest, as well as to any tax audit settlements that are completed during the year. Starting with the third tax year of an electing consolidated group, estimated income tax payments are made on a consolidated basis. In assigning shares of the total Federal corporate income tax liability among group members, say for purposes of computing an affiliate’s E & P balance, the “relative taxable income” and “relative tax liability” methods often are used. Other methods also are allowed by the regulations. 141. The initial basis of the subsidiary is recorded at its acquisition price. If the acquisition takes place as part of a taxdeferred reorganization (see Chapter 7), the amount may be computed as a carryover or substituted basis. The basis is adjusted for the operations of the group. For instance, stock basis is increased by the following items. Powered by Cognero

Page 30


Name:

Class:

Date:

Chapter 08: Consolidated Tax Returns ∙

An allocable share of consolidated taxable income.

An allocable share of the unused portion of the subsidiary’s NOL or net capital loss.

Negative adjustments to stock basis include the following. ∙

An allocable share of a negative consolidated taxable income.

Dividends paid by the subsidiary to the parent out of subsidiary earnings and profits.

An allocable share of deductible carryover NOLs and net capital losses.

142. The parent’s tax basis in the stock of the subsidiary cannot be a negative amount. If excessive taxable losses are incurred to the point that stock basis is zero, the tax law might hold that no further losses can be deducted. This is the treatment for flow-through losses of an S corporation or partnership. The consolidated return rules do not work this way. Subsidiary losses can continue to be deducted, and an excess loss account is generated to account for these amounts. 143. Consolidated taxable income is determined in the following manner. ∙ Taxable income is computed for each group member on a separate basis.

Realized gain/losses from most intercompany transactions are deferred until later tax years. ∙

Certain transactions are accounted for on a group basis using group floors, ceilings, and other computational limits. These items include capital gain/loss and charitable contributions.

A few intercompany transactions (such as dividend payments) are eliminated from all computations of taxable income.

144. Illustrative intercompany transactions include the following.

∙ SubOne purchases accounting and data processing services from Parent. ∙ SubTwo pays a regular cash dividend to Parent. ∙ SubOne pays a regular dividend to Parent, using an asset with a realized gain to SubOne. ∙ Parent sells to SubTwo an asset with a realized loss. 145. Each year, the total consolidated loss is allocated as follows. Consolidated NOL × Powered by Cognero

Member’s separate NOL Members’ aggregate NOLs

= Member’s apportioned NOL Page 31


Name:

Class:

Date:

Chapter 08: Consolidated Tax Returns This allocation of a consolidated operating loss among the affiliates is important when an affiliate is disposed of or leaves the group. In such cases, it takes its share of the loss to its next (separate) tax return. 146. The SRLY limits generally work this way. Loss Year

Deduction Year

Consolidated

Consolidated

Consolidated

Separate

Separate

Consolidated

Applicable Rules Regular NOL rules—Carryforward per usual rules and limitations Carryforward only the member’s apportioned loss to its separate return. Departing member takes its apportioned loss with it to the separate return. New member’s NOL carryforward is available to the group only to the extent of the new member’s cumulative positive contribution to consolidated taxable income. § 382 rules also may apply.

147. The separate return limitation rule (SRLY) rules are designed to keep Federal consolidated return members from acquiring new affiliates chiefly to obtain tax loss and credit carryovers. When, for instance, an NOL is carried forward from a separate return year onto a consolidated return that now includes an acquired loss corporation, the consolidated return can include the loss corporation’s loss from preconsolidation (separate return) years only to the extent of the lesser of its (1) current-year or (2) cumulative positive contribution to consolidated taxable income. The SRLY rules never apply to the consolidated group’s parent corporation. When both provisions apply due to an ownership change (see text Section 7-4c), the § 382 provisions override the SRLY limits. 148. The following items are among those that are computed on a group basis for a Federal corporate consolidated income tax return. ∙ Net long-term capital gain/loss. ∙ Net short-tem capital gain/loss. ∙ § 1231 gain/loss. ∙ Casualty/theft gain/loss. ∙ Charitable contributions. ∙ Dividends received deduction. ∙ Net operating loss. ∙ § 163(j) interest expense limitation. ∙ Various tax credits and recapture amounts. Powered by Cognero

Page 32


Name:

Class:

Date:

Chapter 08: Consolidated Tax Returns

149. Gain or loss realized on property transactions between affiliates may not be recognized immediately. If more than one tax year is involved, the recognized gain or loss is deferred under the matching rule until the sold asset leaves the affiliated group, i.e., by a subsequent sale to a nonaffiliated third party. This prevents group members from manipulating consolidated taxable income merely by moving assets among the members. The acceleration rule applies to a deferred gain/loss when one of the affiliates involved in the transaction leaves the group or if the consolidation election is terminated. These events trigger the full recognition of the deferred gain or loss. 150. Taxpayers have incentives to optimize their overall tax benefits when choosing consolidated return partners. Within the limitations of the Code and of the nontax parameters of the corporation’s business, target corporations might include those of the following with appropriate amounts. ∙

Loss and credit carryovers.

Passive activity income, losses, and credits.

Gains that can be deferred through intercompany sales.

Excess limitation amounts (e.g., concerning charitable contributions).

§ 1231 gains, losses, and look-back profiles.

Powered by Cognero

Page 33


Name:

Class:

Date:

Chapter 09: Partnerships Formation Operations and Basis True / False 1. A partnership is an association formed by two or more taxpayers (which may be any type of entity) to carry on a trade or business. a. True b. False 2. In a limited liability company, all members may participate in management (the operating agreement cannot limit participation), and all entity debts are treated as nonrecourse liabilities for purposes of allocating the LLC’s liabilities to basis. a. True b. False 3. In a limited liability company, all members are protected from the debts of the LLC unless they personally guaranteed the debt. a. True b. False 4. In a limited liability partnership, all members may participate in management and generally have personal liability for entity debts except for malpractice committed by the other partners. a. True b. False 5. A limited partnership (LP) offers all partners protection from claims by the LP’s creditors. a. True b. False 6. The primary purpose of the partnership agreement is to document the various tax elections made by the partners regarding items such as depreciation methods, treatment of research and experimental costs, and the § 754 election. a. True b. False 7. The taxable income of a partnership flows through to the partners, who report the income on their tax returns. a. True b. False 8. An example of the aggregate concept underlying partnership taxation is the fact that the partners (rather than the partnership) pay tax on partnership income. a. True b. False 9. An example of the aggregate concept of partnership taxation is that the partnership makes elections related to depreciation, tax credit calculations (except the foreign tax credit), and whether to claim a § 179 deduction. a. True b. False 10. The partnership agreement might provide, for example, that the first $40,000 of ordinary income is allocated to Powered by Cognero

Page 1


Name:

Class:

Date:

Chapter 09: Partnerships Formation Operations and Basis Partner A. Allocating income in this manner is an example of a separately stated item. a. True b. False 11. A partner's profit-sharing, loss-sharing, and capital-sharing ownership percentages are the same. a. True b. False 12. The inside basis is defined as a partner’s basis in the partnership interest. a. True b. False 13. A partnership reports each partner’s share of income to the partner on a Form 1099-MISC. a. True b. False 14. Section 721 provides that, in general, no gain or loss is recognized by the partnership or the partner on contribution of appreciated or depreciated property to a partnership in exchange for an interest in the partnership. a. True b. False 15. Ken and Linh formed the equal KL Partnership during the current year; Ken contributes $100,000 in cash and Linh contributes land (basis of $60,000, fair market value of $40,000) and equipment (basis of $0, fair market value of $60,000). Linh recognizes a $40,000 gain on the contribution, and her basis in her partnership interest is $100,000. a. True b. False 16. When Kevin and Marshall formed the equal KM LLC, the fair market values of their interests were each $100,000. Kevin contributed $60,000 cash, equipment with a basis of $0 and a fair market value of $10,000, and a small parcel of land in which he had a basis of $50,000 and that was valued at $30,000. Marshall contributed a cash basis account receivable that was valued at $100,000 and in which his basis was $0. Kevin has a basis in his partnership interest of $110,000 and Marshall’s basis is $0. a. True b. False 17. Morgan and Kristen formed an equal partnership on August 1 of the current year. Morgan contributed $60,000 cash and land with a basis of $18,000 and a fair market value of $40,000. Kristen contributed equipment with a basis of $42,000 and a value of $100,000. Kristen and Morgan both have a basis of $100,000 in their partnership interests. a. True b. False 18. Section 721 provides that no gain or loss is recognized on a contribution of property to a partnership in exchange for an interest in the partnership. An exception might apply if the taxpayer receives a cash distribution from the partnership soon after the property contribution is made. a. True Powered by Cognero

Page 2


Name:

Class:

Date:

Chapter 09: Partnerships Formation Operations and Basis b. False 19. George received a fully vested 10% interest in partnership capital and a 20% interest in future partnership profits in exchange for services rendered to the GHP, LLC (not a publicly traded partnership interest). The future profits of the partnership are subject to normal operating risks. George will report ordinary income equal to the fair market value of the profits interest, but the capital interest will not be currently taxed to him. a. True b. False 20. Laura is a real estate developer and owns property that is treated as inventory (not a capital asset) in her business. She contributes a parcel of this land (basis of $15,000) to a partnership, also to be held as inventory. The fair market value of the property is $12,000 at the contribution date. After three years, the partnership sells the land for $10,000. The partnership will recognize a $5,000 ordinary loss on sale of the property. a. True b. False 21. If a partnership properly makes an election for treatment of a specific tax item, the partner is bound by that treatment. a. True b. False 22. JLK Partnership incurred $6,000 of organizational costs and $50,000 of startup costs. JKL may deduct $5,000 each of organizational and startup costs, and the remaining costs ($1,000 of organizational costs and $45,000 of startup costs) may be amortized over 60 months. a. True b. False 23. Syndication costs arise when partnership interests are being marketed to investors. These costs cannot be amortized or deducted on income tax returns. a. True b. False 24. BMR LLC conducted activities that were eligible for a $20,000 credit for increasing research activities. In addition, the LLC paid foreign taxes of $1,200. On the partners’ Schedules K-1, BMR will allocate the $20,000 research credit, and it will provide the necessary information so the partners can calculate the foreign tax credit if they so choose. a. True b. False 25. Seven years ago, Pooja purchased residential rental estate that she has been depreciating as MACRS property over 27.5 years. This year, when her adjusted basis in the property was $250,000, she transferred the property to the newly formed PLA LLC in exchange for a one-third interest in it. PLA incurred $10,000 of transfer taxes and fees related to the property. The LLC must treat the $260,000 basis in the property, fees, and expenses as new MACRS property depreciable over 27.5 years. a. True b. False 26. A partnership cannot use the cash method of accounting if one of its partners is a C corporation. a. True Powered by Cognero

Page 3


Name:

Class:

Date:

Chapter 09: Partnerships Formation Operations and Basis b. False 27. Greene Partnership had average annual gross receipts for the past three years of $25,800,000. One of the partners is Jackson, Inc., a C corporation. Because Greene meets the average annual gross receipts test, it may use the cash method of accounting even though it has a partner that is a C corporation. a. True b. False 28. ABC, LLC is equally owned by three corporations. Two corporations have June 30 fiscal year-ends and the third is a calendar year taxpayer. ABC will use the least aggregate deferral method to determine its taxable year-end. a. True b. False 29. PaulCo, DavidCo, and Sean form a partnership with cash contributions of $80,000, $50,000, and $30,000, respectively, and agree to share profits and losses in the ratio of their original cash contributions. PaulCo uses a January 31 fiscal year-end, whereas DavidCo and Sean use a November 30 and December 31 year-end, respectively. The partnership must use the least aggregate deferral method to determine its year-end. a. True b. False 30. MNO Partnership has three equal partners. Moon, Inc. and Neptune, Inc. each have fiscal years ending March 31. Omega uses the calendar year. MNO's required taxable year-end is March 31 under the majority partner rule. a. True b. False 31. A partnership must provide any information to the partners that they would need to calculate deductions not permitted at the partnership level, such as for oil and gas depletion or the corporate dividends received deduction. a. True b. False 32. Items that are not required to be shown on the partners’ Schedules K-1 include AMT adjustments and preferences and taxes paid to foreign countries because any AMT and the foreign tax credit are calculated by the partnership. a. True b. False 33. The amount of a partnership’s income and loss from operating activities is combined with separately stated income and expenses to determine the partnership’s equivalent of taxable income. This amount is reconciled to book income on the partnership’s Schedule M-1 or Schedule M-3. a. True b. False 34. BRW Partnership reported gross income from operations of $60,000, interest income of $3,000, utilities expense of $20,000, and a charitable contribution of $6,000. On its Schedule K, the partnership reports ordinary business income of $40,000, separately stated interest income ($3,000), and charitable contributions ($6,000). a. True b. False Powered by Cognero

Page 4


Name:

Class:

Date:

Chapter 09: Partnerships Formation Operations and Basis 35. DDP Partnership reported gross income from operations of $125,000, a long-term capital gain of $5,000, a short-term capital loss of $2,000, and a charitable contribution of $5,000. On its Schedule K, the partnership reports ordinary business income of $120,000, and a net long-term capital gain of $3,000. a. True b. False 36. To meet the substantial economic effect tests, a partnership’s allocations of income and deductions to the partners are required to be proportionate to the partners’ percentage ownership of partnership capital. a. True b. False 37. Tom and William are equal partners in the TW Partnership. Just before TW liquidated, Tom’s § 704(b) book capital account balance was $50,000 and William’s § 704(b) book capital account balance was $30,000. To meet the substantial economic effect requirements, any liquidating cash distribution must be allocated in proportion to those ending capital account balances. a. True b. False 38. Blaine contributes property valued at $50,000 (basis of $40,000) in exchange for a 25% interest in the BIKE Partnership. If the property is later sold for $70,000, gain of $15,000 will be allocated to Blaine. a. True b. False 39. The RGBY LLC operating agreement provides that 50% of depreciation expense is allocated to Red, and all remaining income (including the remaining 50% of depreciation) is allocated equally among the four partners. Before guaranteed payments and depreciation, RGBY’s net income is $120,000 for the year. RGBY’s depreciation expense is $20,000, and it paid a guaranteed payment to Yellow of $8,000. Assume that all allocations and payments meet the substantial economic effect rules. After all deductions and special allocations are taken into account, Red is allocated net income of $15,500 from the partnership. a. True b. False 40. Nicholas, a one-third partner, received a guaranteed payment in the current year of $50,000. Partnership income before consideration of the guaranteed payment was $20,000. Assuming that no loss limitation rules apply, Nicholas reports a $10,000 ordinary loss from partnership operations and the $50,000 guaranteed payment as ordinary income. a. True b. False 41. Emma’s basis in her BBDE LLC interest is $60,000 at the beginning of the tax year. Her allocable share of LLC items are as follows: $20,000 of ordinary income, $2,000 tax-exempt interest income, and a $6,000 long-term capital gain. In addition, the LLC distributed $12,000 of cash to Emma during the year. Assuming that the LLC had no liabilities at the beginning or the end of the year, Emma’s ending basis in her LLC interest is $76,000. a. True b. False 42. Stella’s tax basis in her SAW Partnership interest is $200,000 including all adjustments at the beginning of the tax year. Her allocable share of partnership items is: ($120,000) of ordinary loss, $6,000 tax-exempt interest income, and a Powered by Cognero

Page 5


Name:

Class:

Date:

Chapter 09: Partnerships Formation Operations and Basis $14,000 long-term capital gain. In addition, during the year, the LLC distributed $20,000 of cash to Stella. Also during the year, Stella’s share of partnership debt increased by $10,000. Stella’s ending basis in her LLC interest is $80,000. a. True b. False 43. Ashley purchased her partnership interest from Lindsey on the first day of the current year for $40,000 cash. The partnership had no liabilities at that time. Ashley received a $10,000 cash distribution from the partnership during the year, and her share of partnership income is $15,000. Her share of partnership liabilities on the last day of the partnership year is $20,000. Ashley’s outside basis for her partnership interest at the end of the year is $45,000. a. True b. False 44. Julie and Kate form an equal partnership during the current year. Julie contributes cash of $200,000, and Kate contributes property (adjusted basis of $90,000, fair market value of $260,000) subject to a nonrecourse liability of $60,000. As a result of these transactions, Kate has a basis in her partnership interest of $120,000. a. True b. False 45. Debt of a limited liability company is allocated among its members using the nonrecourse debt allocation rules unless an LLC member has personally guaranteed the debt. a. True b. False 46. Belinda owns a 30% profit and loss interest in the BOW LLC, and her basis in the interest is $30,000 excluding her share of the LLC’s liabilities. Belinda guarantees a $40,000 LLC debt. Remaining liabilities (not guaranteed by any of the LLC members) are $100,000. Belinda’s basis in the LLC is $100,000. a. True b. False 47. The sum of the partners’ ending tax basis amounts equals the partners’ ending tax basis capital account balances. These amounts are shown on the partnership's Schedule K. a. True b. False 48. Micah’s beginning tax basis capital account on his Schedule K-1 is $60,000. During the year, he is allocated $20,000 of partnership income, $8,000 of nondeductible expenses, and a $12,000 share of tax-exempt income. His Schedule K-1s show allocations of nonrecourse debt of $20,000 (last year) and $30,000 (this year). Micah’s ending tax basis capital account is $94,000. a. True b. False 49. If a partnership allocates losses to the partners, the partners first apply the passive loss limitations, then the basis limitation, and finally the at-risk limitations. If all three hurdles are met, a partner may deduct the loss. a. True b. False 50. Harry’s basis in his partnership interest was $10,000 at the beginning of the tax year. For the year, his share of the Powered by Cognero

Page 6


Name:

Class:

Date:

Chapter 09: Partnerships Formation Operations and Basis partnership’s loss was $8,000, and he also received a distribution of $4,000. Harry can deduct an $8,000 loss, and he recognizes a gain of $2,000 on the distribution of cash in excess of his remaining basis. a. True b. False 51. Wei is a general partner in the WST partnership. During the current year, she receives a guaranteed payment of $10,000 for services she provides to the partnership, and her distributive share of partnership income is $30,000. Wei is required to pay self-employment tax on the $10,000 guaranteed payment but not on her distributive share of partnership income. a. True b. False 52. Maria owns a 60% interest in the KLM Partnership. Four years ago, her father gave her a parcel of land. The gift basis of the land to Maria is $60,000. In the current year, Maria still had not figured out how to use the land for her own personal or business use; consequently, she sold it to the partnership for $50,000. The partnership immediately started using the land as a parking lot for its employees. Maria may recognize her $10,000 loss on the sale. a. True b. False 53. Gina is a single taxpayer and an active partner in the GMA LLC. Gina’s Schedule K-1 reflects a $20,000 ordinary income share, $2,000 of interest income, and a $10,000 guaranteed payment for services. Gina’s self-employment income from other sources and modified adjusted gross income is about $300,000. With respect to the income from the LLC, Gina is subject to the 0.9% additional Medicare tax on $30,000 and the 3.8% net investment income tax of $2,000. a. True b. False 54. One of the disadvantages of the partnership form is that the partner’s share of the partnership’s taxable income is taxed to the partner even if it is not distributed. a. True b. False 55. The total tax burden on entity income is greater for a partner in a partnership (up to 37% for an individual partner) than on a shareholder in a corporation (21% for an individual shareholder), so partnerships are used only in special situations. a. True b. False 56. The qualified business income deduction is calculated at the partner level. The partnership reports information the partner needs to calculate the deduction, such as W-2 wages and the unadjusted basis of the partnership’s depreciable property. a. True b. False 57. The partner's ending tax basis capital account plus the partner's allocable share of liabilities (both amounts shown on Schedule K-1) will typically equal the partner's ending tax basis in the partnership. a. True Powered by Cognero

Page 7


Name:

Class:

Date:

Chapter 09: Partnerships Formation Operations and Basis b. False Multiple Choice 58. Which of the following entity owners cannot participate in the management of an entity? a. A general partner in a general partnership. b. A member of a limited liability company. c. A partner in a limited liability partnership. d. A limited partner in a limited partnership. 59. Which one of the following statements is true regarding a partner’s personal liability for partnership debts? a. LLC members can never be liable for entity debts. b. In a limited partnership, all partners have limited liability for partnership debts. c. In a limited liability partnership, a partner might be subject to liability for other partners’ malpractice. d. In a general partnership, all partners are liable for entity debts. 60. Which one of the following statements regarding partnership taxation is incorrect? a. A partnership is liable for federal income taxes based upon its taxable income. b. Partnership income is comprised of ordinary partnership income or loss and separately stated items. c. A partnership is required to file a return with the IRS. d. A partner’s profit-sharing percentage may differ from the partner’s loss-sharing percentage. 61. Which of the following is a correct definition of a concept related to partnership taxation? a. The aggregate concept treats partners and partnerships as separate units and gives the partnership its own tax personality. b. A partner’s capital-sharing ratio is defined as the percentage of partnership assets (capital) that would be allocated to the partner upon liquidation of the partnership. c. The partnership’s outside basis is defined as the sum of each partner’s capital account balance. d. A special allocation is defined as an amount that could differently affect the tax liabilities of two or more partners. 62. Which one of the following is an example of a special allocation of partnership income? a. The partnership’s capital gains and losses are shown separately on Schedule K-1. b. Distributions from the partnership to the partner are shown on Schedule K-1 line 20. c. The partnership agreement provides that all charitable contributions will be allocated to a specific partner rather than the partner's 20% distributive share. d. The Schedule K-1 reports each partner’s share of the information they need to calculate the § 199A (qualified business income) deduction. 63. On January 1 of the current year, Anna and Jason form an equal partnership. Anna contributes $50,000 cash and a parcel of land (adjusted basis of $200,000; fair market value of $150,000) in exchange for her interest in the partnership. Jason contributes property (adjusted basis of $180,000; fair market value of $200,000) in exchange for his partnership interest. Which of the following statements is true concerning the income tax results of this partnership formation? a. Jason recognizes a $20,000 gain on his property transfer. b. Jason has a $200,000 tax basis for his partnership interest. Powered by Cognero

Page 8


Name:

Class:

Date:

Chapter 09: Partnerships Formation Operations and Basis c. Anna has a $250,000 tax basis for her partnership interest. d. The partnership has a $150,000 adjusted basis in the land contributed by Anna. 64. Tim, Al, and Pat contributed assets to form the equal TAP Partnership. Tim contributed cash of $40,000 and land with a basis of $80,000 (fair market value of $60,000). Al contributed cash of $60,000 and land with a basis of $50,000 (fair market value of $40,000). Pat contributed cash of $60,000 and a fully depreciated property ($0 basis) valued at $40,000. Which of the following tax treatments is not correct? a. Tim’s basis in his partnership interest is $120,000. b. Al realizes and recognizes a loss of $10,000. c. Pat realizes a gain of $40,000 but recognizes $0 gain. d. TAP has a basis of $80,000, $50,000, and $0 in the land and property (excluding cash) contributed by Tim, Al, and Pat, respectively. 65. Xena and Xavier form the XX LLC. Xena contributes cash of $20,000, land (basis = $40,000; fair market value = $25,000), equipment (basis = $0; fair market value = $35,000), and inventory (basis = $30,000; fair market value = $40,000). Xavier contributed $120,000 of cash. How much is the partnership’s basis in the land, equipment, and inventory, and how much is Xena’s basis in the partnership interest? a. $25,000 land, $0 equipment, $30,000 inventory; $55,000 partnership interest. b. $40,000 land, $0 equipment, $30,000 inventory; $90,000 partnership interest. c. $25,000 land, $35,000 equipment, $30,000 inventory; $105,000 partnership interest. d. $40,000 land, $35,000 equipment, $40,000 inventory; $135,000 partnership interest. 66. In which of the following independent situations is the transaction most likely to be characterized as a disguised sale? a. Partner George contributes appreciated property to the GM Partnership, and three years later GM distributes $100,000 proportionately to the partners. b. Brianna contributes property with a basis of $20,000 and a fair market value of $50,000 to the BGB Partnership in exchange for a 20% interest therein. The partnership agrees to distribute $20,000 to Brianna in 15 months if partnership cash flows from operations exceed $100,000 at that time. The partnership does not expect to produce operating cash flows of over $100,000 for at least five years. c. Luis contributes appreciated property to the BLP Partnership. Thirty months later, he receives a distribution from the partnership of $15,000 cash. None of the other partners received a distribution. There was no agreement that BLP would make the distribution, and Luis would have made the contribution whether or not the partnership made the distribution. d. Partner Skylar contributes appreciated property to the equally-owned four-member SANE LLC in exchange for a 25% interest. After 20 months, the LLC distributes $10,000 to partners Azariah, Nikita, and Eastyn, and $50,000 to Skylar. 67. Tara and Robert formed the TR Partnership four years ago. Because they decided the company needed some expertise in multimedia presentations, they offered Katie a one-third interest in partnership capital if she would come to work for the partnership. On July 1 of the current year, the unrestricted partnership interest (fair market value of $25,000) was transferred to Katie. How should Katie treat the receipt of the partnership interest in the current year? a. Nontaxable. b. Carried interest. c. $25,000 ordinary income. d. $25,000 long-term capital gain. 68. Which of the following would be currently taxable as ordinary income to the service partner if received in exchange Powered by Cognero

Page 9


Name:

Class:

Date:

Chapter 09: Partnerships Formation Operations and Basis for services performed for the partnership? (In all cases, assume that the interest is not sold within two years after the time it is granted to the service partner.) a. A 10% interest in the capital of the partnership that will vest if the partner remains in the partnership for three years. b. A 20% interest in the future profits of the partnership received in exchange for future services to be performed for the partnership. c. A 25% interest in the capital of the partnership when there are no restrictions on transferability of the interest. d. A 30% interest in the capital of the partnership when the partner contributes intangible property with a $0 basis that the partner developed. 69. Which of the following is an election or calculation made by the partner rather than the partnership? a. Calculation of a § 199A (qualified business income) deduction amount. b. Tax treatment (e.g., credit, amortization) of research and experimental costs. c. The partnership’s accounting method (e.g., cash, accrual). d. Claiming a § 179 deduction related to property acquired by the partnership. 70. Which of the following statements is always correct regarding assets acquired by a newly formed partnership? If a partner contributes: a. Depreciable property: The partnership treats the property as newly acquired depreciable property and may claim a § 179 deduction. b. Unrealized (cash-basis) receivables: The partnership will report a capital gain when the receivable is collected. c. Inventory (in the partner’s hands): The partnership reports ordinary income if the property is held as a capital asset and sold within five years of the contribution date. d. Land valued at less than its basis: The partnership reports a § 1231 (ordinary) loss if the property is sold at a loss within five years of the contribution date. 71. The partner (rather than the partnership) will make which of the following elections? a. To claim straight-line depreciation. b. To claim a credit or deduction for foreign taxes paid. c. To claim a low-income housing credit. d. To claim a § 179 deduction for certain property placed in service during the year. 72. Which of the following statements is always true regarding accounting methods available to a partnership? a. If a partnership is a tax shelter, it can use the cash method of accounting. b. If a nontax-shelter partnership had average annual gross receipts of less than $30,000,000 (2024) for the last three tax years, it can use the cash method. c. If a partnership has a partner that is a personal service corporation, it cannot use the cash method. d. If a partnership has a partner that is a C corporation, it cannot use the cash method. 73. ACME Partnership, a consulting firm, has had the following gross receipts since its formation: $22,800,000 in year 1, $28,600,000 in year 2, $39,800,000 in year 3, and $41,700,000 in year 4. ACME is not a tax shelter. Assume the gross receipts threshold for Year 3 is $29,000,000 and Year 4 is $30,000,000. In which tax years must ACME start using the accrual method? a. Year 3 if none of the partners is a C corporation. b. Year 4 if none of the partners is a C corporation. c. Year 3 if one of the partners is a C corporation. Powered by Cognero

Page 10


Name:

Class:

Date:

Chapter 09: Partnerships Formation Operations and Basis d. Year 4 if one of the partners is a C corporation. 74. Fern, Inc., Ivy, Inc., and Jeremy formed a general partnership. Fern owns a 50% interest, and Ivy and Jeremy both own 25% interests. Fern, Inc., files its tax return on an October 31 year-end; Ivy, Inc., files with a May 31 year-end, and Jeremy is a calendar year taxpayer. Which of the following statements is true regarding the taxable year the partnership can choose? a. The partnership must choose the calendar year because it has no principal partners. b. The partnership must choose an October year-end because Fern, Inc., is a principal partner. c. The partnership can request permission from the IRS to use a January 31 fiscal year under § 444. d. The partnership must use the least aggregate deferral method to determine its required taxable year. 75. SQRLY LLC has about 25 LLC members. SwanCo (30% owner) and QuinnCo (16% owner) both have June 30 tax year-ends. Royce, Inc.; Larry, Inc.; and Yolanda, Inc. each own 4% (12% total) and have September 30 taxable year-ends. Each of the other LLC members (42% total) owns interests of 4% or less and use the calendar year (December 31). Which of the following statements is true regarding the LLC’s required taxable year end? a. The taxable year is determined under the least aggregate deferral rule. b. The taxable year is determined under the majority interest rule because a majority of members have the same year-end. c. The taxable year is determined under the principal partner rule because the 5% owners (SwanCo and QuinnCo) have the same taxable year. d. The taxable year ends on December 31 because more LLC members use a calendar year than any other year. 76. In the current year, the POD Partnership received revenues of $200,000 and paid the following amounts: $50,000 in rent and utilities and $20,000 as a distribution to partner Olivia. In addition, the partnership earned $6,000 of long-term capital gains during the year. Partner Donald owns a 50% interest in the partnership. How much income must Donald report for the tax year? a. $68,000 ordinary income. b. $78,000 ordinary income. c. $65,000 ordinary income; $3,000 of long-term capital gains. d. $75,000 ordinary income; $3,000 of long-term capital gains. 77. DIP LLC reports ordinary income (before guaranteed payments) of $120,000, rent expense of $40,000, and interest income of $4,000 for the year. In addition, DIP paid guaranteed payments of $20,000 to partner Percy. If Percy owns a 40% capital and profits interest, how much income will he report for the year and what is its character? a. $24,000 ordinary income. b. $24,000 ordinary income, $1,600 interest income, $20,000 guaranteed payment. c. $32,000 ordinary income, $1,600 interest income. d. $32,000 ordinary income, $1,600 interest income, $20,000 guaranteed payment. 78. Kristie is a 30% partner in the KKM Partnership. During the current year, KKM reported gross receipts of $280,000 and a charitable contribution of $30,000. The partnership paid office expenses of $80,000. In addition, KKM distributed $20,000 each to partners Kaylyn and Megan, and paid partner Kaylyn $20,000 for administrative services. Kristie reports the following income from KKM during the current tax year. a. $54,000 ordinary income; $9,000 charitable contribution. b. $60,000 ordinary income; $9,000 charitable contribution. c. $33,000 ordinary income. Powered by Cognero

Page 11


Name:

Class:

Date:

Chapter 09: Partnerships Formation Operations and Basis d. $54,000 ordinary income. 79. Which of the following is not shown on the partnership’s Schedule K of Form 1065? a. The partnership’s self-employment income. b. The partnership’s separately stated income and deductions. c. The partnership’s tax preference and adjustment items. d. The partnership’s net operating loss carryforward. 80. Concerning a partnership’s Form 1065, which of the following statements is not true? a. The partnership reconciles its "Income (Loss) per Books" with "Income (Loss) per Return" on Schedule M-1 or M-3. b. The partnership balance sheet on Schedule L can be presented on a financial (book) basis. c. All taxable/deductible partnership income and expense items are reported on Form 1065, page 1. d. The partnership’s equivalent of taxable income is reported in the “Analysis of Income (Loss).” 81. ABC LLC reported the following items on the LLC’s Schedule K: ordinary income, $100,000; interest income, $3,000; long-term capital loss, ($4,000); charitable contributions, $1,000; AMT depreciation adjustment, $10,000; and cash distributions to partners, $50,000. How much will ABC show as net income (loss) on its Analysis of Income (Loss)? a. $68,000 b. $78,000 c. $95,000 d. $98,000 82. Which of the following is not a requirement of the substantial economic effect test? a. Income, gains, losses, and deductions must be allocated to the partners in accordance with their capital contributions. b. An allocation of income must increase the partner’s § 704(b) book capital account balance, and an allocation of deduction must decrease the partner’s § 704(b) book capital account balance. c. A partner with a negative § 704(b) book capital account balance must restore that capital account, generally by contributing cash to the partnership. d. On liquidation of the partner’s interest in the partnership, the partner must receive assets that have a fair market value equal to that partner’s (positive) § 704(b) book capital account balance. 83. Which of the following allocations is most likely to meet the substantial test in the substantial economic effect rules? (Assume all economic effect tests are met.) a. The ROY LLC specially allocates $20,000 of income each year to partner Red with no offsetting loss allocations in other years. b. The YGB LLC specially allocates $30,000 of ordinary income this year to partner Green with an offsetting allocation of loss in that same amount next year. c. The BPV LLC specially allocates $10,000 of capital gains to Violet and $10,000 of interest income to Purple because Purple is in a lower tax bracket. d. The PIR LLC specially allocates $60,000 of income to Indigo with no offsetting allocations. Indigo has expiring net operating losses. 84. Brooke and John formed a partnership. Brooke received a 40% interest in partnership capital and profits in exchange for contributing land (basis of $30,000 and fair market value of $120,000). John received a 60% interest in partnership Powered by Cognero

Page 12


Name:

Class:

Date:

Chapter 09: Partnerships Formation Operations and Basis capital and profits in exchange for contributing $180,000 of cash. Three years after the contribution date, the land contributed by Brooke is sold by the partnership to a third party for $150,000. How much taxable gain will Brooke recognize from the sale? a. $102,000 b. $90,000 c. $48,000 d. $36,000 85. Mark and Addison formed a partnership. Mark received a 25% interest in partnership capital and profits in exchange for land with a basis of $40,000 and a fair market value of $60,000. Addison received a 75% interest in partnership capital and profits in exchange for $180,000 of cash. Three years after the contribution date, the land contributed by Mark is sold by the partnership to a third party for $76,000. How much taxable gain will Mark recognize from the sale? a. $0 b. $9,000 c. $16,000 d. $24,000 86. Molly is a 30% partner in the MAP Partnership. During the current tax year, the partnership reported ordinary income of $200,000 before any permitted deduction for guaranteed payments and distributions to partners. The partnership made an ordinary cash distribution of $20,000 to Molly and made guaranteed payments to partners Molly, Amber, and Pat of $20,000 each ($60,000 total guaranteed payments). How much will Molly’s adjusted gross income increase as a result of these items? a. $36,000 b. $42,000 c. $60,000 d. $62,000 87. Stephanie is a calendar year cash basis taxpayer. She owns a 50% profit and loss interest in a cash basis partnership with a September 30 year-end. The partnership’s operating income (after deducting guaranteed payments) was $120,000 ($10,000 per month) and $144,000 ($12,000 per month), respectively, for the partnership tax years ended September 30, 2024 and 2025. The partnership paid guaranteed payments to Stephanie of $2,000 and $3,000 per month during the fiscal years ended September 30, 2024 and 2025. How much will Stephanie’s adjusted gross income be increased by these partnership items for her tax year ended December 31, 2024? a. $60,000 b. $72,000 c. $84,000 d. $90,000 88. Ryan is a 25% partner in the ROCC Partnership. At the beginning of the tax year, his basis in the partnership interest was $90,000, including his share of partnership liabilities. During the current year, ROCC reported net ordinary income of $100,000. In addition, ROCC distributed $10,000 to each of the partners ($40,000 total). At the end of the year, Ryan’s share of partnership liabilities increased by $10,000. His basis in the partnership interest at the end of the year is: a. $90,000. b. $100,000. c. $115,000. d. $125,000. Powered by Cognero

Page 13


Name:

Class:

Date:

Chapter 09: Partnerships Formation Operations and Basis 89. Brad is a 40% member in the BB LLC. At the beginning of the tax year, his tax basis capital account showed a balance of $120,000. In this case, his capital account equals his tax basis in the LLC interest excluding his share of the LLC’s debts. His prior year-end Schedule K-1 showed recourse debt (guaranteed by Brad) and nonrecourse debt of $10,000 and $20,000, respectively. During the current year, BB reported net ordinary income of $200,000 and nondeductible expenses of $2,000. There were no distributions during the year. At the end of the year, Brad’s Schedule K-1 showed recourse (guaranteed) and nonrecourse debt of $20,000 and $30,000, respectively. How much is Brad’s basis in the LLC interest at the end of the year? a. $199,200. b. $200,000. c. $249,200. d. $250,000. 90. Allison is a 40% partner in the BAM Partnership. At the beginning of the tax year, her basis in the partnership interest was $100,000, including her share of partnership liabilities. During the current year, BAM reported an ordinary loss of $60,000 (before the following payments to the partners). In addition, BAM made an ordinary distribution of $8,000 to Allison and paid partner Brian a $20,000 consulting fee. At the end of the year, Allison’s share of partnership liabilities decreased by $10,000. Assuming loss limitation rules do not apply, Allison’s basis in the partnership interest at the end of the year is: a. $2,000. b. $50,000. c. $58,000. d. $70,000. 91. Binita contributed property with a basis of $40,000 and a value of $50,000 to the BE Partnership in exchange for a 20% interest in partnership capital and profits. During the first year of partnership operations, BE had net taxable income of $30,000 and tax-exempt interest income of $10,000. The partnership distributed $10,000 cash to Binita. Her adjusted basis (outside basis) for her partnership interest at year-end is: a. $36,000. b. $38,000. c. $60,000. d. $70,000. 92. At the beginning of the year, Heather’s tax basis capital account balance in the HEP Partnership was $85,000. During the tax year, Heather contributed property with a basis of $6,000 and a fair market value of $10,000. Her share of the partnership’s ordinary income and separately stated income and deduction items was $40,000. At the end of the year, the partnership distributed $15,000 of cash to Heather. In addition, the partnership allocated $12,000 of recourse debt and $10,000 of nonrecourse debt to Heather. What is Heather’s ending capital account balance determined using the tax basis method? a. $116,000 b. $120,000 c. $126,000 d. $128,000 93. At the beginning of the year, Ryan’s tax basis capital account balance in the RUS Partnership (in which he owned a 40% interest) was $200,000. During the year, Ryan contributed cash ($40,000) and property (basis = $20,000, fair market value = $30,000). RUS reported ordinary income of $100,000 and tax-exempt income of $6,000. At the end of the year, the partnership distributed $6,000 of cash to Ryan. On the Schedule K-1, the partnership shows that Ryan had a $50,000 Powered by Cognero

Page 14


Name:

Class:

Date:

Chapter 09: Partnerships Formation Operations and Basis share of nonrecourse LLC debt at the end of the year. How much is Ryan’s ending tax basis capital account balance? a. $294,000. b. $296,400. c. $306,400. d. $346,400. 94. Misty and John formed the MJ Partnership. Misty contributed $50,000 of cash in exchange for her 50% interest in the partnership capital and profits. During the first year of partnership operations, the following events occurred: the partnership had a net taxable income of $20,000; Misty received a distribution of $12,000 cash from the partnership; and Misty had a 50% share in the partnership’s $60,000 of recourse liabilities on the last day of the partnership year. Misty’s adjusted basis for her partnership interest at year end is: a. $48,000. b. $60,000. c. $78,000. d. $88,000. 95. Which of the following statements is correct regarding the manner in which partnership liabilities are reflected in the partners’ bases in their partnership interests? a. Nonrecourse debt is allocated to the partners based on the partners' economic risk of loss. b. Recourse debt is allocated to the partners according to their profit-sharing ratios. c. An increase in partnership debts results in a decrease in the partners’ bases in the partnership interest. d. A decrease in partnership debt is treated as a distribution from the partnership to the partner and reduces the partner’s basis in the partnership interest. 96. AmCo and BamCo form the AB General Partnership at the start of the current year with a land contribution by BamCo and a cash contribution by AmCo. BamCo’s contributed property is subject to a recourse mortgage assumed by the partnership. BamCo has an 80% interest in AB’s profits and losses. The land has been held by BamCo for the past 6 years as an investment. It will be used by AB as an operating asset in its parking lot business. Which of the following statements is correct? a. Immediately after formation, AmCo’s basis in the partnership equals the cash that it contributed. b. Immediately after formation, AmCo’s basis in the partnership equals the cash that it contributed plus AmCo's share of the recourse debt contributed by BamCo. c. Because the debt is recourse, it can be allocated only to the general partners if one of them personally guarantees the debt. d. AB’s basis in the land contributed by BamCo equals BamCo’s basis in the land immediately before the contribution date, less the amount of the recourse debt assumed by the partnership. 97. Sharon contributed property to the newly formed QRST Partnership. The property had a $100,000 adjusted basis to Sharon and a $160,000 fair market value on the contribution date. The property was also encumbered by a $90,000 nonrecourse debt, which was transferred to the partnership on that date. Sharon is treated as a general partner. She is allocated 30% of QRST's profits and 20% of QRST's losses. Sharon's basis in the partnership interest after the formation transaction is: a. $28,000. b. $37,000. c. $88,000. d. $127,000. Powered by Cognero

Page 15


Name:

Class:

Date:

Chapter 09: Partnerships Formation Operations and Basis 98. Which of the following is not a specific adjustment to the partners’ basis in the partnership interest? a. Increased by contributions the partner made to the partnership. b. Decreased by the amount of guaranteed payments shown on the partner’s Schedule K-1. c. Increased by the partner’s share of tax-exempt income. d. Decreased by any decrease in the partner’s share of partnership liabilities. 99. Rebecca is a limited partner in the RST Partnership, which is not publicly traded. Her allocable share of RST’s passive ordinary losses from a nonrealty activity for the current year is ($60,000). Rebecca has a $40,000 adjusted basis (outside basis) for her interest in RST (before deduction of any of the passive losses). Her amount “at risk” is $30,000 (before deduction of any of the passive losses). She also has $25,000 of passive income from other sources. She has no business losses for the year from other sources. How much of her ($60,000) allocable RST loss can Rebecca deduct on her currentyear tax return? a. $25,000 b. $30,000 c. $40,000 d. $60,000 100. At the beginning of the tax year, Zach’s basis for his partnership interest and his amount at risk in the partnership was $30,000. His share of partnership items for the year consisted of tax-exempt interest income of $2,000 and an ordinary loss of $44,000. He also received a distribution of $20,000 cash from the partnership during the year. He is an active general partner and has no passive income or business losses from other sources. For the tax year, Zach will report: a. A nontaxable distribution of $20,000, an ordinary loss of $10,000, and a suspended loss carryforward of $34,000. b. An ordinary loss of $32,000, a suspended loss carryforward of $12,000, and a taxable distribution of $20,000. c. A nontaxable distribution of $20,000, an ordinary loss of $12,000, and a suspended loss carryforward of $32,000. d. An ordinary loss of $44,000 and a nontaxable distribution of $20,000. 101. Meredith is a passive 30% member of the MNO LLC. She is not a managing member and she does not participate in any activities of the LLC. Her interest is more in the nature of an investment. In the current year, Meredith’s distributive share of income from the LLC was $50,000. In addition, she received a guaranteed payment of $40,000 for the use of her capital. Assume that her income from other sources exceeds $500,000. How much of Meredith’s LLC income will be subject to the self-employment (SE) tax and the net investment income (NII) tax? (Disregard the additional Medicare tax on upper-income taxpayers.) a. $0 SE tax; $0 NII tax. b. $0 SE tax; $40,000 NII tax. c. $0 SE tax; $90,000 NII tax. d. $50,000 SE tax; $40,000 NII tax. 102. Paul sells one parcel of land (basis of $100,000) for its fair market value of $160,000 to a partnership in which he owns a 60% capital interest. Paul held the land for investment purposes. The partnership is in the real estate development business and will build residential housing (for sale to customers) on the land (the land is inventory to the partnership). Paul will recognize: a. $0 gain or loss. b. $36,000 ordinary income. c. $36,000 capital gain. d. $60,000 ordinary income. Powered by Cognero

Page 16


Name:

Class:

Date:

Chapter 09: Partnerships Formation Operations and Basis 103. Samuel is the managing general partner of STU in which he owns a 25% interest. For the year, STU reported ordinary income of $400,000 (after deducting all guaranteed payments). In addition, the LLC reported interest income of $12,000. Samuel received a guaranteed payment of $120,000 for services he performed for STU. How much income from self-employment did Samuel earn from STU? a. $100,000 b. $120,000 c. $220,000 d. $223,000 104. Which of the following is not a correct statement regarding the advantages of the partnership entity form over the C corporation form? a. A partnership typically has easier administrative and filing requirements than does a C corporation. b. Partnership income is subject to a single level of taxation; corporate income is double taxed. c. Partnerships may specially allocate income and expenses among the partners provided the substantial economic effect requirements are met; corporate dividends must be proportionate to shareholdings. d. Partners in a general partnership have less personal liability for entity claims than shareholders of a C corporation. 105. Which of the following statements is correct regarding potential partnership or C corporation taxes? a. Partnership income is always taxed at higher rates than for a Subchapter C corporation because the individual tax rates are higher than corporate tax rates. b. Partners pay a single tax on their distributive shares of income at the tax rate that applies to the partner. c. C corporations pay a single level of tax on corporate income at rates up to 35%. d. Partnership income is preferred because the income is not subject to employment taxes and the income might be eligible for the qualified business income deduction. Matching Match each of the following statements with the numbered terms below that provide the best definition. a. Organizational choice of many large accounting firms. b. Partner’s allocation of partnership items, in general. c. Might affect any two partners’ tax liabilities in different ways. d. Amount that might be reported on either form 1065, page 1 or, on Schedule K. e. Transfer of asset to partnership followed by immediate distribution of cash to partner. f. Must have at least one general and one limited partner. g. Long-term capital gain might be recharacterized as ordinary income. h. All partners are jointly and severally liable for entity debts. i. Theory treating the partner and partnership as separate economic units. j. Partner’s basis in partnership interest after tax-free contribution of asset to partnership. k. Partnership’s basis in asset after tax-free contribution of asset to partnership. l. Owners are members. m. Theory treating the partnership as a collection of taxpayers joined in an agency relationship. n. Participates in management. o. Not liable for entity debts. Powered by Cognero

Page 17


Name:

Class:

Date:

Chapter 09: Partnerships Formation Operations and Basis p. No correct match provided. 106. Limited partnership 107. General partner 108. Distributive share 109. Limited liability partnership 110. Carried interest 111. Aggregate concept 112. Substituted 113. Limited liability company 114. Limited partner 115. Publicly traded partnership 116. Disguised sale 117. Interest expense 118. Separately stated item 119. Carryover 120. Entity concept 121. General partnership Match each of the following statements with the terms below that provide the best definition. a. Adjusted basis of each partnership asset. b. Operating expenses incurred after entity is formed but before it begins doing business. c. Each partner’s basis in the partnership. d. Reconciles book income to taxable income. e. Percentage allocation of most nonrecourse debt. f. Tax accounting election made by partnership. g. Tax accounting calculation made by partner. h. Tax accounting election made by partner. i. Rolls forward tax basis capital accounts. j. Designed to prevent excessive deferral of taxation of partnership income. k. Amount that may be received by partner for performance of services for the partnership. l. Concept under which a partnership's recourse debt is shared among the partners. m. Recourse debt, if not guaranteed and all allocations have been proportionate to capital. Powered by Cognero

Page 18


Name:

Class:

Date:

Chapter 09: Partnerships Formation Operations and Basis n. Will eventually be allocated to partner making tax-free property contribution to partnership. o. Partner’s share of partnership items. p. Any allocation to the partner must generally meet this requirement. q. Used to determine liquidating distributions. r. Shown on Schedules K-1 and does not include liabilities. 122. Tax basis capital account 123. Required taxable year 124. Foreign tax credit vs. deduction 125. Loss-sharing ratio 126. Schedule K-1 127. Inside basis 128. Section 704(b) book capital account 129. Start-up costs 130. Schedule M-1 131. § 179 deduction 132. Economic effect test 133. Precontribution gain 134. Profit-sharing ratio 135. Schedule M-2 136. Qualified business income deduction 137. Outside basis 138. Guaranteed payment 139. Economic risk of loss Subjective Short Answer 140. Mateo and James are forming the MJ Partnership. Mateo contributes $600,000 cash and James contributes nondepreciable property with an adjusted basis of $400,000 and a fair market value of $750,000. The property is subject to a $150,000 liability, which is transferred into the partnership and is shared equally by the partners for basis purposes. Mateo and James share in all partnership profits equally except for any precontribution gain, which must be allocated according to the statutory rules for built-in gain allocations. Powered by Cognero

Page 19


Name:

Class:

Date:

Chapter 09: Partnerships Formation Operations and Basis a.

What is James’s adjusted tax basis for his partnership interest immediately after the partnership is formed?

b.

What is the partnership’s adjusted basis for the property contributed by James?

c.

If the partnership sells the property contributed by James for $800,000, how is the tax gain allocated between the partners?

141. Palmer contributes property with a fair market value of $4,000,000 and an adjusted basis of $3,000,000 to AP Partnership. Palmer shares in $3,000,000 of partnership debt under the liability sharing rules, giving him an initial adjusted basis for his partnership interest of $6,000,000. One month after the contribution, Palmer receives a cash distribution from the partnership of $2,000,000. Palmer would not have contributed the property if the partnership had not contractually obligated itself to make the distribution. Assume that Palmer’s share of partnership liabilities will not change as a result of this distribution. a.

Under the IRS’s likely treatment of this transaction, what is the amount of gain or loss that Palmer will recognize because of the $2,000,000 cash distribution?

b.

What is the partnership’s basis for the property after the distribution?

c.

If Palmer is unhappy with this result, can you suggest a possible alternative that may provide him with a better answer?

142. During the current year, MAC Partnership reported the following items of receipts and expenditures: $600,000 sales, $80,000 utilities and rent, $200,000 salaries to employees, $20,000 guaranteed payment to partner Antonio for services to the partnership, investment interest income of $4,000, a charitable contribution of $8,000, and a distribution of $30,000 to partner Carl. Antonio is a 25% general partner. Based on this information, what information will be shown on Antonio’s Schedule K-1? What income and deductions will Antonio report? What taxes and other calculations might Antonio need to report? 143. The LN partnership reported the following items of income and deduction during the current tax year: revenues, $300,000; cost of goods sold, $160,000; tax-exempt interest income, $2,000; salaries to employees, $80,000; and longterm capital gain, $10,000. It paid business interest expense of $18,000 and investment interest expense of $2,000. In addition, the partnership distributed $20,000 of cash to 50% partner Nina and $10,000 of cash to 50% partner Len. What is Nina’s share of ordinary partnership income and separately stated items? 144. Carli contributes land to the newly formed CD Partnership in exchange for a 30% interest. The land has an adjusted basis and fair market value of $300,000 and is subject to a liability of $100,000, which the partnership assumes. None of this liability is repaid at year-end. At the end of the year, the partnership owes payables of $20,000. Assume that all liabilities are allocated proportionately to the partners. Total partnership income for the year is $400,000. What is Carli’s basis in her partnership interest at the end of the year? 145. An examination of the RB Partnership’s tax books provides the following information for the current year. Operating (ordinary) income before guaranteed payments Long-term capital gain Guaranteed payment to Rachel for services Cash distributions to Rachel Interest on Colorado state bonds (exempt interest income) Powered by Cognero

$300,000 6,000 30,000 (20,000) 2,000 Page 20


Name:

Class:

Date:

Chapter 09: Partnerships Formation Operations and Basis Charitable contributions made by partnership Decrease in all partnership liabilities from 1/1-12/31

(10,000) (20,000)

Rachel is a 30% general partner in partnership capital, profits, and losses. Assume that the adjusted basis of her partnership interest (including liability share) is $60,000 at the beginning of the year, and she shares in 30% of the partnership’s liabilities for basis purposes. a.

What is Rachel’s adjusted basis for the partnership interest at the end of the year?

b.

How much income must Rachel report on her tax return for the current year? What deductions might be available? What is the character of the income and what types of tax might apply to it?

146. Katherine invested $80,000 this year to purchase a 30% interest in the KLM Partnership. The partnership reported $200,000 of net income from operations, a $2,000 short-term capital loss, and a $10,000 charitable contribution. In addition, the partnership distributed $20,000 to Katherine and $10,000 each to partners Lauren and Missy. If the partnership has no beginning or ending liabilities, what is Katherine’s basis in her partnership interest at the end of the year? 147. Sarah contributed fully depreciated ($0 basis) property valued at $50,000 to the RSTU Partnership in exchange for a 25% interest in partnership capital and profits. During the first year of partnership operations, RSTU had net taxable income of $200,000 and tax-exempt income of $4,000. The partnership distributed $10,000 cash to Sarah. Her share of partnership recourse liabilities on the last day of the partnership year was $20,000. What is Sarah’s adjusted basis (outside basis) for her partnership interest at the end of the tax year? 148. In the current year, the DOE LLC received revenues of $200,000 and paid the following amounts: $50,000 of business expenses (rent, utilities, wages, depreciation, etc.), a $40,000 guaranteed payment (for services) to 50% member Dave, $10,000 to member Ethan for consulting services, and $10,000 as a distribution to member Olivia. In addition, the LLC earned $2,000 of tax-exempt interest income during the year. Dave is the managing member of the LLC. His basis in his LLC interest was $50,000 at the beginning of the year, which includes a $12,000 share of LLC liabilities. At the end of the year, his share of the LLC’s liabilities was $20,000. a. How much income must Dave report for the tax year and what is the character of the income? b. What is Dave’s basis in his LLC interest at the end of the tax year? c. On what income will Dave’s self-employment tax be calculated? d. What is the maximum amount Dave might be able to deduct for this business under § 199A? What additional information would Dave need to make this calculation? 149. Sharon and Sue are equal partners in the S&S Partnership. On January 1 of the current year, each partner’s adjusted basis in S&S was $80,000 (including each partner’s $20,000 share of the partnership’s $40,000 of liabilities). During the current year, S&S repaid $30,000 of the debt and borrowed $20,000 for which Sharon and Sue are equally liable. In the current year ended December 31, S&S also sustained a net operating loss of $40,000 and earned $10,000 of interest income from investments. If liabilities are shared equally by the partners, on January 1 of the next year, how much is each partner’s basis in her interest in S&S? 150. In the current year, the CAR Partnership received revenues of $400,000 and paid the following amounts: $160,000 in rent, utilities, and salaries; a $40,000 guaranteed payment to partner Ryan; $20,000 to partner Amy for consulting services; and a $40,000 distribution to 25% partner Cameron. In addition, the partnership realized a $12,000 net long-term capital gain. Cameron’s basis in his partnership interest was $60,000 at the beginning of the year which included his Powered by Cognero

Page 21


Name:

Class:

Date:

Chapter 09: Partnerships Formation Operations and Basis $25,000 share of partnership liabilities. At the end of the year, his share of partnership liabilities was $15,000. a.

How much income must Cameron report for the tax year?

b.

What is Cameron’s basis in the partnership interest at the end of the year?

151. In the current year, Derek formed an equal partnership with Cody. Derek contributed land with an adjusted basis of $110,000 and a fair market value of $200,000. Derek also contributed $50,000 cash to the partnership. Cody contributed land with an adjusted basis of $80,000 and a fair market value of $230,000. The land contributed by Derek was encumbered by a $60,000 nonrecourse debt. The land contributed by Cody was encumbered by $40,000 of nonrecourse debt. Assume that the partners share debt equally. Immediately after the formation, what is the basis of Cody’s partnership interest? 152. Morgan is a 50% managing member in the calendar year, cash basis MKK LLC. The LLC received $150,000 income from services and paid the following other amounts. Rent expense Salary expense to employees Payment to Morgan for services per the operating agreement Distributions to partners Kristin and Katie ($6,000 each) Payment to 30% cash basis partner Katie for tax and accounting services

$10,000 40,000 40,000 12,000 10,000

How much will Morgan’s adjusted gross income increase as a result of these items? What other deductions must be considered? What amount will be included in Morgan’s self-employment tax calculation?

153. Cassandra is a 10% limited partner in C&C, Ltd. Her basis in the interest is $60,000 before loss allocations, including her $30,000 share of the partnership’s nonrecourse debt. (This debt is not qualified nonrecourse financing.) Cassandra is also a 10% limited partner in MNOP in which her basis is $30,000. Cassandra is allocated an $80,000 loss from C&C and $20,000 of income from MNOP. How much of the loss from C&C may Cassandra deduct? Under what Code provisions are the remaining losses suspended? Assume that Cassandra has no business losses from other sources. 154. The MOP Partnership is involved in construction activities. On January 1 of the current year, Patricia has an adjusted basis of $650,000 for her partnership interest consisting of the following. Capital account Share of partnership recourse debt Share of partnership nonrecourse debt

$400,000 50,000 200,000 $650,000

During the year, the partnership has an operating loss of $1.2 million and distributes $60,000 of cash to Patricia. Partnership liabilities were the same at the end of the tax year, and the nonrecourse debt is not qualified nonrecourse debt. If she owns a 60% share of partnership profits, capital, and losses, and is an active (material) participant in the partnership, how much of her share of the operating loss can Patricia deduct? (Assume that Patricia is a single taxpayer and has no business losses from other sources.) What Code provisions could cause a suspension of the loss? How would your answer change if MOP were an LLC and Patricia had not personally guaranteed any of the debt? 155. Jeordie and Kendis created the JK Partnership by contributing $100,000 each. The $200,000 cash was used by the Powered by Cognero

Page 22


Name:

Class:

Date:

Chapter 09: Partnerships Formation Operations and Basis partnership to acquire a depreciable asset. The partnership agreement provides that the partners’ capital accounts will be maintained in accordance with Reg. § 1.704-1(b) (the economic effect Regulations) and that any partner with a deficit capital account will be required to restore that capital account when the partner’s interest is liquidated. The partnership agreement provides that MACRS will be allocated 20% to Jeordie and 80% to Kendis. All other items of partnership income, gain, loss, deduction, and credit will be allocated equally between the partners. In the first year, MACRS is $40,000 and no other operating transactions occur. The property is sold at the end of the first year for $160,000 (which, coincidentally, corresponds to JK's basis in the property), and the partnership is liquidated immediately thereafter. To satisfy the economic effect test, how much of the $160,000 cash (from the sale) is allocated each to Jeordie and Kendis? 156. Emma and Laine form the equal EL Partnership. Emma contributes cash of $100,000. Laine contributes

property with an adjusted basis of $40,000 and a fair market value of $100,000. a. How much gain, if any, must Emma recognize on the transfer? Must Laine recognize any gain? If so, how much? b. What is Emma’s tax basis in her partnership interest? Her § 704(b) book basis? c. What is Laine’s tax basis in her partnership interest? Her § 704(b) book basis? d. What tax basis does the partnership take in the assets contributed by Laine and Emma? e.

How will the partnership account for the difference between the basis and value of the property transferred by Laine?

157. The JM Partnership was formed to acquire land and subdivide it as residential housing lots. On March 1,

2022, Jessica contributed land valued at $600,000 to the partnership in exchange for a 50% interest. She had purchased the land in 2015 for $420,000 and held it for investment purposes (capital asset). The partnership holds the land as inventory. On the same date, Matt contributed land valued at $600,000 that he had purchased in 2013 for $720,000. He also became a 50% owner. Matt is a real estate developer, but he held this land personally for investment purposes. The partnership holds this land as inventory. In 2024, the partnership sells the land contributed by Jessica for $620,000. In 2025, the partnership sells the real estate contributed by Matt for $580,000. a. What is each partner’s initial basis in his or her partnership interest?

What is the amount of gain or loss recognized on the sale of the land contributed by Jessica? What is the character of this gain or loss? What is the amount of gain or loss recognized on the sale of the land c. contributed by Matt? What is the character of this gain or loss? How would your answer in part (c) change if the property was sold in d. 2029? b.

158. Sam and Drew are equal partners in SD LLC formed on June 1 of the current year. Sam contributed

land that he inherited from his uncle in 2015. Sam’s uncle purchased the land in 1988 for $30,000. Powered by Cognero

Page 23


Name:

Class:

Date:

Chapter 09: Partnerships Formation Operations and Basis The land was worth $100,000 when Sam’s uncle died. The fair market value of the land was $200,000 at the date it was contributed to the partnership. Drew has significant experience developing real estate. After the LLC is formed, he will prepare a plan for developing the property and secure zoning approvals for the LLC. Drew would normally bill a third party $50,000 for these efforts. Drew also will contribute $150,000 cash in exchange for his 50% interest in the LLC. The value of his 50% interest is $200,000. a.

How much gain or income will Sam recognize on his contribution of the land to the LLC? What is the character of any gain or income recognized?

b. What basis will Sam take in his LLC interest?

How much gain or income will Drew recognize on the formation of the LLC? c. What is the character of any gain or income recognized? Does Drew have a “carried interest”? d. What basis will Drew take in his LLC interest?

Construct a balance sheet for SD LLC assuming that Drew’s services are completed immediately after forming SD. The balance sheet should show two e. numeric columns, including the LLC’s basis in assets and the fair market value of these assets. f.

Outline any planning opportunities that may minimize current taxation to any of the parties.

159. Tom and Missy form TM Partnership, Ltd. (a limited partnership), to own and operate certain real estate.

Tom contributed land, and Missy contributed cash to be used for setting up the entity and creating a plan for developing the property. Once a development plan was in place, the partnership sold interests in the partnership to investors to raise funds for constructing a shopping center. The partnership incurred expenses of $30,000 for forming the entity and $60,000 for starting the business (e.g., setting up the accounting systems, locating tenants, and negotiating leases). It also paid $5,000 in transfer taxes for changing the ownership of the property to the partnership’s name. The brokerage firm that sold the interests to the limited partners charged a 6% commission, which totaled $600,000. The calendar year partnership started business in November this year. How are these initial expenses treated by the partnership? How much is currently deductible, and how is the remainder treated for tax purposes? Show your calculations. 160. The RB LLC is owned equally by Romer and Brad. At the beginning of the year,

Romer’s basis is $40,000 and Brad’s is $32,000. RB reported the following income and expenses for the current tax year. Net ordinary business income (loss) (Form 1065, page 1, line 28) Powered by Cognero

($64,000) Page 24


Name:

Class:

Date:

Chapter 09: Partnerships Formation Operations and Basis Long-term capital gains Distribution to Brad Payment to Great Health Hospital for Romer’s medical expenses

12,000 (30,000) (24,000)

Use the ordering rules of Exhibit 9.2 (and the loss limitation rules) and a. calculate Romer’s basis in his partnership interest at the end of the year.

Based on this calculation, what does Romer report on his tax return? b.

Make the same calculation for Brad. What will Brad report on his tax return?

161. On June 1 of the current tax year, Elisha and Ezra (who are equal partners) contribute property to form the

Double E General Partnership. Elisha contributes cash of $200,000. Ezra contributes a building and land with an adjusted basis and fair market value of $340,000, subject to a liability of $140,000. The partnership borrows $20,000 to finance construction of a parking lot in front of the building. At the end of the first year (December 31), the accrual basis partnership owes $8,200 in trade accounts payable to various creditors. The partnership reported net income of $30,000 for the year that the partners share equally. Assume that Elisha and Ezra share equally in partnership liabilities. How much is Elisha’s basis in the partnership interest on December 31? Ezra’s? 162. Four GRRLs Partnership is owned by four girlfriends. Lacy holds a 40% interest; each of the others owns

20%. Lacy sells investment property to the partnership for its fair market value of $200,000 (Lacy’s basis is $250,000). a. How much loss, if any, may Lacy recognize? b. If the partnership later sells the property for $260,000, how much gain must it recognize? c. How would your answers in parts (a) and (b) change if Lacy owned a 60% interest in the

partnership? d. If Lacy owned a 60% interest and her basis in the investment property was

$120,000 (instead of $250,000), how much, if any, gain would she recognize on the original sale for $200,000? How would the gain be characterized?

Essay 163. On the formation of a partnership, when might a disguised sale occur? How can this treatment be avoided? 164. Your client owns a parcel of land that has depreciated in value. He wants to know if there is a way he can

contribute the property to his partnership, have the partnership sell the property, and convert the existing capital loss into an ordinary loss. He also wants to know if part of the loss would be allocated to his other partners. What is your reaction? Powered by Cognero

Page 25


Name:

Class:

Date:

Chapter 09: Partnerships Formation Operations and Basis 165. What are syndication costs, and how are they treated for tax purposes? 166. Harry and Sally are considering forming a partnership for a new consulting business. Both taxpayers use the calendar year and are cash basis taxpayers. The partnership will not be a tax shelter. The partners are uncertain as to whether the partnership should use the cash or accrual method of accounting. Moreover, the idea of a tax deferral in the first year of operations has led them to consider using a June 30 fiscal year-end for the partnership. As their tax adviser, identify the issues that must be considered in selecting an accounting method and tax year for the partnership. 167. The MOG Partnership reports ordinary income of $60,000, long-term capital gain of $12,000, and tax-exempt income of $12,000. The partnership agreement provides that Molly will receive all long-term capital gains and George will receive all tax-exempt interest income. Their allocation of ordinary income will be reduced accordingly, and Olivia will be allocated a proportionately greater share of ordinary income. (In other words, each partner will receive allocations totaling one-third of the total $84,000 of partnership income.) This allocation was agreed upon because Molly and George are in a high marginal tax bracket and Olivia is in a low marginal tax bracket. a.

Describe the elements that must be included in a partnership agreement for an allocation to have economic effect.

b.

Discuss whether or not the MOG allocation would be permitted and provide your reasoning.

168. What is the difference between a partner’s basis in the partnership interest and a partner’s tax basis capital account? Section 704(b) book capital account? What are the purposes of these amounts? Why are these amounts typically different? 169. Describe how a partnership calculates depreciation on property that is contributed by a partner? If the

partnership incurs additional costs that must be capitalized (i.e., transfer taxes related to changing the title), how are those costs treated? 170. Jasmine Gregory is a 20% member in Sparrow Properties LLC, which is a lessor of residential rental

property. Her share of the LLC’s losses for the current year is $100,000. Immediately before considering the deductibility of this loss, Jasmine’s tax basis capital account (which, in this case, corresponds to her tax basis in the LLC interest, excluding liabilities) reflected a balance of $50,000. Jasmine has personally guaranteed a $10,000 debt of the LLC that is allocated to her as a recourse debt. Her share of the LLC’s nonrecourse debt is $30,000. This debt cannot be treated as qualified nonrecourse financing. Jasmine spends several hundred hours a year working for Sparrow Properties. Jasmine is also a managing member of Starling Rentals LLC, which is engaged in long-term (more than 30 days) equipment rental activities. (This is considered a passive activity.) Jasmine’s share of Starling’s income is $36,000. Jasmine is a single taxpayer. Her modified adjusted gross income before considering the LLCs’ activities is $300,000, and she has no other business losses. The “active participation” rental real estate deduction is not available to Jasmine. Determine how much of Sparrow’s $100,000 loss Jasmine can deduct on her current calendar year return. Using the format (1) facts, (2) issues, (3) conclusion, and (4) law and analysis, draft a memo for the client’s tax file describing the loss limitations. Identify the Code sections, if any, under which losses are suspended. Powered by Cognero

Page 26


Name:

Class:

Date:

Chapter 09: Partnerships Formation Operations and Basis

Powered by Cognero

Page 27


Name:

Class:

Date:

Chapter 09: Partnerships Formation Operations and Basis Answer Key 1. True 2. False 3. True 4. True 5. False 6. False 7. True 8. True 9. False 10. False 11. False 12. False 13. False 14. True 15. False 16. True 17. False 18. True 19. False 20. True 21. True 22. False 23. True 24. True 25. False Powered by Cognero

Page 28


Name:

Class:

Date:

Chapter 09: Partnerships Formation Operations and Basis 26. False 27. True 28. False 29. True 30. True 31. True 32. False 33. True 34. True 35. False 36. False 37. True 38. True 39. True 40. True 41. True 42. False 43. False 44. False 45. True 46. True 47. False 48. False 49. False 50. False Powered by Cognero

Page 29


Name:

Class:

Date:

Chapter 09: Partnerships Formation Operations and Basis 51. False 52. False 53. True 54. True 55. False 56. True 57. True 58. d 59. d 60. a 61. b 62. c 63. c 64. b 65. b 66. d 67. c 68. c 69. a 70. c 71. b 72. b 73. d 74. d 75. c 76. d Powered by Cognero

Page 30


Name:

Class:

Date:

Chapter 09: Partnerships Formation Operations and Basis 77. b 78. a 79. d 80. c 81. d 82. a 83. a 84. a 85. d 86. d 87. c 88. c 89. c 90. b 91. b 92. a 93. b 94. c 95. d 96. b 97. b 98. b 99. a 100. c 101. c Powered by Cognero

Page 31


Name:

Class:

Date:

Chapter 09: Partnerships Formation Operations and Basis 102. d 103. c 104. d 105. b 106. f 107. n 108. b 109. a 110. g 111. m 112. j 113. l 114. o 115. p 116. e 117. d 118. c 119. k 120. i 121. h 122. r 123. j 124. h 125. m 126. o 127. a Powered by Cognero

Page 32


Name:

Class:

Date:

Chapter 09: Partnerships Formation Operations and Basis 128. q 129. b 130. d 131. f 132. p 133. n 134. e 135. i 136. g 137. c 138. k 139. l 140. a. Basis of property contributed Plus: James’s share of partnership liability Less: James’s liability transferred to partnership Basis in partnership interest b.

$400,000 75,000 (150,000) $325,000

Partnership’s basis (carryover basis) is $400,000.

c. Sales price Less: Adjusted basis

$800,000 (400,000)

Total gain on sale

$400,000

James Built-in $350,000 (precontribution) gain Remaining gain 25,000 Gain allocated to $375,000 partner

Powered by Cognero

$

Mateo –0–

25,000 $25,000

Page 33


Name:

Class:

Date:

Chapter 09: Partnerships Formation Operations and Basis 141. a.

b.

c.

Palmer likely will recognize a $500,000 [($4,000,000 – $3,000,000) × 50% ] gain on the transaction. Palmer received a cash payment equal to one-half the value of the property he contributed. The IRS would likely treat this as a disguised sale of the property which is presumed to occur when a contractual agreement requires a contribution by a partner to be followed within two years by a specified distribution by the partnership and the distribution is made without regard to partnership profits. Both these issues occur in this scenario. While Palmer could argue that the intent of this transaction is not to create a disguised sale, it is doubtful that he would be successful. The partnership’s total basis for the property is $3,500,000. Its basis for the purchased property is the $2,000,000 cost of the property (the partnership is deemed to have paid for the property). In addition, the partnership has a $1,500,000 carryover basis for the portion of the property that was contributed rather than purchased. If Palmer can wait for more than two years to receive the distribution and if the distribution is not contractually guaranteed, the contribution and distribution transactions will be presumed not to be a disguised sale. The distribution will be treated as a normal distribution that will not create capital gain for Palmer unless the distribution amount exceeds the adjusted basis for his partnership interest when the distribution is made.

142. Sales Utilities and rent Salaries Guaranteed payment to Antonio Partnership ordinary income

$600,000 (80,000) (200,000) (20,000) $300,000

Separately stated interest income

$ 4,000

Guaranteed payment to partner

$20,000

Separately stated charitable contribution

$ 8,000

The distribution to Carl is not deductible by the partnership and will not affect Antonio's Schedule K-1. Antonio’s share of the partnership’s ordinary income is $75,000 ($300,000 × 25%). He also reports his separately stated share of interest income of [$1,000 ($4,000 × 25%)] and charitable contributions [$2,000 ($8,000 × 25%)]. Antonio’s K-1 will also show his guaranteed payment of $20,000, his net earnings from self-employment ($95,000 = $75,000 ordinary income + $20,000 guaranteed payment), and other information he might need in order to prepare his return (e.g., AMT, QBI, and/or investment income information). Antonio will report his shares of ordinary income, interest income, and the guaranteed payment and might be able to deduct the charitable contributions. He might be able to claim a QBI deduction. He might need to calculate alternative minimum tax, self-employment tax, additional Medicare tax, and the net investment income tax. 143. Revenues Cost of goods sold Powered by Cognero

$300,000 (160,000) Page 34


Name:

Class:

Date:

Chapter 09: Partnerships Formation Operations and Basis Salaries Business interest expense (exactly 30% of net of above) Partnership ordinary income

(80,000) (18,000) $ 42,000

Nina’s share of: Partnership ordinary income ($42,000 × 50%)

$ 21,000

Separately stated tax-exempt income (not reported)

$ 1,000

Separately stated long-term capital gain (reported)

$ 5,000

Separately stated investment interest expense

($ 1,000)

The distributions to the partners are not deductible although the $20,000 distribution to Nina is shown on her Schedule K1 so she can calculate her basis in the partnership interest and determine whether the distribution is taxable (probably not in this situation as it is less than the net $26,000 increase to her basis for current activity) 144. Basis in land contributed to CD Less: Relief of liability assumed by partnership Plus: Share of liability related to land ($100,000 × 30%) Plus: Share of trade accounts payable ($20,000 × 30%) Plus: Share of partnership income ($400,000 × 30%) Ending basis in partnership interest

$300,000 (100,000) 30,000 6,000 120,000 $356,000

145. a. Adjusted basis, beginning of year Plus: Share of income after guaranteed payment ($270,000 × 30%) Long-term capital gain ($6,000 x 30%) Share of interest on Colorado state bonds ($2,000 × 30%) Share of partnership charitable contributions ($10,000 x 30%) Decrease in share of partnership liabilities ($20,000 × 30%) Cash distributions Adjusted basis, 12/31 Powered by Cognero

$60,000 $81,000 1,800 600

83,400

$3,000 6,000 20,000 (29,000) $114,400 Page 35


Name:

Class:

Date:

Chapter 09: Partnerships Formation Operations and Basis

b.

Rachel will report $81,000 of income from the partnership plus a long-term capital gain of $1,800. She may be able to deduct $3,000 of charitable contributions as an itemized deduction. She might also be able to claim the QBI deduction. In addition, Rachel must report the $30,000 guaranteed payment as ordinary income. The bond interest income is nontaxable. The cash distribution and debt reduction are not taxable because they do not exceed her basis before those items. Her guaranteed payment ($30,000) and her distributive share ($81,000) are subject to SE tax (and possibly additional Medicare tax) because she is a general partner. The capital gain might be subject to the net investment income tax.

146. $116,400. Katherine’s initial basis of $80,000 is increased by her 30% share of partnership income from operations ($60,000). Her basis is decreased by her 30% share of the partnership’s charitable contribution ($3,000) and the shortterm capital loss ($600). It is also decreased by the $20,000 distribution she received. The distributions to Lauren and Missy do not affect Katherine’s basis. Katherine’s ending basis, then, is $116,400 ($80,000 + $60,000 – $600 – $3,000 – $20,000). 147. $61,000. Sarah's contributed property has a $0 basis. Sarah is a 25% partner and will share in 25% of the partnership’s taxable income and tax-exempt income. In addition, her basis will include her allocable share of the partnership’s recourse liabilities. Her basis will be reduced by the cash distribution during the year. Beginning basis Plus: Share of partnership ordinary income (25% × $200,000) Plus: Share of tax-exempt income (25% × $4,000) Plus: Share of partnership liabilities Basis before losses and distributions Less: Distribution Ending basis 148. a. Revenues Less: Business expenses Less: Guaranteed payment (for services) to Dave Less: Consulting expenses to Ethan Ordinary business income

$ –0– 50,000 1,000 20,000 $71,000 (10,000) $61,000

$200,000 (50,000) (40,000) (10,000) $100,000

The distribution to Olivia is not deductible. The payment to Ethan is a deductible business expense. Dave’s share of DOE’s ordinary income is $50,000. The $2,000 of tax-exempt interest income is a separately stated item, of which Dave’s share is $1,000. (It is not taxable to Dave but will increase his basis.) In addition, Dave must report the $40,000 guaranteed payment as gross income. Dave's total ordinary income from the partnership, then, is $90,000 (plus the $1,000 of exempt income). b.

Beginning basis Plus: Increase in share of the LLC’s liabilities Plus: Share of ordinary income

Powered by Cognero

$ 50,000 8,000 50,000 Page 36


Name:

Class:

Date:

Chapter 09: Partnerships Formation Operations and Basis Plus: Share of tax- exempt interest income Dave's ending basis

1,000 $109,000

Dave’s guaranteed payment does not affect his basis. Dave’s distributive share of $50,000 and his $40,000 guaranteed payment ($90,000 total) are subject to SE tax.

c.

d. Dave's maximum deduction under § 199A is $10,000, or 20% of Dave's 50% share of DOE's ordinary income. QBI does not include income from guaranteed payments, as they are treated (for this purpose) as being more in the nature of salary/wages or investment income. To determine whether this amount is limited, DOE would need to provide Dave's share of any W-2 wages paid to DOE's employees, and his share of the LLC's unadjusted basis of depreciable property. This information is used to calculate two amounts: The higher of the amounts is the limitation. Dave's QBI for this business is the lesser of $10,000 or the limitation. An additional taxable income limitation applies, plus Dave will have additional calculations if he has additional activities subject to QBI.

149. Beginning basis Plus: Share of interest income Less: Net decrease in share of partnership debt Less: Share of S&S loss Ending basis

$80,000 5,000 (5,000) (20,000) $60,000

150. a. $45,000 ordinary income and $3,000 LTCG. The partnership’s ordinary income is calculated as follows. Revenues Less: Rent, utilities, and salaries Less: Guaranteed payment to Ryan Less: Consulting expenses to Amy Ordinary income

$400,000 (160,000) (40,000) (20,000) $180,000

The distribution to Cameron is not deductible. The guaranteed payment to Ryan and the consulting service payment to Amy are deductible business expenses. Cameron’s 25% share of CAR’s ordinary income is $45,000. The $12,000 net long-term capital gain Powered by Cognero

Page 37


Name:

Class:

Date:

Chapter 09: Partnerships Formation Operations and Basis is a separately stated item of which Cameron’s share is $3,000. Ryan (not Cameron) reports the guaranteed payment as income.

b. Beginning basis Plus: Share of ordinary income Plus: Share of net long-term capital gain Less: Decrease in share of partnership liabilities Less: Cash distribution to Cameron

$60,000 45,000 3,000 (10,000) (40,000) $58,000

Ending basis

151. Basis of land Deemed cash distribution (relief of Cody’s debt) Share of Cody’s debt Share of Derek’s debt Cody’s basis

$80,000 (40,000) 20,000 30,000 $90,000

152. $65,000 income and amount included in SE tax calculation. Morgan might be able to claim a QBI deduction of $5,000 (20% of the $25,000 share of partnership ordinary income). The $40,000 payment to Morgan is a guaranteed payment and is deductible by the partnership. The $10,000 payment to Katie is deductible under § 707(a), because it was an ordinary business expense paid during the year. The distributions to Kristen and Katie are not deductible by the partnership. Income from services Less: Rent expense Salaries to employees Guaranteed payment to Morgan Payment to Katie for services Partnership income

$150,000 $10,000 40,000 40,000 10,000

(100,000) $ 50,000

Of this $50,000 partnership income, 50%, or $25,000, is allocated to Morgan. She must also include the $40,000 guaranteed payment in her gross income this year, because she and the partnership use the same reporting period. The guaranteed payment is not eligible for the QBI deduction. This $65,000 is included in Morgan’s SE tax calculation. 153. Cassandra can deduct a $20,000 loss after application of all rules. Her $80,000 loss from C&C is first limited by the basis rules of § 704(d); $20,000 of the loss ($80,000 loss – $60,000 basis) is limited under this rule. The remaining loss of $60,000 is tested under the at-risk rules. Cassandra’s amount at risk is $30,000 (her basis less the nonrecourse debt); $30,000 of the loss ($60,000 – $30,000) is suspended under the at-risk rules. As a limited partner, the remaining $30,000 loss is treated as a passive loss. That loss can be deducted to the extent of Cassandra’s passive income from MNOP, or $20,000. The remaining $10,000 of loss is suspended under the passive loss rules. Adjusted basis [§ 704(d)] At risk amount (§ 465) Powered by Cognero

Deductible $60,000 30,000

Suspended $20,000 30,000 Page 38


Name:

Class:

Date:

Chapter 09: Partnerships Formation Operations and Basis Passive loss rules

20,000

10,000

154. Patricia can deduct only $305,000 (2024) of her $720,000 share of the partnership’s operating loss on her tax return. Her adjusted basis for her partnership interest immediately before the deduction of any portion of the loss is $590,000 ($650,000 – $60,000 distribution). The amount of the loss that can be deducted is first limited by the $590,000 adjusted basis. Then, the remaining loss is limited by the at-risk amount of $390,000 ($650,000 – $60,000 distribution – $200,000 nonrecourse debt). (Patricia is not at-risk for her $200,000 share of the nonrecourse debt.) The passive loss rules do not apply, because Patricia is a material participant in the partnership. The last hurdle is that the loss cannot exceed the excess business loss limitation. As a single taxpayer, Patricia's limitation is $305,000 in 2024, and she carries forward (as a net operating loss) the $70,000 remaining loss that passed the other hurdles. Therefore, she can deduct a $305,000 loss on her return.

Adjusted basis [§ 704(d)] At risk amount (§ 465) Passive loss rules (§ 469) Excess business loss rules (§ 461(l)) *Net operating loss carryover

Deductible $590,000 390,000 Not applicable $305,000

Suspended $130,000 200,000 $85,000*

If MOP were an LLC, the nominally recourse debt of $50,000 would not be included in Patricia’s amount at risk because she did not personally guarantee the debt. Her loss at that level would be limited to $340,000 ($390,000 – $50,000). The excess business loss remains $305,000, but the net operating loss carryover would be reduced to $35,000 ($340,000 allowed under the at-risk rules - $305,000 limitation for 2024). 155. Jeordie will receive $92,000 and Kendis will receive $68,000. Distributions upon liquidation must follow the § 704(b) book capital accounts for each partner. Depreciation is allocated $8,000 (20%) to Jeordie and $32,000 (80%) to Kendis. After MACRS allocations, Jeordie will have a capital account balance of $92,000 ($100,000 – $8,000) and Kendis’s capital account balance will be $68,000 ($100,000 – $32,000). If the asset is sold for its $160,000 carrying value on the partnership books, no gain or loss will be recognized on the sale and, therefore, no further adjustment needs to be made to the partner’s capital accounts. Upon liquidation, the partners will receive assets valued at the balances in their capital accounts. 156.

Under § 721, neither the partnership nor the partners recognizes any gain on formation of the entity. Emma will take a tax and § 704(b) book basis of $100,000 in b. her partnership interest. Laine will take a substituted tax basis of $40,000 in her partnership interest ($40,000 tax basis in the property c. contributed to the entity). Her § 704(b) book basis will equal the fair market value of the property, or $100,000. The partnership will take a carryover basis in the assets it d. receives ($100,000 basis in cash and $40,000 basis in property). The partnership will treat the $60,000 difference between the e. basis and fair market value of the property Laine contributed a.

Powered by Cognero

Page 39


Name:

Class:

Date:

Chapter 09: Partnerships Formation Operations and Basis as a precontribution gain that must be allocated to Laine when the property is sold or as described under the Regulations if the property is depreciable. 157. a. The partners’ initial bases in their partnership interests are the same

amounts as their bases in the contributed property (§ 722). Jessica’s basis Matt’s basis

$420,000 $720,000

b. The 2024 sale results in ordinary income of $200,000 to the

partnership. Selling price Basis Gain

$620,000 (420,000) $200,000

The gain is ordinary income because the land is held as inventory by the partnership. The land was a capital asset to Jessica, but no Code provision allows treatment of the gain based on Jessica’s use rather than the partnership’s use.

c. The 2025 sale results in a $140,000 loss (including a $120,000 capital

loss and a $20,000 ordinary loss. Selling price Basis Loss

$580,000 (720,000) ($140,000)

As a sale of inventory (determined at the partnership level), the sale in 2025 of the land contributed by Matt would normally result in an ordinary loss. However, § 724 overrides the usual treatment. The character of the precontribution loss, instead, is determined based on the character of the property in Matt’s hands. This sale was within five years of the capital contribution date, so the loss is capital in nature to the extent of the built-in loss at the contribution date, which is: FMV at contribution Basis Capital loss Powered by Cognero

$600,000 (720,000) ($120,000) Page 40


Name:

Class:

Date:

Chapter 09: Partnerships Formation Operations and Basis The remaining $20,000 loss in 2025 is an ordinary loss because the character of the post-contribution loss is based on the partnership’s ownership and use of the property as inventory. If the property Matt contributed was sold by the partnership in 2029, the entire $140,000 loss would be treated as an ordinary loss. A sale in d.

2029 would not be within five years of the contribution date, so the character of the loss would be determined solely by reference to the character of the asset to the partnership. Because the land is inventory to the partnership, the loss in 2029 would be ordinary.

158. a.

None. Under § 721, neither the LLC nor any of the members recognize gain on contribution of property to an LLC in exchange for an interest in the LLC.

$100,000. Sam’s basis in his LLC interest will equal the basis he held in the property he inherited from his uncle. The basis a beneficiary takes in b. property received from an estate generally equals the fair market value of the asset at the date of death or at the alternate valuation date (six months later) if available and elected. Drew will recognize $50,000 of ordinary income. The fair market value of Drew’s 50% LLC interest is $200,000. Because Drew will contribute only $150,000 of property, the difference between the amount contributed and the value of the interest will be treated as being for services rendered c. to the LLC. Services do not constitute “property” for purposes of § 721 nonrecognition treatment. Drew does not have a carried interest because (1) it is a capital interest, and (2) Drew “paid” for the interest by recognizing income equal to the value of the interest. d.

Drew’s basis in his LLC interest will be $200,000 [$150,000 (cash contributed) + $50,000 (the amount of ordinary income recognized for services rendered to the partnership)].

SD’s balance sheet is as follows immediately after formation. (The e. column totals are calculated using the “sum” command.) Powered by Cognero

Page 41


Name:

Class:

Date:

Chapter 09: Partnerships Formation Operations and Basis Assets Cash Land Land Improvements Total assets

Basis $150,000 100,000 50,000 $300,000

FMV $150,000 200,000 50,000 $400,000

Partners’ capital Sam’s capital Drew’s capital Total capital

$100,000 200,000 $300,000

$200,000 200,000 $400,000

SD LLC will capitalize the $50,000 deemed payment for Drew’s services, because the services relate to a capitalizable expenditure. The LLC will reflect this $50,000 in “cost of lots sold” as the development lots are sold. Drew could prepare a development plan and secure zoning permits before the LLC is formed. He could then contribute these plans and permits to SD in addition to the $150,000 cash. Because a completed plan would be considered f. “property,” no portion of his LLC interest would be received in exchange for services if this were done. The entire transaction would be considered under § 721.

159. TM Partnership, Ltd., has incurred costs for organizing the partnership ($30,000), starting the business

($60,000), transferring property ($5,000), and securing investors ($600,000) for the partnership. The organizational costs are treated under § 709. Under this section, the first $5,000 of such expenses are deducted (provided the total is less than $50,000); the remainder is amortized over 180 months. The startup costs are treated under § 195. The first $5,000 of such expenses are deducted, provided the total is less than $50,000. If costs exceed $50,000, the $5,000 deduction is phased out, dollar for dollar, by the amount of costs in excess of $50,000. When total costs equal or exceed $55,000, no portion of the expense is currently deductible. Instead, the full amount is amortized over 180 months. Under these rules, TM deducts $5,278 [$5,000 + ($25,000 × 2/180)] of organizational costs and $667 ($60,000 × 2/180) of startup expenses. The $5,000 transfer tax is treated as a cost of acquiring the land and is added to the partnership’s basis in the land (nondepreciable property). The $600,000 of brokerage commissions is treated as a syndication cost of the partnership. Under § 709, these costs cannot be deducted. If newly capitalized costs arise on a contribution of assets to the partnership (e.g., transfer taxes or legal fees), the partnership treats these costs as newly acquired MACRS property and commences depreciation at the date the partnership places the property in service. 160. a.

Romer’s basis in his partnership interest at the end of the tax year is determined as follows, using the ordering rules in Exhibit 9.2.

Powered by Cognero

Page 42


Name:

Class:

Date:

Chapter 09: Partnerships Formation Operations and Basis Beginning basis Share of separately stated income items: Long-term capital gain Basis before loss allocation and distribution Less: Distribution (partnership payment of medical expenses) Basis before loss allocation Less: Ordinary loss allowed under § 704(d) Ending basis in interest

$40,000 6,000 $46,000 (24,000) $22,000 (22,000) -0-

Romer reports the long-term capital gain as income. Per the ordering rules of Exhibit 9.2, the distribution is considered before the loss. The distribution from the partnership is not taxable because it is less than Romer’s basis after current income items. Romer’s ordinary loss from the partnership is limited under § 704(d) to $22,000. The remaining $10,000 ordinary loss is carried forward (as a suspended loss) until such time as Romer has sufficient basis in his partnership interest to utilize the loss. b. Brad’s basis in his partnership interest at the end of the tax year is

determined as follows.

Beginning basis Share of separately stated income items: Long-term capital gain Basis before loss allocation and distribution Less: Distribution Basis before loss allocation Less: Ordinary loss allowed under § 704(d) Ending basis in interest

$32,000 6,000 $38,000 (30,000) $8,000 (8,000) -0-

Brad reports the long-term capital gain. The distribution is again considered before determining the allowable loss. Brad’s basis is $8,000 lower than Romer’s, and the distribution Brad received is $6,000 higher than the distribution to Romer, so Brad’s deductible loss is $14,000 less than Romer’s. Brad may only deduct $8,000 of the loss. The remaining $24,000 loss is carried forward.

Powered by Cognero

Page 43


Name:

Class:

Date:

Chapter 09: Partnerships Formation Operations and Basis 161. Both Elisha’s and Ezra’s bases in the partnership interests are $299,100 at the end of the year. Elisha’s

initial basis of $200,000 (cash contribution) is increased by a $70,000 share of the liability on the contributed land, a $10,000 share of the construction debt, and a $4,100 share of the accounts payable debt. In addition, Elisha’s basis is increased by the $15,000 share of the partnership’s taxable income. Ezra’s initial basis of $340,000 (building and land basis) is reduced by the $140,000 debt assumed by the partnership and then increased by a $70,000 share of the liability on the contributed land, a $10,000 share of the construction debt, and a $4,100 share of the accounts payable debt. In addition, Ezra’s basis is also increased by the $15,000 share of the partnership’s taxable income. The bases are the same because the fair market value of Ezra’s contributed property was the same as its tax basis and because both partners (logically) contributed net assets with equal net fair market values. 162. a.

$50,000 loss. As a 40% owner, Lacy’s loss on the sale to the partnership is not disallowed.

b. $60,000 gain. The partnership has a cost basis in the property of

$200,000. Lacy would claim no loss. Section 707(b)(1)(A) would apply, and Lacy’s $50,000 realized loss would not be deductible. c.

On the partnership’s later sale of the property, it would recognize a gain of $10,000. Section 267(d) permits the partnership to offset any subsequent gain by the loss previously disallowed ($60,000 gain less $50,000 previously disallowed loss).

$80,000 gain. Lacy’s $80,000 gain would be ordinary under § 707(b)(2) if the investment property immediately after the transfer is d. not a capital asset of the Four GRRLs Partnership; otherwise it would probably be a capital gain.

163. A disguised sale might occur when a partner contributes appreciated property to a partnership and soon thereafter receives a cash distribution from the partnership. Under §§ 721 and 731, the contribution and distribution transactions normally would not be taxable events if the partner has sufficient basis to cover the distribution. If the appearance of the transaction is that the contribution and distribution are related, the IRS may take the position that the partnership form was simply used to accommodate a transaction that was intended to be a sale. Disguised sale treatment can be avoided if the other partners receive similar and proportionate distributions; the partnership is not obligated to make the distribution; at the contribution date, the partner’s rights to future distributions are clearly subject to entrepreneurial risk; or the distribution is more than two years after the contribution occurred. 164. In the short run, it would not be possible to convert the capital loss into an ordinary loss. If the client can wait more than five years for the partnership to sell the property, the character of the loss would be determined by reference to the partnership’s use of the land. The built-in (precontribution) loss would be allocated to the client and any loss arising after Powered by Cognero

Page 44


Name:

Class:

Date:

Chapter 09: Partnerships Formation Operations and Basis the contribution date would be allocated according to the provisions in the partnership agreement. Section 724 provides that when property is sold by a partnership at a loss within five years of the date the property is contributed, any built-in capital loss at the contribution date remains a capital loss, regardless of the partnership’s use of the property. For example, even if the land was considered inventory by the partnership rather than a capital asset, sale of that land within five years would result in a capital loss to the extent of the built-in loss at the contribution date. When a partner contributes property to a partnership, any built-in gain or loss must be tracked and allocated to the contributing partner under § 704(c). Therefore, the built-in loss (whether capital or ordinary) would be allocated to the client when the property is eventually sold; that loss cannot be allocated to the other partners. 165. Syndication costs are costs incurred in bringing an investment partnership to market. These costs include brokerage commissions and fees; registration fees; legal and accounting fees for developing the offering document; and printing and distribution costs for the prospectus, placement memoranda, and related documents. Under § 709, syndication costs cannot be deducted and no amortization is permitted. Upon termination of the partnership, the partners’ bases will theoretically still include those costs, so the partner might have a lower gain or a greater loss at that time. 166. Because neither partner is a C corporation and the partnership is not a tax shelter, the partnership may select any accounting method: cash, accrual, or a hybrid of the two methods. (If one partner was a C corporation, the cash method could still be used unless the average annual gross receipts exceeded a threshold amount ($30,000,000 for 2024). If the partnership uses the accrual method of accounting in determining its income, the partners will be taxed on partnership revenues from all closed transactions. In this regard, it does not matter whether cash has been received by the partnership and whether the partners use the accrual method on their individual tax returns. Thus, if the partnership adopts the accrual method for tax purposes, the partners may be faced with reporting and paying taxes on partnership income long before cash is available for distribution. However, under accrual accounting, expenses incurred but not paid may serve to mitigate or eliminate this possibility. Regarding the July 1 to June 30 fiscal year, the desired tax deferral has little chance of success. Under § 706(b), the partnership must use the calendar year unless Harry and Sally can convince the IRS that a business purpose exists for a tax year other than the calendar year. Nothing in the fact pattern indicates that a valid business purpose exists for a fiscal year. The partnership may also elect to use a tax year other than the required tax year if the deferral period is three months or less (e.g., September, October or November year-end), and if the partnership agrees to deposit tax on the deferred income at a specified tax rate. This election cannot be used in this situation to obtain a July 1 to June 30 fiscal year. 167. a. For partnership allocations to meet the economic effect tests under the § 704(b) Regulations, a partnership agreement should provide that (1) capital accounts will be maintained, (2) liquidating proceeds will be distributed according to capital account balances, and (3) any partner with a deficit capital account balance will contribute cash to the partnership to eliminate the deficit. b.

The MOG allocation will not be permitted. In addition to the three requirements above, the allocations must also meet the "substantial" requirement. Though the partnership agreement may meet the economic effect tests under § 704, the allocation does not produce pre-tax economic consequences to Molly and George. Therefore, the allocations are not effective for tax purposes because they have no function other than the reduction of the partners’ combined tax liability.

168. Tax basis in partnership interest. A partner’s basis in the partnership interest is the tax measure by which any Powered by Cognero

Page 45


Name:

Class:

Date:

Chapter 09: Partnerships Formation Operations and Basis taxable gain or loss is determined upon sale of the interest or receipt of cash distributions from the partnership. Basis is also used as the measure for determining whether the partner’s share of partnership losses can be deducted. Section 704(b) book capital. The partner's § 704(b) book capital account balance is an accounting measure of the partner’s ownership interest in the entity. Section 704(b) book capital accounts are referenced in determining the partnership’s allocations of income, gains, losses, deductions, and credits among the partners under § 704(b). In addition, liquidating distributions must be in accordance with ending § 704(b) book capital account balances under the substantial economic effect rules. Tax basis capital account. A partner's tax basis capital account is a measure of the partner's share of the tax basis of partnership assets. The tax basis capital account is reported on the partner's Schedule K-1. The partner’s initial tax basis (cost, gift, or inherited basis) is increased by her or his contributions to and decreased by distributions from the partnership (including changes in the partner’s share of partnership liabilities). In addition, tax basis is increased by the partner’s share of income and gains and decreased by her or his share of deductions and losses. Section 704(b) book capital accounts are similarly adjusted, except the adjustments are based on fair market value determinations and the calculation generally does not include the partner’s share of partnership liabilities. The partner's tax basis capital accounts are subject to these same increases and decreases, but the amounts are determined on a tax rather than fair market value basis; and again, liabilities are not included. On Schedule K-1, the tax basis capital account plus the partner's share of partnership debt will typically approximate the partner's tax basis in the partnership interest. (An exception would be if a step-up/down event occurred but the partnership decided not to make a § 754 election.) In contrast, the partner's (1) basis in the partnership and (2) § 704(b) book capital account are not typically shown in the partnership's tax return. 169. For property contributed by a partner to a partnership, the partnership “steps into the shoes” of the

contributing partner and continues to use the depreciation schedule used by the partner. For example, assume that at the midpoint of year 4, a calendar year partner contributes nonresidential real property that was purchased in August of year 1. The current year’s cost recovery will be calculated using the 39-year MACRS table for the “eighth month,” fourth recovery year. One-half of the cost recovery would be allocated to the partner, and the remainder would be allocated to the partnership. The following year would be the fifth recovery year, and the cost recovery would be allocated completely to the partnership. If newly capitalized costs arise on a contribution of assets to the partnership (e.g., transfer taxes or legal fees), the partnership treats these costs as newly acquired MACRS property and commences depreciation at the date the partnership places the property in service. 170. TAX FILE MEMORANDUM

DATE FROM SUBJECT

January 30, 2024 Beth Mullins Deductibility of loss from LLC

Facts. Jasmine Gregory reports the following allocations and bases in her LLC interests for the current year. Sparrow Starling Income (loss) allocation ($100,000) $36,000 Basis excluding liabilities 50,000 N/A Recourse liability allocation 10,000 N/A Nonrecourse liability allocation 30,000* N/A Powered by Cognero

Page 46


Name:

Class:

Date:

Chapter 09: Partnerships Formation Operations and Basis *This debt cannot be treated as qualified nonrecourse financing. Jasmine spends significant time working for each LLC during the year. Her modified AGI is $300,000 before considering the LLCs’ activities. She has no other business losses. Issues. How much of the $100,000 loss from Sparrow Properties LLC can Jasmine deduct in the current year? Under what Code provision are any disallowed losses suspended? Conclusion. Jasmine can deduct $36,000 of the $100,000 loss. Of the disallowed loss, $10,000 is suspended under § 704(d). An additional $30,000 is suspended under the at-risk limitations. The remaining suspended loss of $24,000 is suspended under the passive activity loss rules of § 469. The special $25,000 deduction for active participation in a passive real estate activity is not available because Jasmine’s modified AGI is too high. None of the loss is suspended under the excess business loss limitation. Law and analysis. Under § 704(d), Jasmine may deduct a portion of the Sparrow loss equal to her basis in her LLC interest, including recourse and nonrecourse liabilities. The basis is $90,000 ($50,000 + $10,000 + $30,000), and the excess $10,000 loss is suspended. The $30,000 of nonrecourse debt cannot be included in the amount at risk, because it is not qualified nonrecourse indebtedness. Therefore, an additional $30,000 is suspended under the at-risk limitation rules of § 465. (After this limitation, $60,000 of the loss is still available for testing under the passive loss limitation rules.) The activities of both LLCs are treated as rental activities, which, by definition, are passive for purposes of § 469. The Sparrow loss is deductible to the extent of the $36,000 of income from Starling. Also, because Sparrow conducts rental real estate activities and Jasmine is an active participant owning more than 10%, this loss is eligible for an additional $25,000 deduction. However, none of this additional $25,000 deduction is available to Jasmine, because her modified AGI exceeds $150,000 (the top of the phaseout range for the additional deduction allowance). The $36,000 loss allowed under the passive activity loss rules exactly equals the passive income from Starling, so the net deductible loss from all activities is $0. Therefore, the excess business loss limitation does not apply.

Powered by Cognero

Page 47


Name:

Class:

Date:

Chapter 10: Partnerships Distributions Transfer of Interests and Terminations True / False 1. A cash distribution from a partnership to a partner generally is taxable to the partner. a. True b. False 2. For Federal income tax purposes, a distribution from a partnership to a partner is treated the same as a distribution from a C corporation to its shareholders. a. True b. False 3. In a current (nonliquidating) distribution, loss never is recognized by the partnership. a. True b. False 4. Anna and Brad are equal partners in the AB LLC. If AB distributes $10,000 of cash to Anna and a capital asset valued at $10,000 to Brad, and if both Anna and Brad continue to be members of the LLC, the distribution is classified as a proportionate current distribution. a. True b. False 5. Generally, a proportionate distribution of property does not result in a gain to a partner on either a current or liquidating distribution. A situation in which a gain may arise, however, is when a partner contributed appreciated property to the partnership and that property is distributed back to that same contributing partner within seven years of the contribution. a. True b. False 6. Loss cannot be recognized on a current (nonliquidating) distribution from a partnership unless cash, unrealized receivables, and/or § 1231 assets are the only items distributed. a. True b. False 7. Generally, no gain is recognized on a proportionate liquidating or current (nonliquidating) distribution of noncash property, even if the fair market value of property distributed exceeds the partner’s basis in the partnership interest. a. True b. False 8. In a proportionate current (nonliquidating) distribution of cash and a capital asset, the partner recognizes gain to the extent the amount of cash plus the fair market value of property distributed exceeds the partner’s basis in the partnership interest. a. True b. False 9. In a proportionate current (nonliquidating) distribution, cash is deemed to be distributed first, followed by capital and § 1231 assets, and last, unrealized receivables and inventory. a. True b. False Powered by Cognero

Page 1


Name:

Class:

Date:

Chapter 10: Partnerships Distributions Transfer of Interests and Terminations 10. A gain arises only on a distribution from a partnership of cash that exceeds the partner’s basis in the partnership interest. For this purpose, only cash, checks, and credit card charges are treated as cash. a. True b. False 11. The ELF Partnership distributed $20,000 cash to Emma in a proportionate, current (nonliquidating) distribution. Emma’s basis in her partnership interest was $12,000 immediately before the distribution. As a result of the distribution, Emma’s basis is reduced to $0 and she recognizes an $8,000 gain. a. True b. False 12. Scott owns a 30% interest in the capital and profits of the SOS Partnership. Immediately before he receives a proportionate current (nonliquidating) distribution from SOS, the basis of his partnership interest is $40,000. The distribution consists of $30,000 in cash and land with a fair market value of $80,000. SOS’s adjusted basis in the land immediately before the distribution is $50,000. As a result of the distribution, Scott recognizes no gain or loss and his basis in the land is $10,000. a. True b. False 13. Randi owns a 40% interest in the capital and profits of the RAY Partnership. Immediately before she receives a proportionate current (nonliquidating) distribution from RAY, the basis for her partnership interest is $60,000. The distribution consists of $45,000 in cash and land with a fair market value of $72,000. RAY’s adjusted basis in the land immediately before the distribution is $36,000. As a result of the distribution, Randi recognizes a gain of $57,000. a. True b. False 14. Lori, a partner in the JKL partnership, received a proportionate current (nonliquidating) distribution of $10,000 cash, unrealized receivables with a basis of $0 and a fair market value of $15,000, and land with a basis of $6,000 and a fair market value of $10,000. Her basis in the partnership interest immediately before the distributions was $14,000. She will recognize $0 gain on the distribution, and her basis in the receivables and land will be $0 and $4,000, respectively. a. True b. False 15. Matt, a partner in the MB Partnership, receives a proportionate, current (nonliquidating) distribution of property having a fair market value of $16,000 and a partnership basis of $23,000. Matt’s basis in the partnership is $10,000 before the distribution. In this situation, Matt will recognize no gain or loss. He takes a $10,000 basis in the property, and his basis in the partnership interest is reduced to zero. a. True b. False 16. Tim and Darby are equal partners in the TD Partnership, which owns no hot assets. Partnership income for the year is $60,000. Tim needs cash to pay tax on his share of the partnership income, but Darby wants to leave the cash in the partnership for expansion. If the partners agree, it is acceptable for TD to distribute $8,000 to Tim but no cash or other property to Darby. a. True b. False Powered by Cognero

Page 2


Name:

Class:

Date:

Chapter 10: Partnerships Distributions Transfer of Interests and Terminations 17. Marcie is a 40% member of the M&A LLC. Her basis is $10,000 immediately before the LLC distributes to her $30,000 of cash and land (basis to the LLC of $20,000 and fair market value of $25,000). As a result of the proportionate, current (nonliquidating) distribution, Marcie recognizes a gain of $20,000, and her basis in the land is $0. a. True b. False 18. The BAM Partnership distributed the following assets to partner Barbie in a proportionate current (nonliquidating) distribution: $10,000 cash, land parcel A (basis of $5,000, fair market value of $30,000) and land parcel B (basis of $10,000, fair market value of $30,000). Barbie’s basis in her partnership interest was $40,000 immediately before the distribution. Barbie will allocate a basis of $10,000 and $20,000, respectively, to the two land parcels, and her basis in her partnership interest will be reduced to $0. a. True b. False 19. Carl’s basis in his LLC interest is $10,000. In a proportionate current (nonliquidating) distribution, Carl receives land (basis = $10,000; fair market value = $12,000); and inventory (basis = $6,000; fair market value = $8,000). Carl takes a $10,000 basis in the land and a $0 basis in the inventory, and has a $0 basis in the LLC interest. a. True b. False 20. In a liquidating distribution that liquidates the partnership, each partner recognizes gain or loss equal to the difference between the value of assets received less the partner’s basis in the partnership interest. a. True b. False 21. In a proportionate liquidating distribution, RST Partnership distributes to partner Riley cash of $30,000, accounts receivable (basis of $0, fair market value of $40,000), and land (basis of $65,000, fair market value of $50,000). Riley’s basis was $40,000 before the distribution. On the liquidation, Riley recognizes a gain of $0, and her basis is $10,000 in the land and $0 in the accounts receivable. a. True b. False 22. In a proportionate liquidating distribution, WYX Partnership distributes to partner William cash of $40,000, cash basis accounts receivable (basis of $0, fair market value of $10,000), and land (basis of $30,000, fair market value of $50,000). William’s basis was $80,000 before the distribution. On the liquidation, William recognizes a $20,000 gain, and he takes a basis of $10,000 in the accounts receivable and $50,000 in the land. a. True b. False 23. Zach’s partnership interest basis is $100,000. Zach receives a proportionate, liquidating distribution from a liquidating partnership of $50,000 cash and inventory having a basis of $20,000 to the partnership and a fair market value of $30,000. Zach assigns a basis of $20,000 to the inventory and recognizes a $30,000 loss. a. True b. False 24. The JIH Partnership distributed the following assets to partner James in a proportionate liquidating distribution in which the partnership also liquidated: $25,000 cash, land parcel A (basis of $5,000, fair market value of $30,000), and land parcel B (basis of $5,000, fair market value of $15,000). James’s basis in his partnership interest was $85,000 Powered by Cognero

Page 3


Name:

Class:

Date:

Chapter 10: Partnerships Distributions Transfer of Interests and Terminations immediately before the distribution. James will allocate bases of $40,000 to parcel A and $20,000 to parcel B, and he will have no remaining basis in his partnership interest. a. True b. False 25. Carlos receives a proportionate liquidating distribution consisting of $8,000 cash and inventory with a basis to the partnership of $5,000 and a fair market value of $6,000. His basis in his partnership interest was $15,000 immediately before the distribution. Carlos assigns a basis of $7,000 to the inventory and recognizes no gain or loss. a. True b. False 26. In a proportionate liquidating distribution in which the partnership is also liquidated, Ralph received cash of $30,000, accounts receivable (basis of $0, fair market value of $20,000), and land (basis of $1,000, fair market value of $10,000; treated as a capital asset by the partnership). Immediately before the distribution, Ralph’s basis in the partnership interest was $40,000. Ralph realizes and recognizes a loss of $9,000, and his basis is $0 in the accounts receivable and $1,000 in the land. a. True b. False 27. Two years ago, Marcus contributed land with a basis of $6,000 and a fair market value of $20,000 in exchange for a 30% interest in the MNO LLC. The partnership has no hot assets. This year when the value was $25,000, that property was distributed to Jamal whose basis in the LLC interest was $50,000 before the distribution. Marcus recognizes a gain of $14,000 and increases his basis in his LLC interest by that same amount; Jamal takes a $20,000 basis in the distributed property. a. True b. False 28. Normally a distribution of property from a partnership does not result in gain recognition. However, a distribution of marketable securities may trigger gain recognition. a. True b. False 29. The Crimson Partnership is a service-providing consulting firm, and Jana is a 20% general partner with a $100,000 basis in her partnership interest. Crimson's assets consist of unrealized receivables (basis of $0, fair market value of $400,000), cash of $300,000, and land (basis of $200,000, fair market value of $300,000). If Crimson distributes $200,000 cash to Jana in liquidation of her interest in the partnership (and Crimson does not liquidate), Jana will recognize ordinary income of $80,000 and a capital gain of $20,000. a. True b. False 30. Midway through the current tax year, Georgie sells her 40% interest in the GHI Partnership to new partner Kelly for $150,000, including Georgie’s share of partnership liabilities. At the beginning of the tax year, Georgie’s basis in her partnership interest was $40,000 (excluding her share of partnership debt). The partnership reported income of $120,000 for the year, and Georgie’s share of partnership debt was $50,000 at the sale date. (Assume that the partnership uses a monthly proration of income.) On the sale date, the partnership’s assets consist of cash ($195,000), land (basis of $90,000, fair market value of $120,000), and unrealized receivables (basis of $0, fair market value of $60,000). Georgie will recognize ordinary income of $24,000 and a capital gain of $12,000, for a total of $36,000 on the sale. a. True Powered by Cognero

Page 4


Name:

Class:

Date:

Chapter 10: Partnerships Distributions Transfer of Interests and Terminations b. False 31. Mark contributed property to the MDB Partnership in year 1. At the time of the contribution, the basis in the property was $40,000 and its value was $50,000. In year 5, MDB distributed that property to partner Dara. The partnership has no hot assets. Because this is a distribution of precontribution gain property, Dara may recognize a gain. a. True b. False 32. In addition to its non-hot assets, BC LLC owns unrealized receivables (basis = $0, fair market value = $60,000). If BC distributes $60,000 cash to Bart and the unrealized receivable to Charles in a current (nonliquidating) distribution, then Bart will recognize ordinary income of $30,000, and Charles will take a basis of $30,000 in receivables valued at $60,000. a. True b. False 33. Taylor’s basis in his partnership interest is $140,000, including his $60,000 share of partnership debt. Sandy buys Taylor’s partnership interest for $100,000 cash, and she assumes Taylor’s $60,000 share of the partnership’s debt. If the partnership owns no hot assets, Taylor will recognize a capital loss of $40,000. a. True b. False 34. James received a distribution of $110,000 cash in liquidation of his 25% managing (general partner) interest in the JKL LLP. JKL is a service-oriented entity. Before the distribution, the LLP’s assets consisted of $200,000 cash, land (basis of $40,000, value of $100,000), unrealized receivables (basis of $0, value of $100,000), and goodwill (basis of $0, value of $40,000). $10,000 of the total payment is for James' share of partnership goodwill, which is not provided for in the partnership agreement. James will recognize $35,000 of ordinary income from his §736(a) payment; the remaining $75,000 distribution is treated as a § 736(b) property payment for his partnership interest. a. True b. False 35. Nick sells his 25% interest in the LMNO Partnership to new partner Katrina for $67,500. The partnership’s assets consist of cash ($100,000), land (basis of $90,000, fair market value of $110,000), and inventory (basis of $40,000, fair market value of $60,000). Nick’s basis in his partnership interest was $57,500. On the sale, Nick will recognize ordinary income of $5,000 and a capital gain of $5,000. a. True b. False 36. If one partner in a two-partner partnership dies, the estate cannot be a partner, so the partnership is terminated. a. True b. False 37. A partnership has accounts receivable with a basis of $0 and a fair market value of $30,000 and depreciation recapture potential of $20,000. All other assets of the partnership are either cash, capital assets, or § 1231 assets. If a purchaser acquires a 40% interest in the partnership from another partner, the selling partner recognizes ordinary income of $12,000. a. True b. False 38. A § 754 election is made for a tax year in which the partner recognizes gain or loss on a distribution from the partnership or the distributee partner’s basis in distributed property is increased or decreased from the inside basis the Powered by Cognero

Page 5


Name:

Class:

Date:

Chapter 10: Partnerships Distributions Transfer of Interests and Terminations partnership held in those assets. The election is made by the partnership each year in which it is necessary to adjust a partner’s share of the inside basis of partnership assets; in a year in which an unfavorable result would arise, the partnership can forgo making the election. a. True b. False 39. Orson was a 40% partner in the calendar year ORP LLC. When Orson died on June 30, the interest transferred to his estate. On October 1 of that year, Yolanda, an unrelated third party, purchased Orson’s interest from his estate (with the approval of the other LLC members). The LLC’s operating agreement provides that a monthly allocation (annual income divided by 12) will be used to prorate income when required. For the year, the LLC will issue three Schedules K-1 related to this interest: (1) 20% of the year’s income to Orson, (2) 10% to Orson's Estate, and (3) 10% to Yolanda. a. True b. False 40. A partnership must make a downward adjustment to the basis of its assets if a partnership interest is sold and if the total basis of partnership assets exceeds their value by more than $250,000, even if a § 754 election is not in effect. a. True b. False 41. Beth sold her 25% partnership interest to Katie for $50,000 cash on July 1 (the halfway point) of the current tax year. Katie also assumed Beth’s share of the partnership’s liabilities. Beth’s basis in her partnership interest at the beginning of the year was $40,000, including a $15,000 share of partnership liabilities. The partnership’s income for the entire year was $100,000, and Beth’s share of partnership debt was $10,000 as of the date she sold the partnership interest. Assume the calendar year partnership has no hot assets, all of its income is earned evenly throughout the year, and the partnership uses the monthly proration method to allocate its income among the partners. Beth recognizes a gain of $12,500 on the sale. a. True b. False 42. The MBA Partnership makes a § 736(b) cash payment of $20,000 to partner Amanda in liquidation of her interest in the partnership. The partnership owns no hot assets and has no outstanding liabilities. Amanda’s basis in her partnership interest before the distribution was $50,000. If the partnership has a § 754 election in effect, it will record a $30,000 decrease in its inside basis in partnership assets, affecting all the remaining partners in the partnership. a. True b. False 43. Jeremy sold his 40% interest in the HIJ Partnership to Ashley for $400,000. The inside basis of all partnership assets was $600,000 at the time of the sale. If the partnership makes a § 754 election, it will record a $160,000 step-up in the basis of the partnership assets, and the step-up will be attributed solely to Ashley. a. True b. False 44. In the year that a donor gives a partnership interest to a donee, their share of the partnership’s income is prorated between the donor and donee. a. True b. False 45. Bob received a proportionate current (nonliquidating) distribution of land from the BZ Partnership. The land had a fair market value of $15,000 and a basis to the partnership of $10,000. The land was held for investment purposes by the Powered by Cognero

Page 6


Name:

Class:

Date:

Chapter 10: Partnerships Distributions Transfer of Interests and Terminations partnership. Bob’s basis in his partnership interest immediately before the distribution was $6,000. If the partnership has a § 754 election in effect, it will record a $4,000 step-down in the basis of remaining assets, and the step-down will be attributed to all partners in the partnership. a. True b. False 46. A two-person partnership terminates when one partner sells his, her, or its partnership interest to the other partner. This termination triggers a deemed liquidating distribution to the partners, and a second level of tax on the realized gain (fair market value less partnership basis) in the partnership’s assets. a. True b. False 47. A limited liability company generally provides limited liability for those owners that are not active in the management of the LLC but requires owner–managers of the LLC to have unlimited personal liability for LLC debts. a. True b. False 48. Fred and Wilma formed the equally owned FW partnership several years ago. On the last day of the prior tax year, they each transferred a 20% interest in the FW partnership (a capital-intensive business) to their son, Rocky in a bona fide transaction (40% total to Rocky; Fred and Wilma each retained 30% interests). For the current tax year, FW reported income of $200,000. Fred provides services to the partnership valued at $60,000; Wilma and Rocky provide no services. The $200,000 of income will be allocated $102,000 to Fred, $56,000 to Rocky, and $42,000 to Wilma. a. True b. False 49. GHI Partnership is a service-providing partnership owned 60% by Greg and 20% each by Howard and Isaac. Greg is an active general partner and wants to dispose of his partnership interest for $100,000, including $20,000 for partnership goodwill, (A goodwill payment to a retiring partner is not provided for in the partnership agreement). The partnership owns no hot assets. Greg will have the same tax result if he sells his interest to Howard and Isaac for $50,000 each ($100,000 total), or if GHI redeems Greg's interest by distributing $100,000 of cash to Greg. a. True b. False 50. Jeremiah received a proportionate nonliquidating distribution of land from the JZ Partnership. The land had a fair market value of $65,000 and a basis to the partnership of $50,000. The land was held for investment purposes by the partnership. Jeremiah’s basis in his partnership interest immediately before the distribution was $40,000. The partnership’s only remaining assets were cash, another parcel of land, and a building on that land. If the partnership has a § 754 election in effect, it will record a $10,000 basis step-up; a portion of the step-up will be allocated to the building and will be depreciated; and the step-up and any related depreciation expense will be allocated among all the partners in the partnership. a. True b. False Multiple Choice 51. Which of the following would be classified as disproportionate distributions? Assume that both A and B own 50% interests in the AB Partnership, the partnership owns hot assets, and the partners remain partners in AB after the distribution. Powered by Cognero

Page 7


Name:

Class:

Date:

Chapter 10: Partnerships Distributions Transfer of Interests and Terminations a. Partner A receives $6,000 cash and Partner B receives a capital asset valued at $6,000. b. Partner A receives $6,000 cash and Partner B receives inventory items valued at $6,000 (basis $3,500). c. Both partners receive unrealized receivables with a basis of $0 and a value of $6,000. d. Choices a. and c. represent disproportionate distributions. 52. Dan receives a proportionate current (nonliquidating) distribution when the basis of his partnership interest is $30,000. The distribution consists of $10,000 in cash and property with an adjusted basis to the partnership of $24,000 and a fair market value of $26,500. Dan's basis in the noncash property is: a. $26,500. b. $24,000. c. $20,000. d. $10,000. 53. At the beginning of the year, Elsie’s basis in the E&G Partnership interest is $90,000. She receives a proportionate current (nonliquidating) distribution from the partnership consisting of $10,000 of cash, unrealized accounts receivable (basis of $0, fair market value $40,000), and land (basis of $30,000, fair market value of $50,000). After the distribution, Elsie’s bases in the accounts receivable, land, and partnership interest are: a. $0; $30,000; and $50,000. b. $0; $50,000; and $30,000. c. $40,000; $30,000; and $10,000. d. $40,000; $40,000; and $0. 54. Megan’s basis was $120,000 in the MYP Partnership interest just before she received a proportionate current (nonliquidating) distribution consisting of land held for investment (basis of $100,000, fair market value of $130,000) and inventory (basis of $80,000, fair market value of $70,000). After the distribution, Megan’s bases in the received assets are, respectively: a. $100,000 (land) and $20,000 (inventory). b. $120,000 (land) and $0 (inventory). c. $50,000 (land) and $70,000 (inventory). d. $40,000 (land) and $80,000 (inventory). 55. Mark receives a proportionate current (nonliquidating) distribution. At the beginning of the partnership year, the basis of his partnership interest is $80,000 and his share of the partnership's income for the year is $20,000. During the year, he received a cash distribution of $40,000 and a property distribution (basis of $30,000, fair market value of $25,000). In addition, Mark’s share of partnership liabilities was reduced by $10,000 during the year. How much gain or loss does Mark recognize; what is his basis in the property he received; and what is his remaining basis in the partnership interest? a. $25,000 loss; $25,000 basis in property; $0 remaining basis. b. $30,000 loss; $30,000 basis in property; $0 remaining basis. c. $0 gain or loss; $25,000 basis in property; $25,000 remaining basis. d. $0 gain or loss; $30,000 basis in property; $20,000 remaining basis. 56. Mack has a basis in a partnership interest of $200,000, including his share of partnership debt. At the end of the current year, the partnership distributed to Mack, in a proportionate current (nonliquidating) distribution, cash of $20,000, inventory (basis to the partnership of $30,000 and fair market value of $40,000), and land (basis to the partnership of $40,000 and fair market value of $42,000). In addition, Mack’s share of partnership debt decreased by $12,000 during the year. What basis does Mack take in the inventory and land and in the partnership interest (including debt share) following the distribution? Powered by Cognero

Page 8


Name:

Class:

Date:

Chapter 10: Partnerships Distributions Transfer of Interests and Terminations a. $30,000 basis in inventory; $40,000 basis in land, $98,000 basis in partnership. b. $30,000 basis in inventory; $42,000 basis in land, $110,000 basis in partnership. c. $40,000 basis in inventory; $40,000 basis in land, $86,000 basis in partnership. d. $40,000 basis in inventory; $42,000 basis in land, $98,000 basis in partnership. 57. On December 31, the last day of the current taxable year, Jack has a basis in a partnership interest of $300,000, including his $80,000 share of partnership debt. On that date, the partnership pays off its liabilities and makes a proportionate current (nonliquidating) distribution to its partners. Jack receives a parcel of land (partnership basis = $120,000, value = $135,000) and inventory (partnership basis = $160,000, value = $180,000). Following the distribution, what is Jack’s basis in the land, inventory, and the partnership interest? a. $120,000 basis in land, $160,000 basis in inventory, $20,000 basis in partnership interest. b. $40,000 basis in land, $180,000 basis in inventory, $0 basis in partnership interest. c. $60,000 basis in land, $160,000 basis in inventory, $0 basis in partnership interest. d. $60,000 basis in land, $160,000 basis in inventory; $80,000 basis in partnership interest. 58. Frank receives a proportionate current (nonliquidating) distribution from the AEF Partnership. The distribution consists of $10,000 cash and property (adjusted basis to the partnership of $54,000 and fair market value of $60,000). Immediately before the distribution, Frank’s adjusted basis in the partnership interest was $50,000. His basis in the noncash property received is: a. $0. b. $40,000. c. $50,000. d. $54,000. 59. Alyce owns a 30% interest in a continuing partnership. The partnership distributes a $35,000 year-end cash payment to Alyce. In a proportionate current (nonliquidating) distribution, the partnership also distributed property (basis of $20,000, fair market value of $30,000) to Alyce. Immediately before the distributions of cash and property, Alyce’s basis in the partnership interest was $60,000. As a result of the distribution, Alyce recognizes: a. No gain or loss. b. Ordinary loss of $5,000. c. Capital loss of $5,000. d. Ordinary gain of $5,000. 60. Catherine’s basis was $50,000 in the CAR Partnership just before she received a proportionate current (nonliquidating) distribution consisting of land held for investment with a basis to CAR of $40,000 (value of $60,000), and inventory with a basis of $40,000 (value of $60,000). After the distribution, Catherine’s bases in the land and inventory are: a. $40,000 (land); $40,000 (inventory). b. $40,000 (land); $10,000 (inventory). c. $10,000 (land); $40,000 (inventory). d. $60,000 (land); $40,000 (inventory). 61. Misha receives a proportionate current (nonliquidating) distribution when the basis of his partnership interest is $60,000. The distribution consists of $80,000 cash and inventory (adjusted basis to the partnership of $10,000, fair market value of $20,000). How much gain or loss does Misha recognize, and what is his basis in the distributed inventory and in the partnership interest following the distribution? Powered by Cognero

Page 9


Name:

Class:

Date:

Chapter 10: Partnerships Distributions Transfer of Interests and Terminations a. $0 gain or loss; $10,000 basis in inventory; $0 basis in partnership interest. b. $0 gain or loss; $20,000 basis in inventory; $50,000 basis in partnership interest. c. $20,000 capital gain; $0 basis in inventory; $0 basis in partnership interest. d. $20,000 ordinary income; $0 basis in inventory; $20,000 basis in partnership interest. 62. Nicky’s basis in her partnership interest was $150,000, including her $40,000 share of partnership liabilities. The partnership decides to liquidate, and after repaying all liabilities, distributes all remaining assets proportionately to the partners. Nicky receives $30,000 cash and inventory with a $50,000 basis and a $58,000 fair market value to the partnership. What loss does Nicky recognize, and what is her basis in the inventory? a. $70,000 loss; $50,000 basis. b. $30,000 loss; $50,000 basis. c. $22,000 loss; $58,000 basis. d. $0 loss; $80,000 basis. 63. Anthony’s basis in the WAM Partnership interest was $200,000 just before he received a proportionate liquidating distribution consisting of investment land (basis of $90,000, fair market value of $100,000), and inventory (basis of $30,000, fair market value of $70,000). After the distribution, Anthony’s recognized gain or loss and his basis in the land and inventory are: a. $80,000 loss; $90,000 (land); $30,000 (inventory). b. $70,000 loss; $100,000 (land); $30,000 (inventory). c. $30,000 loss; $100,000 (land); $70,000 (inventory). d. $0 gain or loss; $170,000 (land); $30,000 (inventory). 64. Beth has an outside basis of $100,000 in the BJDE Partnership as of December 31 of the current year. On that date, the partnership liquidates and distributes to Beth a proportionate distribution of $50,000 cash and inventory with an inside basis to the partnership of $10,000 and a fair market value of $16,000. In addition, Beth receives an antique desk (not inventory) that has an inside basis (and fair market value) of $5,000. None of the distribution is for partnership goodwill. How much gain or loss will Beth recognize on the distribution, and what basis will she take in the desk? a. $40,000 loss; $0 basis. b. $35,000 loss; $5,000 basis. c. $0 gain or loss; $5,000 basis. d. $0 gain or loss; $40,000 basis. 65. Landis received $90,000 cash and a capital asset (basis of $50,000, fair market value of $60,000) in a proportionate liquidating distribution. His basis in his partnership interest was $120,000 prior to the distribution. How much gain or loss does Landis recognize, and what is his basis in the asset received? a. $0 gain or loss; $30,000 basis. b. $0 gain or loss; $50,000 basis. c. $20,000 gain; $50,000 basis. d. $30,000 gain; $60,000 basis. 66. Suzy owns a 30% interest in the JSD LLC. In liquidation of the entity, Suzy receives a proportionate distribution of $30,000 cash, inventory (basis of $16,000, fair market value of $18,000), and land (basis of $25,000, fair market value of $30,000). Suzy’s basis in the entity immediately before the distribution was $80,000. As a result of the distribution, what is Suzy’s basis in the inventory and land, and how much gain or loss does she recognize? a. $0 basis in inventory; $25,000 basis in land; $0 gain or loss. Powered by Cognero

Page 10


Name:

Class:

Date:

Chapter 10: Partnerships Distributions Transfer of Interests and Terminations b. $16,000 basis in inventory; $34,000 basis in land; $0 gain or loss. c. $16,000 basis in inventory; $25,000 basis in land; $9,000 loss. d. $18,000 basis in inventory; $32,000 basis in land; $0 gain or loss. 67. Jonathon owns a one-third interest in a liquidating partnership that owns no hot assets. Immediately before the liquidation, his basis in the partnership interest is $60,000. The partnership distributes cash of $32,000 and two parcels of land (each with a fair market value of $10,000). Parcel A has a basis of $2,000 to the partnership and Parcel B has a basis of $6,000. Jonathon’s basis in the two parcels of land is: a. Parcel A, $2,000; Parcel B, $6,000. b. Parcel A, $7,000; Parcel B, $21,000. c. Parcel A, $10,000; Parcel B, $10,000. d. Parcel A, $14,000; Parcel B, $14,000. 68. Janella’s basis in her partnership interest was $120,000, including her $150,000 share of partnership debt. At the end of the current year, the partnership pays off its debts and liquidates. Janella receives a proportionate liquidating distribution consisting of $42,000 cash and inventory valued at $24,000 (adjusted basis to the partnership = $20,000). How much gain or loss does Janella recognize, and what is her basis in the distributed property? a. $0 gain or loss; $78,000 basis in property. b. $58,000 capital loss; $20,000 basis in property. c. $30,000 capital gain; $24,000 basis in property. d. $72,000 capital gain; $0 basis in property. 69. Michelle receives a proportionate liquidating distribution when the basis of her partnership interest is $50,000. The distribution consists of $58,000 cash and noninventory property (adjusted basis to the partnership of $10,000 and fair market value of $12,000). The partnership has no hot assets. How much gain or loss does Michelle recognize, and what is her basis in the distributed property? a. $0 gain or loss; $10,000 basis in property. b. $0 gain or loss; $50,000 basis in property. c. $8,000 ordinary income; $0 basis in property. d. $8,000 capital gain; $0 basis in property. 70. In a proportionate liquidating distribution, Sam receives a distribution of $30,000 cash, accounts receivable (basis of $0, fair market value of $50,000), and land (basis of $20,000, fair market value of $50,000). In addition, the partnership repays all liabilities of which Sam’s share was $40,000. Sam’s basis in the entity immediately before the distribution was $120,000. As a result of the distribution, what is Sam’s basis in the accounts receivable and land, and how much gain or loss does he recognize? a. $0 basis in accounts receivable; $50,000 basis in land; $0 gain or loss. b. $0 basis in accounts receivable; $90,000 basis in land; $0 gain or loss. c. $50,000 basis in accounts receivable; $40,000 basis in land; $0 gain or loss. d. $50,000 basis in accounts receivable; $50,000 basis in land; $50,000 gain. 71. In a proportionate liquidating distribution, Ashleigh receives a distribution of $30,000 cash, accounts receivable (basis of $0, fair market value of $40,000), and land (basis of $40,000, fair market value of $50,000). In addition, the partnership repays all liabilities of which Ashleigh’s share was $70,000. Ashleigh’s basis in the entity immediately before the distribution was $60,000. As a result of the distribution, what is Ashleigh’s basis in the accounts receivable and land, and how much gain or loss does she recognize? a. $0 basis in accounts receivable; $30,000 basis in land; $20,000 gain. Powered by Cognero

Page 11


Name:

Class:

Date:

Chapter 10: Partnerships Distributions Transfer of Interests and Terminations b. $0 basis in accounts receivable; $0 basis in land; $40,000 gain. c. $0 basis in accounts receivable; $40,000 basis in land; $0 gain or loss. d. $40,000 basis in accounts receivable; $20,000 basis in land; $20,000 gain. 72. In a proportionate liquidating distribution, Sara receives a distribution of $40,000 cash, accounts receivable (basis of $0, fair market value of $30,000), and inventory (basis of $50,000, fair market value of $60,000). Her basis in the entity immediately before the distribution was $120,000. As a result of the distribution, what is Sara’s basis in the accounts receivable and inventory, and how much gain or loss does she recognize? a. $0 basis in accounts receivable; $50,000 basis in inventory; $30,000 loss. b. $0 basis in accounts receivable; $80,000 basis in inventory; $0 gain or loss. c. $40,000 basis in accounts receivable; $40,000 basis in inventory; $0 gain or loss. d. $30,000 basis in accounts receivable; $50,000 basis in inventory; $30,000 loss. 73. Which of the following statements correctly reflects one of the rules regarding proportionate liquidating distributions? a. A partner’s basis in distributed unrealized receivables is the lesser of the partnership’s basis in the receivables or their fair market value. b. The basis of unrealized receivables cannot be stepped up to their fair market value unless the partner has adequate unabsorbed basis. c. Assets are deemed distributed in the following order: cash, unrealized receivables and inventory, and finally, capital assets. d. The partner can recognize gain but not loss on a proportionate liquidating distribution. 74. Which of the following distributions would never result in gain recognition to the recipient (distributee) partner? a. A distribution of cash that follows a contribution of appreciated property to the partnership. b. A distribution of property to a partner who, three years ago, contributed other property with a built-in gain. c. A distribution to a second partner of property contributed by the first partner two years ago. d. A distribution of inventory property that is proportionate to the partners. 75. Last year, Darby contributed land (basis of $60,000, fair market value of $80,000) to the Seagull LLC in exchange for a 25% interest in the LLC. In the current year, the LLC distributes the land (now worth $82,000) to Shelby, who is also a 25% owner. Immediately prior to the distribution, Darby’s basis in the LLC was $70,000, and Shelby’s basis in the LLC was $110,000. The partnership owns no hot assets. How much gain or loss must be recognized and by whom? What is Shelby’s basis in the property she receives and Darby’s basis in her partnership interest following the distribution? a. No gain or loss; Shelby’s basis in the property is $80,000; Darby’s basis in interest is $70,000. b. $20,000 gain recognized by Darby; Shelby’s basis in the property is $80,000; Darby’s basis in interest is $90,000. c. $22,000 gain recognized by Darby; Shelby’s basis in the property is $82,000; Darby’s basis in interest is $92,000. d. $20,000 gain recognized by Shelby; Shelby’s basis in the property is $80,000; Darby’s basis in interest is $90,000. 76. Which of the following is not typically considered a hot asset? a. Accounts receivable of a cash basis partnership. b. Inventory with a basis of $16,000 and a fair market value of $15,000. c. Depreciation recapture potential. Powered by Cognero

Page 12


Name:

Class:

Date:

Chapter 10: Partnerships Distributions Transfer of Interests and Terminations d. Land held for development. 77. Tom, Tina, Tatum, and Terry are equal owners (treated as general partners) in the 4-Ts LLC, a cash basis service entity. 4-Ts has unrealized receivables of $400,000 (basis of $0) and no other hot assets. A goodwill payment of $50,000 per partner is provided for in the LLC’s operating agreement. If 4-Ts distributes cash of $300,000 to Tom in liquidation of his LLC interest, which of the following statements is correct? a. The $50,000 payment that relates to LLC goodwill cannot be deducted by the LLC. b. The partnership treats the $100,000 payment for Tom's share of unrealized receivables as part of Tom's § 736(b) payment. c. The $150,000 § 736(a) payment will result in a capital gain to Tom. d. The $200,000 § 736(b) payment will be taxed to Tom as ordinary income. 78. The December 31 balance sheet of the RST General Partnership reads as follows.

Cash Receivables Capital and § 1231 assets Total

Adjusted Basis $ 65,000 –0– 55,000 $120,000

FMV $ 65,000 7,500 100,000 $172,500

Roy, capital Sue, capital Ted, capital Total

$ 40,000 40,000 40,000 $120,000

$ 57,500 57,500 57,500 $172,500

The partners share equally in partnership capital, income, gain, loss, deduction, and credit. Ted’s adjusted basis for his partnership interest is $40,000. On December 31, general partner Ted retires from the partnership, receiving a $60,000 cash payment in liquidation of his interest. The partnership agreement states that $2,500 of the payment is for goodwill. Which of the following statements about this distribution is false? a. If capital is not a material income-producing factor to the partnership, the § 736(a) payment will be $2,500. b. If capital is a material income-producing factor, the entire $60,000 payment will be a § 736(b) property payment. c. The payment for Ted’s share of goodwill will create $2,500 of ordinary income to him. d. The partnership can deduct any amount that is a § 736(a) payment because it will be determined without regard to partnership profits. 79. Rajesh contributed appreciated property to the RS Partnership in year 1. In year 4, that property was distributed to Simon. Which one of the following statements best captures the tax consequences of the distribution? Assume the partnership has no hot assets, the property value has increased since the original contribution, and none of the precontribution gain has previously been recognized. a. Distributions are tax-deferred transactions; because no cash is distributed, neither the partners nor the partnership recognize gain on the distribution. b. The partnership recognizes the precontribution gain; Simon’s basis in the property is increased by the amount of recognized gain. c. Simon recognizes the precontribution gain and increases his basis in the partnership interest; the partnership’s Powered by Cognero

Page 13


Name:

Class:

Date:

Chapter 10: Partnerships Distributions Transfer of Interests and Terminations basis in other property is increased by the amount of recognized gain. d. Rajesh recognizes the precontribution gain and increases his basis in the partnership interest; Simon’s basis in the distributed property is increased by the amount of recognized gain. 80. The December 31 balance sheet of GST Services, LLP reads as follows.

Cash Receivables Capital assets Total

Adjusted Basis $300,000 –0– 120,000 $420,000

FMV $300,000 150,000 150,000 $600,000

George, capital Sue, capital Tom, capital Total

$140,000 140,000 140,000 $420,000

$200,000 200,000 200,000 $600,000

The partners share equally in partnership capital, income, gain, loss, deduction, and credit. Capital is not a material income-producing factor to the partnership, and all partners are active in the business. On December 31, general partner Sue receives a distribution of $200,000 cash in liquidation of her partnership interest under § 736. Sue’s outside basis for the partnership interest immediately before the distribution is $150,000. (Her basis does not correspond to her capital account because she purchased the interest a few years ago at a $10,000 premium and the partnership did not make a § 754 election.) How much is Sue’s gain or loss on the distribution and what is its character? a. $50,000 ordinary income. b. $40,000 ordinary income; $10,000 capital gain. c. $40,000 capital gain; $10,000 ordinary income. d. $50,000 ordinary income; $10,000 capital gain. 81. Which of the following statements is true regarding the sale of a partnership interest? a. The selling partner’s share of partnership liabilities is disregarded in determining the proceeds from the sale of a partnership interest. b. For purposes of computing the selling partner’s gain or loss, the partner’s basis in the partnership interest is determined as of the last day of the partnership tax year ending before the year in which the interest is sold. c. If a partner sells an interest in a partnership, income related to that interest for the year of the sale is allocated to the purchaser. d. The selling partner could be required to report both ordinary income and a capital gain or loss on sale of the partnership interest. 82. The BR LLC owns an unrealized receivable with a basis of $0 and fair market value of $200,000 plus cash of $200,000. If BR distributes $20,000 of the receivable to 50% member Brenda and $20,000 of cash to 50% member Renee, which one of the following statements is true? Assume each partner has a partnership interest basis of $100,000. a. Brenda recognizes $20,000 of capital gain. b. The partnership recognizes $20,000 of ordinary income. c. Renee recognizes $10,000 of ordinary income. d. Brenda recognizes $10,000 of ordinary income. Powered by Cognero

Page 14


Name:

Class:

Date:

Chapter 10: Partnerships Distributions Transfer of Interests and Terminations 83. Miguel contributed substantially appreciated property to the MR Partnership in year 1. In year 4, other property was distributed to Miguel. The partnership does not have any hot assets. Which of the following statements is most likely to be true regarding the contribution and/or distribution? a. Miguel recognizes precontribution gain in year 4 when the second property is distributed to him. b. Miguel recognizes the precontribution gain on an amended tax return for year 1 when the second property is distributed to him. c. The partnership recognizes the precontribution gain in year 4 and allocates that gain to all the partners. d. The partnership recognizes the precontribution gain in year 1 and allocates that gain to Miguel. 84. Nicholas is a 25% owner in the DDBN LLC (a calendar year entity). At the end of the last tax year, Nicholas’s basis in his interest was $50,000, including his $20,000 share of LLC liabilities. On July 1 of the current tax year, Nicholas sells his LLC interest to Anna for $80,000 cash. In addition, Anna assumes Nicholas’s share of LLC liabilities, which, at that date, was $15,000. During the current tax year, DDBN’s taxable income is $120,000 (earned evenly during the year and allocated using the monthly proration method). Nicholas’s share of the LLC’s unrealized receivables is valued at $6,000 ($0 basis). At the sale date, what is Nicholas’s basis in his LLC interest, how much gain or loss must he recognize, and what is the character of the gain or loss on the sale of the partnership interest? a. $45,000 basis; $6,000 ordinary income; $44,000 capital gain. b. $60,000 basis; $6,000 ordinary income; $29,000 capital gain. c. $60,000 basis; $0 ordinary income; $35,000 capital gain. d. $75,000 basis; $0 ordinary income; $20,000 capital gain. 85. On June 30 of the current tax year, Sal sells her 40% interest in the STU Partnership to new partner James for $300,000, including Sal’s share of partnership liabilities. At the beginning of the tax year, Sal’s basis in her partnership interest was $80,000 (excluding her share of partnership debt). The partnership reported income of $240,000 for the year, and Sal’s share of partnership debt was $100,000 at the sale date. (Assume the partnership uses a monthly proration of income.) At the sale date, the partnership’s assets consist of cash ($390,000), land (basis of $180,000, fair market value of $210,000), and unrealized receivables (basis of $0, fair market value of $150,000). What is Sal’s basis at the sale date, and how much gain must Sal recognize on sale of the partnership interest, and what is the character of the gain? a. $180,000 basis, $60,000 ordinary income, $60,000 capital gain. b. $128,000 basis, $60,000 ordinary income, $112,000 capital gain. c. $228,000 basis, $60,000 ordinary income, $12,000 capital gain. d. $228,000 basis, $12,000 ordinary income, $60,000 capital gain. 86. The BLM LLC’s balance sheet on August 31 reads as follows.

Cash Receivables Capital assets

Nonrecourse debt Barney, capital Lillie, capital Marshall, capital

Adjusted Basis $ 60,000 –0– 90,000 $150,000

FMV $ 60,000 150,000 300,000 $510,000

$ 90,000 20,000 20,000 20,000 $150,000

$ 90,000 140,000 140,000 140,000 $510,000

The nonrecourse debt is shared equally among the LLC members. On that date, Lillie sells her one-third interest to Robyn Powered by Cognero

Page 15


Name:

Class:

Date:

Chapter 10: Partnerships Distributions Transfer of Interests and Terminations for $170,000, including cash and relief of Lillie’s share of the nonrecourse debt. Lillie’s outside basis for her interest in the LLC is $50,000, including her share of the LLC’s debt and after allocation of Lillie's share of income for the period. How much capital gain and/or ordinary income will Lillie recognize on the sale? a. $100,000 capital gain; $50,000 ordinary income. b. $120,000 capital gain; $0 ordinary income. c. $150,000 capital gain; $0 ordinary income. d. $70,000 capital gain; $50,000 ordinary income. 87. Which of the following statements about the transfer of a partnership interest is not true? a. The seller’s adjusted basis for the partnership interest is increased by the seller’s share of undistributed partnership income (or reduced by partnership loss) for the portion of the partnership’s taxable year ending on the date of the sale. b. The partnership taxable year generally does not close with respect to a partner who transfers a partnership interest at death; all amounts are allocated to the successor. c. The amount realized on the sale of a partnership interest is the sum of any money and the fair market value of any property received for the interest plus the selling partner’s share of partnership liabilities under § 752. d. The selling partner will report (1) income/loss from the Schedule K-1 for the portion of the year in which the interest was owned, and (2) gain/loss on sale of the interest. 88. Brittany, Jennifer, and Daniel are equal partners in the BJD Partnership. The partnership balance sheet reads as follows on December 31.

Cash Unrealized receivables Land Total

Adjusted Basis $ 75,000 –0– 45,000 $120,000

FMV $ 75,000 51,000 63,000 $189,000

Brittany, capital Jennifer, capital Daniel, capital Total

$ 40,000 40,000 40,000 $120,000

$ 63,000 63,000 63,000 $189,000

Partner Daniel has an adjusted basis of $40,000 for his partnership interest. If he sells his entire partnership interest to new partner Amber for $73,000 cash, by how much can the partnership step up the basis of Amber’s share of partnership assets under §§ 754 and 743(b)? a. $6,000 b. $17,000 c. $23,000 d. $33,000 89. Marcella is a 40% nonmanaging member (treated as a limited partner) in the MNO LLC in which capital is a material income-producing factor. MNO has inventory with a basis of $200,000 and a value of $250,000 but no other hot assets. Marcella’s basis in the LLC interest is $80,000. Total value of partnership assets recorded on the books is $450,000. If she receives a distribution of $180,000 cash in liquidation of her interest in the LLC, which one of the following statements is not true? a. Because this is a capital-intensive business, the entire payment is a §736(b) payment. b. Because Marcella is treated as a limited partner, the entire payment is a §736(b) payment. Powered by Cognero

Page 16


Name:

Class:

Date:

Chapter 10: Partnerships Distributions Transfer of Interests and Terminations c. MNO can deduct the $20,000 paid for Marcella’s share of ordinary income related to the inventory. d. None of the $180,000 payment to Marcella is for partnership goodwill. 90. Partner Jordan received a distribution of $90,000 cash from the JKL Partnership in complete liquidation of his partnership interest. If his outside basis immediately before the distribution was $80,000 and if the partnership has made (and not revoked) a § 754 election in a prior year, which of the following statements is true? (Assume the partnership owns no hot assets.) a. The partnership will step down the basis of its assets by $10,000. b. The partnership will step up the basis of its assets by $10,000. c. Jordan will recognize a $10,000 capital gain on the distribution. d. Both b. and c. are true. 91. The RST Partnership makes a proportionate distribution of its assets to Ryan in complete liquidation of his partnership interest. The distribution consists of $40,000 in cash and capital assets with a basis to the partnership of $30,000 and a fair market value of $48,000. None of the payment is for partnership goodwill. At the time of the distribution, Ryan’s partnership basis is $45,000 and the partnership has no liabilities and no hot assets. If the partnership makes an optional basis adjustment election on a timely filed return, the partnership recognizes: a. Capital gain of $25,000 and increases the basis of its remaining assets by $12,500. b. Capital loss of $5,000 and decreases the basis of its remaining assets by $5,000. c. No gain or loss and increases the basis of its remaining assets by $12,500. d. No gain or loss and increases the basis of its remaining assets by $25,000. 92. Cynthia sells her 1/3 interest in the CAR Partnership to Brandon for $95,000 cash. On the date of sale, the partnership balance sheet and agreed-upon fair market values were as follows.

Cash Receivables Land Total

Adjusted Basis $40,000 –0– 50,000 $90,000

FMV $ 40,000 60,000 125,000 $225,000

Cynthia, capital Arnold, capital Ralph, capital Total

$30,000 30,000 30,000 $90,000

$ 75,000 75,000 75,000 $225,000

If the partnership has a § 754 election in effect, the total “step up” in basis of partnership assets allocated to Brandon is: a. $75,000. b. $65,000. c. $45,000. d. $0. 93. BLC Partnership makes a proportionate distribution of its assets to Blake in complete liquidation of his partnership interest. At that time, Blake’s basis in his partnership interest is $160,000 and the partnership has no liabilities and no hot assets. The distribution consists of $100,000 in cash and capital assets with a basis to the partnership of $15,000 and a fair market value of $70,000. Assume that the entire payment is a § 736(b) payment with the same result as under the normal liquidating distribution rules and that none of the payment is for partnership goodwill. If the partnership made an optional Powered by Cognero

Page 17


Name:

Class:

Date:

Chapter 10: Partnerships Distributions Transfer of Interests and Terminations basis adjustment election in a prior tax year, what is the adjustment amount for the current year? a. Increase the basis of remaining assets by $55,000. b. Decrease the basis of remaining assets by $10,000. c. Increase the basis of remaining assets by $45,000. d. Decrease the basis of remaining assets by $45,000. 94. Which of the following transactions will not result in the termination of a partnership for Federal tax purposes? a. The partnership is incorporated. b. The sole surviving partner buys the interest of the deceased second partner through a buy-sell agreement taking place upon the second partner's death. c. Cash is distributed in liquidation of a 60% partner’s interest in a five-partner partnership. d. A 40% interest in partnership capital and profits is sold to the other partner in a two-partner partnership. 95. Which of the following statements is correct regarding termination of a three-person partnership? a. The partnership can terminate only when it distributes all of its assets in a complete liquidation. b. Redemption of one partner’s 60% interest under § 736 will terminate the partnership. c. If one partner dies, the partnership year closes, and the partnership must file a return as of the date of death. d. An incorporation of the partnership will terminate the partnership. 96. Roman was a 40% partner in the calendar year POR LLC. When Roman died on April 30, the interest transferred to his estate. On November 1 of that year, Tiwanda, an unrelated third party, negotiated a buyout and acquired Roman’s interest from the estate (with the approval of the other LLC members). The LLC’s operating agreement provides that a monthly allocation will be used to prorate income when required. If the LLC’s income for the year was $300,000, how will it be allocated? a. $0 to Roman, $0 to Roman's estate, $120,000 to Tiwanda. b. $40,000 to Roman, $60,000 to Roman's estate, $20,000 to Tiwanda. c. $0 to Roman, $100,000 to Roman's estate, $20,000 to Tiwanda. d. $100,000 to Roman, $0 to Roman's estate, $20,000 to Tiwanda. 97. Leonard and Penny, a married couple, formed the equally owned calendar year PL LLC several years ago. Capital is a material income-producing factor for the LLC. On July 1 of this year, they each transferred a 20% interest to their son, Sheldon (after the transfer, 40% is owned by Sheldon; Leonard and Penny each retained 30% interests). For the current tax year, PL reported income of $300,000, which was earned evenly throughout the year. Penny provides uncompensated services to the partnership valued at $100,000; Leonard and Sheldon provide no services. The partnership uses a monthly proration of income. How much income will be allocated to Penny, Leonard, and Sheldon for the year? a. $300,000 to Penny, $0 to Leonard, $0 to Sheldon. b. $90,000 to Penny, $90,000 to Leonard, $120,000 to Sheldon. c. $120,000 to Penny, $120,000 to Leonard, $60,000 to Sheldon. d. $180,000 to Penny, $80,000 to Leonard, $40,000 to Sheldon. 98. On December 31 of last year, Maria gave her daughter, Chelsea, a gift of a 25% interest in a partnership in which capital is a material income-producing factor. For the current calendar year, the partnership’s ordinary income was $100,000. Maria and Chelsea were the only partners, and there were no guaranteed payments. Maria’s services performed for the partnership were worth $60,000, and Chelsea has never performed any services. What is Maria’s distributive share of partnership income for the current year? a. $25,000 Powered by Cognero

Page 18


Name:

Class:

Date:

Chapter 10: Partnerships Distributions Transfer of Interests and Terminations b. $60,000 c. $75,000 d. $90,000 99. Which of the following statements, if any, about a multimember LLC is false? a. A multimember LLC is usually taxed like a partnership. b. Members of an LLC generally have limited personal liability for debts of the LLC except for the managing member who has unlimited liability for LLC debts. c. Members of an LLC can participate in management of the LLC unless the member agrees not to participate. d. Liquidating distributions from an LLC are subject to a single level of taxation in the same manner as a liquidating distribution from a general partnership. 100. The POD Partnership will soon be liquidating the 20% interest of partner Daniel by making a cash distribution of $200,000 to him under § 736. Much of that payment is in return for Daniel’s share of partnership goodwill. Which of the following conditions would have to be true to increase the portion of that payment that POD could claim as a current deduction? a. Daniel is treated as a general partner. b. The partnership agreement is silent regarding payment of partnership goodwill. c. The partnership is an entity in which capital is not a material income-producing factor. d. To maximize the partnership’s deductions under § 736, a., b., and c. would have to be true. 101. On July 1, George is planning to retire from the GDP LLC where he is an active managing member owning a 60% interest. Capital is not a material income-producing factor to GDP. The LLC can either redeem his interest under § 736 or he can sell his interest to Dale, who currently owns a 20% interest. The LLC’s operating agreement is silent regarding treatment of goodwill. As to George’s alternatives, which one of the following statements is true? a. Under either alternative, on July 1, the partnership closes its books and starts a new tax year. b. Payments to George for his share of GDP’s goodwill would be treated the same for either a sale or a redemption. c. George will report ordinary income related to his share of hot assets under either the sale or the redemption scenario. d. If GDP/Dale negotiate payments over several years, either an installment sale or a redemption over time would result in the same tax situation to George. Matching Match the following statements with the best match from the following choices. Choice K may be used more than once. a. Cash basis accounts receivable, for example. b. Fair market value exceeds 120% of basis. c. Inside basis of partnership property can be adjusted to reflect the purchase price paid. d. Terminates the partner’s interest in the partnership. e. Ordinary income-producing items. f. Cash, then inventory and unrealized receivables, and then other assets. g. Does not eliminate the partner’s interest in the partnership. h. Changes the partner’s or the partnership’s ordinary income potential. i. Any partnership assets other than cash, capital, or § 1231 assets. Powered by Cognero

Page 19


Name:

Class:

Date:

Chapter 10: Partnerships Distributions Transfer of Interests and Terminations j. Sometimes treated as an unrealized receivable. k. No correct match provided. 102. Nonqualified distribution 103. Ordering rules 104. Hot assets 105. Substantially appreciated inventory 106. Current distribution 107. Potential depreciation recapture 108. Inventory 109. Unrealized receivable 110. Disproportionate distribution 111. Liquidating distribution 112. Optional adjustment election Match the following statements with the best match from the following choices. Choice M may be used more than once. a. Includes the partner’s share of partnership liabilities. b. Could result from sale of a partnership interest for more than the partner’s share of the inside basis of assets. c. Liquidation payments from this type of partnership are always § 736(b) payments. d. Could arise if a distribution results in loss to the distributee partner. e. May be a § 736(a) payment. f. May receive § 736(a) payments. g. Probably treated as a general partner for § 736 purposes h. Conversion of an LLC to a C corporation i. Liquidation payments from this type of partnership may include § 736(a) payments. j. A § 736(b) payment. k. Adjustment designed to bring inside and outside bases into balance. l. Partnership asset basis is at least $250,000 > FMV. m. No correct match is provided. 113. Limited partner 114. General partner 115. Limited liability partnership 116. Active member of LLC Powered by Cognero

Page 20


Name:

Class:

Date:

Chapter 10: Partnerships Distributions Transfer of Interests and Terminations 117. Unstated goodwill 118. Stated goodwill 119. Capital intensive partnership 120. Service-providing partnership 121. Sales price of partnership interest 122. Section 754 123. Mandatory step down 124. Step up 125. Step down 126. Partnership terminates Match the following independent distribution payments in liquidation of a partner’s interest in an ongoing partnership with the statements below. a. A payment for the partner’s share of partnership income under § 736(a). b. A payment for the partner’s share of partnership property under § 736(b). c. The payment includes both a § 736(a) and a § 736(b) element. 127. Distribution of $10,000 cash to a limited partner for goodwill when goodwill is not provided for in the partnership agreement. 128. Distribution of $10,000 cash to a general partner for goodwill when goodwill is provided for in the partnership agreement, and the partnership derives most of its income from services. 129. Distribution of $25,000 cash for a partner’s share of substantially appreciated inventory. 130. Distribution of $60,000 cash for a partner’s share of unrealized receivables when the partner is a general partner and most of the partnership’s income is derived from services. 131. Distribution of $60,000 cash for a partner’s share of unrealized receivables when the partner is a limited partner and most of the partnership’s income is derived from services. 132. Distribution of $100,000 cash to a managing member (general partner) in a cash basis service-oriented LLC for the member's share of unrealized receivables and land held for investment. 133. Distribution of $100,000 cash representing the partner’s share of the value of partnership equipment that has potential depreciation recapture of $25,000. Match the following independent descriptions with the following statements. a. Hot assets for purposes of distributions, liquidation of a partnership interest under § 736, and sale of a partnership interest. Powered by Cognero

Page 21


Name:

Class:

Date:

Chapter 10: Partnerships Distributions Transfer of Interests and Terminations b. May be a hot asset for some but not all the purposes stated in (a). c. Not a hot asset. 134. Inventory with a basis of $10,000 and a fair market value of $15,000. 135. Marketable securities (not held as inventory). 136. Cash basis accounts receivable. 137. Installment receivables for sale of a capital asset. 138. Highly appreciated land held by the partnership for the purpose of subdividing and selling lots. 139. Inventory with a basis of $10,000 and a fair market value of $10,500. Subjective Short Answer 140. Randy owns a one-fourth capital and profits interest in the calendar year RUSR Partnership. His adjusted basis for his partnership interest was $200,000 when he received a proportionate nonliquidating distribution of the following assets. Cash Inventory

Partnership’s Basis in Asset $120,000 60,000

Asset’s Fair Market Value $120,000 90,000

a.

Calculate Randy’s recognized gain or loss on the distribution, if any. Explain.

b.

Calculate Randy’s basis in the inventory received.

c.

Calculate Randy’s basis for his partnership interest after the distribution.

141. Connie owns a one-third capital and profits interest in the calendar year CAB Partnership. Her adjusted basis for her partnership interest was $120,000 when she received a proportionate current (nonliquidating) distribution of the following assets. Cash Land held for investment

Partnership’s Basis in Asset $140,000 30,000

Asset’s Fair Market Value $140,000 60,000

a.

Calculate Connie’s recognized gain or loss (if any) on the distribution.

b.

Calculate Connie’s basis in the land received.

c.

Calculate Connie’s basis for her partnership interest after the distribution.

142. Karli owns a 25% capital and profits interest in the calendar year KJDV Partnership. Her adjusted basis for her partnership interest on July 1 of the current year is $200,000. On that date, she receives a proportionate current (nonliquidating) distribution of the following assets. Cash Inventory Land (held for investment) Powered by Cognero

Partnership’s Basis in Asset $120,000 50,000 70,000

Asset’s Fair Market Value $120,000 60,000 100,000 Page 22


Name:

Class:

Date:

Chapter 10: Partnerships Distributions Transfer of Interests and Terminations a.

Calculate Karli’s recognized gain or loss on the distribution if any.

b.

Calculate Karli’s basis in the inventory received.

c.

Calculate Karli’s basis in land received.

d.

Calculate Karli’s basis for her partnership interest after the distribution.

143. In a proportionate current (nonliquidating) distribution of his 30% interest in the MNO LLC, Neil received cash ($60,000), land (basis of $40,000 and value of $75,000), and unrealized receivables (basis of $0 and value of $22,000). In addition, Neil is relieved of his $40,000 share of the LLC’s liabilities. Neil’s basis in MNO (including his share of LLC liabilities) was $80,000 immediately prior to this distribution. a.

How much gain or loss does Neil recognize on this distribution?

b.

What is Neil’s basis in the receivables and land he receives in the distribution?

c.

What is Neil’s basis in the LLC interest following the distribution?

144. Melissa is a partner in a continuing partnership. At the end of the current year, the partnership makes a proportionate, current (nonliquidating) distribution to Melissa of $50,000 cash, inventory (basis of $22,000, fair market value of $20,000), and land (basis of $30,000, fair market value of $60,000). Melissa’s basis in the partnership interest was $90,000 before the distribution. What is Melissa’s basis in the inventory, land, and partnership interest following the distribution? Show your calculations. 145. In a proportionate liquidating distribution of his 40% interest in the RST LLC, Stuart received cash ($100,000), land (basis of $60,000 and value of $90,000), and inventory (basis of $30,000 and value of $40,000). (The partnership also liquidates.) In addition, Stuart is relieved of his $80,000 share of the LLC’s liabilities. Stuart’s basis in RST (including his share of LLC liabilities) was $200,000 immediately prior to this distribution. a.

How much gain or loss does Stuart recognize on this distribution?

b.

What is Stuart’s basis in the land and inventory he receives in the distribution?

146. In a proportionate liquidating distribution in which the partnership is liquidated, Bill received cash of $120,000, inventory (basis of $6,000, fair market value of $8,000), and a capital asset (basis and fair market value of $16,000). Immediately before the distribution, Bill’s basis in the partnership interest was $90,000. a.

How much gain or loss will Bill recognize on the distribution?

b.

What is Bill’s basis in the inventory and the capital asset?

147. Josh owns a 25% capital and profits interest in the calendar year GDJ Partnership. His adjusted basis for his partnership interest on October 15 of the current year is $300,000. On that date, the partnership liquidates and makes a proportionate distribution of the following assets to Josh. Cash Inventory a.

Partnership’s Basis in Asset $ 70,000 120,000

Asset’s Fair Market Value $ 70,000 150,000

Calculate Josh’s recognized gain or loss on the liquidating distribution, if any, and his basis in the distributed inventory.

Powered by Cognero

Page 23


Name:

Class:

Date:

Chapter 10: Partnerships Distributions Transfer of Interests and Terminations b.

How would your answer to a. change if the partnership also distributed a small parcel of land it had held for investment to Josh? Assume the land has a $5,000 adjusted basis (FMV is $8,000) to the partnership.

148. The December 31 balance sheet of DBW, LLP, a service-providing partnership, reads as follows.

Cash Receivables Capital assets Total

Adjusted Basis $180,000 –0– 90,000 $270,000

FMV $180,000 60,000 120,000 $360,000

Dana, capital Brooke, capital Whitney, capital Total

$ 90,000 90,000 90,000 $270,000

$120,000 120,000 120,000 $360,000

The partners share equally in partnership capital, income, gain, loss, deductions, and credits. Capital is not a material income-producing factor to the partnership. On December 31, partner Dana (who is an active managing partner in the partnership) receives a distribution of $120,000 cash in liquidation of her partnership interest under § 736. Dana’s outside basis for the partnership interest immediately before the distribution is $90,000. How much is her gain or loss on the distribution and what is its character? 149. The December 31 balance sheet of the calendar-year BCD LLP reads as follows.

Cash Receivables Capital assets Total

Adjusted Basis $210,000 –0– 42,000 $252,000

FMV $210,000 120,000 69,000 $399,000

Ben, capital Christina, capital Danielle, capital Total

$ 84,000 84,000 84,000 $252,000

$133,000 133,000 133,000 $399,000

Each partner shares in 1/3 of the partnership capital, income, gain, loss, deductions, and credits. Capital is not a material income-producing factor to the partnership. On December 31, Christina (treated as a general partner) receives a distribution of $140,000 cash in liquidation of her partnership interest under § 736. Nothing is stated in the partnership agreement about goodwill. Christina’s outside basis for the partnership interest immediately before the distribution is $84,000. How much is Christina’s recognized gain from the distribution and what is the character of the gain? How much, if anything, can the partnership deduct? If a § 754 election is in effect, what (if any) adjustment is made? Show your calculations.

Powered by Cognero

Page 24


Name:

Class:

Date:

Chapter 10: Partnerships Distributions Transfer of Interests and Terminations 150. Jeremy is an active partner who owns a 30% interest in the JS LLP (in which capital is not a material incomeproducing factor). Partnership assets consist of land (fair market value of $200,000, basis of $140,000), accounts receivable (fair market value of $200,000, basis of $0), and cash of $400,000. JS distributes $220,000 of the cash to Jeremy in liquidation of his interest. In addition, Jeremy is relieved of his $40,000 share of the LLP’s liabilities. The total payment includes $20,000 for Jeremy’s share of JS goodwill (not stated in the partnership agreement). Jeremy’s basis in the partnership interest (including his share of the partnership’s liabilities) is $120,000 immediately before the distribution. How much gain or loss does Jeremy recognize and what is its character? How much can the partnership deduct? Are any planning opportunities available to the LLP? 151. Serena owns a 40% interest in the RST LLP. Partnership assets consist of land (fair market value of $100,000, basis of $80,000), accounts receivable (fair market value of $120,000, basis of $0), and cash of $180,000. Serena sells her interest in RST to Jaclyn for cash of $140,000. In addition, Jaclyn assumes Serena’s $40,000 share of the LLP’s liabilities. Serena’s basis in the partnership interest (including her share of the partnership’s liabilities) is $120,000 immediately before the sale. a. How much gain or loss does Serena recognize and what is its character? b.

What is Jaclyn’s basis in the partnership interest?

c.

If the LLP has a § 754 election in effect, how much is the adjustment and to which partner(s) is it allocated?

152. On August 31 of the current tax year, the balance sheet of the RBD General Partnership reads as follows Adjusted Basis FMV Cash $150,000 $150,000 Receivables –0– 90,000 Capital assets 600,000 660,000 Total $750,000 $900,000 Nonrecourse debt Rachel, capital Barry, capital Dale, capital Total

$150,000 200,000 200,000 200,000 $750,000

$150,000 250,000 250,000 250,000 $900,000

On that date, Rachel sells her one-third partnership interest to Lisa for $300,000, consisting of cash and relief of Rachel’s share of the nonrecourse debt. The nonrecourse debt is shared equally among the partners. Rachel’s outside basis for her partnership interest is $250,000 (including her share of partnership debt). How much capital gain and/or ordinary income will Rachel recognize on the sale? 153. Hannah sells her 25% interest in the HIJK Partnership to Alyssa for $120,000 cash. At the end of the year prior to the sale, Hannah’s basis in HIJK was $70,000. The partnership allocates $15,000 of income to Hannah for the portion of the year she was a partner. The partnership had no debt. On the date of the sale, the partnership assets and the agreed fair market values were as follows.

Cash Accounts Receivable Land Total Powered by Cognero

Adjusted Basis $100,000 –0– 240,000 $340,000

FMV $100,000 80,000 220,000 $400,000 Page 25


Name:

Class:

Date:

Chapter 10: Partnerships Distributions Transfer of Interests and Terminations Determine the amount and character of any gain that Hannah recognizes on the sale. 154. Gil’s outside basis in his interest in the GO Partnership is $100,000. In a proportionate current

distribution, the partnership distributes to him cash of $30,000, inventory (fair market value of $40,000, basis to the partnership of $20,000), and land (fair market value of $90,000, basis to the partnership of $40,000). The partnership continues in existence. a. Does the partnership recognize any gain or loss as a result of this distribution? Explain. b. Does Gil recognize any gain or loss as a result of this distribution? Explain.

c. Calculate Gil’s basis in the land, in the inventory, and in his partnership interest

immediately following the distribution.

155. Parker’s basis in his PQ Partnership interest is $180,000. Parker receives a pro rata liquidating distribution

consisting of $20,000 cash, land with a basis of $80,000 and a fair market value of $100,000, and his proportionate share of inventory with a basis of $60,000 to PQ and a fair market value of $75,000. Assume that PQ also liquidates. a. How much gain or loss, if any, must Parker recognize on the distribution? b. What basis will Parker take in the inventory and land? c. What are the tax consequences to the partnership? d. Would your answer to part (a) or (b) change if this had been a current

distribution? Explain. 156. Ten years ago, Dudley contributed land to the Prosperity LLC. His basis in the land was $100,000.

The fair market value at the contribution date was $115,000. This year, when the property’s value was $200,000, the LLC distributed that property to partner Nicki. At the distribution date, Dudley’s basis in his LLC interest was $150,000 and Nicki’s basis was $160,000. Assume that the partnership continues in existence and has no hot assets. a. What gain or loss is recognized as a result of this distribution of precontribution

gain property contributed ten years earlier? b.

What is Dudley’s basis in his partnership interest following the distribution? What is Nicki’s basis in the property received and her partnership interest following the distribution?

c. Would your answer change if Dudley had originally contributed the property

two years ago? Answer conceptually (no calculations required). Powered by Cognero

Page 26


Name:

Class:

Date:

Chapter 10: Partnerships Distributions Transfer of Interests and Terminations Essay 157. Your client has operated a sole proprietorship for several years and is now interested in raising capital for expansion. The client is considering forming either a C corporation or an LLC. a.

Describe the treatment of an LLC and discuss any advantages the LLC offers over the C corporation.

b. Assume instead the client has previously operated as a C corporation. Describe the tax consequences of converting to an LLC. 158. Julie is an active owner of a 52% interest in the JIR LLP, a consulting company (service provider). Her basis in the partnership interest is $100,000, and her share of the partnership’s inside basis in assets is $120,000. Julie can sell her interest in the LLP on the first day of the tax year to Irene and Rachel (the other partners) for $100,000 each ($200,000 total). Alternatively, the LLP can distribute $200,000 of cash to redeem Julie’s interest. Assume the following: $10,000 of the redemption payment would be for the LLP’s goodwill (which is not provided for in the partnership agreement); Julie’s share of JIR’s unrealized receivables is $40,000; and JIR has a § 754 election in effect. What are the advantages and disadvantages of the sale versus the redemption from Julie’s and JIR’s perspective? What is your recommendation? Explain. 159. George (a calendar year taxpayer) owns a 40% interest in the cash basis GLO LLP, which has substantial accounts receivable from its customers. GLO has a natural business year ending March 31. George has found another opportunity and would like to sell his interest on July 1 of the current tax year to new partner Monica. What are some of the issues that should be considered by George, Monica, and GLO? 160. Compare the different tax results (gains, losses, basis) that might arise for a partner in a proportionate current (nonliquidating) distribution versus a proportionate liquidating distribution, under the general tax rules. 161. Your client, Greg, contributed precontribution gain property to BIG LLC on December 31, 2023, in exchange for a 30% interest.

a.

Describe two types of distributions that might result in some or all of this precontribution gain being recognized by Greg.

b.

In general terms, what is the purpose of these rules?

162. Cindy, a 20% general partner in the CDE Partnership, wants to retire and has approached the other partners about having the partnership buy her out. The partnership is a cash basis, service-oriented partnership in which Cindy is an active partner. The partnership’s assets consist primarily of unrealized receivables and cash. The partnership also has substantial going concern value (goodwill) which is probably its most valuable asset. Goodwill payments are not currently provided for in the partnership agreement, but the partnership agreement can be amended to include goodwill payments if that is advantageous. The other partners in the partnership are also active in the business and are not related to Cindy. Discuss from Cindy’s viewpoint how you would structure the liquidation of her interest under § 736. Answer as if you are her advocate. Do you think the other partners will agree with this structure? If not, what structure would they prefer?

Powered by Cognero

Page 27


Name:

Class:

Date:

Chapter 10: Partnerships Distributions Transfer of Interests and Terminations Answer Key 1. False 2. False 3. True 4. True 5. False 6. False 7. True 8. False 9. False 10. False 11. True 12. True 13. False 14. True 15. True 16. True 17. True 18. False 19. False 20. False 21. True 22. False 23. True 24. True 25. False Powered by Cognero

Page 28


Name:

Class:

Date:

Chapter 10: Partnerships Distributions Transfer of Interests and Terminations 26. False 27. True 28. True 29. True 30. True 31. False 32. True 33. False 34. True 35. True 36. False 37. False 38. False 39. True 40. True 41. True 42. True 43. True 44. True 45. False 46. False 47. False 48. True 49. False 50. True Powered by Cognero

Page 29


Name:

Class:

Date:

Chapter 10: Partnerships Distributions Transfer of Interests and Terminations 51. b 52. c 53. a 54. d 55. d 56. a 57. c 58. b 59. a 60. c 61. c 62. b 63. d 64. d 65. a 66. b 67. d 68. d 69. d 70. a 71. b 72. a 73. c 74. d 75. b 76. b Powered by Cognero

Page 30


Name:

Class:

Date:

Chapter 10: Partnerships Distributions Transfer of Interests and Terminations 77. a 78. c 79. d 80. a 81. d 82. c 83. a 84. b 85. c 86. d 87. b 88. d 89. c 90. d 91. d 92. b 93. d 94. c 95. d 96. b 97. d 98. d 99. b 100. d 101. c Powered by Cognero

Page 31


Name:

Class:

Date:

Chapter 10: Partnerships Distributions Transfer of Interests and Terminations 102. k 103. f 104. e 105. b 106. g 107. j 108. i 109. a 110. h 111. d 112. c 113. m 114. f 115. m 116. g 117. e 118. j 119. c 120. i 121. a 122. k 123. l 124. b 125. d 126. h 127. b Powered by Cognero

Page 32


Name:

Class:

Date:

Chapter 10: Partnerships Distributions Transfer of Interests and Terminations 128. b 129. b 130. a 131. b 132. c 133. b 134. a 135. c 136. a 137. c 138. a 139. b 140. a. Randy recognizes no gain or loss. Because the cash received does not exceed his basis in the partnership interest, no gain is recognized. Because this is a nonliquidating distribution, no loss is recognized. b.

Randy’s basis in the inventory is $60,000. The cash distribution reduces his basis to $80,000. When the inventory is distributed, it takes the lesser of a carryover basis of $60,000 or Randy’s remaining basis.

c.

The inventory distribution reduces Randy’s basis from $80,000 (after the cash distribution) to $20,000.

141. a. $20,000 gain. Connie recognizes a gain of $20,000 on the distribution, which is the amount by which the $140,000 cash distribution exceeds her $120,000 outside basis. b.

$0 basis in land. The cash distribution reduces Connie’s basis to $0. When the land is distributed, it takes the lesser of a carryover basis of $30,000 or Connie’s remaining basis of $0.

c.

$0 basis in partnership interest. Because Connie has no remaining basis for her partnership interest after the cash distribution, her outside basis is $0.

142. a. No gain or loss. Karli will not recognize any gain or loss on the distribution because the Powered by Cognero

Page 33


Name:

Class:

Date:

Chapter 10: Partnerships Distributions Transfer of Interests and Terminations $120,000 cash distributed does not exceed her $200,000 outside basis. b.

$50,000 basis in inventory. The inventory has an adjusted basis of $50,000 to Karli. The partnership will distribute the $120,000 cash first, thereby reducing her outside basis for her partnership interest to $80,000 ($200,000 – $120,000). The inventory will be distributed next, taking a carryover basis of $50,000 and reducing the adjusted basis for her partnership interest to $30,000.

c.

$30,000 basis in land. The land parcel is distributed last and takes a $30,000 substituted basis because the basis in the land cannot exceed Karli’s remaining basis in her partnership interest.

d.

$0 basis in partnership interest. Karli’s basis for her partnership interest after the distribution is $0. Her entire $200,000 outside basis has been allocated to the distributed assets in the following amounts. Cash Inventory Land

$120,000 50,000 30,000

143. a. Neil recognizes a gain of $20,000. He received a cash distribution of $100,000 (including his relief of liabilities), against his basis of $80,000. His basis in the partnership interest is reduced to $0 after the cash distribution. b.

Neil has a carryover/substituted basis in the unrealized receivables of $0. He also has a $0 substituted basis in the land.

c.

Neil’s basis in the LLC interest is $0 following the distribution.

144. Beginning basis in Melissa’s interest Less: Cash distribution Basis before property distributions Less: Inventory distribution Less: Land distribution Basis after property distributions

$ 90,000 (50,000) $ 40,000 (22,000) (18,000) $ –0–

Melissa’s basis in the inventory equals the partnership’s basis in the inventory of $22,000. Whether the property is appreciated or depreciated does not matter. Her basis in the land is a substituted basis equal to the basis in the partnership interest following the inventory and cash distributions, or $18,000. 145. a. Stuart recognizes no gain or loss. He received a cash distribution of $180,000 (including his relief of liabilities), which did not exceed his basis of $200,000. b.

Stuart’s basis is reduced to $20,000 by the cash distribution. The inventory is distributed next.

Powered by Cognero

Page 34


Name:

Class:

Date:

Chapter 10: Partnerships Distributions Transfer of Interests and Terminations He has a substituted basis in the inventory of $20,000, and the partnership interest basis is reduced to $0.Therefore, the land takes a $0 basis.

146. a. Bill recognizes a capital gain of $30,000 on the liquidation of his partnership interest. This equals the excess of the cash distribution over Bill’s basis in his partnership interest before the distribution ($120,000 cash – $90,000 basis). b.

Bill’s bases in the inventory and capital asset are both $0. His partnership interest basis is reduced to $0 by the cash distribution so he has no remaining basis to allocate to the other properties.

147. a. Josh takes a carryover basis of $120,000 in the distributed inventory and recognizes a $110,000 capital loss on the distribution. The loss is the difference between his $300,000 adjusted basis and the $190,000 ($70,000 + $120,000) inside basis of the cash and inventory distributed to him. b.

Josh would not recognize any loss on the distribution and the land would take a $110,000 adjusted basis. A loss cannot be recognized if any assets other than cash, unrealized receivables, and inventory are received in the distribution. Because land is generally a capital asset, it will absorb all of the remaining basis of $110,000. The capital loss would be deferred until the land is sold. If, however, the land is converted to business use, its later sale would result in a § 1231 loss, which is generally an ordinary loss.

148. $20,000 ordinary income and $10,000 capital gain. The payment for Dana’s share of the unrealized receivables is a § 736(a) payment, taxed as ordinary income. Dana’s share of the receivables is $20,000 ($60,000 × 1/3). The § 736(b) payment is $100,000, consisting of the amount paid for Dana’s 1/3 share of the partnership cash and capital assets. The § 736(b) payment is treated first as a return of her $90,000 outside basis. The rest of the § 736(b) payment is taxed to Dana as a $10,000 capital gain. 149. Christina will recognize $47,000 of ordinary income and $9,000 of capital gain. The partnership can deduct, as a § 736(a) payment, $47,000, and it will record a step up of $9,000. Under § 736(a), the $40,000 ($120,000 × 1/3) paid for Christina’s share of unrealized receivables will be taxed to her as a $40,000 guaranteed payment. In addition, the $7,000 ($140,000 – $133,000) paid for her share of unstated goodwill will be treated as a guaranteed payment. The remaining $93,000 ($140,000 – $47,000) cash distribution will be treated as a § 736(b) payment. The first $84,000 of this payment will be a return of Christina’s outside basis. The remaining $9,000 will be taxed to her as capital gain. As mentioned, the partnership can deduct, as a guaranteed payment, the amount treated as a § 736(a) payment, which is $47,000. In addition, because a § 754 election is in effect, the partnership records a step up in the basis of its remaining assets for the $9,000 Christina recognizes as a gain. 150. Jeremy recognizes $80,000 of ordinary income and a $60,000 capital gain. The ordinary income equals Jeremy’s $60,000 share of accounts receivable ($200,000 × 30%) and his $20,000 payment for goodwill. [Total asset value is $800,000 ($200,000 + $200,000 + $400,000) and Jeremy’s share is $240,000 ($800,000 × 30%). Because Jeremy receives $260,000 (including relief of $40,000 of partnership debt), the $20,000 excess ($260,000 – $240,000) is for unstated goodwill).] The remaining payment to Jeremy of $180,000 ($260,000 – $80,000 ) is treated as a § 736(b) payment, which is offset by Jeremy’s basis of $120,000. Powered by Cognero

Page 35


Name:

Class:

Date:

Chapter 10: Partnerships Distributions Transfer of Interests and Terminations Jeremy is treated as a general partner and a goodwill payment is not provided for in the partnership agreement. Therefore, JS’s $80,000 payment for Jeremy’s share of receivables and unstated goodwill is a § 736(a) payment that is deductible by the partnership. In addition (as a planning note), the partnership could make a § 754 election to step up the basis of its other assets for the $60,000 of gain Jeremy recognizes. 151. a. Serena recognizes a gain of $60,000, including ordinary income of $48,000 and a capital gain of $12,000. She received $140,000 plus relief of $40,000 of liabilities for a total of $180,000. Her basis in the LLP interest was $120,000, resulting in a net gain of $60,000. Of this gain, she reports ordinary income in the amount of her share of the LLP’s unrealized receivables, or $48,000 ($120,000 × 40%). The remaining gain is a capital gain. b.

Jaclyn’s basis equals the $180,000 paid, including the $40,000 share of the LLP’s liabilities.

c.

If the LLP has a § 754 election in effect, it records an adjustment of $76,000 {$180,000 paid – $104,000 share of inside asset basis [($80,000 + $0 + $180,000) × 40%]}. This step up is allocated to Jaclyn.

152. Rachel’s realized gain is $50,000 ($300,000 received less $250,000 outside basis), consisting of a capital gain of $20,000 and $30,000 of ordinary income. Because the receivables are a § 751 “hot asset,” Rachel is treated as having sold her 1/3 share and, therefore, will recognize $30,000 ordinary income. The rest of the sale is taxed under the general rule of § 741 and generates a capital gain of $20,000. 153. Hannah recognizes $20,000 of ordinary income and a $15,000 capital gain for a total gain of $35,000. Hannah’s basis is increased from $70,000 at the beginning of the year to $85,000 at the sale date as a result of HIJK’s allocation of $15,000 of income to her during the sale year. Her total gain is $35,000 ($120,000 sales price – $85,000 basis). Hannah recognizes $20,000 of ordinary income under § 751(a) and a $15,000 capital gain under § 741. Hannah recognizes ordinary income to the extent of her share of the partnership’s inventory and unrealized receivables. Her 25% share of the receivables is $20,000 (25% × $80,000). The difference between the amount Alyssa paid ($120,000) and Hannah’s share of the value of partnership assets ($100,000) is probably the value of the partnership’s intangible assets or goodwill. Whereas the goodwill might result in ordinary income under § 736, that is not the case for the sale of a partnership interest. 154. a. No. The partnership recognizes no gain or loss as a result of the

distribution per § 731(b). No. Because this is a proportionate current distribution, Gil would

only recognize gain if the cash received exceeded his outside basis. b. He would only recognize loss under limited conditions and only if

his interest in the partnership terminated. Therefore, he has no recognized gain or loss. Gil’s bases are $20,000 (inventory), $40,000 (land), and $10,000

(partnership interest). Gil takes a basis in the inventory equal to the partnership’s basis in the inventory, or $20,000. His $100,000 c. basis in the partnership is reduced first by the $30,000 cash distribution, then by the $20,000 inventory distribution. He takes a basis in the land equal to the partnership’s basis in the Powered by Cognero

Page 36


Name:

Class:

Date:

Chapter 10: Partnerships Distributions Transfer of Interests and Terminations land, or $40,000. The land distribution reduces his basis in the partnership interest to $10,000.

155. No gain or loss. The cash is distributed first, followed by the inventory. The a. land is distributed last and absorbs Parker’s remaining basis in partnership

interest of $100,000 ($180,000 basis – $20,000 cash – $60,000 inventory). Parker’s basis in the inventory is a carryover basis of $60,000. His basis

b.

in the land is $100,000 (see calculation in part a.). c. The partnership has no gain or loss. No change to part a., but the answer for part b. would differ. If this had

been a current distribution, Parker’s basis in the land (part b.) would have been limited to the partnership’s $80,000 basis. Parker would have a remaining basis in the partnership interest of $20,000.

d.

156.

No gain or loss is recognized on the distribution. Dudley’s basis in his partnership interest remains $150,000. Nicki takes a carryforward basis in the property of $100,000, and her basis in the a. partnership interest is reduced to $60,000. Here are the details: Dudley contributed the property more than seven years before the property was distributed to the other partner. Therefore, the precontribution gain distribution rules do not apply, and this is a normal proportionate current distribution. Dudley’s basis in the partnership interestis not affected by the distribution to the other partner. Nicki takes a $100,000 basis in the property, which is b. the lesser of the $100,000 carryover basis in the property or her $160,000 basis in the partnership interest before the distribution. Her basis in the partnership interest following the distribution is $60,000. If the property had been contributed only two years ago, Dudley would

recognize the precontribution gain when the property was distributed to c. Nicki. Nicki’s basis in the property and Dudley’s basis in the partnership interest would be adjusted to reflect the recognized gain. NOTE: The actual amounts (not required) are that Dudley would recognize a $15,000 gain, and his basis in the partnership interest would be increased to $165,000. Powered by Cognero

Page 37


Name:

Class:

Date:

Chapter 10: Partnerships Distributions Transfer of Interests and Terminations Nicki’s basis in the distributed property would be increased to its value at Dudley's contribution date, or $115,000.

157. a. The limited liability company (LLC) generally provides for Federal taxation under Subchapter K (partnership taxation) and limited liability for all owners. Therefore, an LLC provides for the same protection from personal liability as a C corporation while providing for (pass-through) taxation of LLC operations at the owner level. The C corporation double taxation does not apply to an LLC. Partnership provisions such as optional adjustments to basis; special allocations of income, gain, loss, deduction, and credit; and inclusion of all LLC liabilities in outside basis are additional advantages of an LLC over a C corporation. b.

An existing C corporation should carefully weigh the consequences before converting to an LLC. The IRS has ruled that a C corporation must liquidate and re-form as an LLC. When a C corporation liquidates, any appreciation in corporate assets triggers a corporate level gain. This results in immediate double taxation of the gain (at the corporate level and then as a shareholder gain or loss on liquidation of the entity). A partnership, on the other hand, does not liquidate when it converts to an LLC, and any appreciation in partnership assets remains untaxed.

158. Sale. On a sale of the partnership interest for $200,000, Julie recognizes a gain of $100,000, which is classified as $40,000 of ordinary income and a $60,000 capital gain. The partnership reflects a step-up allocated to Irene and Rachel of $80,000 ($200,000 purchase price less $120,000 share of JIR’s inside basis). The goodwill portion of the payment does not affect Julie’s treatment of the sales price. [Instructor note: Although not specifically discussed in the chapter, the existing § 754 election applies with respect to the purchasing partners, allowing (requiring) the step up in basis.] Redemption. On a distribution from JIR to Julie in redemption of her interest, Julie recognizes a gain of $100,000, which is classified as $50,000 of ordinary income and a $50,000 capital gain. Julie is deemed to receive a § 736(a) payment of $50,000, including $40,000 for her share of unrealized receivables and $10,000 for unstated goodwill. (This result applies because Julie is treated as being a general partner in a service-providing partnership, and the goodwill is not provided for in the partnership agreement.). The remaining distribution of $150,000 is a § 736(b) payment in exchange for her interest in partnership property. On this portion of the payment, Julie recognizes a capital gain of $50,000 ($150,000 § 736(b) payment less $100,000 basis). JIR deducts the $50,000 § 736(a) payment. In addition, JIR reflects a step-up equal to the $50,000 capital gain recognized by Julie. Recommendation. In both situations, Irene and Rachel bear the burden of $100,000 of payment, and Julie receives $200,000 of cash before taxes. Therefore, the preferred treatment hinges on the tax ramifications of the payments. Powered by Cognero

Page 38


Name:

Class:

Date:

Chapter 10: Partnerships Distributions Transfer of Interests and Terminations Under § 736, the partnership claims a deduction for $50,000 and a step-up of $50,000 ($100,000 total). For a sale of the interest, the step up is $80,000 (and the partnership claims no current deduction) because the partnership’s inside basis is higher than Julie’s outside basis. In addition, a step up can be recovered only through depreciation, amortization, or sale by the partnership of an underlying asset. From Irene and Rachel’s perspective, the redemption is much preferred. From Julie’s perspective, she reports $100,000 of gain in either situation. In the redemption, she has $10,000 of additional ordinary income and $10,000 less of capital gain. The sale is slightly more favorable for her if capital gains tax rates are preferable to ordinary income rates. 159. Upon the sale of a partnership interest, the income share is allocated between the buyer and the seller. The allocation can be based on an interim closing of the books or the proration method. Because the tax year closes with respect to George, he would report income from two GLO K-1’s that will eventually be prepared as of March 1 and July 1. Income bunching occurs, in that George must report income from both Schedules K-1 in his current-year personal tax return. However, George might not receive the second (July 1) Schedule K-1 in time to file his tax return because GLO is not required to issue a Schedule K-1 until its normal due date (June 15 of the following year, or as extended). Thus, George might have to file an extension for his personal tax return for the year of the sale, or he might have to estimate the income and file an amended tax return later. George recognizes a gain or loss on sale of his partnership interest. George's basis on the sale date must reflect his shares of income from the periods ended March 31 and July 1, and, again, he might not know the July amount in time to calculate the gain or loss on the sale. George's share of partnership liabilities would be included in both the sales price and his basis in the LLP interest. Because the partnership uses the cash method, it probably has substantial unrealized receivables, and this results in ordinary income to George. His remaining gain or loss is from the sale of his partnership interest and is capital in nature. Because of the value inherent in the unrealized receivables, Monica would probably pay a premium over her share of GLO’s inside basis in its assets. Monica should request that the partnership make a § 754 election (if an election is not already in effect). The resulting step up in the inside basis of the assets would be allocated to her, and this reduces her allocable share of income when the receivables are collected. 160. Gains. For both a proportionate liquidating and a current (nonliquidating) distribution, gain generally arises only upon distribution of cash in excess of the partner’s basis in the partnership. Cash includes relief of the partner’s share of partnership liabilities as well as a portion of certain marketable security distributions. Losses. In a current (nonliquidating) distribution, a loss deduction is not permitted. Because the partner remains a partner in the partnership, any eventual loss cannot be determined and, therefore, is not realized. In a liquidating distribution, a realized loss might be deductible if the partner receives only cash and ordinary-income producing property (unrealized receivables and inventory). If other property (capital or § 1231 assets) is distributed, that property absorbs any remaining basis and the loss is not currently deductible. Basis. In a current (nonliquidating) distribution, the basis of distributed property is the lesser of the partnership’s basis in the property or the partner’s remaining basis in the partnership interest. This same result holds for a liquidating distribution except that if third-tier (capital or § 1231) assets are distributed, those assets absorb any remaining basis in the partnership interest. 161. a. Distribution of the precontribution gain property to another partner. Any previously unrealized precontribution gain would be recognized by Greg if BIG distributed the property to another partner before 2031. Distribution of other property to contributing partner. If BIG distributes a second property to Greg before 2031, he might be taxed on any previously unrecognized built-in gain. Powered by Cognero

Page 39


Name:

Class:

Date:

Chapter 10: Partnerships Distributions Transfer of Interests and Terminations b.

Purpose and effect of rules. These rules are designed to ensure that any precontribution gain is taxed to the person who owned the property at the time the gain arose.

162. Cindy will prefer to treat the cash received for her share of the partnership goodwill as a § 736(b) payment. Section 736(b) payments are taxed as a return of basis and result in capital gain to the recipient partner. The goodwill can be treated as a § 736(b) payment if the partnership agreement provides for the goodwill. Unfortunately, the partnership probably will prefer to treat the payment for goodwill as a § 736(a) payment because this will allow the partnership to deduct the amounts paid for Cindy’s share of partnership goodwill. As currently structured, the payment for Cindy’s share of the goodwill will be treated as a § 736(a) payment because the goodwill is not provided for in the partnership agreement. Cindy and the partnership can control how the goodwill will be treated. In situations such as this, the partnership typically has the more powerful position in the negotiations. This typically allows the partnership to dictate the treatment of goodwill as a § 736(a) payment.

Powered by Cognero

Page 40


Name:

Class:

Date:

Chapter 11: S Corporations True / False 1. Liabilities affect the owner’s basis differently in an S corporation than they do in a partnership. a. True b. False 2. An S corporation cannot incur a tax liability at the corporation level. a. True b. False 3. Distributions of appreciated property by an S corporation are not taxable to the entity. a. True b. False 4. A newly formed S corporation does not receive any tax benefit from a C corporation's NOL incurred in its first election tax year. a. True b. False 5. S corporation status allows shareholders to realize tax benefits from corporate losses immediately (assuming sufficient stock basis). a. True b. False 6. NOL carryforwards from C years can be used in an S corporation year against ordinary income. a. True b. False 7. Tax-exempt income at the S corporation level flows through as taxable income to the shareholder. a. True b. False 8. A one-person LLC can be a shareholder of an S corporation. a. True b. False 9. A corporation may alternate between S corporation and C corporation status each year depending on which results in more tax savings. a. True b. False 10. An estate may be a shareholder of an S corporation. a. True b. False 11. A former spouse is treated as being in the same family as the individual to whom they were married for purposes of Powered by Cognero

Page 1


Name:

Class:

Date:

Chapter 11: S Corporations determining the number of S corporation shareholders. a. True b. False 12. A corporation can revoke its S election as of a future date. a. True b. False 13. Most limited liability partnerships can own stock in an S corporation. a. True b. False 14. The AAA begins with a zero balance on the first day of an S corporation’s first tax year. a. True b. False 15. An S corporation's AAA can have a negative balance. a. True b. False 16. A distribution from the other adjustment account (OAA) is not taxable to an S shareholder. a. True b. False 17. An S election made before becoming a corporation is valid only beginning with the first 12-month tax year. a. True b. False 18. At least 51% of the shareholders must consent to an S election. a. True b. False 19. Persons who were S shareholders during any part of the year before the election date but were not shareholders when the election was made also must consent to an S election. a. True b. False 20. The termination of an S election occurs on the day after a corporation ceases to be a qualifying S corporation. a. True b. False 21. The passive investment income of an S corporation includes gains from the sale of securities. a. True b. False 22. Tax-exempt income at the corporate level flows through as exempt income to S shareholders. Powered by Cognero

Page 2


Name:

Class:

Date:

Chapter 11: S Corporations a. True b. False 23. The Section 179 expense deduction is a Schedule K (separately stated) item on the Form 1120S. a. True b. False 24. Depreciation recapture income is a Schedule K (separately stated) item on the Form 1120S. a. True b. False 25. A per-day, per-share allocation of flow-through S corporation items must be used. a. True b. False 26. Any distribution of cash or other property by a corporation that does not exceed the balance of AAA with respect to S stock during a post-termination transition period of approximately one year is applied against and reduces the basis of the S stock. a. True b. False 27. A capital loss allocated to a shareholder always reduces the Other Adjustments Account. a. True b. False 28. An item such as tax-exempt interest that appears in the Other Adjustments Account affects stock basis, but not AAA. a. True b. False 29. An S corporation does not recognize a loss when distributing assets that are worth less than their basis. a. True b. False 30. When loss assets are distributed by an S corporation, a shareholder’s basis is equal to the asset’s fair market value. a. True b. False 31. An S shareholder’s basis is increased by stock purchases and capital contributions. a. True b. False 32. An S shareholder’s basis is decreased by distributions treated as being paid from AAA. a. True b. False 33. An S corporation shareholder’s stock basis includes their direct investments plus a ratable share of any corporate Powered by Cognero

Page 3


Name:

Class:

Date:

Chapter 11: S Corporations liabilities. a. True b. False 34. An S shareholder’s stock basis can be reduced below zero. a. True b. False 35. An S shareholder’s stock basis is reduced by flow-through losses before accounting for distributions. a. True b. False 36. An S shareholder’s stock basis does not include a ratable share of S corporation liabilities. a. True b. False 37. An S corporation can claim a deduction for its operating loss amounts. a. True b. False 38. The corporate-level tax on recognized built-in gains applies when an S corporation disposes of an asset in a taxable disposition within five years after the date on which the S election took effect. a. True b. False 39. The LIFO recapture tax is a variation of the passive investment income penalty tax. a. True b. False 40. The passive investment income of an S corporation includes net capital gains from the sale of stocks and securities. a. True b. False 41. Compensation paid to employees by an S corporation generates a corporate-level FICA tax liability. a. True b. False 42. The exclusion of gain on disposition of small business stock is not available on disposition of S corporation stock. a. True b. False 43. There are no advantages for an S corporation to issue § 1244 stock. a. True b. False Multiple Choice Powered by Cognero

Page 4


Name:

Class:

Date:

Chapter 11: S Corporations 44. An S corporation is subject to the following tax(es). a. Corporate income tax. b. Built-in gains tax. c. Alternative minimum tax. d. None of these choices are correct. 45. Which statement is incorrect? a. S corporations are treated as corporations under state law. b. S corporations are treated as partnerships for Federal income tax purposes. c. Distributions of appreciated property are taxable to the S corporation. d. All of these choices are correct. 46. An S corporation must possess which of the following characteristics? a. Not more than 100 shareholders. b. Corporation organized in the United States. c. Only one class of stock. d. All of these choices are correct. 47. Which entity is eligible to make the S election? a. Non-U.S. corporation. b. One-person limited liability company. c. Insurance company. d. U.S. bank. 48. Identify a disadvantage of being an S corporation. a. Estates can be shareholders. b. Losses flow through immediately to the shareholders. c. The exclusion of gain on disposition of small business stock is not available for S stock. d. Tax-exempt income at the S level does not lose its special tax treatment for shareholders. 49. Which of the following, if any, can be eligible shareholders of an S corporation? a. A Roth IRA. b. A partnership. c. A non-U.S. corporation. d. None of these choices are correct. 50. Which statement is incorrect with respect to filing an S election? a. Form 2553 must be filed. b. All shareholders must consent. c. The election may be filed in the previous year. d. An extension of time is available for filing the application. 51. Which of the following, if any, are eligible shareholders of an S corporation? Powered by Cognero

Page 5


Name:

Class:

Date:

Chapter 11: S Corporations a. A partnership. b. A nonresident alien. c. A three-person LLC. d. The estate of a deceased shareholder. 52. Several individuals acquired assets on behalf of Skip Corporation on May 28, purchased assets on June 3, and began business on June 11. They subscribe to shares of stock, file articles of incorporation for Skip, and become shareholders on June 21. The S election must be filed no later than two and one-half months after: a. May 28. b. June 3. c. June 11. d. June 21. 53. The theoretical maximum number of actual shareholders in an S corporation is: a. 75. b. 100. c. 200. d. Unlimited 54. Which statement is incorrect with respect to the number-of-shareholders test in filing an S election? a. Husband Jaime and wife Maria count as one shareholder. b. Grandmother Adela and granddaughter Maria count as one shareholder. c. Husband Jaime and ex-wife Isabel count as one shareholder. d. All of these statements are correct. 55. A new S corporation shareholder can revoke the S election unilaterally, if they own how much of the existing S corporation’s stock? a. More than 50%. b. 50% or more. c. The election can be revoked only if all of the shareholders consent. d. The election cannot be revoked during the first year of the new shareholder’s ownership. 56. Which statement is incorrect with respect to an S shareholder’s consent? a. Both spouses must consent if one owns the stock as community property. b. An S election requires a consent from all of the S corporation’s shareholders. c. A consent must be in writing. d. All of these statements are correct. 57. Which of the following is not a separately stated item on Schedule K of Form 1120S? a. Tax-exempt interest income. b. Section 1231 gain. c. Section 179 depreciation deduction. d. Depreciation recapture income. 58. Which item is not included in an S corporation’s nonseparately computed income? Powered by Cognero

Page 6


Name:

Class:

Date:

Chapter 11: S Corporations a. Net sales. b. Cost of goods sold. c. Dividends received. d. Depreciation recapture. 59. Which of the following is not a separately stated item on Schedule K of Form 1120S? a. Intangible drilling costs. b. Foreign loss. c. Utilities expense. d. Recovery of a tax benefit. 60. What method is used to allocate S corporation income or losses (unless an election to the contrary is made)? a. Any method agreed to by all of the shareholders. b. Per-day allocation. c. FIFO method. d. LIFO method. 61. If an S corporation’s beginning balance in OAA is zero and the following transactions occur, what is the entity's ending OAA balance? Depreciation recapture income Payroll tax penalty Tax-exempt interest income Nontaxable life insurance proceeds Life insurance premiums paid (nondeductible)

$21,000 4,200 5,300 5,100 2,800

a. $1,300 b. $7,600 c. $23,300 d. $27,500 62. Which transaction affects the Other Adjustments Account on an S corporation’s Schedule M-2? a. Payroll penalty. b. Unreasonable compensation. c. Life insurance proceeds (nontaxable to the recipient S corporation). d. Taxable interest. 63. Which of the following items, if any, decreases an S corporation’s AAA? a. Section 1231 loss. b. Expenses related to tax-exempt income. c. Depletion in excess of basis. d. Distribution from earnings and profits. 64. During the year, Marcus, the sole shareholder of a calendar year S corporation, received a distribution of $16,000. At the end of last year, his stock basis was $4,000. The corporation earned $11,000 ordinary income during the year. It holds a zero balance in accumulated E & P. Which statement is correct? Powered by Cognero

Page 7


Name:

Class:

Date:

Chapter 11: S Corporations a. Marcus recognizes a $1,000 LTCG. b. Marcus’s stock basis is $2,000. c. Marcus’s ordinary income is $15,000. d. Marcus’s tax-free return of capital is $11,000. 65. Amit, Inc., an S corporation, holds an AAA balance of $614,000 at the beginning of the tax year. During the year, the following items occur. Operating income Interest income Dividend income Municipal bond interest income Long-term capital loss from sale of investment land Section 179 depreciation deduction Charitable contributions Cash distributions

$501,000 6,500 13,020 6,000 7,400 6,000 19,000 57,000

Amit’s ending AAA balance is: a. $1,045,120. b. $1,185,150. c. $1,191,150. d. $1,242,150. 66. Kinney, Inc., an electing S corporation, holds $5,000 of AEP and $9,000 in AAA at the beginning of the calendar tax year. Kinney has two shareholders, Eric and Maria, each of whom owns 500 shares of Kinney’s stock. Kinney’s taxable income is $6,000 for the year. Kinney distributes $6,000 to each shareholder on February 1, and it distributes another $3,000 to each shareholder on September 1. How is Eric taxed on the distribution? a. $500 dividend income. b. $1,000 dividend income. c. $1,500 dividend income. d. $3,000 dividend income. 67. Fred is the sole shareholder of an S corporation in Fort Deposit, Alabama. At a time when his stock basis is $20,000, the corporation distributes appreciated property worth $100,000 (basis of $20,000). Fred’s taxable gain is: a. $0. b. $10,000. c. $80,000. d. $100,000. 68. You are given the following facts about a solely owned S corporation. What is the shareholder’s ending stock basis? Increase in AAA Increase in OAA Payroll tax penalty Beginning stock basis Stock purchases Powered by Cognero

$31,000 6,300 2,140 39,800 22,000 Page 8


Name:

Class:

Date:

Chapter 11: S Corporations Tax-exempt life insurance proceeds Life insurance premiums paid (nondeductible)

4,800 2,700

a. $61,800 b. $68,100 c. $99,100 d. $100,100 69. Which of the following reduces a shareholder’s S corporation stock basis? a. Depletion deductions in excess of the basis of property. b. Illegal kickbacks paid. c. Nontaxable income. d. A 20% QBI deduction. 70. You are given the following facts about a 50% owner of an S corporation. Compute her ending stock basis. Increase in AAA Increase in OAA Payroll tax penalty Owner's beginning stock basis Tax-exempt interest income Insurance premiums paid (nondeductible) Owner's additional stock purchases Owner's 20% QBI deduction

$32,000 6,300 2,140 39,800 4,800 2,700 22,000 21,000

a. $80,950. b. $85,750. c. $100,100. d. $106,225. 71. Samantha owned 1,000 shares in Evita, Inc., an S corporation, that uses the calendar year. On October 11, Samantha sells all of her Evita stock. Her stock basis at the beginning of the tax year was $60,000. Evita's ordinary income for the year was $22,000 through the date of sale, and Samantha receives a distribution of $35,000 on May 3rd. Her stock basis at the time of the sale is: a. $117,000. b. $82,000. c. $60,000. d. $47,000. 72. You are given the following facts about a 40% owner of an S corporation. Calculate her ending stock basis. Owner's beginning stock basis Increase in AAA Increase in OAA Payroll tax penalty Tax-exempt interest income Life insurance premiums paid (nondeductible) Powered by Cognero

$36,800 32,000 6,300 2,140 4,800 2,700 Page 9


Name:

Class:

Date:

Chapter 11: S Corporations Owner's purchases of additional stock

22,000

a. $71,600 b. $74,120 c. $76,220 d. $78,920 73. Oxen Corporation incurs the following transactions. Net income from operations Interest income from savings account Long-term capital gain from sale of securities Short-term capital loss from sale of securities

$100,000 3,000 10,000 4,000

Oxen maintains a valid S election and does not distribute any assets (cash or property) to its sole shareholder, Megan. As a result, Megan must recognize (ignore 20% QBI deduction): a. Ordinary income of $103,000. b. Ordinary income of $103,000 and long-term capital gain of $6,000. c. Ordinary income of $103,000, long-term capital gain of $10,000, and $4,000 short-term capital loss. d. Ordinary income of $109,000. 74. On January 1, Bobby and Alice equally own all of the stock of an electing S corporation called Prairie Dirt Delight. The entity incurs a $60,000 loss for the year. On the 200th day of the year (not a leap year), Bobby sells his one-half of the stock to his son, Saul. How much of the $60,000 loss, if any, is allocated to Bobby? a. $-0b. $13,562 c. $16,438 d. $32,877 75. A cash basis calendar year C corporation reports $100,000 of accounts receivable on the date of its conversion to S status on February 7. By the end of the year, $60,000 of these receivables have been collected. Calculate any built-in gains tax, assuming that there is sufficient taxable income. a. $0 b. $12,600 c. $21,000 d. $35,000 76. Mock Corporation converts to S corporation status in 2025. Mock used the LIFO inventory method in 2024 and had a LIFO inventory of $435,000 (FIFO value of $550,000) on the date of the S election. How much tax must be added to Mock’s 2024 corporate tax liability? a. $-0b. $6,038 c. $24,150 d. $115,000 77. A calendar year C corporation reports a $41,000 NOL in 2024, but it elects S status for 2025 and generates an NOL of Powered by Cognero

Page 10


Name:

Class:

Date:

Chapter 11: S Corporations $30,000 in that year. At all times during 2025, the stock of the corporation was owned by the same 10 shareholders, each of whom owned 10% of the stock. Kris, one of the 10 shareholders, holds an S stock basis of $2,300 at the beginning of 2025. How much of the 2025 loss, if any, can she deduct? a. $-0b. $2,300 c. $3,000 d. $7,100 78. Pepper, Inc., an S corporation, holds a $1 million balance in accumulated E&P. The corporation reports sales revenues of $400,000, taxable interest of $380,000, operating expenses of $250,000, and deductions attributable to the interest income of $140,000. What is Pepper’s passive income penalty tax payable, if any? a. $380,000. b. $116,842. c. $24,537. d. $-0-. 79. If the beginning balance in OAA is zero and the following transactions occur, what is the ending OAA balance? Depreciation recapture income Payroll tax penalty Tax-exempt interest Nontaxable life insurance proceeds Insurance premiums paid (nondeductible) Charitable contributions a. $1,300. b. $6,700. c. $23,300. d. $27,500.

$21,000 4,200 5,700 3,900 2,900 17,000

80. At the beginning of the year, the AAA of Rose, Inc. shows a balance of $682,000. During the year, the following items occur. Compute the end-of-year AAA balance. Operating income Interest income Dividend income Municipal bond interest income Short-term capital loss from sale of building Section 179 expense Charitable contributions Cash distributions Depreciation recapture

$452,000 6,550 14,050 12,000 7,400 6,500 19,000 57,000 3,500

a. $1,064,700. b. $1,185,150. c. $1,191,150. d. $1,242,150. 81. This year, Jiang, the sole shareholder of a calendar year S corporation, received a distribution of $17,000. On Powered by Cognero

Page 11


Name:

Class:

Date:

Chapter 11: S Corporations December 31 of the prior year, his stock basis was $3,000. The corporation earned $12,000 ordinary income during the year. The corporation has a zero balance in accumulated E&P. Which statement is correct? Ignore the 20% QBI deduction. a. Jiang recognizes a $2,000 LTCG. b. Jiang’s stock basis will be $2,000. c. Jiang’s ordinary income is $15,000. d. Jiang’s return of capital is $11,000. 82. Which of the following items, if any, has no effect on the stock basis of an S corporation shareholder? a. Operating income. b. Short-term capital gain. c. Advertising expenses. d. The 20% QBI deduction. 83. Lemon Corporation incurs the following transactions. Net income from operations Interest income from saving account Long-term capital gain from sale of securities Short-term capital loss from sale of securities

$110,000 5,000 9,000 4,000

Lemon maintains a valid S election and does not distribute any dividends to its shareholder, Nina. As a result, which of the following must Nina recognize? Ignore the 20% QBI deduction. a. Ordinary income of $115,000 and long-term capital gain of $5,000. b. Ordinary income of $115,000, long-term capital gain of $9,000, and $4,000 short-term capital loss. c. Ordinary income of $120,000. d. Capital gain of $120,000. 84. Lent Corporation, a calendar year taxpayer, converts to S corporation status beginning in 2024. Lent had been using the LIFO inventory method and held a LIFO inventory value of $510,000 (FIFO value of $650,000). How much tax, if any, is added for these items for the final C corporation year? a. $-0-. b. $7,350. c. $29,400. d. $140,000. Completion 85. As with partnerships, the income, deductions, and tax credits of an S corporation ____________________ to the shareholders annually. 86. Some ____________________and

taxation rules apply to an S corporation.

87. The choice of a flow-through entity for a closely held corporation often is between a(n) ____________________ (a Federal tax entity) and a(n) ____________________ (a state tax entity). 88. If any entity electing S status is currently a C corporation, NOL carryovers from prior years Powered by Cognero

Page 12


Name:

Class:

Date:

Chapter 11: S Corporations generally____________________ (can/cannot) be used in an S corporation year. 89. An S corporation is limited to a theoretical maximum of ____________________ shareholders. 90. Shareholders owning a(n) ____________________ of shares (voting and nonvoting) may ____________________ revoke an S election. 91. If an S corporation has C corporation E & P and passive investment income in excess of ____________________ % of its gross receipts for ____________________ consecutive taxable years, the S election is terminated at the beginning of the ____________________ year. 92. Depreciation recapture income is a ____________________ (separately, nonseparately) computed amount. 93. An S corporation’s separately stated items generally are identical to those reported by _________________________. 94. Separately stated items are listed on Schedule _________________________ of the Form 1120S. 95. In the case of a complete termination of an S corporation interest, a ____________________ tax year may occur. 96. Since loss property receives a ____________________ in basis without any loss recognition, S corporation distributions of loss property generally should be ____________________. 97. Nonseparately computed loss ____________________ (increases, reduces) an S shareholder’s stock basis. 98. An S corporation recognizes a ____________________ on any distribution of appreciated property. 99. Stock basis first is increased by income items, then ____________________ by distributions, and finally decreased by ____________________. 100. An S corporation’s LIFO recapture amount equals the excess of the inventory’s value under ____________________ over the ____________________ value. 101. The exclusion of _________________ on the disposition of small business stock (is/is not) _________________ available for S stock. Subjective Short Answer 102. Estella, Inc., a calendar year S corporation, incurred the following items during the tax year. Municipal bond interest Sales Depreciation recapture income Long-term capital gain Cost of goods sold Administrative expenses Depreciation expense Charitable contributions 20% QBI deduction Powered by Cognero

$ 7,000 120,000 14,000 20,000 (42,000) (15,000) (13,000) (10,000) 4,370 Page 13


Name:

Class:

Date:

Chapter 11: S Corporations Calculate Estella’s nonseparately computed income. 103. Towne, Inc., a calendar year S corporation, holds AAA of $627,050 at the beginning of the tax year. During the year, the following items occur. Sales income Loss from real estate operations Officers’ life insurance proceeds Premiums paid for officers’ life insurance Dividend income Interest income Charitable contributions § 179 depreciation expense Administrative expenses Cash distributions to shareholders

$216,000 (4,000) 100,000 (3,600) 17,000 3,000 (22,000) (2,500) (35,000) (73,220)

Calculate Towne’s ending AAA balance. 104. Ridden, Inc., a calendar year S corporation, incurred the following items. Sales Depreciation recapture income Short-term capital gain Cost of goods sold Municipal bond interest income Administrative expenses Depreciation expense Charitable contributions

$130,000 12,000 30,000 (42,000) 7,000 (15,000) (17,000) (14,000)

Calculate Ridden’s nonseparately computed income. 105. Gene Grams is a 45% owner of a calendar year S corporation during the tax year. His beginning stock basis is $230,000, and the S corporation reports the following items. Ordinary income Short-term capital gain § 1231 loss Tax-exempt interest income

$72,000 16,000 6,000 5,000

Calculate Grams’s stock basis at year-end. 106. You are a 60% owner of an S corporation. Calculate your ending stock basis based upon these facts. Beginning stock basis Purchases of additional stock Insurance premiums paid (nondeductible) Tax-exempt interest income Payroll tax penalty Increase in AAA Powered by Cognero

$52,600 15,000 3,600 5,230 3,770 22,400 Page 14


Name:

Class:

Date:

Chapter 11: S Corporations Increase in OAA

5,800

107. On December 31, Erica Sumners owns one share of an S corporation’s 10 outstanding shares of stock. The basis of Erica’s share is $300. The next year, the S corporation incurs a loss of $3,650. Determine the amount of the loss allocated to Erica, and calculate her stock basis at the end of the second year. 108. Alomar, a cash basis S corporation in Orlando, FL, holds the following assets and liabilities on January 1, 2024, the date the S election is made.

Cash Accounts receivable Equipment Land Accounts payable

Adjusted Basis $ 200,000 –0– 110,000 1,800,000 –0–

Fair Market Value $ 200,000 105,000 100,000 2,500,000 110,000

During the year, Alomar collects the accounts receivable and pays the accounts payable. The land is sold for $3 million, and the taxable income for the year is $590,000. Calculate any built-in gains tax. 109. Pepper, Inc., an S corporation in Norfolk, VA, has revenues of $400,000, taxable interest of $380,000, operating expenses of $250,000, and deductions attributable to the interest income of $140,000. Calculate any passive investment income penalty tax payable by this corporation. Essay 110. Discuss two ways that an S election may be terminated. 111. Advise your client how income, expenses, gain, and losses are allocated to shareholders of an S corporation. 112. Explain the OAA concept.

Powered by Cognero

Page 15


Name:

Class:

Date:

Chapter 11: S Corporations Answer Key 1. True 2. False 3. False 4. False 5. True 6. False 7. False 8. True 9. False 10. True 11. True 12. True 13. False 14. True 15. True 16. True 17. False 18. False 19. True 20. False 21. True 22. True 23. True 24. False 25. True Powered by Cognero

Page 16


Name:

Class:

Date:

Chapter 11: S Corporations 26. False 27. False 28. True 29. True 30. True 31. True 32. True 33. False 34. False 35. False 36. True 37. False 38. True 39. False 40. True 41. True 42. True 43. False 44. b 45. b 46. d 47. b 48. c 49. d 50. d Powered by Cognero

Page 17


Name:

Class:

Date:

Chapter 11: S Corporations 51. d 52. a 53. b 54. d 55. a 56. d 57. d 58. c 59. c 60. b 61. b 62. c 63. a 64. a 65. a 66. c 67. c 68. c 69. b 70. a 71. d 72. b 73. c 74. c 75. b 76. b Powered by Cognero

Page 18


Name:

Class:

Date:

Chapter 11: S Corporations 77. b 78. c 79. b 80. a 81. a 82. d 83. b 84. b 85. flow through 86. partnership C corporation 87. S corporation, limited liability company 88. cannot 89. 100 90. majority, voluntarily 91. 25, 3, fourth 92. nonseparately 93. partnerships 94. K 95. short 96. step-down, avoided 97. reduces 98. gain 99. decreased, losses 100. FIFO, LIFO 101. gain, is not Powered by Cognero

Page 19


Name:

Class:

Date:

Chapter 11: S Corporations 102. Sales Depreciation recapture income Less: Cost of goods sold Administrative expenses Depreciation expense Nonseparately computed income 103. Beginning AAA Add: Sales income Dividend income Interest income Less: Real estate loss Charitable contributions § 179 expense Administrative expenses

$120,000 14,000 $134,000 $42,000 15,000 13,000

(70,000) $ 64,000

$627,050 $216,000 17,000 3,000

$236,000

$ 4,000 22,000 2,500 35,000

(63,500)

Less distributions Ending AAA 104. Sales Depreciation recapture income Cost of goods sold Administrative expenses Depreciation expense

$130,000 12,000 $142,000 $42,000 15,000 17,000

Nonseparately computed income 105. Beginning stock basis Ordinary income (45% × $72,000) STCG (45% × $16,000) Tax-exempt interest (45% × $5,000) § 1231 loss (45% × $6,000) Ending basis

172,500 $799,550 (73,220) $726,330

(74,000) $ 68,000

$230,000 32,400 7,200 2,250 (2,700) $269,150

106. $84,520 [$52,600 + $15,000 +0.60($22,400) + 0.60($5,800)]. 107. The loss assigned to each day of the S corporation’s tax year is $10 ($3,650/365 days). For each day, $1 is allocated to each outstanding share ($10/10 shares). Erica is allocated $365 of loss for her one share owned during the tax year. However, she is limited to a loss deduction of $300, i.e., until her stock basis reaches zero. Her stock basis is zero at the end of the year. She has a $65 loss carryforward available for deduction in subsequent years. Powered by Cognero

Page 20


Name:

Class:

Date:

Chapter 11: S Corporations 108. $123,900. The net unrealized built-in gain on the conversion date is $685,000 ($105,000 – $10,000 + $700,000 – $110,000), the maximum amount subject to the § 1374 tax. The recognized built-in gains and losses are: Accounts receivable collected Accounts payable Gain on the land (limited to builtin gain) Total Limited to net unrealized built-in gain

$105,000 (110,000) 700,000 $695.000 $685,000

Taxable income is $590,000 and the built-in gains tax is assessed on the smaller amount ($590,000 × 21% = $123,900). However, there is a carryforward of the built-in gains amount that escapes taxation due to the taxable income limitation to be taxed in subsequent years when taxable income is sufficient. Therefore, $95,000 is carried into the next tax year and treated as a recognized built-in gain then. 109. The S corporation pays a § 1375 penalty tax of $24,537, calculated as follows. ENPI =

Net Passive Income PII

× (PII – 25% GR)

ENPI =

$380,000 – $140,000 $380,000

× [$380,000 – (25% × $780,000)]

$240,000 $380,000

×

0.6315789 × $116,842

×

($380,000 – $195,000) $185,000 0.21 = $24,537

110. Broadly, there are two ways to terminate the S election—voluntary termination and involuntary termination. If shareholders owning a majority of shares consent, an election can be voluntarily terminated. If the revocation is filed by the fifteenth day of the third month of the tax year, the revocation is effective for the entire tax year (unless a prospective effective date is specified). An S election may be involuntarily terminated if a disqualifying event occurs (i.e., issues a second class of stock, too many shareholders, etc.). The loss of the election applies as of the date on which the disqualifying event occurs. 111. In general, S corporation items are divided into (1) nonseparately computed income or losses and (2) separately stated income, losses, deductions, and credits that could uniquely affect the tax liability of any shareholder. Each shareholder is allocated a pro rata portion of nonseparately computed income or loss. Separately stated items (e.g., LTCG, charitable contributions) also are allocated on a pro rata share method. 112. S corporations report changes in the AAA on Schedule M-2 of Form 1120S. Schedule M-2 contains a column labeled Other Adjustments Account (OAA). This account includes items that affect S stock basis but not the AAA, such as taxexempt income and any related nondeductible expenses. For example, life insurance proceeds received and insurance premiums paid are traced through the OAA. Powered by Cognero

Page 21


Name:

Class:

Date:

Chapter 11: S Corporations Distributions are made from the OAA after AEP and the AAA equal zero. Distributions from the OAA generally are taxfree to the receiving shareholder.

Powered by Cognero

Page 22


Name:

Class:

Date:

Chapter 12: Comparative Forms of Doing Business True / False 1. For Federal income tax purposes, a business entity with two or more owners may be conducted as a partnership, C corporation, S corporation, or limited liability company. a. True b. False 2. Daniel, who is single, estimates that the profits of his business for the current tax year will be $200,000; he has no other sources of gross income. As the 21% corporate rate is less than Daniel's marginal rate of 32%, he would save taxes operating the business as a C corporation. a. True b. False 3. A limited liability company (LLC) is a hybrid business form that combines the corporate characteristic of limited liability for the owners with the tax characteristics of a partnership. a. True b. False 4. A sole proprietorship files Schedule C of Form 1040, a partnership files Form 1065, a C corporation files Form 1120, and an S corporation files Form 1120S. a. True b. False 5. Each of the following can pass profits and losses through to the owners: general partnership, limited partnership, S corporation, and limited liability company. a. True b. False 6. A corporation has a greater potential for raising capital than does a partnership. a. True b. False 7. The at-risk provisions and the passive activity loss provisions decrease the tax attractiveness of investments in real estate for non-corporate taxpayers. a. True b. False 8. An S corporation offers less flexibility in terms of the types of owners and capital structure than a C corporation. a. True b. False 9. All of the shareholders of an S corporation have limited liability with respect to their ownership interests in the corporation whereas only limited partners in a limited partnership have such limited liability. a. True b. False 10. S corporation status always avoids double taxation. Powered by Cognero

Page 1


Name:

Class:

Date:

Chapter 12: Comparative Forms of Doing Business a. True b. False 11. C corporations are not subject to AMT but individuals are. a. True b. False 12. An S corporation is not subject to the AMT, but its shareholders are because the S corporation’s AMT adjustments and preferences are passed through to them. a. True b. False 13. The tax treatment of S corporation shareholders with respect to fringe benefits is not the same as the tax treatment for C corporation shareholders, but is the same as the fringe benefit treatment for partners. a. True b. False 14. An S corporation election for Federal income tax purposes also is effective for all states’ income tax purposes. a. True b. False 15. Some fringe benefits provide a double income tax benefit—a deduction for the employer and an exclusion for the employee. a. True b. False 16. Of the corporate types of entities, all are subject to double taxation on current earnings. a. True b. False 17. A C corporation making deductible payments to shareholders can reduce or eliminate double taxation. a. True b. False 18. If lease rental payments to a noncorporate shareholder–lessor are classified as unreasonable, the taxable income of both the corporation and the shareholder will increase. a. True b. False 19. If the IRS reclassifies debt as equity, the repayment of the debt by the corporation to the shareholder will generally be treated as a dividend. a. True b. False 20. Wally contributes land (adjusted basis of $30,000; fair market value of $100,000) to an S corporation in a transaction that qualifies under § 351. The corporation subsequently sells the land for $120,000, recognizing a gain of $90,000 Powered by Cognero

Page 2


Name:

Class:

Date:

Chapter 12: Comparative Forms of Doing Business ($120,000 – $30,000). If Wally owns 30% of the stock, $76,000 [$70,000 + 30%($20,000)] of the $90,000 recognized gain is allocated to Wally. a. True b. False 21. The profits of a business owned by Taylor (60%) and Maggie (40%) for the current tax year are $100,000. If the business is a C corporation or an S corporation, there is no effect on Taylor’s basis in her stock. If the business is a partnership or an LLC, Taylor’s basis in her partnership interest or basis in her stock is increased by $60,000. a. True b. False 22. A shareholder’s basis in the stock of an S corporation is increased by corporate profits and decreased by losses. a. True b. False 23. One benefit of an S corporation when compared with a C corporation is that an S corporation is subject to Federal income tax only in limited circumstances. a. True b. False 24. If an S corporation distributes appreciated property as a dividend, it must recognize gain related to the appreciation. a. True b. False 25. The passive activity loss rules generally limit the recognition of passive losses realized by C corporations. a. True b. False 26. John wants to buy a business whose assets have appreciated in value. If the business is operated as a C corporation, it does not matter to John whether he purchases the assets or the stock. a. True b. False 27. The Net Investment Income Tax (NIIT) is owed by both high-income individuals and corporations. a. True b. False 28. An individual who owes the NIIT cannot also be subject to the additional Medicare tax. a. True b. False Multiple Choice 29. A limited liability company (LLC): a. Is subject to double taxation. b. Is usually taxed as a partnership. Powered by Cognero

Page 3


Name:

Class:

Date:

Chapter 12: Comparative Forms of Doing Business c. Is usually taxed as a corporation. d. Is exempt from Federal income taxation. 30. A limited liability company with 100 unrelated owners: a. May be taxed as a C corporation. b. May be taxed as an S corporation. c. May be taxed as a partnership. d. Only a. and c. are correct. 31. Which of the following statements is incorrect? a. The number of owners of an LLC is not limited. b. If the LLC has three or more corporate characteristics, it will be taxed as a C corporation. c. An LLC can elect to be taxed as a C corporation or as a partnership. d. Only a. and c. 32. Which of the following statements is correct? a. An S corporation has a greater opportunity to raise capital than does a C corporation. b. A general partnership has a greater opportunity to raise capital than does a limited partnership. c. A partnership has a greater opportunity to raise capital than does a sole proprietorship. d. Only a. and b. are correct. 33. Nontax factors that affect the choice of business entity include: a. Ease of capital formation. b. Limited liability. c. Single versus double taxation. d. Only a. and b. 34. Which of the following statements is correct? a. The AMT applies to both the individual taxpayer and all C corporations. b. An individual's AMT rates are 26% and 28%. c. The S corporation AMT rate is 20%. d. Only a. and b. are correct. 35. Techniques that can be used to minimize the current period tax liability include: a. Utilizing special allocations. b. Having favorable treatment of certain fringe benefits. c. Minimizing double taxation. d. All of these can be used for effective tax planning. 36. Audrey holds an ownership interest in a business entity. She is in the 24% tax bracket. The entity incurs $30,000 of lodging expense for Audrey, which she believes qualifies for exclusion treatment as a qualified fringe benefit. Which of the following statements are correct? a. If the entity is a partnership and Audrey has a 60% interest, the effect of the $30,000 expenditure by the partnership on Audrey’s tax liability is an increase of $4,320. b. If the entity is a sole proprietorship, the effect of the $30,000 expenditure by the sole proprietorship on Powered by Cognero

Page 4


Name:

Class:

Date:

Chapter 12: Comparative Forms of Doing Business Audrey’s tax liability is $7,200. c. If the entity is a C corporation, the effect of the $30,000 expenditure by the corporation on Audrey’s tax liability is $0. d. Only b. and c. are correct. 37. Robin Company has $100,000 of income before payment of $100,000 of reasonable salaries to its owners/employees (who are in the 32% bracket). Which form of business results in the least amount of combined income tax being paid by the company and its owners? a. Partnership. b. C corporation. c. S corporation. d. Choices a., b., and c. all result in the same amount of tax. 38. Tyrone purchases a building for $750,000 that is going to be used by his wholly owned corporation. Which of the following statements is correct? a. If Tyrone contributes the building to the corporation, there will be no gross income in the current year and a carryover basis of $750,000. b. If Tyrone leases the building to the corporation, lease-rental payments of $30,000 per year to Tyrone will result in a $30,000 deduction for the corporation. c. If Tyrone leases the building to the corporation, lease-rental payments of $30,000 per year to Tyrone will result in $30,000 of gross income for Tyrone. d. All of these statements are correct. 39. LaTonya contributes $150,000 to Swan, Inc., for 80% of the stock. In addition, she loans Swan $600,000. The maturity date on the loan is five years, and the interest rate is 3%—the same as the Federal rate. Which of the following statements are correct? a. If the loan is reclassified as equity, Swan qualifies for a deduction of $600,000 when the loan is repaid and LaTonya receives dividend income of $600,000 (assuming that Swan’s earnings and profits are at least $600,000). b. If the loan is not reclassified as equity, Swan can deduct interest expense annually of $18,000 and LaTonya includes in gross income annual interest income of $18,000. c. If the loan is reclassified as equity, Swan claims no interest deduction and LaTonya recognizes no income. d. Only a. and b. 40. Malcolm and Cho, both shareholders of Crow, Inc, each loan Crow $50,000 at the market rate of interest. Which of the following statements are false? a. Crow may deduct the interest expense, and the interest income is taxable to Malcolm and Cho. b. If the IRS were successful in reclassifying the notes as equity, the interest payments would not be deductible by Crow, and Malcolm and Cho would still recognize income. c. If the IRS were successful in reclassifying the notes as equity, repayment of the note principal to Malcolm and Cho would not qualify for return of capital treatment and would most likely result in dividend income treatment for Malcolm and Cho. d. All of these are true. 41. Factors that should be considered in making the S corporation election for the current tax year include the following. a. Are greater than 50% of the shareholders willing to consent to the election? Powered by Cognero

Page 5


Name:

Class:

Date:

Chapter 12: Comparative Forms of Doing Business b. Can the requirements for qualification be satisfied by the fifteenth day of the third month of the tax year and also for the period of the tax year that precedes this date? c. Will the corporation have total capital not in excess of $1 million? d. Only b. and c. 42. Tuan and Ella are going to establish a business. They expect the business to be very successful in the long-run, but they project losses of approximately $100,000 for each of the first five years. Due to potential environmental concerns, limited liability is a requisite for the owners. Which form of business entity should they select? a. General partnership. b. Limited partnership. c. C corporation. d. S corporation. 43. Alice and Jose are going to form a business entity. Alice will contribute cash of $200,000 for a 40% ownership interest, and Jose will contribute land worth $300,000 (basis of $180,000) for a 60% ownership interest. Which of the following statements is correct? a. If the entity is a C corporation, Alice has $0 recognized gain and a basis for her stock of $200,000, and Jose has a recognized gain of $120,000 and a basis for his stock of $300,000. b. If the entity is an S corporation, Alice has $0 recognized gain and a basis for her stock of $200,000, and Jose has $0 recognized gain and a basis for his stock of $180,000. c. If the entity is a general partnership, Alice has $0 recognized gain and a basis for her partnership interest of $152,000 ($380,000 × 40%), and Jose has $0 recognized gain and a basis for his partnership interest of $228,000 ($380,000 × 60%). d. Only a. and c. are correct. 44. Both Thu and Juan own one-half of the stock of Wren, Inc., a C corporation. Each shareholder holds a stock basis of $175,000. Wren holds accumulated E & P of $300,000. Wren’s taxable income for the current year is $100,000, and it distributes $75,000 to each shareholder. Thu’s stock basis at the end of the year is: a. $0. b. $100,000. c. $150,000. d. $175,000. 45. Both Tracy and Cabel own one-half of the stock of Finch, Inc., an S corporation with no accumulated E & P. Tracy’s basis in the Finch stock is $225,000. Finch’s taxable income for the current year is $100,000, and it distributes $180,000 to each shareholder. Tracy’s stock basis at the end of the year is: a. $-0-. b. $45,000. c. $95,000. d. $100,000. 46. Chen contributes property with an adjusted basis of $80,000 and a fair market value of $100,000 to a newly formed business entity. If the entity is a C corporation and the transaction qualifies under § 351, the corporation’s basis for the property and the shareholder’s basis for the stock are: Asset Basis a. Powered by Cognero

Stock Basis Page 6


Name:

Class:

Date:

Chapter 12: Comparative Forms of Doing Business

b. c.

$ 80,000

$100,000

$100,000

$ 80,000

$ 80,000

$ 80,000

$100,000

$100,000

d.

47. Ruchi contributes property with an adjusted basis of $80,000 and a fair market value of $100,000 to a newly formed business entity. If the entity is a partnership and the transaction qualifies under § 721, the partnership’s basis for the property and the partner’s basis for the partnership interest are: Asset Basis a. $ 80,000 b.

c.

d.

Stock Basis $100,000

$100,000

$ 80,000

$ 80,000

$ 80,000

$100,000

$100,000

48. Martin contributes property with an adjusted basis of $100,000 and a fair market value of $140,000 to a newly formed business entity. If the entity is an S corporation and the transaction qualifies under § 351, the S corporation’s basis for the property and the shareholder’s basis for the stock are: Asset Basis a. $100,000 b.

c.

d.

Stock Basis $140,000

$140,000

$100,000

$100,000

$100,000

$140,000

$140,000

49. Alice contributes equipment (fair market value of $82,000; adjusted basis of $20,000), subject to a $14,000 liability, to Powered by Cognero

Page 7


Name:

Class:

Date:

Chapter 12: Comparative Forms of Doing Business form Orange Partnership, a general partnership. Mary contributes $68,000 cash. Alice and Mary share equally in partnership profits and losses. What is Alice’s and Mary’s basis for their partnership interests? a. $6,000 to Alice, $68,000 to Mary. b. $6,000 to Alice, $75,000 to Mary. c. $13,000 to Alice, $75,000 to Mary. d. $20,000 to Alice, $68,000 to Mary. 50. Kim contributes land (basis of $190,000; fair market value of $250,000) to a business entity in exchange for 100% of the stock. During the first year of operation, the entity earns a profit of $75,000. At the end of the first year, the entity has outstanding liabilities of $30,000 ($20,000 recourse and $10,000 nonrecourse). Select from the following: a. If the entity is a C corporation, Kim’s basis for her stock at the end of the first year is $265,000 ($190,000 + $75,000) and her at-risk basis is $265,000. b. If the entity is a partnership, Kim’s basis for her partnership interest (outside basis) at the end of the first year is $355,000 ($250,000 + $75,000 + $30,000) and her at-risk basis is $345,000 ($250,000 + $75,000 + $20,000). c. If the entity is an S corporation, Kim’s basis for her stock at the end of the first year is $345,000 ($250,000 + $75,000 + $20,000) and her at-risk basis is $345,000. d. Choice a., b., and c. are incorrect. 51. Albert’s sole proprietorship owns the following assets.

Accounts receivable Inventory Machinery and equipment* Buildings** Land

Adjusted Basis Fair Market Value $ –0– $ 60,000 20,000 30,000 50,000 90,000 120,000 170,000 80,000 140,000 $270,000 $490,000

* Potential § 1245 recapture of $45,000. ** Straight-line depreciation used. Albert sells his sole proprietorship for $500,000. Calculate Albert’s recognized gain or loss and classify it as capital or ordinary. a. $230,000 ordinary income. b. $230,000 capital gain. c. $115,000 ordinary income and $115,000 capital gain. d. $110,000 ordinary income and $120,000 capital gain. 52. Kristine owns all of the stock of a C corporation that owns the following assets.

Accounts receivable Inventory Machinery and equipment* Buildings** Land

Adjusted Basis $ –0– 20,000 50,000 120,000 80,000 $270,000

Fair Market Value $ 60,000 30,000 90,000 170,000 140,000 $490,000

* Potential § 1245 recapture of $45,000. Powered by Cognero

Page 8


Name:

Class:

Date:

Chapter 12: Comparative Forms of Doing Business ** Straight-line depreciation was used. Her adjusted basis for her stock is $270,000. Calculate Kristine’s recognized gain or loss and classify it as capital or ordinary if she sells her stock for $500,000. a. $230,000 ordinary income. b. $230,000 capital gain. c. $115,000 ordinary income and $115,000 capital gain. d. $110,000 ordinary income and $120,000 capital gain. 53. Which of the following statements is correct? a. The sale of an unincorporated sole proprietorship always is treated as the sale of the individual business assets. b. The sale of a partnership is treated as the sale of the individual assets only if the sales transaction is structured as the sale of the individual assets. c. The sale of a corporation is treated either as the sale of the corporate stock or as the sale of the individual assets. d. Choice a., b., and c. are correct. 54. Which of the following statements is incorrect? a. The purchase of an unincorporated sole proprietorship always is treated as the purchase of its assets. b. A taxpayer purchasing a corporation in which the assets are appreciated would prefer to purchase the stock of the corporation. c. The purchase of a corporation always is treated as the purchase of the corporate stock. d. Only a. and b. are correct. Matching Match the following attributes with the different forms. A particular attribute may apply to more than one entity form. a. Ability of all owners to have limited liability. b. Ability to pass tax attributes through to the owners. c. Right of all owners to participate in the management of the business. d. Number of owners is limited. e. Ability to have multiple owners. 55. S corporation 56. C corporation 57. Limited partnership 58. General partnership 59. Sole proprietorship Match the following statements. a. Transaction in this form enables double taxation to be avoided. b. Gain or loss is calculated separately for each asset and is subject to single taxation. c. This is subject to double taxation. Powered by Cognero

Page 9


Name:

Class:

Date:

Chapter 12: Comparative Forms of Doing Business d. The sale is treated as the sale of a capital asset under § 741 but subject to ordinary income potential under § 751. e. This is not subject to double taxation on the sale of corporate stock. 60. Sale of the individual assets of an unincorporated sole proprietorship by the owner. 61. Sale of the corporate assets by the C corporation. 62. Sale of corporate stock by the C corporation shareholders. 63. Sale of corporate stock by the S corporation shareholders. 64. Sale of an ownership interest by a partner. Match the following statements. a. For the corporate taxpayer, taxed using the regular tax rates. b. Must be capitalized, but can be amortized over 180 months. c. For the corporate taxpayer, the rate is 21%. d. For the corporate taxpayer, cannot be deducted at all in the current tax year. e. For the corporate taxpayer, limited to 10% of taxable income before certain deductions. 65. Organization costs 66. Regular tax rate 67. Net capital gain 68. Net capital loss 69. Charitable contributions Match the following. a. Contribution of appreciated property to the business entity by an owner is never subject to taxation. b. Realized gains on the contribution of appreciated property to the entity are not recognized by the contributor when an 80% control requirement is satisfied. c. Realized losses on the contribution of loss property to the entity never are recognized by the contributor. d. Realized losses on the contribution of loss property to the entity are recognized by the contributor unless an 80% control requirement is satisfied. e. Basis of ownership interest to the owner is dependent on whether gain or loss is recognized to the owner on the contribution of assets to the business entity. 70. S corporation 71. C corporation 72. Limited partnership 73. General partnership Match the following statements. Powered by Cognero

Page 10


Name:

Class:

Date:

Chapter 12: Comparative Forms of Doing Business a. Usually subject to single taxation even if the entity is incorporated. b. Not making distributions to shareholders. c. Rate for a corporate taxpayer is 21%. d. Subject to double taxation. e. Eligible for special allocations. 74. Technique for minimizing double taxation 75. Regular tax rate 76. S corporations 77. C corporations 78. Partnerships Subjective Short Answer 79. Jane is going to invest $90,000 in a business entity that she will manage. Her projected share of the loss for the first year is $36,000. Jane’ marginal tax rate is 32%. Determine the cash flow benefit of the loss to her if the business form is: a.

A general partnership.

b.

An S corporation.

c.

An LLC.

d.

A C corporation.

80. Candace, who is in the 32% tax bracket, is establishing a business that could have potential environmental liability problems. She is trying to decide between the C corporation form and the S corporation form. She projects that the business will generate earnings of about $75,000 each year. Advise Candace on the tax consequences of each entity form. 81. Kirby, the sole shareholder of Falcon, Inc., leases a building to the corporation. The taxable income of the corporation for the tax year before deducting the lease payments is projected to be $500,000. a.

What are the tax consequences to Kirby and to Falcon if Kirby leases a building to the corporation for $400,000?

b.

Is there a potential pitfall? How would it change the tax consequences to Kirby and to Falcon?

82. Blue, Inc., records taxable income before salary payments of $700,000 to its president who has a marginal rate of 32%. a.

Calculate the tax liability to Blue if the president’s salary is $400,000 and if it is $100,000.

b.

What tax benefit is there of paying the higher salary to the president?

Powered by Cognero

Page 11


Name:

Class:

Date:

Chapter 12: Comparative Forms of Doing Business c.

What negative tax result may occur associated with the payment of the higher salary?

83. Both Albert and Elva own 50% of the stock of Eagle, Inc. (a C corporation). To cover temporary working capital needs, each shareholder loans Eagle $200,000 using the annual Federal interest rate of 3% and a maturity date of one year. a.

What are the tax consequences to Albert, Elva, and Eagle if the loans are classified as debt?

b.

What are the tax consequences to Albert, Elva, and Eagle if the loans are classified as equity?

84. Swallow, Inc., will distribute cash of $700,000 to shareholder Marjean, who is in the 35% tax bracket. a.

Determine the tax liability to Marjean if the form of the distribution is a dividend.

b.

Determine the tax liability to Marjean if the form of the distribution is a stock redemption. Assume Marjean’s adjusted basis for the stock redeemed is $200,000 and that she has owned the stock for five years.

85. Sang-hoon is establishing a business in year 1 that could have potential environmental liability problems. Therefore, he is trying to decide between the C corporation form and the S corporation form. He projects that the business will generate losses of approximately $100,000 each year for the first three years and then will generate profits of at least $200,000 each year thereafter. All profits will be reinvested in the growth of the business. Sang-hoon projects he will be in the 35% bracket for all tax years. Advise Sang-hoon on which tax form he should select. 86. Melanie and Sonny form Bird Enterprises. Sonny contributes cash of $100,000 and land worth $50,000 (adjusted basis of $30,000). Melanie contributes land and a building worth $280,000 (adjusted basis of $200,000) and performs services worth $20,000 associated with the formation of the entity. Melanie receives a two-thirds ownership interest and Sonny receives a one-third ownership interest. Determine the tax consequences of the contributions to Melanie, Sonny, and Bird if the business is: a.

An S corporation.

b.

A C corporation

c.

A partnership.

87. Sam and Trang are going to establish a business. Sam will contribute cash of $100,000 for a 50% interest, and Trang will contribute land and a building worth $135,000 (adjusted basis of $65,000) for a 50% interest. The land and building are encumbered by a $35,000 mortgage, which the entity assumes. Determine the tax consequences of the contribution to Sam, Trang, and the entity if the business is: a.

An S corporation.

b.

A partnership.

c.

A C corporation.

88. Meg has an adjusted basis of $150,000 for her 40% ownership interest. During the year, the entity earns a profit of $100,000 and liabilities increased by $70,000. Determine Meg’s adjusted basis for her ownership interest. a.

If the entity is a partnership.

Powered by Cognero

Page 12


Name:

Class:

Date:

Chapter 12: Comparative Forms of Doing Business b.

If the entity is a C corporation.

c.

If the entity is an S corporation.

89. Anne contributes property to the TCA Partnership, which was formed eight years ago by Clark and Tara. Anne’s basis for the property is $90,000 and the fair market value is $220,000. Anne receives a 25% interest for her contribution. Because the TCA Partnership is unsuccessful in having the property rezoned from agricultural to commercial, it sells the property 14 months later for $225,000. a.

Determine the tax consequences to Anne and to the partnership on the contribution of the property to the partnership.

b.

Determine the tax consequences to Anne and the other partners on the sale of the property.

c.

Would the tax consequences in b. differ if the entity were an S corporation?

90. Wren, Inc. is owned by Tucker (30%) and Maribel (70%). Tucker’s marginal tax rate is 22% and Maribel’s marginal tax rate is 35%. Wren’s taxable income for the current tax year is $300,000. Determine the amount of the distribution that Wren would make to enable Tucker and Maribel to pay their tax liabilities associated with Wren’s $300,000 taxable income.

a. If Wren is an S corporation.

b. If Wren is a C corporation. 91. Lee owns all the stock of Vireo, Inc., a C corporation for which he has an adjusted basis of $150,000. The assets of Vireo are recorded as follows.

Cash Accounts receivable Inventory Building Land

Adjusted Basis $35,000 20,000 22,000 28,000 40,000

FMV $35,000 20,000 25,000 30,000 90,000

Lee sells his stock to Katrina for $300,000. Determine the tax consequences to: a.

Lee.

b.

Katrina.

c.

Vireo

92. Luis owns all the stock of Silver, Inc., a C corporation for which his adjusted basis is $225,000. Luis founded Silver 12 years ago. The assets and liabilities of Silver are recorded as follows. Assets Cash Accounts receivable Powered by Cognero

Basis $ 15,000 –0–

FMV $ 15,000 25,000 Page 13


Name:

Class:

Date:

Chapter 12: Comparative Forms of Doing Business Inventory Machinery and equipment* Land

30,000 70,000 60,000 $175,000

35,000 90,000 150,000 $315,000

Liabilities Accounts payable Notes payable

Basis $ 5,000 10,000 $15,000

FMV $ 5,000 10,000 $15,000

*Accumulated depreciation of $55,000 has been deducted. Luis has agreed to sell the business to Marilyn and they have agreed on a purchase price of $350,000 less any outstanding liabilities. They are both in the 35% tax bracket. a.

Advise Luis on whether the form of the sales transaction should be a stock sale or an asset sale.

b.

Advise Marilyn on whether the form of the purchase transaction should be a stock purchase or an asset purchase.

Essay 93. Lisa is considering investing $60,000 in a limited partnership that is raising additional capital. According to the prospectus, for the past 10-year period, the average earnings have been 12% and for the past 5-year period, the average earnings have been 9%. Lisa is in the 24% tax bracket. a.

List some factors Lisa should consider in making a decision on the potential investment.

b.

Assuming the partnership finances its activities with equity rather than debt, what is the maximum cash flow benefit Lisa can receive if the partnership generates losses?

94. Mai is going to contribute the following assets to a business entity in exchange for an ownership interest. Adjusted Basis Cash $100,000 Land and building 60,000 What are the tax consequences of the contribution to Mai if the business entity is a(n): a.

Sole proprietorship?

b.

General partnership?

c.

Limited partnership?

d.

C corporation?

e.

S corporation?

Powered by Cognero

FMV $100,000 95,000

Page 14


Name:

Class:

Date:

Chapter 12: Comparative Forms of Doing Business 95. Which of the following business entity forms are subject to single taxation on the profits, and which are subject to double taxation? a.

Sole proprietorship.

b.

General partnership.

c.

Limited partnership.

d.

C corporation.

e.

S corporation.

f.

LLC.

96. List some techniques that can be used to avoid and/or reduce double taxation for a C corporation. 97. List some techniques for reducing and/or avoiding double taxation by transferring funds to the shareholders that are deductible to the corporation. 98. To which of the following entities does the AMT apply? ∙

Sole proprietorship.

General partnership.

Limited partnership.

LLC.

S corporation.

C corporation.

99. How can double taxation be avoided or reduced by a shareholder owning necessary business assets outside a C corporation? 100. Ori has been operating his business as a C corporation for the past five years. The corporation pays him a reasonable salary. The profits of the corporation, after paying Federal income tax, are distributed to him each year as a dividend. He is considering electing S status for his corporation, to avoid double taxation. What factors should he consider assuming after-tax earnings will continue to be distributed to him? 101. Mabel and Alan, who both are in the 32% Federal income tax bracket, recently acquired a fast-food

franchise. They both will work in the business and receive a salary of $175,000 each. They anticipate that the annual profits of the business, after deducting salaries, will be approximately $450,000. The entity will distribute enough cash each year to Mabel and Alan to cover their Federal income taxes associated with the franchise. a. What amount will the entity distribute if the franchise operates as a C corporation? b. What amount will the entity distribute if the franchise operates as an S corporation? c. What will be the amount of the combined entity/owner tax liability in parts (a) and (b)? Powered by Cognero

Page 15


Name:

Class:

Date:

Chapter 12: Comparative Forms of Doing Business 102. Jiang contributes land with an adjusted basis of $85,000 and a fair market value of $100,000 to a business entity in which she is an 80% owner on the first day of the tax year. Discuss the tax consequences to Jiang if the entity sells the land six months later for $120,000 if: a.

The entity is a partnership.

b.

The entity is a C corporation.

c.

The entity is an S corporation.

103. Dudley holds a 20% ownership interest in a business for which his basis is $100,000. During the year, the entity earns profits of $90,000 and makes cash distributions of $50,000 to the owners. How do these transactions affect Dudley’s basis if: a.

The entity is a C corporation?

b.

The entity is a general partnership?

c.

The entity is an S corporation?

104. A business entity has appreciated land (basis of $50,000 and fair market value of $75,000) which it is going to distribute to Chun, one of its owners. The entity has earned substantial profits during its 15 years of operations and has reinvested most of them in the business. What are the tax consequences of the distribution to the business entity and to Chun if the business entity is a(n): a.

C corporation?

b.

S corporation?

c.

Partnership?

105. With respect to special allocations, is the S corporation treated more like a partnership or a C corporation? Elaborate. 106. Ling owns a sole proprietorship for which the assets have appreciated in value. If he is going to sell the business to Abner, should Ling structure the sale as (1) a sale of the individual assets or (2) a sale of the sole proprietorship? 107. Corey is going to purchase the assets of Kathlyn’s sole proprietorship. The assets of Kathlyn’s sole proprietorship have appreciated in value. From Corey’s perspective, does it matter whether the purchase is structured as the purchase of (1) the individual assets or (2) the sole proprietorship? 108. Ralph wants to purchase either the stock or the assets of Red, Inc., a C corporation. Under what circumstances would Ralph prefer to purchase: a.

The stock from the shareholders?

b.

The assets from the corporation?

109. Wang wants to sell his wholly owned C corporation, Cream, Inc. The fair market value of his stock exceeds the corporation’s adjusted basis for the assets. Should Wang sell his stock or have Cream sell its assets and make a liquidating distribution to him? Powered by Cognero

Page 16


Name:

Class:

Date:

Chapter 12: Comparative Forms of Doing Business 110. Alejandro, a single individual, owns a sole proprietorship business. For 2024, his net income from this activity was $170,000 after considering all business-related deductions. Assume Alejandro has other income equal to his standard deduction. a. Compute any qualified business income deduction for Alejandro. b. Given a FICA earnings limit of $168,600 for 2024, calculate and identify all of the employment taxes Alejandro is subject to for 2024. c. Which taxes in b., if any, may Alejandro deduct on his Form 1040? d. How would your answers to a. and b. change if Alejandro were instead an employee-owner of a C corporation with a salary of $170,000? 111. For which type of business entity is an owner least likely to have a cash flow problem as to Federal income taxes? Explain.

Powered by Cognero

Page 17


Name:

Class:

Date:

Chapter 12: Comparative Forms of Doing Business Answer Key 1. True 2. False 3. True 4. True 5. True 6. True 7. True 8. True 9. True 10. False 11. False 12. True 13. True 14. False 15. True 16. False 17. True 18. False 19. True 20. False 21. False 22. True 23. True 24. True 25. False Powered by Cognero

Page 18


Name:

Class:

Date:

Chapter 12: Comparative Forms of Doing Business 26. False 27. False 28. False 29. b 30. d 31. b 32. c 33. d 34. b 35. d 36. d 37. d 38. d 39. b 40. d 41. b 42. d 43. b 44. d 45. c 46. c 47. c 48. c 49. c 50. d Powered by Cognero

Page 19


Name:

Class:

Date:

Chapter 12: Comparative Forms of Doing Business 51. d 52. b 53. d 54. b 55. a 56. a 57. b 58. b 59. b 60. b 61. c 62. a 63. a 64. d 65. b 66. c 67. a 68. d 69. e 70. b 71. b 72. a 73. a 74. b 75. c 76. a Powered by Cognero

Page 20


Name:

Class:

Date:

Chapter 12: Comparative Forms of Doing Business 77. d 78. e 79. a. Under the conduit concept applicable for a general partnership, Jane will deduct the $36,000 loss on her Form 1040. Thus, the cash flow benefit to her will be $11,520 ($36,000 × 32%). b.

Under the conduit concept applicable for an S corporation, Jane will deduct the $36,000 loss on her Form 1040. Thus, the cash flow benefit to her will be $11,520 ($36,000 × 32%).

c.

Under the conduit concept applicable for an LLC, Jane will deduct the $36,000 loss on her Form 1040. Thus, the cash flow benefit to her will be $11,520 ($36,000 × 32%).

d.

Under the entity concept applicable for C corporations, the corporate loss is not passed through to the shareholders. Thus, the cash flow benefit to her will be $0.

80. If the form selected is an S corporation, the corporation has a zero tax liability, and Candace has a tax liability associated with the business of $24,000 ($75,000 × 32%) each year. A distribution of the $75,000 of earnings to Candace would result in no additional tax liability. If the form selected is a C corporation, the corporation has a tax liability of $15,750 each year. A distribution of the aftertax earnings of $59,250 ($75,000 – $15,750) would result in an additional tax liability for Candace of $8,888 ($59,250 × 15%) each year. Thus, the total tax liability on the $75,000 would be $24,638 ($15,750 + $8,888) each year. The C corporation could follow a no-dividend distribution policy if the earnings are invested in the growth of the business. Thus, a significant factor affecting the choice between a C corporation and an S corporation for Candace is the projected distribution policy. 81. a.

b.

Kirby would include the $400,000 of lease income in his gross income. By deducting the lease payment, Falcon would reduce its taxable income to $100,000 ($500,000 – $400,000).

If the IRS determines that Falcon’s lease payments of $400,000 are not reasonable in amount, it will reclassify part of the lease payments as a dividend. Since dividends are not deductible by the corporation, the corporate taxable income increases by the amount of lease payments deemed unreasonable. Kirby’s gross income would not change. His lease income decreases and dividend income increases by the amount of the lease payments reclassified as a dividend. Note that Kirby’s dividend income would be eligible for the beneficial tax rate (15% or 20%).

82. a. Taxable income before salary Salary Taxable income Powered by Cognero

$700,000

$700,000

(400,000) $300,000

(100,000) $600,000 Page 21


Name:

Class:

Date:

Chapter 12: Comparative Forms of Doing Business Tax liability (21% rate)

b.

c.

$63,000

$126,000

The salary is included in the gross income of the recipient. Therefore, for the $400,000 salary, the president’s tax liability will increase by $128,000 ($400,000 × 32%), whereas for the $100,000 salary, their tax liability will increase by $32,000 ($100,000 × 32%). Even though the president’s tax liability increase (32% rate) in either case is greater than the corporation’s tax savings (21%) rate, the salary provides a way to get funds out of the corporation to shareholder/employees with the corporation deducting the salary. A payment to the president in the form of a dividend would not be deductible by the corporation and would still be included in their gross income. Qualified dividends are taxed to noncorporate shareholders at a beneficial 15% or 20% rate. If the IRS should classify part of the president’s salary as unreasonable compensation, the president would have dividend income rather than salary income. While the president’s taxable income would not change as a result of this, their tax liability would decrease (i.e., dividend income is taxed at a 15% or 20% rate whereas salary income is taxed at a 32% rate). However, the corporation would not receive a deduction of the amount classified as a dividend. Thus, the corporate tax liability would increase.

83. a.

If the loans are classified as debt, Eagle would deduct interest expense of $12,000 ($400,000 × 3%). Albert and Elva would each include $6,000 ($200,000 × 3%) of interest income in gross income. The repayment of the $400,000 in one year hence by Eagle would be the repayment of a liability by Eagle and a nontaxable return of capital to Albert and Elva.

b.

If the loans are reclassified as equity by the IRS, Eagle would treat the $12,000 annual payment as a nondeductible dividend. Albert and Elva would each include $6,000 of dividend income in gross income. The repayment of the $400,000 in one year by Eagle (assuming adequate E & P) would be treated as the payment of a nondeductible dividend. Albert and Elva would each include $200,000 of dividend income in gross income.

84. a. If the distribution is a dividend, Marjean’s tax liability is $105,000 ($700,000 × 15%). Given the amount of this dividend though, Marjean would have some of this qualified dividend taxed at the 20% capital gains rate. b.

If the distribution is a stock redemption, Marjean’s recognized gain is calculated as follows. Amount realized Adjusted basis Realized gain

$700,000 (200,000) $500,000

Recognized gain

$500,000

Powered by Cognero

Page 22


Name:

Class:

Date:

Chapter 12: Comparative Forms of Doing Business The gain is classified as a long-term capital gain. Using the LTCG tax rate of 15%, Marjean’s tax liability is $75,000 ($500,000 × 15%). Again, given the amount of this gain, Marjean would have some of it taxed at the 20% capital gains rate. 85. If the business operates as a C corporation, the $300,000 of projected losses will not benefit Sang-hoon on his individual tax return. Instead, the corporation will carry the losses forward to offset against future profits. Assuming profits of $200,000 in year 4 and thereafter, $160,000 of the net operating loss will be used in year 4 and the remaining $140,000 will be used in year 5. From a cash flow perspective, this will result in tax savings to the corporation in years 4 and 5 of $63,000.

Income before NOL carryforward NOL carryforward Taxable income Tax liability (21%)

Year 4 $200,000 (160,000) $40,000 $8,400

Year 5 $200,000 (140,000) $60,000 $12,600

For year 6 and thereafter, the annual tax liability will be $42,000 ($200,000 x 21%). If the S election is made, the $100,000 losses for each of the first three years can be passed through to Sang-hoon and deducted on his individual tax return. This will result in tax savings to him of $35,000 in each of tax years 1, 2, and 3. When the time value of money concept is considered, the benefit of the S election is even greater compared with C corporation status. At the end of year 3, Sang-hoon may want to terminate the S corporation election. If the election is maintained, Sanghoon’s annual tax liability will be increased by $70,000 ($200,000 × 35%) in tax year 4 and each year thereafter for the S corporation earnings. If the election is terminated, the annual tax liability of the C corporation will be $42,000. This assumes that the after-tax earnings of the C corporation will be reinvested in the growth of the business rather than distributed to the shareholders as dividends. 86. a.

b.

c.

Under § 351, no gain or loss is recognized at the time of the contribution of the assets to the S corporation. Since Melanie and Sonny satisfy the 80% control requirement, § 351 applies. Sonny’s basis for his stock is $130,000 ($100,000 + $30,000) and Melanie’s basis for her stock is $220,000 ($200,000 + $20,000). The corporation’s basis for its assets is a carryover basis (i.e., $100,000 for the cash, $30,000 for the land, $200,000 for the land and building, and $20,000 for the capitalized organization costs). Melanie has $20,000 of ordinary income for the services provided. The tax consequences for Melanie, Sonny, and the C corporation are the same as in a., (i.e., § 351 applies to both C corporations and S corporations). Under § 721, no gain or loss is recognized at the time of the contribution of the assets to the partnership. Note that § 721, unlike § 351, does not have a control requirement. Sonny’s basis for his partnership interest is $130,000 ($100,000 + $30,000), and Melanie’s basis for her partnership interest is $220,000 ($200,000 + $20,000). The partnership’s basis for its assets is a carryover basis (i.e., $100,000 for the cash, $30,000 for the land, $200,000 for the land and building, and $20,000 for the organization costs). Melanie has $20,000 of ordinary income for the services she provided.

Powered by Cognero

Page 23


Name:

Class:

Date:

Chapter 12: Comparative Forms of Doing Business 87. a.

Under § 351, no gain or loss is recognized at the time of the contribution of the assets to the S corporation. Since Sam and Trang satisfy the 80% control requirement, § 351 applies. Sam’s basis for his stock is $100,000, and Trang’s basis for her stock is $30,000 ($65,000 – $35,000), a carryover basis. The corporation’s basis for its assets is a carryover basis (i.e., $100,000 for the cash and $65,000 for the land and building).

b.

Under § 721, no gain or loss is recognized at the time of the contribution of the assets to the partnership. Note that § 721, unlike § 351, does not have a control requirement. Sam’s basis for his partnership interest is $117,500 [$100,000 + (50% × $35,000)], and Trang’s basis for her partnership interest is $47,500 ($65,000 – $17,500), a carryover basis. The partnership’s basis for its assets is a carryover basis (i.e., $100,000 for the cash and $65,000 for the land and building).

c.

The tax consequences for Sam, Trang, and the C corporation are the same as in a. (i.e., § 351 applies to both C corporation and S corporations).

88. Beginning adjusted basis + Share of profits + Share of liability increase Ending adjusted basis

Partnership $150,000 40,000 28,000 $218,000

C Corporation $150,000 –0– –0– $150,000

S Corporation $150,000 40,000 –0– $190,000

89. Anne has no recognized gain under § 721 and a carryover basis for a. her partnership interest of $90,000. The partnership has a carryover basis for Anne’s property of $90,000. b. Amount realized Basis for property Recognized gain

$225,000 (90,000) $135,000

The precontribution appreciation of $130,000 ($220,000 – $90,000) is allocated to Anne. Of the $5,000 balance, $1,250 ($5,000 × 25%) is allocated to Anne and $3,750 ($5,000 × 75%) is allocated to the other partners. Thus, the total gain allocated to Anne is $131,250 ($130,000 + $1,250). If the entity were an S corporation, the recognized gain would be c. allocated based on stock ownership. Thus, $33,750 ($135,000 × 25%) would be allocated to Anne and $101,250 ($135,000 × 75%) would be allocated to the other shareholders. The precontribution appreciation rule that applies to contributions made to partnerships does not apply to contributions made to S corporations. 90. a. If Wren is an S corporation, it is the tax reporter and Tucker and Maribel are Powered by Cognero

Page 24


Name:

Class:

Date:

Chapter 12: Comparative Forms of Doing Business the taxpayers. The tax liabilities on their shares of Wren’s $300,000 taxable income would be determined as follows. Tucker ($300,000 × 30% × 22%) Maribel ($300,000 × 70% × 35%)

$19,800 $73,500

Thus, Wren would need to distribute $93,300 ($19,800 + $73,500) to enable Tucker and Maribel to pay their tax liabilities associated with Wren. b. If Wren is a C corporation, Wren is the taxpayer. Its tax liability would be $63,000 ($300,000 x 21%). There would be no need to make any distributions to Tucker and Maribel since they are taxed only if they receive distributions from Wren.

91. a. Lee reports a recognized gain of $150,000 ($300,000 amount realized – $150,000 adjusted basis). The gain is a capital gain and is a long-term capital gain if the holding period for the stock is more than one year. b.

Katrina has an adjusted basis for her stock of $300,000.

c.

Vireo was not involved in the transaction. Therefore, the adjusted basis for its assets remains unchanged.

92. Luis would prefer that the form of the transaction be a stock sale, to avoid a. double taxation. Amount realized ($350,000 – $15,000) Stock basis Recognized gain (LTCG)

$335,000 (225,000) $110,000

A LTCG of $110,000 on the sale of the stock will result in an increase of Luis’s tax liability of $16,500 ($110,000 LTCG × 15%). Liquidating Silver, Inc., (i.e., an asset sale) would result in the following recognized gain and the related classification at the corporate level.

Asset Cash Accounts receivable Inventory Machinery and equipment Land Goodwill ($335,000 - 315,000) Powered by Cognero

Classification Gain OI LTCG $ –0– $ –0– 25,000 25,000 5,000 5,000 20,000 20,000 90,000 $90,000* 20,000 ______ 20,000 $160,000 $50,000 $110,000 Page 25


Name:

Class:

Date:

Chapter 12: Comparative Forms of Doing Business *If this is the only § 1231 transaction, the § 1231 gain is LTCG. The sale of Silver, Inc., assets will result in a corporate tax liability of $33,600 ($160,000 × 21%). In addition, when Silver distributes the available cash to Luis, his tax liability will be as follows. Amount realized ($350,000 – $15,000 – $33,600) Stock basis Recognized gain (LTCG)

$301,400 (225,000) $ 76,400

The LTCG of $76,400 to Luis produces a tax liability of $11,460 ($76,400 LTCG × 15%). Thus, the combined tax liability associated with the liquidation of Silver is $45,060 ($33,600 + $11,460). b.

93. a.

b.

Marilyn would prefer to purchase the assets so she can step up the basis of each asset to the purchase price (i.e., FMV) of $335,000. With a stock purchase, the assets of the corporation will have a carryover basis of $175,000.

Above all, the investment should make economic sense. Among the nontax factors, Lisa should consider the following: ∙

What has caused the decline in the average earnings rate for the 10-year average of 12% to the 5-year average of 9%?

What is the projected earnings rate for the future?

To what extent do the limited partners participate in profits and losses?

Why does the limited partnership need additional owner financing?

Are there any contingent liabilities that could affect the limited partner’s ability to recoup their investment?

What distributions, if any, can limited partners expect?

What other investment options does Lisa have?

If the partnership generates losses, the maximum amount Lisa will be able to deduct on her individual income tax return is her investment of $60,000. From a cash flow perspective, this could generate tax savings of $14,400 ($60,000 × 24%). A related issue that needs to be addressed by Lisa is the risk that she will not recover her investment.

Powered by Cognero

Page 26


Name:

Class:

Date:

Chapter 12: Comparative Forms of Doing Business 94. a. Since the business entity is a sole proprietorship, transactions between Mai and the entity are not taxable. The sole proprietorship has a carryover basis for the assets received from her (i.e., cash = $100,000, land = $60,000). b.

c. d.

e.

Contributions by a partner to a partnership are not subject to taxation under § 721. Mai’s basis for her ownership interest is a carryover basis of $160,000 under § 722. The partnership’s basis for the assets is a carryover basis (i.e., cash = $100,000, land and building = $60,000) under § 723. The answer is the same as that in b., for a general partnership. If Mai and any other shareholders involved in the transaction satisfy the § 368(c) control requirement (i.e., 80%), § 351 provides that realized gain is not recognized. In this case, Mai’s basis for her stock is a carryover basis of $160,000. The C corporation’s basis for the assets is a carryover basis (i.e., cash = $100,000, land and building = $60,000). However, if the control requirement is not satisfied, then Mai’s realized gain of $35,000 is recognized. In this case, Mai’s basis for her stock is the fair market value of the assets of $195,000 contributed. The C corporation’s basis for the assets also is the fair market value (i.e., cash = $100,000, land and building = $95,000). The tax consequences for the S corporation are the same as those for the C corporation in d..

95. The sole proprietorship, partnership (both general and limited), and S corporation are subject to single taxation (i.e., the owners rather than the entity are subject to Federal income tax). The C corporation is subject to double taxation (i.e., the entity is subject to Federal income tax on its profits and the owners are taxed on distributions (dividends) received from the corporation). The LLC could be subject to either single taxation or double taxation depending on what election is made under the check-the-box Regulations (i.e., normally single taxation since the partnership form usually is selected). 96. Techniques that can be used to avoid and/or reduce double taxation include: ∙ ∙

Transferring funds to the shareholders that are deductible to the corporation. (such as interest expense on a bona fide loan from the shareholder) Not making distributions to the shareholders.

Making distributions that qualify for return of capital treatment.

Making the S corporation election.

97. Transferring funds to the shareholders that are deductible to the corporation include the following: ∙

Salary payments to shareholder-employees.

Lease rental payments to shareholder-lessors.

Interest payments to shareholder-creditors.

To be deductible, the amounts must be reasonable and legitimate (for example, for interest payments, there must be a Powered by Cognero

Page 27


Name:

Class:

Date:

Chapter 12: Comparative Forms of Doing Business valid note between the parties calling for an appropriate interest rate and regular payments). 98. The AMT applies directly to a sole proprietorship and very large C corporations (average annual adjusted financial statement income for any three-year period in excess of $1 billion). The AMT applies indirectly to the other four passthrough entities in that the AMT attributes flow through to the individual’s tax return where the AMT is imposed. 99. The shareholder can lease the assets to the corporation. Assuming such rental payments are deductible (because there is a business purpose for the rental payments and the amounts are reasonable), the corporate taxable income is reduced and the corporation gets cash out of the corporation to the shareholder. Although the shareholder must include the rental income in gross income, single taxation is achieved. 100. Ori should consider the following factors. ∙

Are all the shareholders willing to consent to the S election?

Can the qualification requirements for an S election be satisfied at the time of the election?

∙ ∙ ∙

Since the S qualification requirements become maintenance requirements, can these requirements continue to be satisfied? For what period will the conditions that make the S election beneficial continue to prevail? Will the corporate distribution policy create wherewithal to pay problems at the shareholder level?

An analysis of Ori’s situation indicates that none of these factors should deter him from electing S status for his corporation. A factor normally considered that is not present for Ori is the existence of NOLs for the C corporation. An S corporation generally cannot offset C corporation NOLs against S corporation profits.

101.

The corporate tax liability on taxable income of $450,000 is $94,500 ($450,000 × 21%) for the C corporation. Since the tax liability on the $450,000 is assessed at the corporate level, there is no need to a. make a dividend distribution to Mabel and Alan to cover their share of entity-level taxes. They will each receive a salary of $175,000. The tax liability is assessed at the shareholder level rather than at the corporate level for the S corporation. Mabel and Alan each incur a Federal income tax liability of $72,000 ($225,000 × 32%) associated with their respective shares of b. the corporate taxable income of $450,000. Therefore, the corporation will need to distribute $72,000 each to Mabel and Alan to cover the Federal income tax liability. The parties can agree to reduce this amount, if desired, by factoring in the potential qualified business income deduction they claim. They also receive their salary of $175,000 each. c. The combined entity/owner tax liability in part a. is: C corporation Shareholders on distribution

$ 94,500 –0–

Shareholders on salaries ($350,000 × 32%)

112,000

Combined tax liability

$206,500

Powered by Cognero

Page 28


Name:

Class:

Date:

Chapter 12: Comparative Forms of Doing Business

The combined entity/owner tax liability in part b. is: S corporation Shareholders taxed on S corporation earnings

$ –0–

($450,000 × 32%) Shareholders on salaries ($350,000 × 32%)

144,000 112,000 $256,000

Combined tax liability

102. a. For a partnership, the precontribution gain of $15,000 ($100,000 – $85,000) must be allocated to Jiang. Thus, her share of the recognized gain of $35,000 ($120,000 – $85,000) on the sale of the land by the partnership is $31,000 [$15,000 + ($20,000 × 80%)]. b.

None of the $35,000 gain is allocated to the shareholders. All of the gain is taxed to the corporation.

c.

The $35,000 gain is passed through to the shareholders of the S corporation based on the stock ownership. Thus, $28,000 ($35,000 × 80%) is allocated to Jiang. The precontribution appreciation rule that applies to contributions made to partnerships does not apply to contributions made to S corporations.

103. a. If the entity is a C corporation, Dudley’s stock basis remains at $100,000. The profits are taxed at the C corporation level. Since the $50,000 distribution is a dividend (i.e., current E & P is $90,000), this transaction does not affect Dudley’s stock basis. b. If the entity is a general partnership, Dudley’s basis for his ownership interest is affected by both the profits and by the distribution. Beginning basis Plus: Share of profits ($90,000 × 20%) Less: Distribution received ($50,000 × 20%) Ending basis

$100,000 18,000 (10,000) $108,000

c. If the entity is an S corporation, Dudley’s stock basis is affected by both the profits and the distribution. Beginning stock basis Plus: share of profits ($90,000 × 20%) Less: Distribution received ($50,000 × 20%) Ending basis

$100,000 18,000 (10,000) $108,000

104. a. Gain of $25,000 [$75,000 (fair market value) – $50,000 (adjusted basis)] is recognized by Powered by Cognero

Page 29


Name:

Class:

Date:

Chapter 12: Comparative Forms of Doing Business the corporation under § 311(b) on the distribution of the land. Since the corporation apparently has substantial earnings and profits, Chun recognizes dividend income of $75,000. b.

c.

Gain of $25,000 [$75,000 (fair market value) – $50,000 (adjusted basis)] is recognized by the S corporation under § 311(b) on the distribution of the land. This gain is passed through to the shareholders to report on their tax returns and increases their stock basis. Assuming that Chun’s stock basis is at least $75,000, he incurs no additional recognized gain due to the distribution and reduces his stock basis by $75,000. If his stock basis is less than $75,000, he reduces his stock basis to zero, and the excess of the distribution over the stock basis is capital gain. The distribution of the land does not result in recognition of gain to the partner or partnership. If Chun’s basis in his partnership interest is at least $50,000, he reduces the basis by $50,000 and assigns a $50,000 basis to the land. If his basis for his partnership interest is less than $50,000, he reduces the basis to zero and assigns the amount of the partnership interest basis before the distribution to the land (e.g., if his partnership interest basis was $42,000, then his basis for the land is $42,000).

105. With respect to special allocations, an S corporation is treated more like a C corporation than a partnership. For example, the § 704(c)(1) allocations of built-in gain or loss are mandatory for partnerships. Such a special allocation is not permitted for an S corporation under the per share/per day rule. 106. The form of the sale does not matter to Ling for Federal income tax purposes. The sales transaction will be treated as the sale of the individual assets. Thus, ordinary and capital gain or loss will result depending on the nature of the asset sold. 107. No. Regardless of the legal form of the purchase transaction, the basis of the individual assets will be assigned a cost (same as fair market value) basis. If the purchase price exceeds the FMV of the assets, such excess is assigned to goodwill. 108. a. If the fair market value of the corporate assets does not exceed the adjusted basis of the corporate assets, a stock purchase is preferable. The adjusted basis of the corporate assets will remain unchanged. b.

If the fair market value of the corporate assets exceeds the adjusted basis of the corporate assets, an asset purchase from Red is preferable. Then Ralph could contribute the assets to a corporation under § 351. The adjusted basis of the corporate assets will equal the amount Ralph paid for them (i.e., FMV).

109. Selling the stock will result in single taxation at the beneficial capital gain rate. Selling the assets will result in double taxation with only Wang’s recognized gain qualifying for the beneficial capital gain rate. Therefore, Wang should sell his stock. 110. a. QBI deduction is $34,000 ($170,000 x 20%). This deduction is not allowed in computing self-employment tax. b. $168,600 x 15.3% = $1,400 x 2.9% = Powered by Cognero

$25,735 41 Page 30


Name:

Class:

Date:

Chapter 12: Comparative Forms of Doing Business Total self-employment tax

$25,776

c. Alejandro may deduct one-half of his self-employment tax ($12,888). This amount was already subtracted to derive his sole proprietorship earnings of $170,000. d. Both Alejandro and the C corporation (employer) would pay half of the $25,776 as payroll taxes. Neither Alejandro nor the C corporation would be allowed to claim the qualified business income deduction. 111. An owner of a C corporation is least likely to have a wherewithal to pay problem. The shareholder is not taxed on their distributive share of entity earnings. Instead, the shareholder would be taxed only on wages (if an employee), dividends, or other payments, such as for rent or interest income. In each situation, assuming the transfer is in cash, the owner has funds to pay any tax on the income. A sole proprietor may have a minor wherewithal to pay issue. A sole proprietor is taxed on the net earnings of the business. If the business uses the accrual method of accounting and there are significant accounts receivable, it is possible that funds are insufficient for the owner to pay taxes. A partner must pay tax on their distributive share of partnership income even if none of the earnings or capital is distributed to the owner. This can present a severe wherewithal to pay problem for them. An S corporation shareholder also must pay tax on their distributive share of corporate income. If the distribution is less than the tax on these earnings, the owner has a wherewithal to pay problem. However, unlike a partnership, the S corporation owner who also works for the entity is an employee and is to be paid reasonable wages. These wages also decrease the owner’s distributive share of income. The receipt of wages allows the shareholder to pay tax on such income.

Powered by Cognero

Page 31


Name:

Class:

Date:

Chapter 13: Taxes in the Financial Statements True / False 1. Temporary differences involve items that appear in both the GAAP financial statements and the Federal income tax return but not in the same reporting period. a. True b. False 2. Permanent differences include items that appear in both the GAAP financial statements and the Federal income tax return but not in the same reporting period. a. True b. False 3. In general, the purpose of ASC 740 is to ensure that all of the income tax consequences related to current-year book income are reported in the current-year financial statements. a. True b. False 4. The current tax expense reported on the GAAP financial statements generally represents the tax liabilities reflected on all current-year tax returns. a. True b. False 5. A deferred tax liability represents a current tax liability associated with income or expense to be reported in future-year GAAP financial statements. a. True b. False 6. A deferred tax asset is the expected future tax benefit (savings) associated with items reported in the current-year GAAP financial statements. a. True b. False 7. An example of a book-tax difference resulting in a deferred tax asset is the excess of accelerated MACRS depreciation over GAAP straight-line depreciation. a. True b. False 8. The valuation allowance can reduce either a deferred tax asset or a deferred tax liability. a. True b. False 9. If a valuation allowance is decreased in the current year, the corporation’s effective tax rate is lower than had the valuation allowance not decreased. a. True b. False 10. A valuation allowance expresses in the financial statements that there exists uncertainty that the taxpayer will be able Powered by Cognero

Page 1


Name:

Class:

Date:

Chapter 13: Taxes in the Financial Statements to recover a deferred tax asset. a. True b. False 11. A valuation allowance expresses in the financial statements that there exists uncertainty regarding the level of authoritative support for a tax position taken. a. True b. False 12. The income tax footnote to the GAAP financial statements includes a reconciliation of (1) a corporation’s hypothetical tax on book income as if it were taxed in full at the applicable U.S. income tax rates and (2) its reported tax expense. a. True b. False 13. A deferred tax liability represents a potential future tax benefit associated with items reported in the current-year GAAP financial statements. a. True b. False 14. The tax rate reconciliation included in the income tax footnote uses temporary book-tax differences to reconcile the reported tax expense to what would be expected if all book income were currently taxable at the U.S. tax rate. a. True b. False 15. Kling Corporation has a $150,000 favorable temporary book-tax difference. This item has the same effect on Kling’s current-year effective tax rate as a $150,000 favorable permanent book-tax difference. a. True b. False 16. ASC 740 addresses how an entity should report uncertain tax positions in its financial statements. a. True b. False 17. The release of a valuation allowance may relate to a tax planning strategy adopted by the taxpayer that will produce taxable income in the future. a. True b. False 18. A chief financial officer (CFO) probably prefers a tax planning strategy that produces a favorable temporary book-tax savings difference to one that produces a favorable permanent difference. a. True b. False 19. The income tax expense recorded by a corporation on its GAAP financial statements includes its Federal and state income taxes but not its foreign income taxes. a. True Powered by Cognero

Page 2


Name:

Class:

Date:

Chapter 13: Taxes in the Financial Statements b. False 20. The current tax expense reported on the GAAP financial statements generally represents the taxes actually payable to domestic or foreign governmental authorities. a. True b. False Multiple Choice 21. Which of the following items produces a temporary book-tax difference? a. Municipal bond interest. b. Federal income tax paid. c. Addition to bad debt allowance. d. Nondeductible penalties. 22. Phyllis, Inc., earns book net income before tax of $600,000. Phyllis puts into service a depreciable asset this year, and its first-year tax depreciation exceeds book depreciation by $120,000. Phyllis has recorded no other temporary or permanent book-tax differences. Assuming that the applicable tax rate is 21%, what is Phyllis’s total income tax expense reported on its GAAP financial statements? a. $151,200 b. $126,000 c. $100,800 d. $25,200 23. Gravel, Inc., earns book net income before tax of $600,000. Gravel puts into service a depreciable asset this year, and its first-year tax depreciation exceeds book depreciation by $120,000. Gravel has recorded no other temporary or permanent book-tax differences. Assuming that the applicable tax rate is 21%, what is Gravel’s current income tax expense reported on its GAAP financial statements? a. $151,200 b. $126,000 c. $100,800 d. $25,200 24. Clipp, Inc., earns book net income before tax of $600,000. Clipp puts into service a depreciable asset this year, and its first-year tax depreciation exceeds book depreciation by $120,000. Clipp has recorded no other temporary or permanent book-tax differences. Assuming that the applicable tax rate is 21%, what is Clipp’s deferred tax liability reported on its GAAP financial statements? a. $151,200 b. $126,000 c. $100,800 d. $25,200 25. Jogg, Inc., earns book net income before tax of $600,000. It puts into service a depreciable asset this year, and its firstyear tax depreciation exceeds book depreciation by $120,000. Jogg has recorded no other temporary or permanent booktax differences. Assuming that the applicable tax rate is 21%, and that this is Jogg’s first year of operations, what is Jogg’s balance in its deferred tax asset and deferred tax liability accounts at year-end? Powered by Cognero

Page 3


Name:

Class:

Date:

Chapter 13: Taxes in the Financial Statements a. $25,200 and $0. b. $0 and $0. c. $0 and $25,200. d. $25,200 and $25,200. 26. Qute, Inc., earns book net income before tax of $500,000. In computing its book income, Qute expenses $50,000 more in warranty expense for book purposes than it is allowed to deduct for tax purposes. Qute records no other temporary or permanent book-tax differences. Assuming that the applicable tax rate is 21% and no valuation allowance is required, what is Qute’s total income tax expense reported on its GAAP financial statements? a. $115,500 b. $105,000 c. $94,500 d. $10,500 27. Morrisson, Inc., earns book net income before tax of $500,000. In computing its book income, Morrisson expenses $50,000 more in warranty expense for book purposes than it is allowed to deduct for tax purposes. Morrisson records no other temporary or permanent book-tax differences. Assuming that the applicable tax rate is 21% and no valuation allowance is required, what is Morrisson’s current income tax expense reported on its GAAP financial statements? a. $115,500 b. $105,000 c. $94,500 d. $10,500 28. Never, Inc., earns book net income before tax of $500,000. In computing its book income, Never expenses $50,000 more in warranty expense for book purposes than it is allowed to deduct for tax purposes. Never records no other temporary or permanent book-tax differences. Assuming that the applicable tax rate is 21% and no valuation allowance is required, what is Never’s deferred tax asset reported on its GAAP financial statements? a. $115,500 b. $105,000 c. $94,500 d. $10,500 29. South, Inc., earns book net income before tax of $400,000 in year 1. It acquires a depreciable asset in year 1, and its first-year tax depreciation exceeds book depreciation by $50,000. In year 2, South earns $500,000 book net income before tax, and its book depreciation exceeds tax depreciation by $20,000. South records no other temporary or permanent booktax differences. Assuming that the applicable tax rate is 21%, what is South’s total provision for income tax expense reported on its GAAP financial statements for year 2? a. $4,200 b. $94,500 c. $105,000 d. $109,200 30. South, Inc., earns book net income before tax of $400,000 in year 1. It acquires a depreciable asset in year 1, and its first-year tax depreciation exceeds book depreciation by $50,000. In year 2, South earns $500,000 book net income before tax, and its book depreciation exceeds tax depreciation by $20,000. South records no other temporary or permanent book-tax differences. Assuming that the applicable tax rate is 21% in both years, what is South’s current income tax expense reported on its GAAP financial statements for year 2? Powered by Cognero

Page 4


Name:

Class:

Date:

Chapter 13: Taxes in the Financial Statements a. $4,200 b. $94,500 c. $105,000 d. $109,200 31. South, Inc., earns book net income before tax of $400,000 in year 1. It acquires a depreciable asset in year 1, and its first-year tax depreciation exceeds book depreciation by $50,000. In year 2, South earns $500,000 book net income before tax, and its book depreciation exceeds tax depreciation by $20,000. South records no other temporary or permanent book-tax differences. Assuming that the applicable tax rate is 21% in both years, what is South’s balance in its deferred tax liability account at the end of year 2? a. $0 b. $4,200 c. $6,300 d. $10,500 32. Van Dyke, Inc., tentatively expects to report total tax expense of $150,000 in its current-year financial statements. This amount consists of $200,000 in current tax expense and a $50,000 tax benefit related to the expected future use of an NOL. If management determines that a valuation allowance of $20,000 must be placed against Van Dyke’s deferred tax assets, what will be the total tax expense reported in Van Dyke's financial statements? a. $150,000 b. $170,000 c. $200,000 d. $250,000 33. Cold, Inc., reported $100,000 total tax expense for financial statement purposes in year 1. This total expense consisted of $150,000 in current tax expense and a deferred tax benefit of $50,000. The deferred tax benefit consisted of $90,000 in deferred tax assets reduced by a valuation allowance of $40,000. In year 2, Cold reports $600,000 in book net income before tax. Cold records no other permanent or temporary book-tax differences. At the end of year 2, Cold’s management determines that the existing valuation allowance of $40,000 should be reduced to zero. What is Cold’s total tax expense for year 2? a. $250,000 b. $126,000 c. $86,000 d. $40,000 34. Which of the following items is not included in the GAAP financial statement income tax footnote’s effective tax rate reconciliation? a. Hypothetical tax on book income at U.S. Federal corporate tax rate. b. Total tax expense per the GAAP financial statements. c. Tax effect of temporary differences. d. Tax effect of permanent differences. 35. Which of the following items is not included in the income tax footnote for a publicly traded company? a. Rate reconciliation. b. Analysis of deferred tax assets and liabilities. c. Breakdown of income tax between foreign and domestic. Powered by Cognero

Page 5


Name:

Class:

Date:

Chapter 13: Taxes in the Financial Statements d. Breakdown of income tax among U.S. states. 36. Hot, Inc.’s primary competitor is Cold, Inc. When comparing its deferred tax assets and liabilities with Cold's, which of the following benchmarking activities should Hot undertake? a. Scale the deferred tax assets and liabilities by total sales or total assets. b. Compare raw dollar amounts of deferred tax assets and liabilities. c. Ignore deferred tax assets and liabilities and focus on overall effective tax rate. d. Ignore all tax information other than the current tax expense. 37. Healy, Inc., reports an effective tax rate in its income tax footnote of 6%. The only reconciling item with regard to the hypothetical tax at 21% is a valuation allowance reversal of negative 15%. Which of the following statements is true concerning comparing Healy's effective tax rate with its competitors, all of whom have an effective tax rate between 20 and 24%? a. Healy is managing its tax burden in a more efficient manner than its competitors are. b. Healy's structural effective tax rate is actually quite close to that of its competitors. c. Healy earned more cash profits because of its lower effective tax rate. d. Healy is likely to be engaged in tax shelter activities. 38. Which of the following statements is true? a. The breakdown of tax expense between current and deferred may provide useful information regarding the comparison of tax burdens between companies. b. An analysis of earnings before interest, taxes, depreciation, and amortization (EBITDA) is often a better approach to comparing the operating results of two companies. c. One-time effects within a company’s effective tax rate should be removed before comparing the effective tax rates across companies (or across years for the same company). d. All these observations are correct. 39. The best estimate of the income tax reflected on a corporation's current-year tax returns is the: a. Change in the deferred tax asset account balance. b. Change in the deferred tax liability account balance. c. Current tax expense. d. Total tax expense. 40. From a tax planning perspective, a taxpayer would generally prefer to see a(n): a. Increase in the current tax expense. b. Increase in the deferred tax asset account balance. c. Decrease in the deferred tax liability account balance. d. Increase in the deferred tax liability account balance. 41. An increase to an entity’s valuation allowance might be called for when: a. The entity’s last book profit occurred 10 years ago. b. The entity’s assets have appreciated greatly in value, but there is no inclination to sell them. c. Both a. and b. d. Neither a. nor b. Powered by Cognero

Page 6


Name:

Class:

Date:

Chapter 13: Taxes in the Financial Statements 42. A GAAP valuation allowance relates to a: a. Deferred tax liability. b. Deferred tax asset. c. Both a. and b. d. Neither a. nor b. 43. Income tax information can be found in an entity’s GAAP financial statements in the: a. Balance sheet. b. Income statement. c. Both a. and b. d. Neither a. nor b. 44. A GAAP financial statement includes footnotes that: a. Give estimates of the dates on which the deferred tax liability will be paid. b. Show the journal entries to determine the deferred income tax expense. c. Break down the state-by-state profitability of the entity. d. Include a reconciliation of the book effective tax rate with the applicable statutory tax rate. 45. JiangCo constructs the following table related to its filing position for a research activities credit. Its book tax provision for the year, including $3 million for the credit, is $10 million. Resulting Estimated Tax Benefit $3 million $2 million $1.6 million $1.1 million

Probability of Realizing the Cumulative Probability Estimated Benefit of Realization 10% 10% 35% 45% 40% 85% 15% 100%

JiangCo’s book income tax expense is: a. $13 million. b. $11.6 million. c. $11.4 million. d. $10 million. 46. JuarezCo constructs the following table related to its filing position for a research activities credit. Its book-tax provision for the year, including $3 million for the credit, is $10 million. Resulting Estimated Tax Benefit $3 million $2 million $1.6 million $1.1 million

Probability of Agreement between JuarezCo and IRS 10% 45% 40% 5%

Cumulative Probability of Realization 10% 55% 95% 100%

JuarezCo’s book income tax expense is: Powered by Cognero

Page 7


Name:

Class:

Date:

Chapter 13: Taxes in the Financial Statements a. $13 million. b. $12 million. c. $11.4 million. d. $11 million. 47. An investment analyst would prefer to see that an entity’s effective tax rate is: a. Sustainable over time. b. Less than that of competitors. c. Both a. and b. d. Neither a. nor b. 48. OldTown, Inc. claimed $100,000 of rehabilitation credits on its prior year tax return. There was sufficient tax authority to claim the credits. However, due to uncertainty regarding the eligibility of some of the expenditures, OldTown only recognized $70,000 of the benefits in its prior year financial statements. Upon the audit of its tax return in the current year, the IRS challenged the eligibility of some of the expenditures and OldTown agreed to pay $45,000 additional tax to settle the dispute. By how much does the settlement impact OldTown’s current-year book tax expense? a. $0. b. $15,000. c. $30,000. d. $45,000. 49. An increase to an entity’s valuation allowance might be called for when: a. The entity’s last book profit occurred 10 years ago. b. The entity’s assets have appreciated greatly in value, but there is no inclination to sell them. c. Both a. and b. d. Neither a. nor b. Subjective Short Answer 50. Budlow, Inc., reported the following results for the current year.

Book income (before tax) Tax depreciation in excess of book Warranty expense, not deductible currently Municipal bond interest income

$500,000 75,000 17,500 10,000

Determine Budlow’s current tax expense for the year assuming a combined federal, state, and foreign tax rate of 25%. 51. Black, Inc., is a domestic corporation with the following balance sheets for book and tax purposes at the end of the year. Assume a 21% corporate tax rate and no need for a valuation allowance. Tax Debit/(Credit) Assets Cash Accounts Receivable Powered by Cognero

$

300 5,000

Book Debit/(Credit) $ 300 5,000 Page 8


Name:

Class:

Date:

Chapter 13: Taxes in the Financial Statements Buildings Accumulated Depreciation Furniture & Fixtures Accumulated Depreciation Total Assets

300,000 (150,000) 40,000 (21,000) $174,300

Liabilities Accrued Litigation Expense Note Payable Total Liabilities Stockholders’ Equity Paid in Capital Retained Earnings Total Liabilities and Stockholders’ Equity

300,000 (80,000) 40,000 (15,000) $250,300

$ –0– (116,000) ($116,000)

($ 27,000) (116,000) ($143,000)

($ 1,000) (57,300)

($ 1,000) (106,300)

($174,300)

($250,300)

Black, Inc.’s, gross deferred tax assets and liabilities at the beginning of Black’s year are as follows: Beginning of Year $20,000 $20,000 × 21% $ 4,200

Accrued litigation expense Subtotal Applicable tax rate Gross deferred tax asset Building – Accumulated depreciation Furniture & fixtures – Accumulated depreciation Subtotal Applicable tax rate Gross deferred tax liability

($61,000) (3,000) ($64,000) × 21% ($13,440)

Black, Inc.’s, book income before tax is $6,000. Black records two permanent book-tax differences. It earned $250 in tax-exempt municipal bond interest, and it incurred $500 in nondeductible entertainment expense. Determine the change in Black’s deferred tax assets for the current year. 52. Black, Inc., is a domestic corporation with the following balance sheets for book and tax purposes at the end of the year. Assume a 21% corporate tax rate and no valuation allowance. Tax Debit/(Credit) Assets Cash Accounts Receivable Buildings Powered by Cognero

$

300 5,000 300,000

Book Debit/(Credit) $

300 5,000 300,000 Page 9


Name:

Class:

Date:

Chapter 13: Taxes in the Financial Statements Accumulated Depreciation Furniture & Fixtures Accumulated Depreciation Total Assets Liabilities Accrued Litigation Expense Note Payable Total Liabilities Stockholders’ Equity Paid-in Capital Retained Earnings Total Liabilities and Stockholders’ Equity

(150,000) 40,000 (21,000) $174,300

(80,000) 40,000 (15,000) $250,300

$ –0– (116,000) ($116,000)

($ 27,000) (116,000) ($143,000)

($ 1,000) (57,300)

($ 1,000) (106,300)

($174,300)

($250,300)

Black, Inc.’s, gross deferred tax assets and liabilities at the beginning of Black’s year are as follows: Beginning of Year Accrued litigation expense $20,000 Subtotal $20,000 Applicable tax rate × 21% Gross deferred tax asset $ 4,200 Building – Accumulated depreciation Furniture & fixtures – Accumulated depreciation Subtotal Applicable tax rate Gross deferred tax liability

($61,000) (3,000) ($64,000) × 21% ($13,440)

Black, Inc.’s, book income before tax is $6,000. Black records two permanent book-tax differences. It earned $250 in tax-exempt municipal bond interest, and it incurred $500 in nondeductible entertainment expense. Determine the net deferred tax asset or net deferred tax liability at year-end. 53. Black, Inc., is a domestic corporation with the following balance sheet for book and tax purposes at the end of the year. Assume a 21% corporate tax rate and no need for a valuation allowance. Tax Debit/(Credit) Assets Cash Accounts receivable Buildings Accumulated depreciation Furniture & fixtures Accumulated depreciation Total Assets Liabilities Accrued litigation expense Powered by Cognero

Book Debit/(Credit)

$

300 5,000 300,000 (150,000) 40,000 (21,000) $174,300

$ –0–

$

300 5,000 300,000 (80,000) 40,000 (15,000) $250,300

($ 27,000) Page 10


Name:

Class:

Date:

Chapter 13: Taxes in the Financial Statements Note payable Total liabilities Stockholders’ Equity Paid-in capital Retained earnings Total liabilities and stockholders’ equity

(116,000) ($116,000)

(116,000) ($143,000)

($ 1,000) (57,300)

($ 1,000) (106,300)

($174,300)

($250,300)

Black, Inc.’s, gross deferred tax assets and liabilities at the beginning of Black’s year are listed below. Accrued litigation expense Subtotal Applicable tax rate Gross deferred tax asset

Beginning of Year $20,000 $20,000 × 21% $ 4,200

Building – Accumulated depreciation Furniture & fixtures – Accumulated depreciation Subtotal Applicable tax rate Gross deferred tax liability

($61,000) (3,000) ($64,000) × 21% ($13,440)

Black, Inc.’s, book income before tax is $6,000. Black records two permanent book-tax differences. It earned $250 in tax-exempt municipal bond interest and incurred $500 in nondeductible entertainment expense. Determine the change in Black’s gross deferred tax liabilities for the current year.

54. Black, Inc., is a domestic corporation with the following balance sheet for book and tax purposes at the end of the year. Assume a 21% corporate tax rate and no valuation allowance. Tax Debit/(Credit) Assets Cash Accounts receivable Buildings Accumulated depreciation Furniture & fixtures Accumulated depreciation

Book Debit/(Credit)

$

Total Assets

300 5,000 300,000 (150,000) 40,000 (21,000) $174,300

300 5,000 300,000 (80,000) 40,000 (15,000) $250,300

Liabilities Accrued litigation expense Note payable Total liabilities

$ –0– (116,000) ($116,000)

($ 27,000) (116,000) ($143,000)

Powered by Cognero

$

Page 11


Name:

Class:

Date:

Chapter 13: Taxes in the Financial Statements

Stockholders’ Equity Paid-in capital Retained earnings Total liabilities and stockholders’ equity

($ 1,000) (57,300)

($ 1,000) (106,300)

($174,300)

($250,300)

Black, Inc.’s, gross deferred tax assets and liabilities at the beginning of Black’s year are as follows:

Accrued litigation expense Subtotal Applicable tax rate Gross deferred tax asset Building – Accumulated depreciation Furniture & fixtures – Accumulated depreciation Subtotal Applicable tax rate Gross deferred tax liability

Beginning of Year $20,000 $20,000 × 21% $ 4,200 ($61,000) (3,000) ($64,000) × 21% ($13,440)

Black, Inc.’s, book income before tax is $6,000. Black records two permanent book-tax differences. It earned $250 in tax-exempt municipal bond interest, and it incurred $500 in nondeductible entertainment expense. Calculate Black’s current tax expense. 55. Black, Inc., is a domestic corporation with the following balance sheet for book and tax purposes at the end of the year. Assume a 21% corporate tax rate and no valuation allowance. Tax Book Debit/(Credit) Debit/(Credit) Assets Cash $ 300 $ 300 Accounts receivable 5,000 5,000 Buildings 300,000 300,000 Accumulated depreciation (150,000) (80,000) Furniture & fixtures 40,000 40,000 Accumulated depreciation (21,000) (15,000) $174,300 $250,300 Total assets Liabilities Accrued litigation expense Powered by Cognero

$ –0–

($ 27,000) Page 12


Name:

Class:

Date:

Chapter 13: Taxes in the Financial Statements Note payable Total liabilities Stockholders’ Equity Paid-in capital Retained earnings Total liabilities and stockholders’ equity

(116,000) ($116,000)

(116,000) ($143,000)

($ 1,000) (57,300)

($ 1,000) (106,300)

($174,300)

($250,300)

Black, Inc.’s, gross deferred tax assets and liabilities at the beginning of Black’s year are as follows: Accrued litigation expense Subtotal Applicable tax rate Gross deferred tax asset Building – Accumulated depreciation Furniture & fixtures – Accumulated depreciation Subtotal Applicable tax rate Gross deferred tax liability

Beginning of Year $21,000 $21,000 × 21% $4,410 ($61,000) (3,000) ($64,000) × 21% ($13,440)

Black, Inc.’s, book income before tax is $6,000. Black records two permanent book-tax differences. It earned $250 in tax-exempt municipal bond interest and incurred $500 in nondeductible entertainment expense. Provide the income tax footnote rate reconciliation for Black, using either dollars or percentages. 56. At the beginning of the year, Schrader, Inc., holds a net operating loss carryforward, and its balance sheet shows a related deferred tax asset of $500,000. At the end of the year, the balance in the deferred tax asset account has not changed, but Schrader’s management believes a $90,000 valuation allowance is needed, because of a persistent downturn in Schrader’s profitability. Provide the journal entry to record the valuation allowance. 57. At the beginning of the year, the balance sheet of Schrader, Inc., shows a $500,000 deferred tax asset relating to a net operating loss carryforward offset by a $90,000 valuation allowance. At the end of the year, Schrader’s management believes there is sufficient positive evidence to support releasing $20,000 of the allowance. Provide the journal entry to record this change in the valuation allowance. Essay 58. A corporation’s taxable income almost never is the same as its GAAP financial accounting income. Discuss the types of differences that exists between the two. Use the terms permanent and temporary book-tax differences in your answer. Give at least two examples of each type of book-tax difference. 59. Jacobsen Corporation has determined the appropriate changes to its deferred tax assets and liabilities. It is now considering whether to place a valuation allowance against the deferred tax asset on the balance sheet. List some of the factors that Jacobsen should consider in this regard. 60. You are the tax adviser to a publicly traded U.S. corporation. How might you use a “benchmarking” analysis to begin your review of the entity’s tax situation and planning opportunities? 61. Bryden Corporation is considering two tax planning strategies. Both would produce a $1 million tax savings. One of Powered by Cognero

Page 13


Name:

Class:

Date:

Chapter 13: Taxes in the Financial Statements the strategies involves a temporary book-tax difference, and the other would generate a permanent difference. In general, which plan would the Bryden CFO prefer? Which would a stock analyst reviewing the Bryden valuation prefer? 62. The tax expense reported by a corporation in its financial statements is unlikely equal to the tax reported on

its Federal income tax return for the same period. Identify and briefly explain the items that cause the two to differ.

Powered by Cognero

Page 14


Name:

Class:

Date:

Chapter 13: Taxes in the Financial Statements Answer Key 1. True 2. False 3. True 4. True 5. False 6. True 7. False 8. False 9. True 10. True 11. False 12. True 13. False 14. False 15. False 16. True 17. True 18. False 19. False 20. True 21. c 22. b 23. c 24. d 25. c Powered by Cognero

Page 15


Name:

Class:

Date:

Chapter 13: Taxes in the Financial Statements 26. b 27. a 28. d 29. c 30. d 31. c 32. b 33. c 34. c 35. d 36. a 37. b 38. d 39. c 40. d 41. a 42. b 43. c 44. d 45. c 46. d 47. c 48. b 49. a 50. Budlow reports net income before tax of $500,000 on its GAAP financial statement, but it must adjust this amount for book-tax differences. Tax depreciation in excess of book is a tax deduction not deducted for book purposes, and warranty Powered by Cognero

Page 16


Name:

Class:

Date:

Chapter 13: Taxes in the Financial Statements expense is deductible for book purposes but not yet deductible for tax. Both these items are temporary differences because they eventually reverse (with book depreciation eventually exceeding tax depreciation and the warranty expense ultimately deducted for tax when incurred). The municipal bond adjustment is a permanent difference because this income never is subject to Federal income tax. Book income (before tax) Tax depreciation in excess of book Nondeductible warranty expense Municipal bond interest income Taxable income (Form 1120) Tax rate Current tax expense

$500,000 (75,000) 17,500 (10,000) $432,500 x 25% $108,125

51.

Accrued litigation expense Subtotal Applicable tax rate Gross deferred tax asset Change in deferred tax Asset

Beginning Current of Year Year Difference End of Year $20,000 $7,000 $27,000 $20,000 $7,000 $27,000 × 21% × 21% $ 4,200 $ 5,670 $1,470

52. Tax Debit/(Credit) Assets Cash Accounts receivable Buildings Accumulated depreciation Furniture & fixtures accumulated Depreciation Total Assets

Book Debit/(Credit)

Difference

$ 300 5,000 300,000

$ 300 5,000 300,000

(150,000)

(80,000)

40,000

40,000

(21,000)

(15,000)

(6,000)

$174,300

$250,300

($76,000)

$ –0– (116,000) ($116,000)

($ 27,000)

$27,000

(116,000) ($143,000)

$27,000

($70,000)

Liabilities Accrued litigation Expense Note payable Total liabilities Powered by Cognero

Page 17


Name:

Class:

Date:

Chapter 13: Taxes in the Financial Statements Stockholders’ Equity Paid-in capital Retained earnings Total liabilities & stockholders’ equity

($ 1,000) (57,300)

($ 1,000) (106,300)

($174,300)

($250,300)

Given these basis differences, the gross DTA and gross DTL are calculated as follows. Gross deferred tax asset ($27,000 × 21%) Gross deferred tax liability ($76,000 × 21%) Net deferred tax liability

$ 5,670 (15,960) ($10,290)

53. Building – Accumulated depreciation Furniture & fixtures – Accumulated depreciation Subtotal Applicable tax rate Gross deferred tax liability Change in deferred tax Liability

Beginning Current of Year Year Difference ($61,000) ($ 9,000)

54. Pre-tax book income

End of Year ($70,000)

(3,000)

(3,000)

(6,000)

($64,000) × 21% ($13,440)

($12,000)

($76,000) × 21% ($15,960)

($ 2,520)

$6,000

Book-tax adjustments Permanent Items Tax exempt income Nondeductible entertainment expense Temporary differences Building depreciation Furniture & fixtures depreciation Accrued litigation expenses Taxable income Current tax expense (21%)

(250) 500

(9,000) (3,000) 7,000 $1,250 $ 263

55. Powered by Cognero

Page 18


Name:

Class:

Date:

Chapter 13: Taxes in the Financial Statements $ 1,260 (53) 105 1,312

Tax on book income at statutory rate Tax-exempt income Nondeductible entertainment expense Provision for income tax expense

56. Income Tax Expense (provision), Current Year

% 21.00 (0.88) 1.75 21.87

$90,000

Valuation Allowance 57. Valuation Allowance

$90,000

$20,000

Income Tax Expense (provision), Current Year

$20,000

58. Temporary differences are caused by income and expenses appearing in both the financial statement and tax return, but in different periods (i.e., a timing difference). Permanent differences are caused by items appearing in the financial statement or the tax return, but not both. Examples of temporary differences include the following. ∙

Cost recovery on fixed assets.

Compensation related expenses where under GAAP, corporations must accrue the future expenses related to providing postretirement benefits other than pension (e.g., health insurance coverage), but these expenses are deductible for tax purposes only when paid.

Warranty expenses accrued for book purposes but not deductible for tax purposes until incurred.

Inventory write-offs accrued for book but not deductible for tax until incurred.

Goodwill is not amortizable for book purposes unless and until impairment is recorded.

Examples of permanent differences include the following. ∙

Municipal bond interest income.

The disallowed portion of business meals expense.

Certain penalties.

Tax credits.

59. To determine whether a valuation allowance is required, both positive and negative evidence must be evaluated. The key issue is whether the taxpayer will generate sufficient income (and tax liability) in the future to use the deferred tax asset before any benefits expire. Examples of negative evidence (i.e., evidence suggesting that the deferred tax asset will Powered by Cognero

Page 19


Name:

Class:

Date:

Chapter 13: Taxes in the Financial Statements not be realized) include: ∙

History of losses.

Expected future losses.

Short tax credit carryback/carryforward periods.

Adverse tax and legal results such as the expiration of a patent or trademark.

History of tax credits expiring unused.

Examples of positive evidence (i.e., support for realizing the current benefit of future tax savings) include: ∙

Strong earnings history.

Existing contracts.

Unrealized appreciation in assets.

Sales backlog of profitable orders.

Turnaround of temporary differences that produce future taxable income.

60. Companies may benchmark their tax situation to other companies within the same industry or to their own previous tax years. The starting point for a benchmarking exercise is the data from the income tax rate reconciliation. When comparing effective tax rates it is important to consider which components of the effective rate produce one-time effects and which components represent structural (long-lasting) effects. In addition to comparing effective tax rates, companies can compare levels of deferred tax assets and liabilities. 61. In most cases, the CFO would prefer the permanent difference because that approach would reduce Bryden’s effective tax rate for the year, probably increasing its share price and the CFO’s possibility for a performance bonus. The stock analyst would prefer the strategy employing a permanent difference as it generates a $1 million overall increase in cash flows rather than a deferral of an expense. However, an analyst would like even more a plan that generates sustainable tax reductions, not one-time refunds. 62. The two may differ if the company operates in a foreign country or in a state or local jurisdiction that

imposes an income tax. Income tax expense reported on a corporation’s financial statements is the combination of Federal, state, local, and foreign income taxes. The two may also differ because financial reporting requires an inter-period allocation of tax expense, recognizing the tax consequences of all items reflected in current-year financial income regardless of when they are reflected in taxable income. While the tax reflected on the current-year tax return is captured in current tax Powered by Cognero

Page 20


Name:

Class:

Date:

Chapter 13: Taxes in the Financial Statements expense, the tax consequences related to items included in current-year book income but in the taxable income of another (i.e., book-tax temporary differences) are nonetheless captured in deferred tax expense. The two may also differ because of the different standards that must be met before a tax benefit may be recognized. A corporation may be unable to recognize in its financial statements tax saving legally taken on its tax return that nonetheless fail to meet the criteria to be recognized for financial accounting purposes (i.e., unrecognized tax benefits due to uncertain tax position). Finally, the two may differ due to the intra-period allocation of tax expense for financial reporting purposes. Any tax expense or benefit attributable to items included in comprehensive income but not in income from operations must be netted with those items when reported on the income statement. Therefore, what is reported as tax expense in the financial statements may omit the tax consequences of some items included in taxable income but not in pre-tax book income.

Powered by Cognero

Page 21


Name:

Class:

Date:

Chapter 14: Exempt Entities True / False 1. A church is one of the types of exempt organizations. a. True b. False 2. All organizations that are exempt from Federal income tax are "charities." a. True b. False 3. An exempt entity is not subject to any Federal income tax. a. True b. False 4. To satisfy the “not-for-profit” requirement for exempt status, the entity may not be engaged in a trade or business. a. True b. False 5. General requirements for exempt status include the organization serving the common good and the organization being a not-for-profit entity. a. True b. False 6. A church can elect to make a limited amount of lobbying activities. a. True b. False 7. The League of Women Voters is a § 501(c)(3) organization. a. True b. False 8. Section 501(c)(3)-exempt organizations are permitted to engage in substantial lobbying activities. a. True b. False 9. Relative to the election to engage in limited lobbying activities, grassroots expenditures are made to influence the opinions of voting legislators. a. True b. False 10. An intermediate sanction in the form of an excise tax imposed by the IRS on an exempt organization is a more stringent sanction than the revocation of exempt status. a. True b. False 11. A feeder organization is exempt from Federal income taxation because it carries on a trade or business for the benefit Powered by Cognero

Page 1


Name:

Class:

Date:

Chapter 14: Exempt Entities of an exempt organization and remits its profits to the exempt entity. a. True b. False 12. Theater, Inc., an exempt organization, owns a media company, Printers, Inc., that remits 85% of its profits (i.e., taxable income of $100,000) to Theater. Since Printers remits at least 85% of its profits to Theater, neither Theater nor Printers must pay Federal income tax on this $85,000. a. True b. False 13. The tax consequences to a donor of making a charitable contribution to a private foundation may be less favorable than the tax consequences to a donor of making a charitable contribution to a public charity. a. True b. False 14. To satisfy the broadly supported provision to avoid classification as a private foundation, an exempt organization must satisfy an internal support test. Under the internal support test, more than one-third of the exempt organization’s support for the taxable year must come from gross investment income and unrelated business taxable income. a. True b. False 15. Excise taxes, such as the tax on self-dealing and the tax on excess business holdings, are imposed on exempt organizations classified as private foundations. These taxes are not imposed on exempt organizations classified as public charities. a. True b. False 16. Some of the excise taxes on private foundations may be imposed on both the private foundation and the foundation management. a. True b. False 17. The purpose of the excise tax imposed on a private foundation for failure to distribute sufficient levels of income is to motivate the foundation to distribute more of its income to outside recipients. a. True b. False 18. The unrelated business income tax (UBIT) is calculated by multiplying unrelated business taxable income by the appropriate Federal income tax rate. a. True b. False 19. Federal agencies are not subject to the unrelated business income tax (UBIT). a. True b. False 20. For purposes of the unrelated business income tax (UBIT), a trade or business consists of any activity conducted for Powered by Cognero

Page 2


Name:

Class:

Date:

Chapter 14: Exempt Entities the production of income through the sale of merchandise or from the performance of services on which profits have been earned during at least three of the five previous tax years. a. True b. False 21. The trade or business of selling merchandise where substantially all of the merchandise has been received as contributions or gifts is not subject to the unrelated business income tax. a. True b. False 22. If an exempt organization conducts a trade or business that is regularly carried on by the organization and the business relates to the organization's exempt purpose, the organization is subject to the unrelated business income tax (UBIT). a. True b. False 23. For an exempt organization to be subject to the unrelated business income tax, the trade or business must not be substantially related to the exempt purpose of the organization. a. True b. False 24. A corporate sponsorship payment that is contingent on attendance at a sporting event increases the recipient's unrelated business income. a. True b. False 25. The income from a bingo game or a casino game conducted by an exempt organization may be unrelated business income. a. True b. False 26. Revenue generated by an exempt organization from the distribution of low-cost items to its donors is not income from an unrelated trade or business. a. True b. False 27. If an exempt organization sells or rents its donor or membership list to another exempt organization, such trade or business is an unrelated trade or business. a. True b. False 28. If personal property is leased with real property and more than 45% of the rent income under the lease is from personal property, all of the rent income is subject to the unrelated business income tax. a. True b. False 29. If the unrelated business income of an exempt organization is $25,000 or less, the unrelated business income tax (UBIT) is $0. Powered by Cognero

Page 3


Name:

Class:

Date:

Chapter 14: Exempt Entities a. True b. False 30. Debt-financed property consists of all real property of a tax-exempt organization on which there is a mortgage. a. True b. False 31. Only certain exempt organizations must obtain IRS approval to obtain exempt status. a. True b. False 32. Even though a church is not required to obtain IRS approval of its exempt status, it still must annually file Form 990. a. True b. False 33. The due date for the Exempt Organization Business Income Tax Return (Form 990-T) is the fifteenth day of the third month after the end of the taxable year. a. True b. False 34. All exempt organizations that are subject to unrelated business income tax must file Form 990-T (Exempt Organization Business Income Tax Return). a. True b. False 35. A § 501(c)(3)-exempt organization (excluding churches and private foundations) must make its current Form 990 available to the general public. a. True b. False Multiple Choice 36. Which of the following is not an example of an exempt organization? a. Public high school. b. Private high school. c. Farmers' cooperative. d. Stock exchange. 37. Which of the following attributes are associated with exempt organizations? a. The organization serves some type of common good. b. The organization is not a for-profit entity. c. Net earnings do not benefit the members of the organization. d. All of these statements are true. 38. Garden, Inc., a qualifying § 501(c)(3) organization, incurs lobbying expenditures of $210,000 during the taxable year. Exempt purpose expenditures are $900,000. If Garden makes the election under § 501(h) to make lobbying expenditures Powered by Cognero

Page 4


Name:

Class:

Date:

Chapter 14: Exempt Entities on a limited basis, its tax liability resulting from the lobbying expenditures is: a. $0. b. $12,500. c. $40,000. d. $50,000. 39. Which of the following statements regarding intermediate sanction via excise taxes is correct? a. Intermediate sanctions are self-assessing (i.e., calculated and paid by the taxpayer, not imposed by the IRS). b. The excise tax is imposed on the exempt organization and on disqualified persons. c. Both a first-level tax and a second-level tax may apply. d. The corporate Federal income tax rates apply in calculating the amount of the tax liability. 40. A § 501(c)(3) organization that otherwise would be classified as a private foundation can avoid such a classification if it satisfies: a. Only an external support test. b. Only an internal support test. c. Both an external support test and an internal support test. d. An external support test, an internal support test, and a good faith test. 41. Blue, Inc., receives its support from the following sources. Governmental unit A for services rendered $18,000 General public for services rendered 25,000 Gross investment income 8,000 Contributions from individual disqualified persons 19,000 Which of the following statements is correct? a. Blue is a private foundation because it satisfies the external support test and fails the internal support test. b. Blue is not a private foundation because it fails both the internal and external support tests. c. Blue is a private foundation because it satisfies both the external support test and the internal support test. d. Blue is not a private foundation because it satisfies both the external support test and the internal support test. 42. Which of the following statements is correct? a. A private foundation is, in general, exempt from Federal income tax. b. A private foundation may be subject to certain types of Federal income tax. c. If the organization satisfies two support tests, an exempt organization that otherwise would be classified as a private foundation is instead classified as a public charity. d. Choices a., b., and c. are correct. 43. Which of the following excise taxes are imposed on private foundations? a. Tax on failure to distribute income. b. Tax on excess business holdings. c. Tax on excess charitable contributions. d. Only a. and b. 44. Which of the following taxes can be imposed on private foundations? a. Tax on self-dealing. Powered by Cognero

Page 5


Name:

Class:

Date:

Chapter 14: Exempt Entities b. Tax on failure to distribute income. c. Tax on excess business holdings. d. All of these. 45. Teal, Inc., is a private foundation that failed to distribute an adequate amount of income. Which of the following statements is correct? a. An excise tax in the form of an initial tax at the rate of 10% may be imposed on Teal. b. An excise tax in the form of an initial tax at the rate of 5% may be imposed on the foundation manager. c. An excise tax in the form of an additional tax at the rate of 100% may be imposed on Teal. d. An excise tax in the form of an additional tax at the rate of 50% may be imposed on the foundation manager. 46. Which of the following statements regarding unrelated business income tax is not correct? a. Unrelated business income is income from activities not related to the exempt purpose of the organization. b. Unrelated business income tax is levied because the exempt organization is engaging in substantial commercial activities unrelated to its exempt purpose. c. If unrelated business income tax were not levied, taxable organizations would be placed at a substantial disadvantage when trying to compete with the exempt organization. d. A flat 30% tax rate is applied to unrelated business taxable income. 47. Which of the following statements is correct regarding unrelated business income tax (UBIT)? a. To be subject to UBIT, the exempt organization must conduct a trade or business, the trade or business is not substantially related to the exempt purpose of the organization, and the trade or business is regularly carried on by the organization. b. To be subject to UBIT, the exempt organization must conduct a trade or business, the trade or business must be substantially related to the exempt purpose of the organization, and the trade or business must be regularly carried on by the organization. c. To be subject to UBIT, the exempt organization must conduct a trade or business, the trade or business is not substantially related to the exempt purpose of the organization, and the trade or business is carried on during more than half the year. d. An exempt entity that conducts a business that competes with for-profit businesses automatically is subject to UBIT. 48. Third Church operates a gift shop in its parish house. The total income of the church is $800,000. Of this amount, $300,000 comes from offerings and $500,000 comes from the net income of the gift shop. The gift shop operations are conducted by six full-time, paid employees. Which of the following statements is correct? a. $800,000 is unrelated business income. b. The $500,000 of gift shop net income is unrelated business income. c. The $300,000 is unrelated business income because the gift shop is a feeder organization. d. None of the $800,000 is unrelated business income. 49. Which of the following statements regarding unrelated business income tax is correct? a. Churches are subject to unrelated business income tax. b. Bingo games are not subject to unrelated business income tax if they are conducted by an exempt organization. c. The exchange or rental of membership lists with other exempt and nonexempt organizations is not an unrelated trade or business. d. All of these statements are correct. Powered by Cognero

Page 6


Name:

Class:

Date:

Chapter 14: Exempt Entities 50. Which of the following are qualified corporate sponsorship payments? a. The amount of the advertiser's payment to the exempt organization is contingent on the attendance at one or more events. b. The corporation's payment is determined by the number of likes on the exempt organization's Facebook page for the advertiser's products. c. The advertiser's payment to the exempt organization results in its corporate logo appearing in the exempt organization’s daily Instagram post. d. Only b. and c. 51. Which of the following requirements must be satisfied for a bingo game to be classified as not being an unrelated trade or business? a. The bingo game is legal under both state and local laws. b. The bingo game is conducted by volunteers. c. For-profit bingo games ordinarily are not permitted in the jurisdiction. d. Only a. and c. must be satisfied. 52. Which of the following statements regarding the distribution of low-cost articles by an exempt entity is correct? a. The distribution of low-cost articles can be classified as not being an unrelated trade or business. b. For 2024, a low-cost article is one that costs the entity $13.20 or less. c. Any contributions received that trigger the distribution of low-cost articles is included in unrelated business income. d. Only a. and b. are correct. 53. Tan, Inc., a tax-exempt organization, records $65,000 of current year net unrelated business income. Total charitable contributions by Tan total $7,500. Assuming that the $7,500 was deducted for these gifts in calculating net unrelated business income, what is Tan’s unrelated business taxable income? a. $65,000 b. $65,250 c. $66,000 d. $72,500 54. Maroon, Inc., a tax-exempt organization, leases a building and equipment to Brown Partnership. The rental income from the building is $100,000 and from the equipment is $9,000. Rental expenses are $40,000 for the building and $4,000 for the equipment. What adjustment must be made to the net unrelated business income? a. $-0b. ($60,000) c. ($65,000) d. ($109,000) 55. For purposes of unrelated business income tax (UBIT), land that is acquired by an exempt organization for later exempt use is excluded from debt-financed property if certain requirements are satisfied. Which of the following is not such a requirement? a. The principal purpose of acquiring the land is for use (substantially all) in achieving the organization’s exempt purpose. b. The fair market value of the land is not over 50% of the fair market value of land presently owned by the Powered by Cognero

Page 7


Name:

Class:

Date:

Chapter 14: Exempt Entities exempt organization. c. The use of the land by the exempt organization will begin within 10 years of the acquisition date. d. At the date the land is acquired, the land is located in the neighborhood of other exempt-purpose property of the organization. 56. Which of the following is a requirement that will enable mortgaged land acquired by an exempt organization for later exempt use to be excluded from debt-financed property, for purposes of unrelated business income tax? a. The principal purpose of acquiring the land is for investment. b. The land is used within 10 years of the acquisition date in the organization’s exempt purpose. c. The land is located in the United States. d. Only a. and b. are correct. 57. Which of the following exempt organizations are required to file Form 990? a. Federal agencies. b. Churches. c. Exempt organizations whose annual gross receipts do not exceed $50,000. d. None of these entities must file Form 990. 58. Which of the following statements is/are correct? a. If an exempt organization records annual gross receipts of less than $50,000, it files Form 990-N. b. Private foundations must file Form 990-PF (Return of Private Foundation). c. An exempt organization with less than $250,000 in gross receipts may file Form 990-EZ. d. Only a. and b. are correct. 59. Which of the following statements regarding exempt organization filing requirements is incorrect? a. A church is required to file Form 990 (Return of Organization Exempt from Income Tax) only if its annual gross receipts exceed $50,000. b. The due date for Form 990 (Return of Organization Exempt from Income Tax) is the fifteenth day of the fifth month after the end of the taxable year, whereas for private foundations the due date for Form 990-PF (Return of Private Foundation) is the fifteenth day of the fourth month after the end of the tax year. c. Exempt organizations whose annual gross receipts do not exceed $100,000 can file an e-Postcard (Form 990N). d. Choices a., b., and c. are incorrect. 60. Which of the following statements is correct regarding the disclosure of tax documents by exempt entities? a. Posting the required tax forms on the Internet is an acceptable technique for satisfying the “widely available" requirement. b. Forms 990 and 1023 must be readily available to the general public. c. Copies of the three most recent Forms 990 must be made available to the public. d. Choices a., b., and c. are all correct. 61. Both of the organizations listed below are exempt from Federal income tax. A donor can claim a charitable contribution for a gift to: a. The Dubuque Art Museum. b. The Buffalo VFW (Veterans of Foreign Wars) post. Powered by Cognero

Page 8


Name:

Class:

Date:

Chapter 14: Exempt Entities c. Both a. and b. d. Neither a. nor b. 62. A museum’s lobbying nontaxable amount is $600,000. Its grass roots nontaxable amount is: a. $600,000. b. $300,000. c. $150,000. d. $0. 63. Ballet’s lobbying nontaxable amount is $600,000. Its lobbying expenditures ceiling is: a. $0. b. $600,000. c. $900,000. d. $1,200,000. 64. To provide the greatest Federal income tax benefits for its donors, Orchestra would prefer to be a: a. Public charity. b. Private foundation. c. C corporation. d. Feeder organization. 65. By default, an exempt entity is a: a. Public charity. b. Private foundation. c. Private charity. d. Feeder organization. 66. A private foundation can be subject to an excise tax on: a. Making jeopardizing investments. b. Engaging in self-dealing. c. Both a. and b. d. Neither a. nor b. 67. The Jimenez Group, a private foundation, has been found guilty of self-dealing. To minimize its Federal excise taxes from this activity, Jimenez should: a. Surrender its tax-exempt status. b. Reorganize as a C corporation. c. Pay the second-level excise tax immediately. d. Undo (i.e., “correct”) the violation immediately. 68. Hospital wants to avoid the UBIT on its gift shop sales. To the extent possible, Hospital should: a. Sell on-site flowers and other items that will cheer up its patients. b. Sell through its website inspirational books and music to customers around the world. c. Both a. and b. Powered by Cognero

Page 9


Name:

Class:

Date:

Chapter 14: Exempt Entities d. Neither a. nor b. 69. Riverside Show Choir reports $20,000 of sales of pizzas at the local high school ballgames this year. Related expenses total $6,000. UBTI equals: a. $-0-. b. $13,000. c. $14,000. d. $20,000. 70. Form 990-N: a. Is filed by newly exempt entities. b. Records the asset holdings and liabilities of the exempt organization. c. Is also known as the e-Postcard. d. No longer exists. 71. Gallery, a public charity, reports annual gross receipts of $100,000 and has total assets of $425,000. It should file: a. Form 990. b. Form 990-PF. c. Form 990-N (e-Postcard). d. Form 990-EZ. 72. School has been paying UBIT every year. To reduce or eliminate this year’s tax liability, School might: a. Relate the profit-making activities to its educational purpose. b. Conduct the profit-making activities on a regular basis. c. Both a. and b. d. Neither a. nor b. 73. School has been paying UBIT every year. To reduce or eliminate this year’s tax liability, school might: a. Accelerate gross income from the profit-making activity into the current tax year. b. Accelerate deductions from the profit-making activity into the current tax year. c. Both a. and b. d. Neither a. nor b. Matching Give an example of the indicated types of exempt organizations. a. League of Women Voters. b. Teachers’ union. c. American Plywood Association. d. Six Flags over Texas theme park. e. Salvation Army. 74. § 501(c)(3) organization 75. § 501(c)(4) civic league Powered by Cognero

Page 10


Name:

Class:

Date:

Chapter 14: Exempt Entities 76. § 501(c)(5) labor organization 77. § 501(c)(6) business league 78. Not an exempt organization For each of the following taxes that are imposed on private foundations and/or foundation managers, match the appropriate initial tax or additional tax. a. 10% initial tax and 25% additional tax on private foundation. b. 30% initial tax and 100% additional tax on private foundation. c. 10% initial tax and 200% additional tax on private foundation. d. 10% initial tax and 100% additional tax on disqualified person. e. 10% initial tax and 200% additional tax on the disqualified person. 79. Tax on self-dealing 80. Tax on failure to distribute adequate amounts of income 81. Tax on excess business holdings 82. Tax on jeopardizing investments Match the following statements. a. May be subject to some Federal income taxation, and classification may adversely affect the amount of charitable contributions received. b. Tax imposed for engaging in transactions with disqualified persons. c. Tax imposed for making investments that are too risky. d. Tax imposed on investments that enable a private foundation to control unrelated for-profit businesses. 83. Private foundation 84. Tax on self-dealing 85. Tax on excess business holdings Match the following statements. a. Exempt from tax on unrelated business. b. Inappropriate definition. c. Exempt organization may be subject to the tax on unrelated business income. d. Annual information return of an exempt organization that is required to file a return and which is not a private foundation. e. Appropriate definition. f. Annual information return of a private foundation. 86. The trade or business is not substantially related to the exempt purpose of the organization. 87. The trade or business consists of selling merchandise, and substantially all of the merchandise has been received as Powered by Cognero

Page 11


Name:

Class:

Date:

Chapter 14: Exempt Entities gifts or contributions to the entity. 88. Unrelated business income is generally that derived from the unrelated trade or business reduced by the deductions directly connected with the conduct of the unrelated trade or business. 89. Debt-financed income is the net income from leveraged assets that produce UBI. 90. Form 990. 91. Form 990-PF. Match the following tax forms. a. Return of Private Foundation. b. Application for Recognition of Exemption under § 501(c)(3). c. Return of Organization Exempt from Income Tax. d. Return of organization reporting UBI. 92. Form 990 93. Form 990-PF 94. Form 1023 95. Form 990-T Match the following statements. a. Distribution of such items to donors is not considered an unrelated trade or business if they are "low-cost items". b. Is considered an unrelated trade or business if the amount received is contingent upon the level of attendance at one or more events c. Is considered an unrelated trade or business if legally it can be conducted by commercial (for-profit) entities. d. A trade or business that consists of either renting or exchanging these with another exempt organization is not an unrelated trade or business. 96. Bingo games 97. Corporate sponsorship payments 98. Branded tote bags 99. Membership lists Subjective Short Answer 100. Miracle, Inc., is a § 501(c)(3) organization involved in medical research. Based on its expectation that proposed legislation will adversely affect the funding supporting its mission, Miracle hires a lobbyist to work in Washington to represent its views. Miracle is eligible for and thus makes the § 501(h) election. Its exempt purpose expenditures are $500,000. The total lobbying expenditures for the year were $115,000. Is Miracle in danger of losing its tax exemption? Why or why not? Calculate Miracle’s tax on excess lobbying expenditures. Powered by Cognero

Page 12


Name:

Class:

Date:

Chapter 14: Exempt Entities 101. Help, Inc., a tax-exempt organization, incurs lobbying expenses of $275,000 during the tax year. Help makes the § 501(h) lobbying expenditure election. During the year, Help spends $1,200,000 carrying out its exempt purpose. a.

Will the lobbying expense result in Help losing its exempt status?

b.

Calculate the amount of any tax that Help must pay associated with its lobbying expenses.

102. Restful, Inc., a § 501(c)(3) exempt organization, hires a registered lobbyist to promote its position on pending legislation. For the year, its lobbying expenses are $100,000. Restful makes the § 501(h) election. The lobbying nontaxable amount is $90,000. a.

Will the lobbying expenses result in Restful losing its exempt status?

b.

Calculate the amount of any tax that Restful must pay associated with the lobbying expenses.

103. Medical, Inc., a § 501(c)(3) exempt organization, engages in an excess benefit transaction such that the intermediate sanction rules may apply. The amount of the excess benefit is $50,000. For the organization management, the participation in the excess benefit transaction was not willful and was supported by reasonable cause. Calculate the amount of the excise tax (first-level tax only) imposed under the intermediate sanctions provision. 104. All of the stock of Hot Dog, Inc., a fast-food franchise operating in nine southeastern states, is owned by Winsome America, Inc., a § 501(c)(3) organization. The stock was received last year as an inheritance from Rob, the entrepreneur who founded the chain. During the current year, Hot Dog reports profits before taxes and taxable income of $8 million. Hot Dog distributes $5 million to its parent, as required under the bequest, and it retains the balance for expansion purposes. a. What are the tax consequences to Hot Dog and to Winsome America?

b. How would your answer in a. change if Hot Dog distributes $8 million to Winsome, rather than $5 million? 105. Assist, Inc., a § 501(c)(3) organization, receives the following sources of support during the tax year.

General public for services rendered Governmental unit A for services rendered Governmental unit B for services rendered Governmental unit C for services rendered Gross investment income Contributions from individual disqualified persons Is Assist, Inc., classified as a private foundation? Explain.

$22,000 30,000 3,000 8,000 19,000 18,000

106. Spirit, Inc., a § 501(c)(3) organization, is classified as a private foundation. It reports investment income of $175,000. Calculate Spirit’s tax on its investment income. 107. Well, Inc., a private foundation, makes a speculative investment of $400,000 that puts the foundation's assets at excessive risk. Calculate the tax on jeopardizing investments. Well Inc. made corrective action so that a second-level tax does not apply. Powered by Cognero

Page 13


Name:

Class:

Date:

Chapter 14: Exempt Entities 108. First Americans, Inc., a § 501(c)(3) organization, operates a museum that depicts the lives of a tribe of Native Americans. It charges an admission fee but also finances its operations through endowment income, contributions, and gift shop sales. The gift shop is operated by 50 volunteers, and the museum is operated by 15 employees. The entity's revenue by source is: Admission fees Endowment income Gift shop net income Contributions

$700,000 75,000 300,000 100,000

a.

Determine the amount of First Americans’ unrelated business income (UBI).

b.

Determine the amount of First American’s unrelated business income tax (UBIT).

109. Rattler, Inc., an exempt organization, trains individuals with disabilities to design smartphone apps. Rather than hold a traditional graduation exercise, the graduates compete in an app design contest. Such activities are held four times each year. An admission fee of $10 is charged to the general public to attend the contest awards ceremony. Eight hundred people attended the ceremony this year, and prizes of $2,000 were given to the three graduates with the highest app ratings. Calculate the amount of Rattler’s unrelated business income from this activity. 110. The Dispensary is a pharmacy that is part of a § 501(c)(3) hospital. Its primary mission is to distribute medicines for hospital patients. In addition, the pharmacy dispenses medicines to former hospital patients for a period of up to 30 days after discharge from the hospital. It does this for the dual purpose of convenience to the former patients (i.e., the closest pharmacy is six miles away), and to ensure that the former patients receive the medicines that have been prescribed for them immediately after discharge. The Dispensary carries out the policy of the hospital board that no more than 25% of its gross revenues come from former-patient medicine sales. If necessary, in December of each year, sales to former patients are curtailed to ensure compliance with this policy. Sales revenue from each of the two sources is as follows for the tax year. Medicine dispensed to hospital patients Medicine sales to former patients

$900,000 100,000

Calculate the amount of The Dispensary’s unrelated business income. 111. Midnight Basketball, Inc., an exempt entity that organizes and conducts basketball games for youths ages 18-25, receives a $20,000 contribution from Brock Shoe Company. Midnight agrees to put Brock’s logo on the cover of its monthly newsletter.

a.

Is the contribution unrelated business income to Midnight?

b.

Assume that instead of putting Brock’s logo on the cover of its newsletter, Midnight agrees to include a statement in the newsletter that youth playing in the games conducted by Midnight only wear shoes manufactured by Brock. Is the contribution unrelated business income to Midnight?

c.

Would your answer in b. change if the amount of the payment was $7,500 instead of $20,000?

112. Faith Church, a § 501(c)(3) organization, operates a bingo game two times each week to raise money to support the Powered by Cognero

Page 14


Name:

Class:

Date:

Chapter 14: Exempt Entities youth activities of the church. For the current year, net proceeds from the bingo game are $900,000. a.

b.

Determine the tax consequences for Faith Church if the bingo game is conducted in a jurisdiction where for-profit bingo games are illegal and where the bingo game is legal for exempt entities. Determine the tax consequences for Faith Church if the bingo game is conducted in a jurisdiction where bingo games are legal for both for-profit and exempt entities.

113. Watch, Inc., a § 501(c)(3) exempt organization, solicits contributions through its website. An executive who recently completed an executive MBA degree program recommends that a $10 gift card be distributed to those who make a gift. What is the effect of the gift card program on Watch’s unrelated business income? 114. Ice, Inc., a § 501(c)(3) organization, has been leasing a building to Soft, Inc., a taxable entity, for 15 years. The lease terminates in the current tax year. Ice’s adjusted basis for the building is $225,000. It sells the building to the Development Partnership, a taxable entity, for $440,000. Selling expenses are $26,400. Calculate Ice's realized gain and any effect of the sale on Ice’s UBTI.

115. Arbor, Inc., an exempt organization, leases land, building, and machinery to a tenant for a 5-year period. The rent income for the land and building is $400,000 per year and that from the related machinery is $80,000 per year. Expenses incurred by Arbor for the land and building during the year are $60,000, and those for the machinery are $36,000. Net unrelated business income, which includes the above rental income and expenses, is $800,000. Calculate Arbor’s unrelated business taxable income from these items. 116. Pearl Inc., a tax-exempt organization, leases land, building, and machinery to Purple Partnership for a 5-year period. This year, the rental income from the land and building is $100,000, with related expenses of $40,000. The rental income from the machinery is $9,000 with related expenses of $3,000. What adjustment must be made in computing any net unrelated business income from these items? Essay 117. Why are some organizations exempt from Federal income tax? 118. Are organizations that qualify for exempt organization status completely exempt from Federal income taxation? 119. Name at least three common characteristics of organizations that receive exempt status. 120. Discuss benefits for which an exempt organization may be eligible, other than exemption from Federal income tax. 121. Describe how a public charity can be eligible to make lobbying expenditures without losing its tax exemption. Be specific. 122. An eligible § 501(c)(3) organization has made the § 501(h) election to participate in lobbying on a limited basis. If the lobbying nontaxable amount is exceeded, what are the potential tax consequences to the exempt organization? 123. What are intermediate sanctions and to what types of exempt organizations do they apply? 124. Agnes is aware that a feeder organization is subject to Federal income taxation. She wonders whether an organization Powered by Cognero

Page 15


Name:

Class:

Date:

Chapter 14: Exempt Entities that is otherwise taxable as a feeder organization can avoid such status if it remits at least 80% of its profits to the § 501(c)(3) entity. 125. Define a private foundation. 126. How can an exempt organization otherwise classified as a private foundation become a public charity? 127. Discuss any negative tax consequences that result from an exempt organization being classified as a private foundation. 128. List at least three of the excise taxes imposed on private foundations. Why are they imposed? 129. Construct the tax model for unrelated business taxable income. 130. What is the purpose of the unrelated business income tax? 131. Which requirements must be satisfied for an exempt organization to be classified as an unrelated trade or business? Is being classified as an unrelated trade or business good or bad? Explain. 132. Define a qualified corporate sponsorship payment. 133. Under what circumstances are bingo games not treated as an unrelated trade or business? 134. If an exempt organization distributes “low-cost items” as an incidental part of its solicitation for charitable contributions, the activity is not considered an unrelated trade or business. a.

At what dollar amount is a distribution no longer a "low-cost item"?

b.

Provide examples of low-cost items.

135. A § 501(c)(3) organization exchanges its membership lists with another exempt organization. What are the Federal income tax consequences of such an exchange? 136. What tax forms are used to apply for exempt status? Be specific. 137. If an exempt organization is required to file an annual information return, on what form is it filed? Be specific.

Powered by Cognero

Page 16


Name:

Class:

Date:

Chapter 14: Exempt Entities Answer Key 1. True 2. False 3. False 4. False 5. True 6. False 7. False 8. False 9. False 10. False 11. False 12. False 13. True 14. False 15. True 16. True 17. True 18. True 19. True 20. False 21. True 22. False 23. True 24. True 25. True Powered by Cognero

Page 17


Name:

Class:

Date:

Chapter 14: Exempt Entities 26. True 27. False 28. False 29. False 30. False 31. True 32. False 33. False 34. False 35. True 36. d 37. d 38. b 39. c 40. c 41. d 42. d 43. d 44. d 45. c 46. d 47. a 48. b 49. a 50. c Powered by Cognero

Page 18


Name:

Class:

Date:

Chapter 14: Exempt Entities 51. d 52. d 53. b 54. c 55. b 56. b 57. d 58. d 59. d 60. d 61. a 62. c 63. c 64. a 65. b 66. c 67. d 68. a 69. b 70. c 71. d 72. a 73. b 74. e 75. a 76. b Powered by Cognero

Page 19


Name:

Class:

Date:

Chapter 14: Exempt Entities 77. c 78. d 79. e 80. b 81. c 82. a 83. a 84. b 85. d 86. c 87. a 88. e 89. b 90. d 91. f 92. c 93. a 94. b 95. d 96. c 97. b 98. a 99. d 100. Since Miracle’s lobbying nontaxable amount is $100,000 ($500,000 * 20%), its $115,000 of lobbying do not exceed the ceiling on lobbying expenses of $150,000 ($100,000 × 1.5). Thus, Miracle is not in danger of forfeiting its exempt status as a result of incurring the lobbying expenses. The excise tax on excess lobbying expenditures is calculated as follows. Powered by Cognero

Page 20


Name:

Class:

Date:

Chapter 14: Exempt Entities Lobbying expenditures Lobbying nontaxable amount Excess lobbying expenditures × 25% rate Tax on excess lobbying expenditures

$115,000 (100,000) $ 15,000 × 25% $ 3,750

101. a. No. Help is not in danger of losing its exempt status because of the lobbying expenses, because it makes the § 501(h) lobbying expenditure election and does not exceed the lobbying expenditures ceiling. 150%[$175,000 + 10%($1,200,000 – $1,000,000)] = $292,500 Help’s lobbying expenses of $275,000 are below the ceiling amount. b. Help’s tax liability of $20,000 is calculated as follows. Lobbying expenditures Lobbying nontaxable amount [$175,000 + 10%($1,200,000 – $1,000,000)] Excess lobbying expenditures Statutory rate Tax on excess lobbying expenditures

102. a.

$ 275,000 (195,000) $ 80,000 × 25% $ 20,000

Restful made the § 501(h) election to lobby to a limited extent. The permitted ceiling on lobbying expenses is $135,000 (150% × $90,000). Since Restful’s lobbying expenses of $100,000 do not exceed this amount, Restful’s exempt status is not jeopardized by the lobbying expenditures.

b.

Restful’s tax liability associated with the lobbying expenses is calculated as follows. Lobbying expenses Lobbying nontaxable amount Excess lobbying expenses Tax rate Tax on excess lobbying expenses

Powered by Cognero

$100,000 (90,000) $ 10,000 × 25% $ 2,500 Page 21


Name:

Class:

Date:

Chapter 14: Exempt Entities 103. Under the intermediate sanctions provision, the excise tax rate of 25% on the disqualified person is applied to the excess benefit. The resulting tax equals $12,500 ($50,000 × 25%). A zero amount of first-level tax is imposed on the exempt organization management. 104. a. Hot Dog is a feeder organization. Therefore, it is subject to Federal income tax on its taxable income of $8 million using the C corporation tax rate of 21%. $8 million × 21% = $1.68 million Since Winsome is an exempt entity, it is not subject to Federal income tax on the dividend distribution it receives from Hot Dog. b. The tax consequences are the same as in a. Hot Dog's corporate income tax is based on its taxable income, regardless of its distributions. 105. To not be classified as a private foundation, Assist must receive broad public support. To receive broad public support, Assist must satisfy both an external support test and an internal support test. The external support test requires that more than one-third of Assist’s support for the taxable year come from the governmental units and the general public. For this purpose, support from a governmental unit is counted only to the extent of the greater of $5,000 or 1% of the total support. Governmental unit A Governmental unit B Governmental unit C General public External support

$ 5,000 3,000 5,000 22,000 $35,000

Since the $35,000 exceeds one-third of the total support of $100,000, the external support test is satisfied. The internal support test requires that no more than one-third of the total support be from gross investment income and unrelated business taxable income (net of the related tax). The internal support test also is satisfied since $19,000 does not exceed one-third of $100,000. Therefore, Assist, Inc., is not classified as a private foundation because it is broadly supported. It is a public charity. 106. Investment income × Rate Tax on its investment income

$175,000 × 1.39% $ 2,433

107. The initial tax on the private foundation is $40,000 ($400,000 × 10%). The initial tax on the foundation manager is Powered by Cognero

Page 22


Name:

Class:

Date:

Chapter 14: Exempt Entities $40,000 ($400,000 × 10%) but limited to $10,000. 108. a. The only potential source of UBI for First Americans, Inc., is the gift shop net income of $300,000. However, since the individuals performing substantially all the work of the gift shop do so without compensation (i.e., they are volunteers), the gift shop net income of $300,000 is not classified as unrelated business income. Thus, First Americans, Inc., reports zero UBI. b.

Because First Americans has no UBI, its unrelated business income tax (UBIT) is $0.

109. The contest activity is not classified as an unrelated trade or business. To be so classified, the following characteristics must be present. ∙

The organization conducts a trade or business.

The trade or business is not substantially related to the exempt purpose of the organization.

The trade or business is regularly carried on by the organization.

Based on the data provided, neither the “not substantially related” nor the “regularly carried on” characteristic is satisfied. Thus, Rattler incurs zero unrelated business income from the contest because the contest is not an unrelated trade or business. 110. For any of the activities of The Dispensary to be classified as unrelated business income, the three following requirements must be satisfied. ∙

The organization conducts a trade or business.

The trade or business is not substantially related to the exempt purpose of the organization.

The trade or business is regularly carried on by the organization.

All of these conditions are satisfied for the sales by The Dispensary to former patients. Thus, this part of the pharmacy activity of the hospital is an unrelated trade or business and the unrelated business gross income is $100,000. The 25% policy of the hospital board is not relevant in classifying part of the pharmacy as an unrelated trade or business. 111. a. No. The $20,000 payment is a qualified corporate sponsorship payment. Thus, it is not unrelated business income to Midnight. b.

Yes. The $20,000 payment does not satisfy the requirements for a qualified sponsorship payment because of the endorsement statement. Thus, it is unrelated business income to Midnight.

c.

No. The amount of the payment is not relevant to its classification as a sponsorship payment.

112. a. In this case, the $900,000 is exempt from Federal income taxation because both of the Powered by Cognero

Page 23


Name:

Class:

Date:

Chapter 14: Exempt Entities following requirements are satisfied.

b.

The bingo game is legal under both state and local law.

Commercial bingo games (conducted for a profit motive) ordinarily are not permitted in the jurisdiction.

In this case, the Federal income tax liability to Faith Church is $189,000 ($900,000 × 21%). Since commercial bingo games are permitted in the jurisdiction, the proceeds of $900,000 from the bingo game sponsored by Faith Church are subject to Federal income taxation as unrelated business income.

113. The gift cards are low-cost articles (permitted indexed amount for 2024 is $13.20). There is no effect on Watch’s unrelated business income, since the distribution of the gift cards is not an unrelated trade or business. 114. Amount realized ($440,000 – $26,400) Adjusted basis Realized gain Recognized gain (UBI)

$ 413,600 (225,000) $ 188,600 $ –0–

None of the realized gain of $188,600 is unrelated business income. Asset sales of this sort are not UBI.

115. Net unrelated business income Net rent income from land and building ($400,000 – $60,000) Unrelated business taxable income

$800,000 (340,000) $460,000

The net rent income from the land and building is a negative adjustment in computing UBTI. The net rent income from the machinery does not qualify as a negative adjustment because it is not incidental (i.e., $80,000 gross rental income is greater than 10% of $480,000 total gross rental income). 116. The net rental income of $66,000 ($60,000 + $6,000) from the land and building ($100,000 – $40,000) and the machinery ($9,000 – $3,000) is not included in calculating unrelated business taxable income. This amount is a negative adjustment in computing Pearl's UBTI. The personal property (machinery) is leased with the real property (land and building), and the personal property rental income ($9,000) is not greater than 10% of the total gross rental income of $109,000 ($100,000 + $9,000) under the lease. 117. The Code’s treatment of exempt organizations is based on a variety of objectives. While the major objective of the Code is to raise revenue, social, economic, and political objectives also are present. Social objectives are served by the provision of exempt status for organizations who serve the general welfare, and by so doing, may reduce the financial burden required of the Federal government (e.g., a charitable hospital may decrease the need for a government hospital.) 118. No. Such organizations may still be subject to Federal taxes. First, if the exempt organization engages in a prohibited Powered by Cognero

Page 24


Name:

Class:

Date:

Chapter 14: Exempt Entities transaction, it is subject to a variety of excise taxes. Second, feeder organizations are subject to Federal income tax. Third, private foundations are subject to certain excise taxes. Fourth, an exempt organization may be subject to tax on its unrelated business taxable income. 119. Many organizations that qualify for exempt status share the following characteristics: ∙

The organization serves some type of common goal.

The organization is not a for-profit entity.

Net earnings of the organization do not benefit the members of the organization.

The organization does not exert political influence.

120. In addition to being exempt from Federal income tax, an exempt organization may be eligible for the following benefits: ∙

Exemption from state income tax, state franchise tax, sales tax, and property tax.

Receive discounts on postage rates.

Donors of property to the exempt organization may qualify for charitable contribution deductions on their Federal or state income tax returns.

121. Public charities generally are not permitted to engage in lobbying activities. But certain public charities can lobby on a limited basis by making the § 501(h) election. Such lobbying expenses are subject to a ceiling. Exceeding the ceiling can lead to the forfeiture of exempt status. (Expenditures in excess of a nontaxable floor amount are subject to a 25% tax.) Churches and private foundations are not eligible to make the § 501(h) election. For those entities and for qualifying public charities that do not make the § 501(h) election, only a nonsubstantial part of the organization’s activities can include lobbying, without jeopardizing the tax exemption. 122. First, a tax is levied on lobbying expenditures that exceed the nontaxable amount. Second, lobbying expenditures could result in revocation of exempt status by the IRS if the amount exceeds a lobbying ceiling repeatedly. 123. Intermediate sanctions apply to public charities (i.e., not to private foundations). They are excise taxes imposed on individuals who are in a position to exercise substantial influence over the affairs of the exempt organization (like executives and directors) and who engage in excess benefit transactions. There is also a tax on exempt organization managers who participate in such a transaction knowing that it is improper. Intermediate sanctions provide the IRS with an option in dealing with such exempt organizations who engage in prohibited transactions (that is less drastic than revoking the entity's exempt status). 124. No. While a feeder organization carries on a trade or business for the benefit of an exempt organization and remits its profits to the exempt organization, its Federal corporate income tax liability is not related to the percentage of its profits that it remits each year. 125. By default, all exempt entities are private foundations. The following § 501(c)(3) organizations are not private foundations: Powered by Cognero

Page 25


Name:

Class:

Date:

Chapter 14: Exempt Entities a.

Churches; educational institutions; hospitals and medical research organizations; charitable organizations receiving a major portion of their support from the general public or the United States, a state, or a political subdivision thereof that is operated for the benefit of a college or university; and governmental units (favored activities category).

b.

Organizations that are broadly supported by the general public (excluding disqualified persons), by governmental units, or by organizations described in choice a.

c.

Entities organized and operated exclusively for the benefit of organizations described in choice a. or b., like a fund-raising or research affiliate [a supporting organization].

126. An exempt organization that otherwise would be classified as a private foundation can avoid private foundation status if it receives broad public support. To meet the broadly supported requirement, the exempt organization must satisfy both an external support test and an internal support test. 127. Two potential negative tax consequences can result from an exempt organization being classified as a private foundation. First, the classification may have an adverse impact on the amount of contributions received by the donee exempt organization. This occurs because the tax consequences for donors may not be as favorable as those for entities not classified as private foundations. Second, the classification may result in certain Federal excise taxes being imposed on the exempt organization. 128. The excise taxes imposed on private foundations include the following: ∙

Tax based on investment income.

Tax on self-dealing.

Tax on failure to distribute income.

Tax on excess business holdings.

Tax on investments that put the private foundation's assets at excessive risk.

The tax based on investment income effectively is an audit fee, for which the chief purpose is to defray IRS expenses. The other taxes restrict the permitted activities of private foundations. By its nature, a private foundation may be conceived or operate with a narrower definition of the common good (and therefore less broad public support) than other exempt organizations. 129. The tax model for unrelated business taxable income is as follows. Gross unrelated business income – Deductions for unrelated business income = Net unrelated business income + Adjustments (positive and negative) - $1,000 "standard deduction" = Unrelated business taxable income

Powered by Cognero

Page 26


Name:

Class:

Date:

Chapter 14: Exempt Entities 130. The unrelated business income tax is designed to treat the exempt entity as if it were subject to the corporate income tax on its unrelated business income. Without such a tax, nonexempt organizations (regular taxable business entities) would be at a substantial disadvantage when trying to compete with the exempt organization. Thus, the UBIT is intended to neutralize the exempt entity’s tax advantage. 131. If the following requirements are satisfied, an exempt organization is classified as an unrelated trade or business:

The organization conducts a trade or business.

The trade or business is not substantially related to the exempt purpose of the organization.

The trade or business is regularly carried on by the organization.

From a tax perspective, an exempt organization usually would prefer not to operate an unrelated trade or business since doing so results in the imposition of the Federal income tax. 132. A payment qualifies as a qualified sponsorship payment if it meets the following requirements: ∙

There is no arrangement or expectation that the trade or business making the payment will receive any substantial benefit other than the use or acknowledgement of its name, logo, or product lines in connection with the activities of the exempt organization.

Such use or acknowledgment does not include advertising or endorsement of the payor’s products or services.

The payment does not include any payment for which the amount is contingent on the level of attendance at one or more events, broadcast ratings, or other factors indicating the degree of public exposure to one or more events.

133. A bingo game is not an unrelated trade or business for purposes of the UBIT if the following requirements are satisfied: ∙

The bingo game is legal under both state and local law.

Commercial bingo games (conducted for a profit motive) are not permitted in the jurisdiction.

134. a. For 2024, a low-cost item is one that costs $13.20 (indexed annually) or less. b.

Examples of low-cost items include pens, tote bags, recordings, stationery, and address labels.

Powered by Cognero

Page 27


Name:

Class:

Date:

Chapter 14: Exempt Entities 135. If an exempt organization conducts a trade or business that consists of either exchanging with or renting the organization’s donor or membership list to other exempt organizations, the activity is not an unrelated trade or business. 136. An organization that is exempt under § 501(c)(3) uses Form 1023 or Form 1023-EZ to apply for exempt status. Form 1024 is used by most other types of exempt organizations to apply for exempt status. 137. Exempt organizations that are not private foundations file Form 990. Forms 990-N and 990-EZ might be available for smaller entities. Private foundations file Form 990-PF. An exempt entity files Form 990-T if the entity generates UBI. The IRS requires electronic filing of all of these forms.

Powered by Cognero

Page 28


Name:

Class:

Date:

Chapter 15: Multistate Corporation Taxation True / False 1. Roughly 20% of all taxes paid by businesses in the United States are to state, local, and municipal jurisdictions. a. True b. False 2. Usually a business chooses a location where it will build a new plant based chiefly on tax considerations. a. True b. False 3. Politicians frequently use tax credits and exemptions to create economic development incentives. a. True b. False 4. All of the U.S. states have adopted a tax based on the net taxable income of corporations. a. True b. False 5. Property taxes generally are collected by local taxing jurisdictions, not the state or Federal governments. a. True b. False 6. State and local politicians tend to apply new and increased taxes to taxpayers who are nonresident visitors to the jurisdiction, such as a tax on auto rentals and hotel stays, because the taxpayer cannot vote to reelect (or oust) the lawmaker. a. True b. False 7. Most states begin the computation of corporate taxable income with an amount from the Federal income tax return. a. True b. False 8. If a state follows Federal income tax rules, the state’s tax compliance and enforcement become easier to accomplish. a. True b. False 9. A typical state taxable income subtraction modification is the interest income earned from another state’s bonds. a. True b. False 10. A typical state taxable income addition modification is for the state's NOL allowed the taxpayer for the tax year. a. True b. False 11. A state can levy an income tax on a business only if the business was incorporated in the state. a. True Powered by Cognero

Page 1


Name:

Class:

Date:

Chapter 15: Multistate Corporation Taxation b. False 12. Typical indicators of income tax nexus include the presence of customers in the state. a. True b. False 13. Under P.L. 86-272, the taxpayer is exempt from state taxes on income resulting from the mere solicitation of orders for the sale of stocks and bonds. a. True b. False 14. In most states, a taxpayer’s income is apportioned on the basis of a formula measuring the extent of business contact and allocated according to the location of property owned or used. a. True b. False 15. All of the U.S. states use an apportionment formula based on the sales, property, and payroll factors. a. True b. False 16. Double weighting the sales factor effectively decreases the corporate income tax burden on taxpayers based in a state such as entities with in-state headquarters. a. True b. False 17. An assembly worker earns a $50,000 salary and receives a fringe benefit package worth $15,000. The payroll factor assigns $65,000 for this employee. a. True b. False 18. A service engineer spends 80% of her time maintaining the employer’s productive business property and 20% maintaining the employer’s nonbusiness rental properties. This year, her compensation totaled $90,000. The payroll factor assigns $90,000 to the state in which the employer is based. a. True b. False 19. The property factor includes land and buildings used for business purposes. a. True b. False 20. The property factor includes business assets that the taxpayer owns and those used under a lease agreement. a. True b. False 21. A unitary business applies a combined apportionment formula, including data from operations of all affiliates. a. True Powered by Cognero

Page 2


Name:

Class:

Date:

Chapter 15: Multistate Corporation Taxation b. False 22. By making a water’s edge election, a multinational taxpayer can limit the reach of unitary principles to the apportionment factors and income of its U.S. and E.U. affiliates. a. True b. False 23. A unitary group of entities files a combined return that includes all of the affiliates’ income and apportionment data. a. True b. False 24. In most states, Federal S corporations must make a separate state-level election of the flow-through status. a. True b. False 25. S corporations flow through income amounts to its shareholders, and most states require a withholding of shareholder taxes on the allocated amounts. a. True b. False 26. An LLC apportions and allocates its annual taxable income in the same manner used by any other business operating in the state. a. True b. False 27. Almost all of the states assess some form of consumer-level sales/use tax. a. True b. False 28. The use tax is designed to complement the sales tax. A use tax typically covers purchases made out of state and brought into the jurisdiction. a. True b. False 29. A taxpayer automatically has nexus with a state for sales and use tax purposes if it has income tax nexus with the same state. a. True b. False 30. Most states’ consumer sales taxes are to be paid by the final purchaser of the taxable asset. a. True b. False 31. Typically included in the sales/use tax base is the purchase of tablet computers and cell phone equipment by a large manufacturing firm whose sales force uses the items. a. True Powered by Cognero

Page 3


Name:

Class:

Date:

Chapter 15: Multistate Corporation Taxation b. False 32. Typically exempt from the sales/use tax base is the purchase of prescription medicines by an individual. a. True b. False 33. Typically exempt from the sales/use tax base is the purchase of lumber by a do-it-yourself homeowner when she builds a deck onto her patio. This exemption is known as the homestead rule. a. True b. False 34. Typically exempt from the sales/use tax base is a symphony orchestra's purchase of printed music for its musicians. Symphonies are organized as tax-exempt organizations. a. True b. False 35. Most states exempt consumer purchases of groceries from the collection of the local sales tax. a. True b. False 36. Typically exempt from the sales/use tax base is the purchase of tools by a manufacturer to make the widgets that it sells. a. True b. False 37. Typically exempt from the sales/use tax base is the purchase of clothing from a neighbor’s garage sale. a. True b. False 38. The individual seller of shares of stock in Netflix is liable for sales tax on the transaction. a. True b. False 39. The typical state sales/use tax falls on sales of both real and personal property. a. True b. False 40. Sales/use tax in most states applies to a restaurant meal. a. True b. False 41. In most states, legal and accounting services are exempt from the sales/use tax base. a. True b. False 42. A capital stock tax usually is structured as an excise tax imposed on a corporation’s net worth, using financial Powered by Cognero

Page 4


Name:

Class:

Date:

Chapter 15: Multistate Corporation Taxation statement data to compute the tax. a. True b. False 43. The typical local property tax falls on both an investor’s principal residence and her stock portfolio. a. True b. False 44. A city might assess a recording tax when a business takes out a mortgage on its real estate. a. True b. False Multiple Choice 45. Adams Corporation owns and operates two manufacturing facilities, one in State X and the other in State Y. Due to a temporary decline in the corporation’s sales, Adams has rented 20% of its Y facility to an unaffiliated corporation. Adams generated $1,000,000 net rental income and $5,000,000 income from manufacturing. Adams is incorporated in Y. For X and Y purposes, rental income is classified as allocable nonbusiness income. By applying the statutes of each state, Adams determined that its apportionment factors are 0.65 for X and 0.35 for Y. Adams’s income attributed to X is: a. $0. b. $3,250,000. c. $3,900,000. d. $5,000,000. 46. Flint Corporation is subject to a corporate income tax only in State X. The starting point in computing X taxable income is Federal taxable income which is $750,000. This amount includes a $50,000 deduction for state income taxes. During the year, Flint received $10,000 interest on Federal obligations. X tax law does not allow a deduction for state income tax payments. Flint’s taxable income for X purposes is: a. $810,000 b. $800,000 c. $790,000 d. $750,000 47. Ramirez Corporation, which is subject to income tax only in State A, generated the following income and deductions. Federal taxable income State A income tax expense Depreciation allowed for Federal tax purposes Depreciation allowed for state tax purposes

$500,000 45,000 300,000 250,000

Federal taxable income is the starting point in computing A taxable income. State income taxes are not deductible for A tax purposes. Ramirez’s A taxable income is: a. $495,000 b. $500,000 Powered by Cognero

Page 5


Name:

Class:

Date:

Chapter 15: Multistate Corporation Taxation c. $545,000 d. $595,000 48. In determining a corporation’s taxable income for state income tax purposes, which of the following does not constitute a subtraction modification from Federal income? a. Interest on U.S. obligations. b. Expenses that are directly or indirectly related to state and municipal interest that is taxable for state purposes. c. The amount by which the state depreciation deduction exceeds the corresponding Federal amount. d. The amount by which the Federal depreciation deduction exceeds the corresponding state amount. 49. In determining state taxable income, all of the following are adjustments to Federal income except: a. Federal net operating loss. b. State income tax expense. c. Fringe benefits paid to officers and executives. d. Dividends received from other U.S. corporations. 50. Zhao Company sold an asset on the first day of the tax year for $500,000. Zhao’s Federal tax basis for the asset was $300,000. Because of differences in cost recovery schedules, the state regular-tax basis in the asset was $350,000. What modification, if any, should be made to Zhao’s Federal taxable income in determining the correct taxable income for the typical state? a. $0 b. ($50,000) c. $50,000 d. $150,000 51. Federal taxable income is used as the starting point in computing the state’s income tax base, but various state adjustments or modifications generally are required to: a. Reflect differences between state and Federal tax statutes. b. Remove income that a state is constitutionally prohibited from taxing. c. Allow for all states to use the same definition of taxable income. d. Choices a. and b. 52. The model law relating to the assignment of income among the states for corporations is: a. Public Law 86-272. b. The Multistate Tax Treaty. c. The Multistate Tax Commission (MTC). d. The Uniform Division of Income for Tax Purposes Act (UDITPA). 53. Multistate income tax planning can be effective for the taxpayer because: a. Different states use different definitions of taxable income. b. State income tax rates generally are steeply progressive. c. Both a. and b. d. Neither a. nor b. Powered by Cognero

Page 6


Name:

Class:

Date:

Chapter 15: Multistate Corporation Taxation 54. In the context of Federal and state corporate income taxation: a. Many states collect their taxes using a piggyback on the Federal return. b. Most states require taxpayers to report to the state taxing agency the changes made to a return in a Federal audit. c. Both a. and b. d. Neither a. nor b. 55. Under P.L. 86-272, which of the following transactions by itself would create nexus with a state? a. Order solicitation for a plot of real estate approved and filled from another state. b. Order solicitation for a computer approved and filled from another state. c. Order solicitation for a machine with credit approval from another state. d. Providing an automobile to a salesperson. 56. Under P.L. 86-272, which of the following transactions by itself would create nexus with a state? a. Having a sales employee inspect customer’s inventory for specific product lines. b. Using a manufacturer’s representative for the taxpayer through a sales office in the state. c. Executing a sales campaign using an advertising agency acting as an independent contractor for the taxpayer. d. Maintaining inventory in the state by an independent contractor under a consignment plan. 57. Which of the following is not immune from state income taxation even if P.L. 86-272 is in effect? a. Sale of office equipment that is used in the taxpayer’s business. b. Sale of office equipment that constitutes inventory to the purchaser. c. Sale of a warehouse used in the taxpayer’s business. d. All of these are protected by P.L. 86-272 immunity provisions. 58. To determine whether an out-of-state entity has income tax nexus, some states use: a. A factor-presence test. b. An economic presence test. c. Both a. and b are used by certain states. d. Neither a. nor b is used by the states. 59. Public Law 86-272: a. Was written by the Multistate Tax Commission. b. Provides nexus definitions for sales of stocks and bonds. c. Provides nexus definitions for the sale of medical and legal services. d. Was enacted by Congress. 60. In applying the typical apportionment formula: a. The aggregate of state taxable incomes equals Federal taxable income. b. The aggregate of state taxable incomes might not equal Federal taxable income. c. When Federal taxable income is positive, all states’ taxable incomes are positive. d. When Federal taxable income is negative, aggregate state taxable incomes total to zero. Powered by Cognero

Page 7


Name:

Class:

Date:

Chapter 15: Multistate Corporation Taxation 61. Marquardt Corporation realized $900,000 taxable income from the sales of its products in States X and Z. Marquardt’s activities establish nexus for income tax purposes in both states. Marquardt’s sales, payroll, and property among the states include the following. State X State Z Totals Sales $1,000,000 $3,000,000 $4,000,000 Property 2,000,000 –0– 2,000,000 Payroll 1,000,000 –0– 1,000,000 Z utilizes an equally weighted three-factor apportionment formula. Marquardt is incorporated in X. How much of Marquardt’s taxable income is apportioned to Z? a. $0 b. $225,000 c. $675,000 d. $3,000,000 62. Generally, a taxpayer’s business income is: a. Apportioned. b. Allocated. c. Both a. and b. d. Neither a. nor b. 63. Typically, state taxable income includes: a. Apportionable income only. b. Nonapportionable income only. c. Both a. and b. d. Neither a. nor b. These are not the terms typically used in the computation. 64. The most commonly used state income tax apportionment formula is: a. Sales factor only. b. Sales factor double-weighted. c. Sales factor equally weighted with property and payroll. d. Payroll factor only. 65. In the income tax apportionment formula, market-sourcing the sales factor for sales of services means that: a. Sales are sourced to the state of the seller. b. Sales are sourced to the state of the customer. c. Sales are sourced to the state of the seller’s corporate headquarters (i.e., where the marketing department works). d. Sales are sourced to the state(s) where the seller incurred most of its related costs. 66. José Corporation realized $900,000 taxable income from the sales of its products in States X and Z. José’s activities in both states establish nexus for income tax purposes. José’s sales, payroll, and property among the states include the following. Powered by Cognero

Page 8


Name:

Class:

Date:

Chapter 15: Multistate Corporation Taxation

Sales Property Payroll

State X State Z Totals $1,500,000 $1,000,000 $2,500,000 500,000 –0– 500,000 2,000,000 –0– 2,000,000

Z utilizes a double-weighted sales factor in its three-factor apportionment formula. How much of José’s taxable income is apportioned to Z? a. $1,000,000 b. $900,000 c. $180,000 d. $0 67. José Corporation realized $900,000 taxable income from the sales of its products in States X and Z. José’s activities in both states establish nexus for income tax purposes. José’s sales, payroll, and property among the states include the following. Sales Property Payroll

State X $1,500,000 500,000 2,000,000

State Z $1,000,000 –0– –0–

Totals $2,500,000 500,000 2,000,000

X utilizes an equally weighted three-factor apportionment formula. How much of José’s taxable income is apportioned to X? a. $120,000 b. $450,000 c. $780,300 d. $900,000 68. Chipper Corporation realized $1,000,000 taxable income from the sales of its products in States X and Z. Chipper’s activities establish nexus for income tax purposes only in Z, the state of its incorporation. Chipper’s sales, payroll, and property among the states include the following. Sales Property Payroll

State X $1,000,000 200,000 100,000

State Z $2,000,000 2,300,000 1,900,000

Totals $3,000,000 2,500,000 2,000,000

X utilizes a sales-only factor in its three-factor apportionment formula. How much of Chipper’s taxable income is apportioned to X? a. $0 b. $333,333 c. $500,000 d. $1,000,000 69. Helene Corporation owns manufacturing facilities in States A, B, and C. State A uses a three-factor apportionment formula under which the sales, property, and payroll factors are equally weighted. State B uses a three-factor apportionment formula under which sales are double-weighted. State C employs a single-factor apportionment formula based solely on sales. Powered by Cognero

Page 9


Name:

Class:

Date:

Chapter 15: Multistate Corporation Taxation Helene’s operations generated $1,000,000 of apportionable income, and its sales and payroll activity and average property owned in each of the three states is as follows. Sales Payroll Property

State A $400,000 100,000 200,000

State B $800,000 150,000 200,000

State C $300,000 50,000 200,000

Totals $1,500,000 300,000 600,000

Helene’s apportionable income assigned to State A is: a. $0 b. $266,667 c. $311,100 d. $1,000,000 70. Simpkin Corporation owns manufacturing facilities in States A, B, and C. State A uses a three-factor apportionment formula under which the sales, property, and payroll factors are equally weighted. State B uses a three-factor apportionment formula under which sales are double-weighted. State C employs a single-factor apportionment formula, based solely on sales. Simpkin’s operations generated $1,000,000 of apportionable income, and its sales and payroll activity and average property owned in each of the three states is as follows. Sales Payroll Property

State A $400,000 100,000 200,000

State B $800,000 150,000 200,000

State C $300,000 50,000 200,000

Totals $1,500,000 300,000 600,000

Simpkin’s apportionable income assigned to State B is: a. $1,000,000 b. $533,333 c. $475,000 d. $0 71. Cruz Corporation owns manufacturing facilities in States A, B, and C. State A uses a three-factor apportionment formula under which the sales, property, and payroll factors are equally weighted. State B uses a three-factor apportionment formula under which sales are double-weighted. State C employs a single-factor apportionment formula based solely on sales. Cruz’s operations generated $1,000,000 of apportionable income, and its sales and payroll activity and average property owned in each of the three states is as follows. Sales Payroll Property

State A $400,000 100,000 200,000

State B $800,000 150,000 200,000

State C $300,000 50,000 200,000

Totals $1,500,000 300,000 600,000

Cruz’s apportionable income assigned to State C is: a. $1,000,000 b. $273,333 c. $200,000 Powered by Cognero

Page 10


Name:

Class:

Date:

Chapter 15: Multistate Corporation Taxation d. $0 72. Boot Corporation is subject to income tax in States A and B. Boot’s operations generated $200,000 of apportionable income, and its sales and payroll activity and average property owned in each of the states is as follows. Sales Payroll Property

State A $200,000 100,000 200,000

State B $600,000 50,000 50,000

Totals $800,000 150,000 250,000

How much more (less) of Boot’s income is subject to State A income tax if, instead of using an equally weighted threefactor apportionment formula, State A uses a formula with a double-weighted sales factor? a. ($50,000) b. $50,000 c. $16,100 d. ($16,100) 73. General Corporation is taxable in a number of states. This year, General made a $100,000 sale from its State A headquarters to the State B office of the Federal Bureau of Investigation. In which state(s) will the sale be included in the sales factor numerator? a. $0 in A and $0 in B. b. $50,000 in A with the balance exempted from other states’ sales factors under the Altria doctrine. c. $100,000 in A. d. $100,000 in B. 74. General Corporation is taxable in a number of states. This year, General made a $100,000 sale from its State A headquarters to a customer in State B. General has not established nexus with State B. State A does not apply a throwback rule. In which state(s) will the sale be included in the sales factor numerator? a. In all of the states, according to the apportionment formulas of each, as the U.S. government is present in all states. b. $100,000 in State A. c. $100,000 in State B. d. $0 in State A and $0 in State B. 75. General Corporation is taxable in a number of states. This year, General made a $100,000 sale from its State A headquarters to a customer in State B. This activity is not sufficient for General to create nexus with State B. State A applies a throwback rule but State B does not. In which state(s) will the sale be included in the sales factor numerator? a. $0 in both State A and State B. b. $100,000 in State A. c. $100,000 in State B. d. In both State A and State B, according to the apportionment formulas of each. 76. General Corporation is taxable in a number of states. This year, General made a $100,000 sale from its State A headquarters to a customer in State B. This activity is not sufficient for General to create nexus with State B. State B applies a throwback rule but State A does not. In which state(s) will the sale be included in the sales factor numerator? a. $0 in State A and $0 in State B. b. $100,000 in State A. Powered by Cognero

Page 11


Name:

Class:

Date:

Chapter 15: Multistate Corporation Taxation c. $100,000 in State B. d. In both States A and B, according to the apportionment formulas of each. 77. Britta Corporation’s entire operations are located in State A. Of Britta’s sales, 80% ($800,000) are made in State A and the remaining sales ($200,000) are made in State B, which has not adopted a corporate income tax. If State A has adopted a throwback rule, the numerator of Britta’s State A sales factor is: a. $0. b. $200,000. c. $800,000. d. $1,000,000. 78. Given the following transactions for the year, determine Comp Corporation’s D payroll factor denominator. State D has adopted the principles of UDITPA. Compensation of sales force Compensation paid to independent contractors Compensation paid to managers of nonbusiness rental property Total compensation a. $1,000,000 b. $900,000 c. $700,000 d. $600,000

$ 600,000 300,000 100,000 $1,000,000

79. Ting, a regional sales manager, works from her office in State W. Her region includes several states as indicated in the following sales report. Determine how much of Ting’s $300,000 compensation is assigned to the payroll factor of State W. State U V W

Sales Generated $ 1,000,000 5,000,000 4,000,000 $10,000,000

Ting’s Time Spent There 15% 55% 30% 100%

a. $0. b. $90,000. c. $120,000. d. $300,000. 80. Trayne Corporation’s sales office and manufacturing plant are located in State X. Trayne also maintains a manufacturing plant and sales office in State W. For purposes of apportionment, State X defines payroll as all compensation paid to employees, including elective contributions to § 401(k) deferred compensation plans. Under the statutes of State W, neither compensation paid to officers nor contributions to § 401(k) plans are included in the payroll factor. Trayne incurred the following personnel costs.

Wages and salaries for employees other than officers Salaries for officers Contributions to § 401(k) plans Powered by Cognero

State X

State W

Totals

$ 500,000

$300,000

$ 800,000

300,000 200,000

150,000 50,000

450,000 250,000 Page 12


Name:

Class:

Date:

Chapter 15: Multistate Corporation Taxation Totals Trayne’s payroll factor for State X is: a. 100.00%. b. 66.67%. c. 62.50%. d. 50.00%.

$1,000,000

$500,000

$1,500,000

81. Net Corporation’s sales office and manufacturing plant are located in State X. Net also maintains a manufacturing plant and sales office in State W. For purposes of apportionment, State X defines payroll as all compensation paid to employees, including contributions to § 401(k) deferred compensation plans. Under State W's statutes, neither compensation paid to officers nor contributions to § 401(k) plans are included in the payroll factor. Net incurred the following personnel costs.

Wages and salaries for employees other than officers Salaries for officers Contributions to § 401(k) plans Totals Net’s payroll factor for State W is: a. 50.00%. b. 37.50%. c. 33.33%. d. 0.00%.

State X

State W

Totals

$ 500,000

$300,000

$ 800,000

300,000 200,000 $1,000,000

150,000 450,000 50,000 250,000 $500,000 $1,500,000

82. Bert Corporation, a calendar year taxpayer, owns property in States M and O. Both states require that the average value of assets be included in the property factor. State M requires that the property be valued at its historical cost, and State O requires that the property be included in the property factor at its net depreciated book value.

Inventories Building & machinery (cost) Accumulated depreciation Land Totals

Account Balances at Beginning of Year State M State O Totals $ 200,000 $300,000 $ 500,000 700,000 300,000 1,000,000 (150,000) (50,000) (200,000) 400,000 200,000 600,000 $1,150,000 $750,000 $1,900,000

Inventories Building & machinery (cost) Accumulated depreciation Land Totals

Account Balances at Year-End State M State O Totals $ 400,000 $ 100,000 $ 500,000 800,000 500,000 1,300,000 (300,000) (100,000) (400,000) 400,000 200,000 600,000 $1,300,000 $ 700,000 $2,000,000

Bert’s State M property factor is: a. 75.0%. Powered by Cognero

Page 13


Name:

Class:

Date:

Chapter 15: Multistate Corporation Taxation b. 66.7%. c. 64.9%. d. 64.4%. 83. Valdez Corporation, a calendar year taxpayer, owns property in States M and O. Both states require that the average value of assets be included in the property factor. State M requires that the property be valued at its historical cost, and State O requires that the property be included in the property factor at its net depreciated book value.

Inventories Building & machinery (cost) Accumulated depreciation Land Totals

Account Balances at Beginning of Year State M State O Totals $ 200,000 $300,000 $ 500,000 700,000 300,000 1,000,000 (150,000) (50,000) (200,000) 400,000 200,000 600,000 $1,150,000 $750,000 $1,900,000

Inventories Building & machinery (cost) Accumulated depreciation Land Totals

Account Balances at Year-End State M State O Totals $ 400,000 $ 100,000 $ 500,000 800,000 500,000 1,300,000 (300,000) (100,000) (400,000) 400,000 200,000 600,000 $1,300,000 $ 700,000 $2,000,000

Valdez’s O property factor is: a. 35.0%. b. 37.2%. c. 39.5%. d. 53.8%. 84. In the broadest application of the unitary theory, the U.S. unitary business files a combined tax return using factors and income amounts for all affiliates: a. Organized in the United States. b. Organized in the U.S., Canada, and Mexico. c. Organized anywhere in the world. d. As dictated by the tax treaties between the United States and other countries. 85. Application of the unitary principle generally works to the taxpayer’s benefit when: a. The other affiliates generate net operating losses. b. The other affiliates operate in low-tax states. c. Both a. and b. d. Neither a. nor b. 86. In most states, a consolidated corporate income tax return: a. Is filed by a unitary group. b. Is required by most of the states. Powered by Cognero

Page 14


Name:

Class:

Date:

Chapter 15: Multistate Corporation Taxation c. Both a. and b. d. Neither a. nor b. 87. A taxpayer wishing to reduce the negative tax effects of the application of the unitary theory might: a. Affiliate with a service division that shows an operating loss, such as one in research and development. b. Acquire a unitary affiliate in a country with a high wage structure. c. Add a profitable entity to the unitary group. d. Both a. and b. 88. Guilford Corporation is subject to franchise tax in State Z. The tax is imposed at a rate of 2.5% of the taxpayer’s net worth that is apportioned to the state by use of a two-factor (sales and property equally weighted) formula. The property factor includes real and tangible personal property valued at net book value at the end of the taxable year. Of Guilford’s sales, 60% are attributable to State Z, and $200,000 of the net book value of its tangible personal property is located in State Z. Determine the State Z franchise tax payable by Guilford this year given the following end-of-the year balance sheet. Cash Equipment Accumulated Depreciation Furniture and Fixtures Accumulated Depreciation Intangible Assets Total Assets

$ 100,000 $800,000 (200,000) $150,000 (50,000)

Accounts and Taxes Payable Long-term Debt Common Stock Additional Paid-in Capital Retained Earnings Total Liabilities and Equity

600,000 100,000 200,000 $1,000,000 $ 150,000 300,000 10,000 600,000 (60,000) $1,000,000

a. $-0-, due to the negative retained earnings b. $6,187 c. $8,250 d. $13,750 89. When the taxpayer has exposure to a capital stock tax: a. The pricing of inventory sales should reflect no more than inflation increases. b. Subsidiary operations should be funded through direct capital contributions. c. Dividends should be paid regularly to a parent based in a low-tax state. d. Expansions should be funded with retained earnings. 90. In most states, a limited liability company (LLC) is subject to the state income tax: Powered by Cognero

Page 15


Name:

Class:

Date:

Chapter 15: Multistate Corporation Taxation a. As though it were a C corporation. b. As though it were a unitary business. c. As a flow-through entity, similar to its Federal income tax treatment. d. LLCs typically are exempted from state income taxation. 91. A state sales tax usually falls upon: a. Sales of groceries. b. Sales of widgets made to out-of-state customers. c. Sales of widgets made to an in-state final consumer of the product or service. d. Sales of real estate. 92. A state sales tax usually falls upon: a. The sale of a used dinette set sold at a rummage sale. b. The sale of a dinette set by the manufacturer to a furniture retailer. c. The sale of a case of Bibles by the publisher to a church bookstore. d. The sale of a Bible to a member of the church. e. All of these choices are exempt transactions. 93. A use tax applies when a State A resident purchases: a. A new automobile from a State A dealership. b. A used automobile from the website of a State A dealership. c. A new automobile from a State B dealership and then uses the car back at home in State A. d. A new automobile that is purchased from an online seller. 94. Typically, a local property tax: a. Applies to the inventories of a business. b. Applies to an individual’s stock portfolio. c. Both a. and b. d. Neither a. nor b. 95. With respect to typical sales/use tax laws: a. Exemptions are granted for purchases of clothing. b. Exemptions are granted for fuel-efficient automobiles. c. Fees paid for legal services are exempt. d. Restaurant meals are exempt. 96. A state’s escheat laws: a. Allow the state to take over property that a taxpayer has not claimed. b. Are applied only if the entity has nexus with the customer's state. c. Both a. and b. d. Neither a. nor b. Powered by Cognero

Page 16


Name:

Class:

Date:

Chapter 15: Multistate Corporation Taxation 97. In conducting multistate tax planning, the taxpayer should: a. Review tax opportunities in light of their effect on the overall business. b. Exploit inconsistencies among the taxing statutes and formulas of the states. c. Consider the tax effects of the plan after accounting for any new compliance and administrative costs that it generates. d. All of these choices are true. 98. When the taxpayer operates in one or more unitary states, all of these statements are true except: a. Apportionment factors are computed on a groupwide basis. b. The tax incentive of creating nexus in a high-tax state is enhanced. c. The tax benefit of a passive investment subsidiary holding company is neutralized. d. The use of a water’s edge election should be considered. 99. The benefits of a passive investment company employed in a nonunitary state typically include: a. Reduced state income taxes. b. Isolation of the entity’s portfolio income from taxation in other nonunitary states. c. Exclusion of the subsidiary’s portfolio income from the parent corporation’s apportionment formula denominator in other nonunitary states. d. All of these choices are benefits. 100. Parent Corporation owns all of the stock of Junior Corporation, a Delaware passive investment company. Parent operates strictly in nonunitary State B, which levies a 9% income tax. This year, Junior earned $200,000 of portfolio interest income and paid a $150,000 dividend to Parent. In which state(s) will the interest income create an income tax liability? a. Only in State B. b. Only in Delaware. c. In neither state. d. In both State B and Delaware, according to the apportionment formulas of each. 101. Parent and Junior form a unitary group of corporations. Parent is located in a state with an effective tax rate of 3% and Junior’s effective tax rate is 9%. Acting in concert to reduce overall tax liabilities, the group should: a. Execute an intercompany loan such that Junior pays deductible interest to Parent. b. Have Parent charge Junior an annual management fee. c. Shift Parent’s high-cost assembly and distribution operations to Junior. d. Do none of these choices are correct. 102. Parent and Minor form a non-unitary group of corporations. Parent is located in a state with an effective tax rate of 3% and Minor’s effective tax rate is 9%. Acting in concert to reduce overall tax liabilities, the group should: a. Have Parent charge Minor an annual management fee. b. Shift Parent’s high-cost assembly and distribution operations to Minor. c. Execute an intercompany loan such that Minor pays deductible interest to Parent. d. Do all of these choices are correct. 103. Hinojosa Corporation sells widgets in two states. State A levies a 9% effective tax rate, and State B levies a 3% rate. A and B have adopted sales-factor-only apportionment formulas. To reduce overall multistate income tax liabilities, Hinojosa should: Powered by Cognero

Page 17


Name:

Class:

Date:

Chapter 15: Multistate Corporation Taxation a. Move its home office from B to A. b. Remove all stored inventory from A. c. Establish a personal training center in A. d. Convert to employee status the independent contractors that it uses to sell widgets in A. 104. For most taxpayers, which of the traditional apportionment factors yields the greatest opportunities for tax reduction? a. Payroll. b. Property. c. Sales (gross receipts). d. Unitary. 105. Hopper Corporation’s property holdings in State E are as follows. Compute the numerator of Hopper’s E property factor. Item Manufacturing equipment Land held for potential appreciation Manufacturing equipment that is not currently needed and sits idle Manufacturing equipment that is not currently needed and is leased to another taxpayer

Property Factor Valuation ($M) $100 25 15 20

a. $100 million. b. $135 million. c. $140 million. d. $160 million. Completion 106. The starting point in computing state taxable income generally is ____________________. 107. In determining taxable income for state income tax purposes, interest income from Federal bonds typically constitutes a(n) ____________________ modification. 108. In determining taxable income for state income tax purposes, interest income from another state’s bonds typically constitutes a(n) ____________________ modification. 109. ____________________ describe(s) the degree of business activity that must be present before a taxing jurisdiction has the right to impose a tax on an out-of-state entity’s income. 110. Under Public Law 86-272, a state is prohibited from income taxing the income of a business if the only connection with the state is the _____________________ of orders for sales of tangible personal property that are sent outside the state for approval or rejection. 111. P.L. 86-272 ____________________ (does/does not) create income-tax nexus when the sales representative approves a sale at the customer’s location. 112. P.L. 86-272 ____________________ (does/does not) create income tax nexus when the seller's website allows an out-of-state customer to place an order and obtain online assistance. Powered by Cognero

Page 18


Name:

Class:

Date:

Chapter 15: Multistate Corporation Taxation 113. Apportionment is a means by which a corporation’s _________________________ taxable income is divided among the states in which it conducts business. 114. Allocation is a method under which a corporation’s _________________________ taxable income is directly assigned to the specific states where the income is derived. 115. Although apportionment formulas vary among jurisdictions, most states use the same three factors in the formula. The factors are ____________________, ____________________, and ____________________. 116. In the apportionment formula, most states assign more than a one-third weight to the ____________________ factor. 117. State Q has adopted sales-factor-only apportionment for its corporate income tax. As a result, a ____________________ (larger/smaller) percentage of an out-of-state corporation’s income is assigned to tax in the state. 118. Under the UDITPA’s ____________________ concept, sales are assumed to take place at the point of delivery as opposed to the location at which the shipment originates. 119. When a _________________________ rule is in effect, out-of-state sales that are not subject to tax in the destination state are pulled back into the sales factor numerator of the origination state. 120. In computing the property factor, property owned by the corporation typically is valued at its ____________________, but without adjusting for depreciation. 121. When included in the property factor, leased property usually is valued at ____________________ times its annual rental even though the taxpayer does not own the asset. 122. A(n) ____________________ business operates in concert with its affiliated companies. As a result, the affiliates’ data are included in the parent’s apportionment computations. 123. In ____________________ states, a(n) ____________________ election permits a multinational corporation to elect to limit the reach of the state’s taxing jurisdiction to activities occurring within the boundaries of the United States. 124. Overall tax liabilities typically ____________________ (increase/decrease) if the members of a unitary group begin to include affiliates that generate net operating losses. 125. Under common terminology, a unitary group files a ____________________ (combined/consolidated) state income tax return. 126. In some states, an S corporation must withhold Federal income tax for the proportionate flow-through income of its shareholders who ____________________ (are/are not) state residents. 127. Almost all of the states allow ____________________ treatment to an LLC for income tax purposes. 128. Several states allow the S corporation to file a(n) ____________________ income tax return, usually in the form of a state-by-state spreadsheet, on behalf of its out-of-state shareholders. 129. Typically, a sales/use tax is applied to a retail sale of ____________________ property. 130. A state sales/use tax is designed to be collected by the ____________________ (seller/purchaser) of the product and then remitted to the state. Powered by Cognero

Page 19


Name:

Class:

Date:

Chapter 15: Multistate Corporation Taxation 131. A ____________________ tax is designed to complement the local sales tax structure, to prevent the consumer from making no- or low-tax purchases in another state, outside the United States, or online and then bringing the asset into the state. 132. The ____________________ tax usually is applied at the city or county level as its main source of revenue. 133. If a gift card is unused after a specified period of time, the state can collect the amount of the card proceeds from the seller as ____________________ property. 134. A state might levy a(n) ____________________ tax when an investor sells shares of stock. 135. The ____________________ tax levied by a state usually is based on the book value of a corporation’s net worth. Matching Match each of the following terms with the appropriate description in the state income tax formula. Apply the UDITPA rules in your responses. a. Addition modification b. Subtraction modification c. No modification 136. Treasury bond interest income. 137. Dividend income from P&G stock held. 138. State income tax expense. 139. Federal depreciation deduction in excess of state amount. 140. State-level NOL. 141. Federal general business credit. 142. Deduction for advertising expenditures. Match each of the following items with the appropriate description in applying the P.L. 86-272 definition of solicitation. a. More than solicitation, creates nexus b. Solicitation only, no nexus created 143. Owning a tablet computer that is used on sales trips to the state. 144. Making a decision as to the creditworthiness of customers. 145. Training administrative personnel to use an update to ordering software. 146. Carrying a free sample of a product to the customer’s premises. 147. Purchasing ads that appear on search-result screens of internet browsers. Powered by Cognero

Page 20


Name:

Class:

Date:

Chapter 15: Multistate Corporation Taxation 148. Operating a warehouse for inventory that is held in the state. 149. Checking the customer’s inventory to determine whether a reorder is needed. Match each of the following events considered independently to its likely effect on WillCo’s various apportionment factors. WillCo is based in Q and has customers in Q, R, and S. To this point, WillCo has not established nexus with S. More than one choice may be correct. a. No change in apportionment factors b. Q apportionment factor increases c. Q apportionment factor decreases d. R apportionment factor increases e. R apportionment factor decreases f. S apportionment factor increases g. S apportionment factor decreases 150. Q adopts a throwback rule. 151. Q adopts a sales-only apportionment formula. 152. WillCo completes the construction of production facilities in S. 153. R adopts an increase in its statutory corporate income tax rates. Match each of the following items with the appropriate description in determining whether sales/use tax typically must be collected. a. Taxable b. Not taxable 154. A garment purchased for resale. 155. A garment purchased by a self-employed actress. 156. A garment purchased by an employee for wear at an office job. 157. A textbook purchased by a State University student. 158. A new auto purchased in Europe and shipped by the owner to her home state in the United States. 159. Prescription drugs and medicines purchased by a consumer. 160. Computing services purchased by a business. 161. Computer equipment purchased by a charity. 162. Groceries purchased by an individual and taken home for meal preparation there. 163. A meal eaten at a restaurant while conducting business. 164. Earthmoving equipment used by the purchaser in a construction business. Powered by Cognero

Page 21


Name:

Class:

Date:

Chapter 15: Multistate Corporation Taxation 165. Fees paid to an accounting firm for the conduct of a financial audit. Subjective Short Answer 166. Compute Still Corporation’s State Q taxable income and tax liability for the year. Addition modifications Allocated income – total Allocated income – State Q Allocated income – State P Apportionment percentage – State Q Federal taxable income State tax credits Subtraction modifications Tax rate

$70,000 $80,000 $60,000 $20,000 40% $500,000 $11,000 $30,000 5%

167. Hill Corporation is subject to tax only in State X. Hill generated the following income and deductions. State income taxes are not deductible for X income tax purposes. Sales Cost of sales State X income tax expense Depreciation allowed for Federal tax purposes Depreciation allowed for state tax purposes Interest income on Federal obligations Interest income on X obligations Expenses related to carrying X obligations

$5,000,000 2,000,000 160,000 1,000,000 800,000 50,000 200,000 10,000

a.

The starting point in computing the State X income tax base is Federal taxable income. Derive this amount.

b.

Determine Hill’s State X taxable income, assuming that interest on X obligations is exempt from State X income tax.

c.

Determine Hill’s taxable income, assuming that interest on X obligations is subject to State X income tax.

168. Provide the required information for Orange Corporation whose Federal taxable income totals $100 million. Orange apportions 70% of its business income to State C. Orange generates $10 million of nonbusiness income each year. Of that income, 40% is attributable to rentals of buildings located in C. Orange’s business income this year totals $90 million. a. b. c.

State C taxes how much of Orange’s business income? State C taxes how much of Orange’s nonbusiness income? Explain your results.

169. Condor Corporation generated $450,000 of state taxable income from selling its product in States A and B. For the taxable year, the corporation’s activities within the two states were as follows. Powered by Cognero

Page 22


Name:

Class:

Date:

Chapter 15: Multistate Corporation Taxation Sales Property Payroll

State A State B Total $800,000 $200,000 $1,000,000 300,000 –0– 300,000 200,000 800,000 1,000,000

Condor has determined that it is subject to tax in both States A and B. Both states utilize a three-factor apportionment formula that equally weights sales, property, and payroll. The rates of corporate income tax imposed in States A and B are 5% and 3%, respectively. Determine Condor’s state income tax liability. 170. Stanton Corporation owns and operates two facilities that manufacture paper products. One of the facilities is located in State D, and the other is located in State E. Stanton generated $1,200,000 of taxable income, composed of $1,000,000 of income from its manufacturing facilities and a $200,000 gain from the sale of nonbusiness property located in State E. State E does not distinguish between business and nonbusiness property. State D apportions business income. Stanton’s activities within the two states as follows. Sales of paper products Property Payroll

State D $4,500,000 3,500,000 1,500,000

State E $1,500,000 2,500,000 1,000,000

Total $6,000,000 6,000,000 2,500,000

Both States D and E utilize a three-factor apportionment formula, under which sales, property, and payroll are equally weighted. Determine the amount of Stanton’s income that is subject to income tax by each state. 171. Dott Corporation generated $300,000 of state taxable income from selling its mapping software in States A and B. For the taxable year, the corporation’s activities within the two states were as follows. Sales Property Payroll

State A $500,000 250,000 200,000

State B $1,500,000 –0– 300,000

Total $2,000,000 250,000 500,000

Dott has determined that it is subject to tax in both A and B. Both states utilize a three-factor apportionment formula that equally weights sales, property, and payroll. The rates of corporate income tax imposed in States A and B are 7% and 10%, respectively. Determine Dott’s total state income tax liability. 172. Drieser Corporation’s manufacturing facility, distribution center, and retail store are located in State K. Drieser sells its products to residents located in States K, M, and N. Sales to residents of K are conducted through a retail store. Sales to residents of M are obtained by Drieser’s sales representative, who has the authority to solicit, accept, and approve sales orders in State M. Residents of N can purchase Drieser’s product only if they place an order online and arrange to take delivery of the product at Drieser’s shipping dock. Drieser’s sales this year were reported as follows: Sales to residents of State K Sales to residents of State M Sales to residents of State N Total

$1,000,000 600,000 900,000 $2,500,000

Drieser’s activities within the three states are limited to those described above. All of the states have adopted a throwback provision and utilize a three-factor apportionment formula under which sales, property, and payroll are equally weighted. State K sources dock sales to the destination state. Powered by Cognero

Page 23


Name:

Class:

Date:

Chapter 15: Multistate Corporation Taxation Determine Drieser’s sales factors for States K, M, and N. 173. Basu Corporation, headquartered in State F, sells wireless computer devices, including keyboards and bar code readers. Basu’s degree of operations is sufficient to establish income tax nexus only in States E and F. Determine its sales factor in those states. State E applies a throwback rule to sales, but State F does not. State G has not adopted an income tax to date. Basu reported the following sales for the year. All of the goods were shipped from Basu’s F manufacturing facilities. Customer NorCo Tools, Inc. UniBell U.S. Department of Defense Total

Customer’s Location E F G All 50 U.S. States

This Year’s Sales $ 60,000,000 20,000,000 50,000,000 20,000,000 $150,000,000

174. Garcia Corporation is subject to income tax in States G, H, and I. Garcia’s compensation expense includes the following. State G State H State I Total Salaries and wages for nonofficers $600,000 $500,000 $500,000 $1,600,000 Officers’ salaries –0– –0– 800,000 800,000 Officers’ salaries are included in the payroll factor for States G and H but not for I. Compute Garcia’s payroll factors for G, H, and I. 175. Kang Corporation, a calendar year taxpayer, has manufacturing facilities in States A and B. A summary of Kang’s property holdings follows.

Inventory Plant and equipment Accumulated depreciation: plant and equipment Land Rental property* Accumulated depreciation: rental property

Inventory Plant and equipment Accumulated depreciation: plant and equipment Powered by Cognero

Beginning of Year State A State B $ 300,000 $ 200,000 2,200,000 1,500,000

Total $ 500,000 3,700,000

(1,200,000) 500,000 900,000

(500,000) 600,000 300,000

(1,700,000) 1,100,000 1,200,000

(200,000)

(50,000)

(250,000)

End of Year State A State B $ 400,000 $ 100,000 2,500,000 1,200,000

Total $ 500,000 3,700,000

(1,500,000)

(1,950,000)

(450,000)

Page 24


Name:

Class:

Date:

Chapter 15: Multistate Corporation Taxation Land 600,000 Rental property* 900,000 Accumulated depreciation: rental property (300,000) *Unrelated to Kang’s regular business and operations.

400,000 300,000

1,000,000 1,200,000

(100,000)

(400,000)

Determine Kang’s property factors for the two states. State A’s statutes provide that the average historical cost of business property is to be included in the property factor. State B’s statutes provide that the property factor is based on the average depreciated basis of in-state business property.

176. You are completing the State A income tax return for Quaint Company, LLC. Quaint operates in various states with the following results. Ordinary income Net capital loss Interest income, IBM bond

$800,000 (60,000) 40,000

In State A, all interest is treated as apportionable income. State A uses a sales-only apportionment factor. Compute Quaint’s State A taxable income.

Sales Property (average cost) Payroll

State A $800,000 250,000 300,000

All Other States $1,200,000 2,000,000 700,000

Total $2,000,000 2,250,000 1,000,000

177. Proton Corporation is a merchandiser. It purchases overstock garments from various suppliers and sells the goods in its State L retail store. Determine the total sales that are subject to the State L sales tax. Sales to State L residents Sales to homeless shelter operated by a local church Sales to residents who cross the border from nearby State M Sales to a similar merchandiser located in another State L town

$600,000 80,000 100,000 20,000

178. Indicate for each transaction whether a sales (S) or use (U) applies or whether the transaction is nontaxable (N). Where the laws vary among various states, assume that the most common rules apply. All taxpayers are individuals. a.

A resident of State A purchases a computer in A.

b.

A resident of State A purchases prescription medicine in A.

c.

A resident of State B purchases a computer in A.

d.

A church purchases office supplies in A.

e.

A State A retailer purchases in B an item that will be in the inventory of its business.

f.

A resident of State A purchases hardware from a retail home improvement store in A.

Powered by Cognero

Page 25


Name:

Class:

Date:

Chapter 15: Multistate Corporation Taxation g.

A business based in State A purchases vacant A land to be held for a future expansion project.

h.

A business based in State A purchases repair services from an A plumbing contractor.

179. Hambone Corporation is subject to the State E capital stock tax. The tax is levied at 2% of the entity’s net worth that is apportioned to State E. Hambone conducts 30% of its operations in State E. Hambone’s current book balance sheet is as follows with amounts in millions. Compute Hambone’s liability for the State E capital stock tax. Cash Receivables Equipment, net of accumulated depreciation Buildings, net of accumulated depreciation Land Total assets

$ 40 20

Payables Mortgages, principal

$ 50 400

140

Common stock

10

300

Additional paid-in capital

190

250 $750

Retained earnings Total liabilities and equity

100 $750

180. Franz Corporation is based in State A (corporate income tax rate 10%). It sells its goods to customers in both State A and State B (corporate income tax rate 4%). Franz’s state taxable income for the year is $1 million, 45% of which relates to State B customers. Franz’s level of activities in State B is insufficient to create nexus there, but State A has adopted a throwback rule as to multistate sales. Would Franz reduce its total state income tax liability b y creating nexus with State B, for example, by allowing its sales force to make credit decisions? Elaborate. Essay 181. Anders, a local business, wants your help in making a decision about a large capital investment. To assist your client, list several tax and nontax implications of the decision. 182. Compost Corporation has finished its computation of Federal taxable income. In State Q, the derivation of state corporate taxable income starts with the Federal amount and makes a number of modifications. List at least five such modifications that Compost is likely to encounter. In this regard, follow the general UDITPA rules, and list both addition and subtraction modifications. 183. What is the significance of the term nexus when discussing state income taxation? 184. Discuss how a multistate business divides its corporate taxable income among the states in which it operates. Hint: use the terms allocation and apportionment in your comments. 185. State Q wants to increase its income tax collections, but politically it would be unwise to raise taxes on in-state individuals or businesses. State Q currently follows all UDITPA rules and employs an equally weighted three-factor apportionment formula. State Q allocates nonbusiness income amounts. Identify some changes to the income tax apportionment formula that would shift the scheduled income tax increases to out-of-state businesses. 186. In international taxation, we discuss income sourcing rules and the permanent establishment doctrine. In multistate taxation, an analogous term might be the ultimate destination concept. Define this term, and identify at least two of the Powered by Cognero

Page 26


Name:

Class:

Date:

Chapter 15: Multistate Corporation Taxation most important exceptions to the general rule. 187. Singh spends time working at the offices of her employer as a consultant to clients who are located in three different U.S. states. Singh’s compensation is assigned to the payroll factor(s) of which state(s)? Apply the general UDITPA rules. 188. The sales/use tax that is employed by most U.S. states does not fall on all retail transactions. Identify at least five sales/use tax exemptions that states often allow that eliminate certain transactions from the tax base. 189. You are preparing to make a presentation to a client who has some familiarity with state corporate income tax rules. At a general level, list for the client the principles of multistate income tax planning.

Powered by Cognero

Page 27


Name:

Class:

Date:

Chapter 15: Multistate Corporation Taxation Answer Key 1. False 2. False 3. True 4. False 5. True 6. True 7. True 8. True 9. False 10. False 11. False 12. False 13. False 14. True 15. False 16. True 17. True 18. False 19. True 20. True 21. True 22. False 23. True 24. False 25. False Powered by Cognero

Page 28


Name:

Class:

Date:

Chapter 15: Multistate Corporation Taxation 26. True 27. True 28. True 29. False 30. True 31. True 32. True 33. False 34. True 35. True 36. True 37. True 38. False 39. False 40. True 41. True 42. True 43. False 44. True 45. b 46. c 47. d 48. d 49. c 50. b Powered by Cognero

Page 29


Name:

Class:

Date:

Chapter 15: Multistate Corporation Taxation 51. d 52. d 53. a 54. b 55. a 56. d 57. c 58. c 59. d 60. b 61. b 62. a 63. c 64. a 65. b 66. c 67. c 68. a 69. c 70. c 71. c 72. d 73. c 74. d 75. b 76. a Powered by Cognero

Page 30


Name:

Class:

Date:

Chapter 15: Multistate Corporation Taxation 77. d 78. d 79. d 80. b 81. b 82. d 83. b 84. c 85. c 86. d 87. a 88. b 89. c 90. c 91. c 92. d 93. c 94. d 95. c 96. a 97. d 98. b 99. d 100. c 101. d Powered by Cognero

Page 31


Name:

Class:

Date:

Chapter 15: Multistate Corporation Taxation 102. d 103. b 104. c 105. a 106. Federal taxable income 107. subtraction 108. addition 109. Nexus 110. solicitation 111. does 112. does 113. business 114. nonapportionable nonbusiness 115. sales, property, payroll property, payroll, sales payroll, sales, property 116. sales 117. larger 118. ultimate destination 119. throwback 120.

original cost historical cost 121. eight 8 122. unitary 123. unitary, water’s edge 124. decrease Powered by Cognero

Page 32


Name:

Class:

Date:

Chapter 15: Multistate Corporation Taxation 125. combined 126. are not 127. flow-through pass-through partnership 128. composite block 129. tangible personal 130. seller 131. use 132. property 133. unclaimed escheated 134.

stock transfer transfer 135. capital stock 136. b 137. b 138. a 139. a 140. b 141. c 142. c 143. b 144. a 145. a 146. b Powered by Cognero

Page 33


Name:

Class:

Date:

Chapter 15: Multistate Corporation Taxation 147. b 148. a 149. b 150. b 151. c 152. c 153. a 154. b 155. a 156. a 157. a 158. a 159. b 160. b 161. b 162. b 163. a 164. b 165. b 166. State Q taxable income is computed as follows. Federal taxable income Addition modifications Subtraction modifications State tax base Allocated income – total Apportionable income Apportionment percentage Apportioned income Allocated income – instate State taxable income Tax rate Powered by Cognero

$500,000 + 70,000 – 30,000 $540,000 – 80,000 $460,000 × 40% $184,000 + 60,000 $244,000 × 5% Page 34


Name:

Class:

Date:

Chapter 15: Multistate Corporation Taxation Gross income tax Credits Tax liability

$ 12,200 – 11,000 $ 1,200

167. a. Sales Cost of sales Cost recovery (Federal) Interest income (Federal only) X income tax expense Federal taxable income

$ 5,000,000 –2,000,000 –1,000,000 +50,000 – 160,000 $ 1,890,000

b. Federal taxable income X income tax expense Depreciation modification ($800,000 – $1,000,000) Interest income on Federal obligations X taxable income

$1,890,000 +160,000 +200,000 – 50,000 $2,200,000

c. Federal taxable income X income tax expense Depreciation modification ($800,000 – $1,000,000) Interest income on Federal obligations Interest income on X obligations Expenses related to X obligations X taxable income

$1,890,000 +160,000 +200,000 –50,000 +200,000 – 10,000 $2,390,000

168. a. $63,000,000 (70% × $90 million). b.

$4,000,000 (40% × $10 million).

c.

Business income is apportioned to the state using the apportionment formula. Nonbusiness income is allocated to the state using a dollar-for-dollar assignment.

169. State A Income Tax Liability Taxable income Apportionment Formula Sales $800,000/$1,000,000 = Property $300,000/$300,000 = Payroll $200,000/$1,000,000 = Total State A apportionment factor (200.00%/3) Taxable income apportioned to A A tax rate A tax liability Powered by Cognero

$450,000 80.00% 100.00% 20.00% 200.00% × 66.67% $300,000 × 5.00% $ 15,000 Page 35


Name:

Class:

Date:

Chapter 15: Multistate Corporation Taxation State B Income Tax Liability Taxable income

$450,000

Apportionment Formula Sales $200,000/$1,000,000 = Property $0/$300,000 = Payroll $800,000/$1,000,000 = Total

20.00% –0–% 80.00% 100.00%

State B apportionment factor (100.00%/3) Taxable income apportioned to B B tax rate B tax liability

× 33.33% $150,000 × 3.0% $ 4,500

Total State Tax Liability A tax liability B tax liability Total tax liability

$15,000 4,500 $19,500

170. State D Taxable Income Income subject to apportionment (business income only)

$1,000,000

Apportionment Formula Sales $4,500,000/$6,000,000 = Property $3,500,000/$6,000,000 = Payroll $1,500,000/$2,500,000 = Total

75.00% 58.33% 60.00% 193.33%

State D apportionment factor (193.33%/3) Taxable income apportioned to D Plus: Income allocated to D* State D taxable income

× 64.44% $ 644,400 –0– $ 644,400

*Since the property for which the $200,000 gain was derived was located in State E, such income is not allocated to State D. State E Taxable Income Income subject to apportionment (business and nonbusiness income) Apportionment formula Sales Property Payroll Total

$1,700,000*/$6,200,000* = $2,500,000/$6,000,000 = $1,000,000/$2,500,000 =

Taxable Income State E apportionment factor (109.09%/3) State E taxable income

$1,200,000

27.42% 41.67% 40.00% 109.09% $1,200,000 × 36.36% $ 436,320

*Includes $200,000 gain on sale of nonbusiness property. Powered by Cognero

Page 36


Name:

Class:

Date:

Chapter 15: Multistate Corporation Taxation 171. State A Income Tax Liability Taxable income Apportionment Formula Sales Property Payroll Total

$300,000 $500,000/$2,000,000 = $250,000/$250,000 = $200,000/$500,000 =

25.00% 100.00% 40.00% 165.00%

Taxable Income State A apportionment factor (165.00%/3) Taxable income apportioned to A A tax rate A tax liability

$300,000 × 55.00% $165,000 × 7.00% $ 11,550

State B Income Tax Liability Taxable income

$300,000

Apportionment Formula Sales Property Payroll Total

$1,500,000/$2,000,000 = $0/$250,000 = $300,000/$500,000 =

Taxable Income State B apportionment factor (135.00%/3) Taxable income apportioned to B B tax rate B tax liability Total State Tax Liability A tax liability B tax liability Total tax liability

75.00% –0–% 60.00% 135.00% $300,000 ×45.00% $ 135,000 ×10.00% $ 13,500 $11,550 13,500 $25,050

172. Sales Factor for State K Sales to residents of K Sales thrown back to K Sales attributed to K

$1,000,000 900,000 * $1,900,000

Sales factor: $1,900,000/$2,500,000 = 76.00% * The throwback rule is an exception to the ultimate destination test. This rule ensures the taxation of sales made to a destination state in which the seller is not taxable. Since Drieser is not subject to tax in N (the destination state), dock sales made to residents of N are treated as in-state sales of K (the origination state). Sales Factor for State M Sales to residents of M Powered by Cognero

$600,000 Page 37


Name:

Class:

Date:

Chapter 15: Multistate Corporation Taxation Sales factor: $600,000/$2,500,000 = 24.00% Drieser’s salesperson has the authority to accept and approve sales to customers located in M. Accordingly, the activities of the salesperson exceed that immune under Public Law 86-272, thereby creating nexus. Drieser is subject to tax in M. Sales Factor for State N Since Drieser’s connection with State N is limited to residents of that state taking delivery of Drieser’s product at Drieser’s shipping dock after placing an online order, insufficient nexus is established to subject Drieser to taxation in N. Accordingly, a zero sales factor is created for N. Summary Because K has adopted a throwback provision, sales to residents of N are attributed to K, and they are used in determining the K sales factor. The sales factors total to 100%. Drieser has no “nowhere sales.” 173. Because F has not adopted a throwback rule, the sales to customers in G are not included in either state’s sales factor. The sales to the U.S. government are sourced to F, the state in which the sale is made. Basu creates $50 million in “nowhere sales.” E Sales factor = $60 million/$150 million = 40.00% F Sales factor = $40 million/$150 million = 26.67% 174. G Payroll factor H Payroll factor I Payroll factor Total of payroll factors

$600,000/$2,400,000 $500,000/$2,400,000 $500,000/$1,600,000

= = =

25.00% 20.83% 31.25% 77.08%

175. Kang Corporation’s property factor is 61.90% for State A, and its property factor for State B is 44.53%. Under the statutes of States A and B, nonbusiness property (i.e., the rental property) is not taken into consideration in computing the property factor. The basis for determining the average property in State A is historical cost, whereas the value employed for State B is depreciated cost. Historical Cost—Excluding Nonbusiness Property Average Property in State A

Inventory Plant and equipment Land Total

Beginning of Year $ 300,000 2,200,000 500,000

End of Year

Total

Average

$ 400,000 2,500,000 600,000

$ 700,000 4,700,000 1,100,000

$ 350,000 2,350,000 550,000 $3,250,000

End of Year

Total

Average

$ 100,000 1,200,000

$ 300,000 2,700,000

$ 150,000 1,350,000

Average Property in State B

Inventory Plant and equipment Powered by Cognero

Beginning of Year $ 200,000 1,500,000

Page 38


Name:

Class:

Date:

Chapter 15: Multistate Corporation Taxation Land Total

600,000

400,000

1,000,000

500,000 $2,000,000

End of Year

Total

Average

$ 400,000 2,500,000

$ 700,000 4,700,000

$ 350,000 2,350,000

(1,500,000) 600,000

(2,700,000) 1,100,000

(1,350,000) 550,000 $1,900,000

End of Year

Total

Average

$ 100,000 1,200,000

$ 300,000 2,700,000

$ 150,000 1,350,000

(450,000) 400,000

(950,000) 1,000,000

(475,000) 500,000 $1,525,000

Property Factor for State A

Depreciated Basis—Excluding Nonbusiness Assets Average Property in State A

Inventory Plant and equipment Accumulated depreciation: plant & equipment Land Total

Beginning of Year $ 300,000 2,200,000 (1,200,000) 500,000

Average Property in State B

Inventory Plant and equipment Accumulated depreciation: plant & equipment Land Total

Beginning of Year $ 200,000 1,500,000 (500,000) 600,000

Property factor for State B

176. LLCs allocate and apportion state taxable income in the same manner as do other entities. No entity-level tax liability is assessed. Quaint members with nexus to State A will report, as determined in the LLC’s operating agreement, the following amounts on the entity’s Schedule K. The A apportionment percentage is 40% ($800,000/$2,000,000). Ordinary income Net capital loss Interest income

Powered by Cognero

$320,000 (24,000) 16,000

Page 39


Name:

Class:

Date:

Chapter 15: Multistate Corporation Taxation 177. Proton must collect State L sales tax on $700,000. No tax is collected from sales to charities or to other entities who will then resell the goods. The sales to State M residents instead create a State M use tax because the goods presumably are worn by the customers in State M. State M residents should file a return to pay the State M sales tax and then claim a credit for the payment actually made to State L. 178. a. S. b.

N. Probably exempt as a necessities item.

c.

U.

d.

N. Probably exempt because of charitable status.

e.

N. Probably exempt under the resale rule.

f.

S.

g.

N, not tangible personal property.

h.

N, not tangible personal property.

179. Hambone’s corporate net worth ($750 – $50 – $400) $300 X E apportionment factor × 0.30 X E capital stock tax rate × 0.02 Tax liability $ 1.80 180. Creating nexus with State B would be an effective multistate tax planning move. If No Nexus with State B

If Nexus with State B

State A income tax

($550,000 A taxable income + $450,000 B taxable income $550,000 × 10% = $55,000 after throwback) × 10% tax rate = $100,000

State B income tax

$0, no nexus

Total state income tax $100,000

$450,000 × 4% = $18,000 $73,000

181. Businesses operating in a multistate environment seldom make critical decisions based on tax factors alone. The following events might bring up important state and local tax issues. ∙ Location of a plant expansion or distribution center. ∙

Increases in spending for high-tech or other equipment. Powered by Cognero

Page 40


Name:

Class:

Date:

Chapter 15: Multistate Corporation Taxation ∙ Concentration of sales activities using catalog and Internet exposure. One must examine the implications of this decision on Anders’s state and local corporate income taxes, including the payroll, property, and sales factors; sales/use tax obligations and collection responsibilities; property tax and other business licenses and fees; and income tax effects on its employees and contractors. In addition, the investment might bring about changes as to Anders’s market share for its products; price levels for wages and supplies; quality of local labor; transportation and communications costs; and fee structures and regulation climate of the state and local environment. 182. State income tax modifications include the following commonly encountered items. Every state’s rules are unique, so thorough research is needed to complete a final list for Compost’s location. Addition modifications ∙ Municipal bond interest, especially for out-of-state issuances. ∙

Cost recovery deductions when the Federal deduction exceeds the state’s.

State income tax expense.

Federal and out-of-state net operating losses.

Subtraction modifications ∙ U.S. Treasury interest income. ∙

Cost recovery deductions when the Federal deduction is less than the state’s.

State income tax refunds.

Federal income tax expense.

183. A U.S. state cannot levy an income tax on an out-of-state entity unless that entity has conducted a significant degree of business activity within the state’s borders. Each state defines differently the degree of nexus (connection) that is required before the right to tax the business arises. Typically, sufficient nexus is present when a corporation sells goods or services within the state, owns or leases in-state property, employs personnel in the state, or holds physical or financial capital there. Lacking sufficient nexus, the state cannot place an income tax on the business. Note that a state may define nexus differently for sales/use tax purposes. 184. Generally, business income is apportioned by a formula to the state(s) in which the income is derived, and nonbusiness income is allocated to the state(s) of its situs. These income assignments are made into states with which the taxpayer has established nexus. An apportionment formula generally is an average of the relative sales, property, and payroll activities of the taxpayer in a particular state. Allocation usually is made with respect to the rental income, interest, dividends, and capital gains of the taxpayer. 185. ∙ Overweighting the sales factor. ∙

Sales-factor only apportionment.

Adoption of a unitary approach to out-of-state affiliates. Powered by Cognero

Page 41


Name:

Class:

Date:

Chapter 15: Multistate Corporation Taxation 186. Under the ultimate destination concept, a sale of a tangible asset is sourced only to the sales factor of the state of the point of delivery, not the location where the shipment is initiated. Exceptions to the ultimate destination concept include the following. ∙

Different rules apply for sales of intangible assets and real estate.

When a dock sale shipment is used, the sale is assigned to the purchaser’s state regardless of the shipping patterns.

If a state has adopted the throwback rule, the sale is sourced to the seller’s state where the ultimate destination concept would not subject the sale to income taxation. ∙

Sales to the U.S. government are sourced to the seller's state.

187. The compensation of an employee generally is not split among the states where the employee’s services are rendered. Rather, it is assigned to one state in full with that state identified in this order. ∙

To the state of the employee’s base of operations.

If the compensation has not yet been assigned, then to the state from which the services are directed or controlled.

If the compensation has yet to be assigned, then to the state of the employee’s residency.

188. Most state and local governments allow the following exemptions from a consumer sales/use tax base. ∙

Sales of many services rather than of tangible personal property.

Sales for resale.

Casual or occasional sales.

Sales to tax-exempt organizations including governments and charities.

Sales of necessities such as groceries and health care goods.

Sales of targeted items such as computing and agricultural items.

Sales of manufacturing equipment.

Packaging and shipping materials used by a manufacturer.

189. •

State and local income tax planning often involves modifications in the legal, functional, or technical means by which the taxpayer conducts business, but these modifications should be undertaken only after considering their

Powered by Cognero

Page 42


Name:

Class:

Date:

Chapter 15: Multistate Corporation Taxation nontax effects. •

Use of independent contractors instead of employees may reduce the payroll factor for a state.

Moving assets into a leasing subsidiary in a low-or no-tax state might reduce the property factory in a high-tax state.

Establishing nexus in a low- or no-tax state often optimizes the multistate tax liability and eliminating nexus in high-tax, nonunitary states can accomplish the same objective.

Holding companies for investment assets can take advantage of the states’ exemptions for certain interest and dividend income.

Powered by Cognero

Page 43


Name:

Class:

Date:

Chapter 16: Taxation of International Transactions True / False 1. The United States has in force income tax treaties with about 70 countries. a. True b. False 2. Interest paid to an unrelated party by a U.S. corporation that historically earns more than 50% of its gross income each year from the conduct of an active trade or business outside the United States is foreign-source income. a. True b. False 3. Dividends received from Hernandez Corp., a foreign corporation that earns 70% of its income from U.S. business activities, are 70% U.S.-source income. a. True b. False 4. Serena, a nonresident alien, is employed by GlobalCo, a non-U.S. corporation. She works in the United States for three days during the year, receiving a gross salary of $2,500 for this period. GlobalCo is not engaged in a U.S. trade or business. Under the commercial traveler exception, the $2,500 is not classified as U.S.-source income. a. True b. False 5. Plantco, a foreign corporation, uses a formula to manufacture pharmaceuticals. The formula was developed and is owned by DrugCo, a U.S. entity. Royalties paid by PlantCo to DrugCo for the use of the formula are U.S.-source income. a. True b. False 6. Julio, a nonresident alien, realizes a gain on the sale of commercial real estate located in the United States. The real estate was sold to Mariana, Julio’s cousin, who is also a nonresident alien. Julio recognizes foreign-source income from the sale because his home country is not the United States. a. True b. False 7. The residence of seller rule is used in determining the sourcing of all gross income and deductions of a U.S. multinational business. a. True b. False 8. A U.S. business conducts international communications activities between the United States and Spain. The resulting income is sourced 100% to the United States, the residence of the taxpayer. a. True b. False 9. The sourcing rules of Federal income taxation apply to deductions as well as to income items. a. True b. False Powered by Cognero

Page 1


Name:

Class:

Date:

Chapter 16: Taxation of International Transactions 10. In allocating interest expense between U.S. and foreign sources, a taxpayer can elect to use either the tax book value of the income-producing assets or their fair market values. a. True b. False 11. The IRS can use § 482 reallocations to ensure that transactions between related parties are properly reflected in a tax return. a. True b. False 12. A qualified business unit of a U.S. corporation that operates in Germany generally uses the Euro as its functional currency. a. True b. False 13. LocalCo merges into HeirCo, a non-U.S. entity, in a transaction that would qualify as a “Type A” reorganization. The resulting realized gain is tax-deferred under U.S. income tax law using §§ 351 and 368. a. True b. False 14. Inbound and offshore asset transfers by a U.S. business can be subject to immediate Federal income taxation under § 367. a. True b. False 15. A “U.S. shareholder” for purposes of CFC classification is any U.S. person who owns directly, indirectly, and/or constructively at least 50% of the voting power of a foreign corporation. a. True b. False 16. Twenty unrelated U.S. persons equally own all of the stock of Quigley, a foreign corporation. Quigley is a CFC. a. True b. False 17. Hendricks Corporation, a U.S. corporation, owns 40 percent of Shane Corporation and 55 percent of Ferrell Corporation, both foreign corporations. Ferrell owns the other 60 percent of Shane Corporation. Both Shane and Ferrell are CFCs. a. True b. False 18. Kipp, a U.S. shareholder under the CFC provisions, owns 40% of a CFC. If the CFC’s Subpart F income for the taxable year is $200,000, Kipp is taxed on receipt of a constructive dividend of $80,000. a. True b. False 19. ForCo, a non-U.S. corporation based in Sweden, purchases widgets from USCo, Inc., its U.S. parent corporation. The widgets are sold by ForCo to an unrelated foreign corporation in Sweden. The income from the sale of the widgets by Powered by Cognero

Page 2


Name:

Class:

Date:

Chapter 16: Taxation of International Transactions ForCo is Subpart F foreign base company sales income. a. True b. False 20. ForCo, a subsidiary of a U.S. corporation incorporated in Belgium, manufactures widgets in Belgium and sells the widgets to its 100%-owned subsidiary in Germany. The income from the sale of widgets is not Subpart F foreign base company sales income. a. True b. False 21. Subpart F income includes portfolio income such as dividends and interest. a. True b. False 22. Jokerz, a CFC of a U.S. parent, generated $80,000 Subpart F foreign base company services income in its first year of operations. The next year, Jokerz distributes $50,000 cash to the parent, from those service profits. The parent is taxed on $0 in the first year (tax deferral rules apply) and $50,000 in the second year. a. True b. False 23. Jaime received gross foreign-source dividend income of $250,000. Foreign taxes withheld on the dividend were $25,000. Jaime’s total U.S. tax liability is $840,000 (the 21% tax rate applies). Jaime’s current-year FTC is $52,500. a. True b. False 24. Waltz, Inc., a U.S. taxpayer, pays foreign taxes of $50,000 on foreign-source general basket income of $90,000. Waltz’s worldwide taxable income is $450,000, on which it owes U.S. taxes of $94,500 before FTC. Waltz’s FTC is $50,000. a. True b. False 25. Unused foreign tax credits are carried back two years and then forward 20 years. a. True b. False 26. A U.S. taxpayer may take a current FTC equal to the greater of the FTC limit or the actual foreign taxes (direct or indirect) paid or accrued. a. True b. False 27. Freda was born and continues to live in Uruguay. She exports widgets to U.S. customers. The United States does not have in force an income tax treaty with Uruguay. Freda’s net U.S. income from the widgets is subject to a flat 30% Federal income tax rate. a. True b. False 28. Nico lives in California. She was born in Peru but holds a green card. Nico is a nonresident alien (NRA). Powered by Cognero

Page 3


Name:

Class:

Date:

Chapter 16: Taxation of International Transactions a. True b. False 29. Carol, a citizen and resident of France, reports gross income that is effectively connected with a U.S. business. No deductions are allowed against this income, and Carol’s U.S. tax rate is a flat 30%. a. True b. False 30. A domestic corporation is one whose assets are primarily located in the United States. For this purpose, the primarily located test (greater than 50%) applies. a. True b. False 31. Gains on the sale of U.S. real property held directly or indirectly through U.S. stock ownership by NRAs and foreign corporations are subject to tax at capital gains rates under FIRPTA. a. True b. False 32. In year 1, George renounces his U.S. citizenship and moves to Andorra, where income tax rates are very low. George is a multimillionaire and says he “has had it” with high Federal income taxes on wealthy individuals like himself. In year 4, George’s U.S.-source income is $1.5 million. That income escapes Federal income taxes. a. True b. False 33. The purpose of the transfer pricing rules is to ensure that taxpayers have ultimate flexibility in shifting profits between related entities. a. True b. False 34. The U.S. system for taxing income earned inside its borders by non-U.S. persons is referred to as inbound taxation because such foreign persons are earning income by coming into the United States. a. True b. False 35. GILTI is a tax assessed on a controlled foreign corporation on tested earnings in excess of a 10% return on

its tangible assets. a. True b. False 36. GILTI was enacted with the purpose of encouraging U.S. corporations to repatriate their foreign earnings. a. True b. False 37. The general purpose of Form 5471 is to provide the IRS with information about the shareholders, income, balance sheet, and activities of foreign corporations owned by US shareholders. a. True Powered by Cognero

Page 4


Name:

Class:

Date:

Chapter 16: Taxation of International Transactions b. False 38. Form 5472 is required to be filed by foreign corporations engaged in a trade or business in the United States. a. True b. False Multiple Choice 39. Without the foreign tax credit, double taxation would result when: a. The United States taxes the U.S.-source income of a U.S. resident. b. A foreign country taxes the foreign-source income of a nonresident alien. c. The United States and a foreign country both tax the foreign-source income of a U.S. resident. d. Terms of a tax treaty assign income taxing rights to the United States. 40. U.S. income tax treaties typically: a. Provide for taxation exclusively by the source country. b. Provide for taxation exclusively by the country of residence. c. Provide rules by which multinational taxpayers avoid double taxation. d. Provide that the country with the highest tax rate will be allowed exclusive tax collection rights. 41. Which of the following statements is false in regard to the U.S. income tax treaty program? a. There are about 70 bilateral income tax treaties between the United States and other countries. b. Tax treaties generally provide for primary taxing rights that require the other treaty partner to allow a credit for the taxes paid on the twice-taxed income. c. U.S. income tax treaties are written to set up a “network” of up to five foreign countries that are covered by the treaty language. d. None of these statements is false. 42. ForCo, a foreign corporation, receives interest income of $100,000 from USCo, an unrelated U.S. corporation. USCo has historically earned 85% of its income from foreign sources. What amount of ForCo’s interest income is U.S. source? a. $0 b. $50,000 c. $85,000 d. $100,000 43. John, an NRA, is employed by Fisher, Inc., a foreign corporation. In November, John spends 10 days in the United States performing consulting services for Fisher’s U.S. branch. He earns $5,000 per month. A month includes 20 workdays. a. John has $2,500 U.S.-source income, which is exempt from U.S. taxation, because he is in the United States for 90 days or less. b. John has $2,500 U.S.-source income, which is exempt from U.S. taxation, because the amount paid to him is less than $3,000. c. John has $2,500 U.S.-source income, because his foreign employer has a U.S. branch. d. John has no U.S.-source income under the commercial traveler exception. 44. USCo, a U.S. corporation, purchases inventory from distributors within the United States and resells this inventory to Powered by Cognero

Page 5


Name:

Class:

Date:

Chapter 16: Taxation of International Transactions customers outside the United States, with title passing outside the United States. Profit on the sale is $10,000. What is the sourcing of the USCo’s inventory sales income? a. $5,000 U.S. source and $5,000 foreign source. b. $5,000 U.S. source and $5,000 sourced based on location of the pertinent manufacturing assets. c. $10,000 U.S. source. d. $10,000 foreign source. 45. Liang, an NRA, is sent to the United States by Fulston Corporation, her non-U.S. employer. She spends 50 days in the United States and earns $20,000 for a two-month period. This amount is attributable to 40 U.S. working days and 10 nonU.S. working days. Fulston does not have a United States trade or business, and Liang spends no other time in the United States for the tax year. Liang’s U.S.-source taxable income is: a. $20,000. b. $16,000. c. $3,000. d. $0. 46. Olaf, a citizen of Norway with no trade or business activities in the United States, sells at a gain 200 shares of MicroShift, Inc., a U.S. company. The sale takes place through Olaf’s broker in Oslo. How is this gain treated for U.S. tax purposes? a. It is foreign-source income subject to U.S. taxation. b. It is foreign-source income not subject to U.S. taxation. c. It is U.S.-source income subject to U.S. taxation. d. It is U.S.-source income exempt from U.S. taxation. 47. During the current year, USACo (a U.S. corporation) sold equipment to FrenchCo, a non-U.S. corporation, for $350,000 with title passing to the buyer in France. USACo purchased the equipment several years ago for $100,000 and took $80,000 of depreciation deductions on the equipment, all of which were allocated to U.S.-source income. USACo’s adjusted basis in the equipment is $20,000 on the date of sale. What is the sourcing of the $330,000 gain on the sale of this equipment? a. $330,000 foreign source. b. $330,000 U.S. source. c. $250,000 foreign source and $80,000 U.S. source. d. $250,000 U.S. source and $80,000 foreign source. 48. Section 482 is used by the U.S. Treasury to: a. Force taxpayers to use arms length transfer pricing on transactions between related parties. b. Reallocate income, deductions, etc., to a related taxpayer to minimize tax liability. c. Increase information that is reported about U.S. corporations with non-U.S. owners. d. All of these choices are correct. 49. An advance pricing agreement (APA) is used between: a. Two or more governments. b. Two related taxpayers. c. The taxpayer and the IRS. d. The IRS and non-U.S. non-taxing authorities. Powered by Cognero

Page 6


Name:

Class:

Date:

Chapter 16: Taxation of International Transactions 50. Flapp Corporation, a U.S. corporation, conducts all of its transactions in the U.S. dollar. It sells inventory for $1 million to a Canadian company when the exchange rate is $1US: $1.2Can. The Canadian company pays for the inventory when the exchange rate is $1US: $1.3Can. What is Flapp’s exchange gain or loss on this sale? a. Flapp does not have a foreign currency exchange gain or loss, as it conducts all of its transactions in the U.S. dollar. b. Flapp’s account receivable for the sale is $1 million (when the exchange rate is $1US: $1.2Can.) and it collects on the receivable when the exchange rate is $1US: $1.3Can. Flapp has an exchange gain of $100,000. c. Flapp’s account receivable for the sale is $1 million (when the exchange rate is $1US: $1.2Can.). It collects on the receivable at $1US: $1.3Can. Flapp has an exchange loss of $10,000. d. Flapp’s foreign currency exchange loss is $100,000. 51. In which of the following independent situations would Slane, a foreign corporation, be classified as a controlled foreign corporation? The Slane stock is directly owned 12% by Jen, 10% by Kathy, 12% by Ling, 10% by David, 8% by Ben, and 48% by Mia. a. Jen, Kathy, Ling, David, Ben, and Mia are all U.S. citizens. b. Jen, Kathy, Ling, David, and Ben are all U.S. citizens. David is married to Kathy. Mia is a foreign resident and citizen. c. Jen, Kathy, Ling, David, and Ben are all U.S. citizens. Ben is Mia’s son. Mia is a foreign resident and citizen. d. Jen, Kathy, Ling, David, and Ben are all U.S. citizens. Mia is a foreign resident and citizen. 52. The following persons own Schlecht Corporation, a non-U.S.entity. Jim, U.S. individual Gina, U.S. individual Marina, U.S. individual Pedro, U.S. individual Chee, non-U.S. individual

35% 15% 8% 12% 30%

None of the shareholders are related. Subpart F income for the tax year is $300,000. No distributions are made. Which of the following statements is correct? a. Schlecht is not a CFC. b. Chee includes $90,000 in gross income. c. Marina is not a U.S. shareholder for purposes of determining whether Schlecht is a CFC. d. Marina includes $24,000 in gross income. 53. Wellington, Inc., a U.S. corporation, owns 30% of a CFC that has $50 million of earnings and profits for the current year. Included in that amount is $20 million of Subpart F income. Wellington has been a CFC for the entire year and makes no distributions in the current year. Wellington must include in gross income: a. $0. b. $6 million. c. $20 million. d. $50 million. 54. A controlled foreign corporation (CFC) realizes Subpart F income from: a. Purchase of inventory from an unrelated U.S. person and sale outside the CFC country. b. Purchase of inventory from a related U.S. person and sale outside the CFC country. Powered by Cognero

Page 7


Name:

Class:

Date:

Chapter 16: Taxation of International Transactions c. Services performed for the U.S. parent in a country in which the CFC was organized. d. Services performed on behalf of an unrelated party in a country outside the country in which the CFC was organized. 55. Purchase of inventory from a U.S. parent followed by which of the following income items does not represent Subpart F income if it is earned by a controlled foreign corporation in Spain? a. Sale to anyone outside Spain. b. Sale to anyone inside Spain. c. Sale to a related party outside Spain. d. Sale to a nonrelated party outside Spain. 56. OutCo, a controlled foreign corporation in Ireland, earns $600,000 in net interest and dividend income from investments in the bonds and stock of unrelated companies. All of the dividend payors are located in Ireland. OutCo’s Subpart F income for the year is: a. $0. b. $0 only if OutCo is engaged in a trade or business in Ireland. c. $600,000. d. $600,000 only if OutCo is engaged in a trade or business in Ireland. 57. Peanut, Inc., a U.S. corporation, receives $500,000 of foreign-source interest income on which foreign taxes of $5,000 are withheld. Peanut’s worldwide taxable income is $900,000, and its U.S. Federal income tax liability before FTC is $189,000. What is Peanut’s foreign tax credit? a. $500,000 b. $189,000 c. $105,000 d. $5,000 58. Which of the following is a specific separate income “basket” for purposes of the foreign tax credit limitation calculation? a. Certain intangible income. b. Portfolio income. c. Business income. d. All of these choices are separate FTC limitation baskets. 59. Kunst, a U.S. corporation, generates $100,000 of foreign-source income in the general income basket and $40,000 of foreign-source income in the passive income basket. Kunst’s worldwide taxable income is $1,200,000, and its U.S. tax liability before FTC is $240,000. Foreign taxes attributable to the general income basket are $60,000 and to the passive income basket are $4,000. What is Kunst’s foreign tax credit for the tax year? a. $64,000 b. $24,000 c. $20,000 d. $4,000 60. Luisa, a non-U.S. person with a green card, spends the following days in the United States. Year 1

360 days

Powered by Cognero

Page 8


Name:

Class:

Date:

Chapter 16: Taxation of International Transactions Year 2 Year 3

210 days 20 days

Luisa’s residency status for year 3 is: a. U.S. resident because she has a green card. b. U.S. resident as she was a U.S. resident for the past immediately preceding two years. c. Not a U.S. resident because Luisa was not in the United states for more than 30 days during year 3. d. Not a U.S. resident as, using the three-year test, Luisa is not present in the United States for at least 183 days. 61. Zhang, an NRA who is not a resident of a treaty country, receives taxable dividends of $50,000 from U.S. corporations. Zhang does not conduct a U.S. trade or business. Zhang’s dividends are subject to withholding by the payor of: a. 35%. b. 30%. c. 15%. d. 0%. 62. Which of the following statements regarding foreign persons not engaged in a U.S. trade or business is true? a. They are subject to potential withholding taxes on the gross amount of U.S.-source investment income. b. If they have any U.S.-source income, they are taxed on net investment income (after expenses). c. They are not subject to U.S. tax if not engaged in a U.S. trade or business. d. If they have only U.S.-source investment income, they are exempt from U.S. tax. 63. The following income of a foreign corporation is not subject to the regular U.S. corporate income tax rates. a. FIRPTA gains. b. Capital gains effectively connected with a U.S. trade or business. c. Net long-term capital gains for which no U.S. trade or business exists. d. Interest income effectively connected with a U.S. trade or business. 64. Which of the following statements regarding the taxation of U.S. real property gains recognized by non-U.S. persons not engaged in a U.S. trade or business is false? Gains from the disposition of U.S. real property are: a. Not taxed to non-U.S. persons because real property gains are specifically exempt from U.S. taxation. b. Taxed to non-U.S. persons without regard to whether such non-U.S. persons are engaged in a U.S. trade or business. c. Taxed in the United States because such gains are treated as if they are effectively connected to a U.S. trade or business. d. Taxed to non-U.S. persons notwithstanding the general exemption of capital gains from U.S. taxation. 65. ForCo, a foreign corporation not engaged in a U.S. trade or business, recognizes a $3 million gain from the sale of land located in the United States. The amount realized on the sale was $50 million. Absent any exceptions, what is the required withholding amount on the part of the purchaser of this land? a. $0 b. $300,000 c. $3 million d. $5 million Powered by Cognero

Page 9


Name:

Class:

Date:

Chapter 16: Taxation of International Transactions 66. Which of the following situations requires the filing of an information return with the U.S. government? a. A U.S. corporation that is 25% or more foreign owned. b. A foreign corporation carrying on a trade or business in the United States. c. U.S. persons who acquire or dispose of an interest in a foreign partnership. d. All of these taxpayers must file a U.S. return. 67. In working with the foreign tax credit, a U.S. corporation may be able to alleviate the problem of excess foreign taxes by: a. Deducting the excess foreign taxes that do not qualify for the credit. b. Repatriating more foreign income to the United States in the year there is an excess limitation. c. Generating “same basket” foreign-source income that is subject to a tax rate higher than the U.S. tax rate. d. Generating “same basket” foreign-source income that is subject to a tax rate lower than the U.S. tax rate. 68. Dark, Inc., a U.S. corporation, operates Dunkel, an unincorporated branch manufacturing operation in Germany. Dark reports $100,000 of taxable income from Dunkel on its U.S. tax return along with $400,000 of taxable income from its U.S. operations. Dark paid $30,000 in German income taxes related to the $100,000 of Dunkel income. Assuming a U.S. tax rate of 21%, what is Dark’s U.S. tax liability after any allowable foreign tax credits? a. $21,000 b. $75,000 c. $84,000 d. $105,000 69. Which of the following statements regarding the U.S. taxation of non-U.S. persons is true? a. A non-U.S. person’s effectively connected U.S. business income is taxed by the United States only if it is portfolio income. b. A non-U.S. person’s effectively connected U.S. business income is subject to U.S. income taxation. c. A non-U.S. person may earn income from selling U.S. real property without incurring any U.S. income tax. d. A non-U.S. person must spend at least 183 days in the United States before any effectively connected income is subject to U.S. taxation. 70. Which of the following statements regarding a non-U.S. person’s U.S. tax consequences is true? a. Non-U.S. persons may be subject to withholding tax on U.S.-source investment income even if not engaged in a U.S. trade or business. b. Non-U.S. persons are subject to U.S. income or withholding tax only if they are engaged in a U.S. trade or business. c. Non-U.S. persons are not taxed on gains from U.S. real property as long as such property is not used in a U.S. trade or business. d. Once a non-U.S. person is engaged in a U.S. trade or business, the non-U.S. person’s worldwide income is subject to U.S. taxation. 71. Which of the following is not a U.S. person? a. U.S. corporation. b. Citizen of Turkey with U.S. permanent residence status (i.e., green card). c. U.S. corporation 100% owned by a foreign corporation. d. Foreign corporation 100% owned by a U.S. corporation. Powered by Cognero

Page 10


Name:

Class:

Date:

Chapter 16: Taxation of International Transactions 72. Which of the following is not a foreign person? a. A foreign corporation 51% owned by U.S. shareholders. b. A foreign corporation 100% owned by a U.S. corporation. c. A citizen of Germany with U.S. permanent resident status (i.e., green card). d. A citizen of Italy who spends 14 days vacationing in the United States. 73. Yvonne is a citizen of France and does not have permanent resident status in the United States. During the last three years, she has spent a number of days in the United States. Current year – 150 days First prior year – 150 days Second prior year – 90 days Is Yvonne treated as a U.S. resident for the current year? a. No, because Yvonne is a citizen of France. b. No, because Yvonne was not present in the United States at least 183 days during the current year. c. No, because although Yvonne was present in the United States at least 31 days during the current year, she was not present at least 183 days in a single year during the current or prior two years. d. Yes, because Yvonne was present in the United States at least 31 days during the current year and 215 days during the current and prior two years (using the appropriate fractions for the prior years). 74. Magdala is a citizen of Italy and does not have permanent resident status in the United States. During the last three years, she has spent a number of days in the United States. Current year – 120 days First prior year – 150 days Second prior year – 150 days Is Magdala treated as a U.S. resident for the current year? a. Yes, because Magdala was present in the United States at least 31 days during the current year and 195 days during the current and prior two years (using the appropriate fractions for the prior years). b. No, because Magdala is a citizen of Italy. c. No, because Magdala was not present in the United States at least 183 days during the current year. d. No, because although Magdala was present in the United States at least 31 days during the current year, she was not present at least 183 days in a single year during the current or prior two years. 75. Which of the following is a principle used in applying the income-sourcing rules under U.S. tax law? a. The rules should be acceptable to both countries. b. The rules should favor the U.S. Treasury. c. The rules should favor the treasury of the non-U.S. country. d. The rules should apply to income items only; deductions need not be sourced in this way. 76. Which of the following statements regarding income sourcing is correct? a. Everything else being equal, a larger foreign-source income decreases the foreign tax credit limitation for U.S. persons. b. Everything else being equal, a larger foreign-source income increases the foreign tax credit limitation for U.S. persons. c. Everything else being equal, a larger U.S.-source income increases the foreign tax credit limitation for U.S. Powered by Cognero

Page 11


Name:

Class:

Date:

Chapter 16: Taxation of International Transactions persons. d. Everything else being equal, changing foreign-source income does not change the foreign tax credit limitation for U.S. persons. 77. Which of the following statements regarding income sourcing is not correct? a. Concerning the foreign tax credit, most U.S. persons benefit from earning low-tax foreign-source income. b. Foreign persons generally benefit from avoiding U.S.-source income classification. c. U.S. persons are not concerned with source of income because all their income is subject to U.S. tax under a worldwide system. d. Foreign persons may be subject to tax on U.S.-source income without regard to their actual presence in the United States. 78. ForCo, a foreign corporation, receives interest income of $50,000 from USCo, an unrelated U.S. corporation. USCo historically has earned 79% of its gross income from active foreign-source business income. What amount of ForCo’s interest income is U.S.-source? a. $0 b. $10,500 c. $39,500 d. $50,000 79. WorldCo, a foreign corporation not engaged in a U.S. trade or business, receives $50,000 in interest income from deposits with the foreign branch of a U.S. bank. The U.S. bank earns 78% of its income from foreign sources. How much of WorldCo’s interest income is U.S. source? a. $0 b. $11,000 c. $39,000 d. $50,000 80. GlobalCo, a foreign corporation not engaged in a U.S. trade or business, receives $80,000 in interest income from deposits with the foreign branch of a U.S. bank. The U.S. bank earns 24% of its income from foreign sources. How much of GlobalCo’s interest income is U.S. source? a. $0 b. $19,200 c. $60,800 d. $80,000 81. Which of the following statements is true concerning the sourcing of income from inventory produced by the taxpayer in the United States and sold outside the United States? a. Because the inventory is manufactured in the United States, all of the inventory income is U.S. source. b. If title passes on the inventory outside the United States, all of the inventory income is foreign source. c. The taxpayer may source one-half the income based on title passage and one-half the income based on where the sale negotiation takes place. d. The taxpayer may source one-half the income based on title passage and one-half the income based on location of production assets. 82. AirCo, a U.S. corporation, purchases inventory for resale from unrelated distributors within the United States and Powered by Cognero

Page 12


Name:

Class:

Date:

Chapter 16: Taxation of International Transactions resells this inventory to customers outside the United States with title passing outside the United States. What is the sourcing of AirCo’s inventory sales income? a. 100% U.S. source. b. 100% foreign source. c. 50% U.S. source and 50% foreign source. d. 50% foreign source and 50% sourced based on location of manufacturing assets. 83. WaterCo, a U.S. corporation, purchases inventory for resale from unrelated distributors outside the U.S. It resells this inventory to U.S. customers with title passing inside the United States. What is the sourcing of WaterCo’s inventory sales income? a. 100% U.S. source. b. 100% foreign source. c. 50% U.S. source and 50% foreign source. d. 50% foreign source and 50% sourced based on location of manufacturing assets. 84. RainCo, a U.S. corporation, owns a number of patents related to designing umbrellas. RainCo licenses these patents to unrelated parties. TexCo, a U.S. corporation, paid RainCo $100,000 in royalties related to these licenses. TexCo uses the patent information in its manufacturing process in its Canadian plant. IrishCo, an Irish corporation, paid RainCo $25,000 in royalties related to the licenses. IrishCo uses the patent information in its manufacturing process in its Michigan manufacturing plant. How much U.S.-source royalty income did RainCo earn from these licenses? a. $0 b. $25,000 c. $100,000 d. $125,000 85. BunCo, a U.S. corporation, owns a number of patents related to designing sunglasses. BunCo licenses these patents to unrelated parties. SpainCo, a Spanish corporation, paid BunCo $78,000 in royalties related to these licenses. SpainCo uses the patent information in its manufacturing process in its Texas plant. WiscCo, a U.S. corporation, paid BunCo $32,000 in royalties related to the licenses. WiscCo uses the patent information in its manufacturing process in its Germany manufacturing plant. How much U.S.-source royalty income did BunCo earn from these licenses? a. $0 b. $32,000 c. $78,000 d. $110,000 86. Which of the following statements regarding the sourcing of dividend income is true? a. Dividends are sourced based on the residence of the recipient. b. Dividends from a U.S. corporation are U.S.-source based on the percentage of U.S.-source income earned by the U.S. payor. c. Dividends from a U.S. corporation are U.S.-source based without regard to where the U.S. corporation generated the E & P. d. Dividends from a U.S. corporation are foreign-source based on the percentage of foreign-source income earned by the U.S. payor. 87. Which of the following statements regarding the sourcing of dividend income is true? a. Dividends are sourced based on the residence of the recipient. Powered by Cognero

Page 13


Name:

Class:

Date:

Chapter 16: Taxation of International Transactions b. Dividends from non-U.S. corporations are always foreign source. c. Dividends from non-U.S. corporations are foreign-source only to the extent that 80% or more of the non-U.S. corporation’s gross income for the three years preceding the year of the dividend payment was effectively connected with the conduct of a non-U.S. trade or business. d. A percentage of dividends from non-U.S. corporations are U.S. source to the extent that 25% or more of the non-U.S. corporation’s gross income for the three years preceding the year of the dividend payment was effectively connected with the conduct of a U.S. trade or business. 88. Which of the following statements best describes the primary purpose of the Subpart F income provisions? a. They allow for a deferral of non-U.S.-source income from U.S. taxation. b. They provide certainty as to the U.S. income tax treatment of cross-border transactions. c. They prevent shifting of income from the United States to high-tax non-U.S. jurisdictions. d. They prevent shifting of income from the United States to low-tax non-U.S. jurisdictions. 89. USCo, a U.S. corporation, receives $700,000 of foreign-source passive income on which foreign taxes of $70,000 are withheld. Its worldwide taxable income is $1,500,000, and its U.S. tax liability before the foreign tax credit is $315,000. What is USCo’s allowed foreign tax credit? a. $70,000 b. $147,000 c. $315,000 d. $385,000 90. Which of the following foreign taxes paid by a U.S. corporation may be eligible for the foreign tax credit? a. Real property taxes. b. Value added taxes. c. Sales taxes. d. Dividend withholding taxes. 91. Under a territorial income tax system, a country assesses an income tax on: a. Income of all entities earned within its borders. b. Income of its citizens earned in other countries. c. Both a. and b. d. Neither a. nor b. 92. U.S. income tax treaties: a. Involve three to seven countries as treaty partners. b. Are renewable upon expiration every five years. c. Are rare with countries in Africa. d. Are rare with countries in Europe. 93. U.S. income tax treaties can be described as: a. Napoleonic. b. Spoke-and-Wheel. c. Balanced. d. Bilateral. Powered by Cognero

Page 14


Name:

Class:

Date:

Chapter 16: Taxation of International Transactions 94. Identifying an income item as U.S source creates U.S. taxable income for a: a. Non-U.S. taxpayer. b. U.S. taxpayer. c. Both a. and b. d. Neither a. nor b. 95. An effective transfer pricing strategy would: a. Have a parent entity assess a management fee from a subsidiary in a low-tax country. b. Decrease the price of inventory that is sold by a parent to a subsidiary in a low-tax country. c. Both a. and b. d. Neither a. nor b. 96. A foreign currency gain or loss might occur in a year when: a. A U.S. person makes a sale that is denominated in U.S. dollars. b. A sales contract is signed in year 1 and the transaction is completed in year 2. c. Both a. and b. d. Neither a. nor b. 97. A U.S. parent can deduct the current operating loss of a non-U.S. entity that it owns if the non-U.S. entity is operated as a: a. Branch. b. Subsidiary. c. Both a. and b. d. Neither a. nor b. 98. Blue Co., a U.S. corporation, owns 100% of Red Co., a controlled foreign corporation. Red Co. has tested

income of $1,000,000, qualified business asset investment of $500,000, and specified interest expense of $10,000. How much of Red's income must Blue recognize immediately in its own taxable income (i.e., what is Blue Co's GILTI)? a. $960,000 b. $950,000 c. $500,000 d. $490,000 Matching Match the definition with the correct term. a. Inbound b. Outbound c. Allocation and apportionment d. Qualified business unit e. Income tax treaty f. Section 482 Powered by Cognero

Page 15


Name:

Class:

Date:

Chapter 16: Taxation of International Transactions 99. Bilateral agreement between two countries related to tax issues. 100. U.S. taxpayers earning income outside the United States. 101. Foreign taxpayers earning income inside the United States. 102. Method for sourcing income and deductions. 103. Treasury powers to recast the results of transfer pricing. 104. A business operation that accounts for profits and losses using its functional currency. Match the definition with the correct term. Not all of the terms have a match. A definition can be used more than once. a. Foreign base company income b. Foreign personal holding company income c. Controlled foreign corporation d. U.S. shareholder e. Previously taxed income f. More than 10 percent g. More than 50 percent h. More than 80 percent 105. Owner of shares counted in determining whether a foreign corporation is a controlled foreign corporation. 106. A CFC’s profits from sales of goods and services. 107. Portfolio income treated as Subpart F income. 108. A non-U.S. subsidiary whose income may be taxed to the U.S. parent before repatriation. 109. Upon repatriation to a CFC, it does not create dividend income. 110. Ownership threshold for U.S. shareholders to be deemed a controlled foreign corporation. Match the definition with the correct term. a. Expatriate b. Resident c. Nonresident alien d. U.S. trade or business e. Effectively connected income 111. Activity that creates the potential for effectively connected income. 112. Individual who is not a U.S. citizen or resident. 113. A non-U.S. citizen who holds a “green card.” 114. An individual who gives up U.S. citizenship to avoid U.S. income taxes. Powered by Cognero

Page 16


Name:

Class:

Date:

Chapter 16: Taxation of International Transactions 115. Income of foreign person taxed through filing of a U.S. tax return with deductions allowed against gross income. Subjective Short Answer 116. During year 4, Josita, an NRA, receives interest income of $50,000 from Talmadge, Inc., an unrelated U.S. corporation. Considering the following facts related to Talmadge’s operations, what is the sourcing of the interest income received by Josita?

Tax Year Year 1 Year 2 Year 3 Totals

U.S.-source Income $200,000 50,000 100,000 $350,000

Active Foreign Business Income $ 500,000 950,000 900,000 $2,350,000

Total Gross Income $ 700,000 1,000,000 1,000,000 $2,700,000

Year 4

$150,000

$ 950,000

$1,100,000

117. Goolsbee, Inc., a U.S. corporation, generates U.S.-source and foreign-source gross income. Goolsbee’s assets (tax basis) are as follows. Generating U.S.-source income Generating foreign-source income Total

$15,000,000 25,000,000 $40,000,000

Goolsbee incurs interest expense of $200,000. Apportion interest expense to foreign-source income. 118. Arendt, Inc., a U.S. corporation, purchases equipment for use in its manufacture of custom pianos. The equipment is acquired in Ireland at a cost of 200,000 euros when 1€: $1.25. Payment is due in 90 days. Arendt acquires 200,000 euros and pays for the machine when 1€: $1.15. What is the basis of the asset to Arendt and what is the foreign currency exchange gain or loss, if any? 119. Given the following information, determine whether FanCo, a foreign corporation, is a CFC. Shareholders of Voting Foreign Corporation Power Murray 24% Nancy 20% Otto 40% Patricia 16% Patricia is Murray’s daughter.

Classification U.S. person U.S. person Foreign person U.S. person

120. Britta, Inc., a U.S. corporation, reports foreign-source income and pays foreign taxes as follows. Passive category General limitation category

Income $200,000 800,000

Taxes $ 10,000 350,000

Britta’s worldwide taxable income is $1,600,000 and U.S. taxes before FTC are $336,000 (21% tax rate). What is Britta’s U.S. tax liability after the FTC? Powered by Cognero

Page 17


Name:

Class:

Date:

Chapter 16: Taxation of International Transactions 121. Given the following information, determine whether Greta, an alien, is a U.S. resident for year 3. Greta cannot establish a tax home in or a closer connection to a foreign country.

Year 3 2 1

Number of Days in the United States 120 150 240

Essay 122. With respect to income generated by non-U.S. persons, does the U.S. apply a “worldwide” or a “territorial” approach. Be specific. 123. Discuss the primary purposes of income tax treaties. 124. In international corporate income taxation, what are the uses of the “sourcing rules” in computing Federal taxable income? 125. The § 367 cross-border transfer rules seem to counteract other favorable tax provisions that allow the taxpayer to defer gross income (e.g. §§ 351 and 368.) What is the rationale for eliminating this deferral? Provide two examples of transactions to which § 367 would apply. 126. Your client holds foreign tax credit (FTC) carryforwards (i.e., it is in an “excess credit” position.) Give at least three planning ideas that the client should implement to free up the suspended FTCs.

Powered by Cognero

Page 18


Name:

Class:

Date:

Chapter 16: Taxation of International Transactions Answer Key 1. True 2. False 3. True 4. True 5. False 6. False 7. False 8. False 9. True 10. False 11. True 12. True 13. False 14. True 15. False 16. False 17. True 18. True 19. False 20. True 21. True 22. False 23. False 24. False 25. False Powered by Cognero

Page 19


Name:

Class:

Date:

Chapter 16: Taxation of International Transactions 26. False 27. False 28. False 29. False 30. False 31. False 32. False 33. False 34. True 35. False 36. True 37. True 38. True 39. c 40. c 41. c 42. a 43. c 44. d 45. b 46. b 47. c 48. a 49. c 50. a Powered by Cognero

Page 20


Name:

Class:

Date:

Chapter 16: Taxation of International Transactions 51. a 52. c 53. b 54. b 55. b 56. c 57. d 58. d 59. b 60. a 61. b 62. a 63. c 64. a 65. d 66. d 67. d 68. c 69. b 70. a 71. d 72. c 73. d 74. a 75. a 76. b Powered by Cognero

Page 21


Name:

Class:

Date:

Chapter 16: Taxation of International Transactions 77. c 78. d 79. a 80. a 81. a 82. b 83. a 84. b 85. c 86. c 87. d 88. d 89. a 90. d 91. a 92. c 93. d 94. a 95. b 96. d 97. a 98. a 99. e 100. b 101. a Powered by Cognero

Page 22


Name:

Class:

Date:

Chapter 16: Taxation of International Transactions 102. c 103. f 104. d 105. d 106. a 107. b 108. c 109. e 110. g 111. d 112. c 113. b 114. a 115. e 116. Talmadge meets the 80% active foreign business requirement; thus, the interest income received by Josita is entirely foreign source. The 80% test is calculated by taking Talmadge’s active foreign business income for the immediately preceding three tax years over its total gross income for those years ($2,350,000/$2,700,000 = 87%). 117.

118. No foreign currency exchange gain or loss is recognized until the payment is made. The cost of $250,000 (200,000€ × $1.25) is recorded as the basis of the equipment. When payment is made, the foreign currency exchange gain is $20,000 ($250,000 – $230,000). Arendt effectively pays $230,000 for the 200,000 euros needed to make payment (200,000€ × $1.15) before accounting for the tax on the currency gain. 119. Shareholder Murray Nancy Patricia Powered by Cognero

Voting Power Held Directly 24% 20% 16% 60%

Voting Power Held Constructively 16% (via Patricia) 24% (via Murray)

Total Voting Power 40% 20% 40% Page 23


Name:

Class:

Date:

Chapter 16: Taxation of International Transactions Murray, Nancy, and Patricia are U.S. shareholders for purposes of CFC determination because all own 10% or more (directly or indirectly) of the corporation’s voting power. Murray owns 40% (24% directly and 16% constructively through Patricia). Nancy owns 20% directly. Patricia also owns 40% (16% directly and 24% constructively through Murray). The corporation is a CFC because U.S. shareholders own 60% of the voting power. Constructive voting power is not counted twice in making this determination. It is counted only in determining whether the U.S. persons are U.S. shareholders. Voting power held indirectly (i.e., through a foreign corporation) is counted in determining whether a foreign corporation is a CFC. If Patricia were not related to Murray or Nancy, Patricia would still be a U.S. shareholder (she holds 10% or more of the voting power directly), and the corporation would still be a CFC.

120. The FTC is computed separately for each income basket. Total FTC = $178,000 ($10,000 + $168,000). Net U.S. tax liability = $158,000 ($336,000 – $178,000). FTC—Passive Basket FTC is the lesser of foreign taxes paid ($10,000) or the limitation: $336,000 × $200,000/$1,600,000 = $42,000. FTC = $10,000 FTC—General Basket FTC is the lesser of foreign taxes paid ($350,000) or the limitation: $336,000 × $800,000/$1,600,000 = $168,000. FTC = $168,000 121. In general, for Federal income tax purposes, an individual is considered a U.S. resident if the individual is present in the United States for at least 183 days during a year. If this test is not met, a separate computation determines residency based on the current and prior two years (with current-year days counting in full, preceding year days counting as 1/3, and second preceding year days counting as 1/6). In year 3, Greta is only present in the United States 120 days. Under the separate computation, Greta is a U.S. resident for year 3, because she was physically present for 210 days during the three-year testing period. Year 1: 1/6 × 240 days = Year 2: 1/3 × 150 days = Year 3: 1/1 × 120 days =

40 days 50 days 120 days 210 total days

If Greta would prefer a different result, she could establish a tax home in and a closer connection to a foreign country for year 3, she could avoid U.S. residency; she is present in the United States for fewer than 183 days during year 3. 122. U.S. persons are subject to worldwide taxation with the foreign tax credit and tax treaty provisions allowing for a mitigation of some exposure to double taxation. Large multinational C corporations can apply provisions that resemble territorial taxation, resulting in a hybrid model. Non-U.S. persons generally are subject to territorial approach (i.e., being taxed only on U.S.-sited business and portfolio income items.) 123. The primary purpose of an income tax treaty is to eliminate or reduce the double taxation of persons resident in one Powered by Cognero

Page 24


Name:

Class:

Date:

Chapter 16: Taxation of International Transactions country earning income from sources within the treaty-partner country. Tax treaties can override the Code and generally provide lower tax burdens as compared with statutory tax provisions of a country. 124. The sourcing of income and deductions inside and outside the United States has a direct bearing on a number of tax provisions affecting both U.S. and foreign taxpayers. For example, foreign taxpayers generally are taxed only on income sourced inside the United States, and U.S. taxpayers receive relief from double taxation under the foreign tax credit rules based on their foreign-source income. Accordingly, an examination of sourcing rules is often the starting point in addressing international tax issues. 125. Section 367 provides for the immediate taxation of transactions that would otherwise be tax deferred under U.S. law (e.g., § 351 transfers, § 332 liquidations, etc.). Because assets with potential gain are being transferred outside the U.S. taxing jurisdiction, § 367 preserves the ability of the United States to tax this gain currently or, in some circumstances, in the future. Example transactions include an incorporation of a foreign branch, transfer of U.S. assets to a foreign subsidiary, liquidation of a U.S. subsidiary into a foreign parent, and liquidation of a foreign subsidiary into a U.S. parent. 126. •

Generate “same basket” foreign-source income that is subject to a tax rate lower than the taxpayer’s marginal U.S. tax rate.

Time the repatriation of foreign-source earnings to coincide with “excess limitation” years.

Deduct foreign taxes in years when the deduction benefit exceeds the FTC benefit.

Convert deductions related to foreign-source income so that they now relate to U.S.-source income instead.

Powered by Cognero

Page 25


Name:

Class:

Date:

Chapter 17: Tax Practice and Ethics True / False 1. The tax professional can do more than just tax compliance work. They can work with the client in consultation over the strategy and tactics of dealing with a Federal tax audit. a. True b. False 2. The IRS employs more than 79,000 personnel, making it one of the largest Federal agencies. a. True b. False 3. Recently, the overall Federal income tax audit rate for the Form 1040 has been under 1%. a. True b. False 4. The IRS targets high-income individuals for an audit rate that is higher than that of the general populace. a. True b. False 5. IRS computers use document matching programs for both individuals and business taxpayers to keep the audit rate low. a. True b. False 6. The IRS’s top legal officer is known as the Chief Counsel. a. True b. False 7. The IRS is organized according to the industry classification of the taxpayer. One of the operating divisions of the IRS deals exclusively with manufacturing and exporting businesses. a. True b. False 8. In a letter ruling, the IRS responds to a taxpayer's request concerning the tax treatment of a proposed transaction. a. True b. False 9. An IRS letter ruling might determine that an employee’s compensation is unreasonable in amount. a. True b. False 10. Taxpayers must pay a significant fee to have a letter ruling issued by the IRS. a. True b. False 11. The IRS can require that the taxpayer produce its financial accounting records to determine whether taxable income is computed correctly. Powered by Cognero

Page 1


Name:

Class:

Date:

Chapter 17: Tax Practice and Ethics a. True b. False 12. When a tax issue is taken to court, the burden of proof is on the IRS to show that its audit adjustments are correct. a. True b. False 13. When the IRS issues a notice of tax due, the taxpayer has 90 days to either pay the tax or file a petition with the Tax Court. This is conveyed in the “90-day letter.” a. True b. False 14. Most IRS audits take place strictly through correspondence with the taxpayer. a. True b. False 15. After a tax audit, the taxpayer receives the Revenue Agent’s Report as part of the “30-day letter.” a. True b. False 16. The government can appeal a decision of the Tax Court Small Cases Division, but the taxpayer cannot. a. True b. False 17. Cheng filed an amended return this year, claiming a refund relative to her tax computation on a prior-year return. When the IRS approves the amended return and issues the refund 50 days later, it also pays Cheng interest with respect to the overpayment. a. True b. False 18. During any month in which both the failure-to-file and failure-to-pay penalties apply, both penalties are paid in full. a. True b. False 19. Maria and Miguel Blanco are in the midst of negotiating a divorce. Because both parties are unwilling to share any current financial information, their joint Form 1040 for the year is not filed until October 31 of the next year, when the respective divorce attorneys forced them to cooperate. The Blancos should not be subject to any Federal late-filing penalties, because the reasonable cause exception applies to their family discord. a. True b. False 20. A negligence penalty is assessed when the taxpayer is found not to have made a reasonable attempt to comply with the tax law. a. True b. False Powered by Cognero

Page 2


Name:

Class:

Date:

Chapter 17: Tax Practice and Ethics 21. Jaime’s negligence penalty will be waived under the reasonable cause exception. He told the court, “My taxes were wrong because I couldn’t understand the tax law.” a. True b. False 22. Because he undervalued property that he transferred by gift, Dan owes additional gift taxes of $4,000. The penalty for undervaluation does not apply in this situation, because the tax understatement was too small. a. True b. False 23. In the context of civil tax fraud litigation, the burden of proof is on the taxpayer to show the court by a preponderance of the evidence that they were not acting with an intent to evade a tax. a. True b. False 24. In a criminal tax fraud case, the burden is on the taxpayer to show that they were innocent beyond the shadow of any reasonable doubt. a. True b. False 25. Yang, a calendar year taxpayer, did not file a tax return for 2020 because she honestly believed that no additional tax was due. In 2025, Yang is audited by the IRS, and the agent assesses a deficiency of $17,000. Yang need not pay this deficiency, since the three-year statute of limitations expired on April 15, 2024, meaning that the IRS no longer can adjust Yang’s tax. a. True b. False 26. Kevin did not report the gain on the sale of his vacation home. He mistakenly thought that it was not subject to tax. As a result, the gross income on his tax return was only two-thirds of what it should have been. The statute of limitations for Kevin’s return is six years. a. True b. False 27. In the case of bad debts and worthless securities, the statute of limitations on claims for refund is three years. a. True b. False 28. Jenny has prepared Steve’s income tax returns for no compensation since he got his first job. Jenny is Steve’s mother. This year, the IRS notifies Steve that it will audit his returns for a prior tax year. Jenny cannot represent Steve during the audit of the returns, because she is not a registered tax return preparer. a. True b. False 29. Circular 230 applies to all paid tax practitioners. But attorneys, CPAs, and enrolled agents are exempt from the Circular 230 rules because each of the groups has its own code of professional conduct. a. True b. False Powered by Cognero

Page 3


Name:

Class:

Date:

Chapter 17: Tax Practice and Ethics 30. Circular 230 requires that a paid tax preparer apply a quality review system in preparing tax returns for her or his clients. a. True b. False 31. Under Circular 230, Basu cannot complete a client’s original Form 1040 and charge a fee equal to one-third of the resulting refund. a. True b. False 32. As part of a tax return engagement for XYZ Partnership, Enrolled Agent Wang can draft an amendment to the XYZ partnership agreement. a. True b. False 33. Circular 230 compliance implies that a tax preparer provide training for the tax staff as to the latest changes in the tax law. a. True b. False 34. Under Circular 230, tax preparer Winston cannot disclose to a mortgage banker the income level of her client Pickett or other information acquired by preparing the return without Pickett’s permission. a. True b. False 35. Fiona, a VITA volunteer for her college’s tax clinic, is not a tax preparer as defined by the Code. Thus, Fiona is exempted from the Code’s tax preparer penalties. a. True b. False 36. Latrelle, who prepares the tax return for Whitehall Corporation includes a $5,000 deduction on the return. This type of deduction previously has been disallowed by the Tax Court, although there is a 15% chance that the holding will be reversed on an appeal by Whitehall. The return does not make any special disclosure that the deduction is being claimed. Whitehall paid Latrelle a fee of $8,000 for preparing the Form 1120. Latrelle will be assessed a preparer penalty of $4,000 for taking an unreasonable position on the Whitehall return. a. True b. False 37. CPA Norma and her client Colin hold a privilege of confidentiality from the IRS as to their tax planning discussions about completing Colin’s tax returns. The IRS cannot successfully subpoena records concerning these discussions. a. True b. False 38. The Statements on Standards for Tax Services (SSTS) apply to members of the AICPA, a state bar association, and all enrolled agents. a. True Powered by Cognero

Page 4


Name:

Class:

Date:

Chapter 17: Tax Practice and Ethics b. False 39. The AICPA’s Statements on Standards for Tax Services (SSTS) provide principles-based guidelines as to how all tax professionals should conduct business. a. True b. False 40. A CPA can take a tax return position for a client that is contrary to current IRS interpretations of the law. a. True b. False 41. Faye, a CPA, is preparing Judith’s tax return. Last year, Judith’s return included dividend income from the P&Q Company. This year, Judith reports no such income. Faye should inquire as to whether Judith sold the P&Q stock during the year. a. True b. False 42. In preparing a tax return, a CPA should verify to the penny every item of information submitted by a client about its deduction for repairs and maintenance. a. True b. False 43. Last year, Ned’s property tax deduction on his residence was $22,500. Although he lives in the same house, he tells his CPA that this year’s taxes will be only $7,500. The CPA can use this estimate in computing Ned’s itemized deductions under the Statements of Standards for Tax Services. a. True b. False 44. Ling has hired CPA Gracie to complete this year’s Form 1040. Ling uses online accounting software to keep the books for her civil engineering sole proprietorship. Ling tells Gracie that a $5,000 amount for business supplies is “close enough” to report for this year’s deduction. Gracie can use this estimate in completing Form 1040. a. True b. False 45. CPA Liam discovers that last year’s Form 1120 for his client PollCo claimed a $100,000 advertising deduction for a gift to candidates of the Green Party. AICPA tax ethics rules require that Liam file an amended return immediately because political expenditures are not deductible. a. True b. False 46. CPA Shearer is required by AICPA tax ethics rules to inform her client Martinez of significant changes in the Federal income tax laws. a. True b. False 47. It typically is advisable that an IRS audit be conducted at the office of the tax adviser, not of the client. a. True Powered by Cognero

Page 5


Name:

Class:

Date:

Chapter 17: Tax Practice and Ethics b. False 48. The tax professional can reduce the chances that staff personnel will incur IRS preparer penalties by adopting a “tone at the top” that stresses integrity, diligence, and other elements of an ethical tax practice. a. True b. False Multiple Choice 49. The Commissioner of the IRS is appointed by the: a. Secretary of the Treasury Department. b. U.S. President. c. U.S. House of Representatives. d. SEC Commissioner. 50. This official does not report directly to the IRS Commissioner. a. Treasury Secretary. b. Director of Compliance Analytics. c. Director of the Whistleblower Office. d. Chief Appeals Officer. 51. The National Taxpayer Advocate reports to the: a. Treasury Secretary. b. IRS Commissioner. c. IRS Whistleblower Office. d. IRS Director of Professional Responsibility. 52. After receiving a 90-day letter, the taxpayer has 90 days to: a. Pay any outstanding tax, interest, and penalties. b. Appeal the dispute to the Tax Court. c. Either a. or b. d. Neither a. nor b. 53. Which statement is correct as to the conduct of IRS income tax audits? a. A correspondence audit is conducted by mail. b. Field audits are most common for Forms 1040. c. The most common type of Federal income tax audit is an office audit. d. A correspondence audit usually is concluded after a meeting with the taxpayer at the IRS auditor’s office. 54. If a former employee wants to give information to the IRS about shady tax dealings about the employer when the disputed amount is $2,500,000, the best approach is to use the: a. Informants Program. b. Whistleblower Program. c. Verification Program. d. Compromise Program. Powered by Cognero

Page 6


Name:

Class:

Date:

Chapter 17: Tax Practice and Ethics 55. Which statement is incorrect as to the conduct of IRS income tax audits? a. The taxpayer can obtain an audio recording of their interview with an IRS auditor. b. The taxpayer can suspend an interview with an IRS auditor, so that their CPA can be brought in. c. The IRS publishes the factors that its computers use for audit selection purposes annually in the Commissioner’s Report. d. For a Form 1040 that is filed on April 11, if the taxpayer has not received an audit notification from the IRS by the end of the year, the return still may be audited. 56. With respect to the Small Cases Division of the Tax Court,: a. The taxpayer (but not the IRS) can appeal a contrary judgment. b. The IRS (but not the taxpayer) can appeal a contrary judgment. c. Either the IRS or the taxpayer can appeal a contrary judgment. d. Neither the IRS nor the taxpayer can appeal a contrary judgment. 57. Singh has agreed with the IRS that she owes $70,000 in delinquent tax, penalties, and interest. As to her application for an installment agreement by which to pay the amounts due: a. Singh can file an online application for the installment plan. b. The IRS must adopt Singh’s proposal for the installment plan. c. Both a. and b. d. Neither a. nor b. 58. Which of the following statements correctly reflects the rules governing interest to be paid on an individual’s Federal tax deficiency or claim for refund? a. The IRS has full discretion in determining the rate that will apply. b. The simple interest method for calculating interest is used. c. IRS interest compounds daily. d. Congress sets the IRS interest rate twice each year. 59. Which of the following statements does not reflect the rules governing the accuracy-related penalty for negligence? a. The penalty rate is 20%. b. The penalty applies whenever the taxpayer takes a return position that is contrary to a court decision. c. The penalty applies when the taxpayer does not keep records adequate to compute the tax correctly. d. The penalty is waived if the taxpayer uses Form 8275 to disclose a return position that is reasonable but contrary to the IRS position. 60. An individual is likely to be incarcerated upon a conviction for: a. Using an improper appraisal to compute a tax deduction. b. Filing a tax refund claim that the Tax Court denies. c. Committing criminal tax fraud. d. Failure to pay to the Treasury one pay period’s withholdings from employees. 61. On July 1, Hernandez files a refund claim with the IRS, and the IRS agrees with his computations. The IRS can avoid paying any interest to Hernandez if: a. The total tax involved does not exceed $5,000 (indexed for inflation). Powered by Cognero

Page 7


Name:

Class:

Date:

Chapter 17: Tax Practice and Ethics b. The IRS pays Hernandez the amount owed on August 10. c. Both a. and b. d. Neither a. nor b. 62. The penalty for substantial understatement of tax liability does not apply if: a. The taxpayer has substantial authority for the treatment taken on the tax return. b. The relevant facts affecting the treatment are adequately disclosed in the return or on Form 8275. c. The IRS failed to meet its burden of proof in showing the taxpayer’s error. d. All of these statements are correct. 63. Juarez (a calendar year taxpayer) donates a painting to a local art museum (a qualified charity). The painting cost Juarez $2,000 10 years ago and, according to one of Juarez’s friends (an amateur artist), now is worth $40,000. On his income tax return, Juarez deducts $40,000 as a Form 1040 charitable contribution. Upon later audit by the IRS, it is determined that the true value of the painting was $30,000. Juarez is subject to a 24% marginal Federal income tax rate, so his penalty for overvaluation is: a. $10,000 (minimum penalty). b. $5,000. c. $2,400. d. $2,000. e. $0. 64. Malik, Inc., a calendar year C corporation subject to a 21% marginal income tax rate, claimed a Form 1120 charitable contribution deduction of $30,000 for a sculpture that the IRS later valued at $10,000. The applicable overvaluation penalty is: a. $0. b. $4,200. c. $10,000 (minimum penalty). d. $20,000. 65. Michelle, a calendar year taxpayer subject to a 24% marginal Federal income tax rate, claimed a Form 1040 charitable contribution deduction of $275,000 for a sculpture that the IRS later valued at $200,000. The applicable overvaluation penalty is: a. $0. b. $3,600. c. $7,200. d. $18,000. 66. Lisa, a calendar year taxpayer subject to a 32% marginal Federal income tax rate, claimed a Form 1040 charitable contribution deduction of $250,000 for a sculpture that the IRS later valued at $160,000. The applicable overvaluation penalty is: a. $0. b. $5,760. c. $10,000 (maximum penalty). d. $11,520. 67. Georgio, a calendar year taxpayer subject to a 32% marginal Federal income tax rate, claimed a Form 1040 charitable Powered by Cognero

Page 8


Name:

Class:

Date:

Chapter 17: Tax Practice and Ethics contribution deduction of $300,000 for a sculpture that the IRS later valued at $120,000. The applicable overvaluation penalty is: a. $-0-. b. $10,000 (maximum penalty). c. $11,520. d. $23,040. e. $60,000. 68. Lola, a calendar year taxpayer subject to a 40% marginal Federal gift tax rate, made a gift of a sculpture to Redd, valuing the property at $70,000. The IRS later valued the gift at $100,000. The applicable undervaluation penalty is: a. $0. b. $1,000 (minimum penalty). c. $2,400. d. $12,000. 69. Gadsden, who is subject to a 40% marginal Federal gift tax rate, made a gift of a sculpture to Marvin, valuing the property at $150,000. The IRS later valued the gift at $400,000. The applicable undervaluation penalty is: a. $0. b. $20,000. c. $25,000 (maximum penalty). d. $40,000. 70. Juanita, who is subject to a 40% marginal Federal gift tax rate, made a gift of a sculpture to Bianca, valuing the property at $150,000. The IRS later valued the gift at $300,000. The applicable undervaluation penalty is: a. $24,000. b. $12,000. c. $10,000 (maximum penalty). d. $0. 71. The tax penalty imposed on appraisers: a. Can be as much as 200% of the appraisal fee that was charged. b. Is waived if the taxpayer also was charged with their own valuation penalty. c. Equals 25% of the appraised value of the property with a $10,000 minimum penalty. d. Applies if the appraiser knew that the appraisal would be used in preparing a Federal income tax return. 72. Concerning the penalty for civil tax fraud: a. The burden of proof is on the taxpayer to establish that no fraud was committed. b. Fraudulent behavior is more than mere negligence on the part of the taxpayer. c. The penalty is 100% of the underpayment. d. Fraud is defined in Code §§ 6663(b) and (f). 73. Concerning a taxpayer’s requirement to make quarterly estimated tax payments: a. A C corporation must make estimated payments if its Federal income tax liability for the year will exceed $250. b. The due dates of the payments for a calendar year C corporation are March, June, September, and December Powered by Cognero

Page 9


Name:

Class:

Date:

Chapter 17: Tax Practice and Ethics 15. c. A C corporation’s estimates must total at least 90% of the current-year tax to avoid the penalty. d. An individual must make estimated payments if their balance due for the Federal income tax for the year will exceed $1,000. 74. Mickey, a calendar year taxpayer, was not required to file a Federal income tax return last year because his AGI was very low. For this tax year, his AGI is $120,000 and his tax liability is $10,000. To avoid a penalty for tax underpayments for the current year, Mickey must make aggregate estimated tax payments of at least: a. $10,000. b. $9,000. c. $1,000 (minimum amount). d. $0. 75. Minnie, a calendar year taxpayer, filed a return correctly showing a zero Federal income tax liability for last year because her Form 1040 showed various deductions and credits. For this tax year, Minnie’s AGI is $120,000, and her tax liability is $20,000. To avoid a penalty for the current year, Minnie must make aggregate estimated tax payments of at least: a. $20,000. b. $18,000. c. $1,000 (minimum amount). d. $0. 76. The usual three-year statute of limitations on additional tax assessments applies in the following situation(s). a. No return at all is filed. b. An investment in a marketable security is worthless. c. A taxpayer discovers an inadvertent overstatement of deductions equal to 30% of gross income. d. A taxpayer inadvertently omits an amount of gross income equal to 30% of the gross income stated on the return. 77. Jake, an individual calendar year taxpayer, incurred the following transactions. Gross receipts Less: Cost of sales Net business income Capital gain Capital loss Total income

$800,000 (300,000) $500,000 $30,000 (90,000)

(60,000) $440,000

Assuming that any error in timely reporting these amounts was inadvertent, how much omission from gross income would be required before the six-year statute of limitations would apply? a. More than $110,000. b. More than $132,500. c. More than $200,000. d. More than $207,500. 78. Young-Eagle files her 2023 Form 1040 on July 1, 2024. The Federal tax statute of limitations for this return expires on: Powered by Cognero

Page 10


Name:

Class:

Date:

Chapter 17: Tax Practice and Ethics a. April 15, 2027. b. July 1, 2027. c. July 1, 2030. d. Never for a late-filed return. 79. Young-Eagle files her 2023 Form 1040 on March 1, 2024. The Federal tax statute of limitations for this return expires on: a. March 1, 2027. b. April 15, 2027. c. April 15, 2028. d. April 15, 2030. 80. AICPA SSTS #1 requires that a client’s tax position: a. Have a realistic possibility of being sustained if challenged by the Treasury or in the courts. b. Be supported by substantial authority. c. Agree with all pertinent IRS interpretations of the tax law. d. Follow all interpretations of the tax law that were reflected on the immediately prior tax year’s return. 81. The tax law requires that all paid tax preparers: a. Have a current Preparer Tax Identification Number (PTIN). b. Hold a bachelor’s degree. c. Hold a bachelor’s degree in business or economics. d. Complete at least 20 hours or continuing education courses every year. 82. Vera is audited by the IRS for three tax years. Her returns were prepared by the following parties to each of whom Vera paid a professional fee. Tax Year Preparer 1 Sally (Vera’s niece, a dentist) 2 Wesley (an enrolled agent) 3 Alex (a CPA) Vera wants help in appearing before the IRS Revenue Agent for the audit. Which of the following statements is correct? a. Sally may represent Vera for all tax years involved. b. Wesley may represent Vera, but only for tax year 2. c. Alex can represent Vera, but only for tax year 3. d. Vera may represent herself for all tax years involved. 83. Circular 230 allows a tax preparer to: a. Take a position on a tax return that is contrary to a decision of the U.S. Supreme Court. b. Avoid signing a tax return that is likely to be audited. c. Charge a $5,000 fee to prepare a Form 1040EZ. d. Operate the “Tax Nerd’s Blog” on the Internet. 84. A tax preparer is in violation of Circular 230 if they: a. File a tax return that includes a math error. Powered by Cognero

Page 11


Name:

Class:

Date:

Chapter 17: Tax Practice and Ethics b. Fail to inform the IRS of an error on the client’s prior-year return. c. Charge a fee to prepare an original Form 1120 equal to one-third of the taxpayer’s refund due. d. All of these choices are Circular 230 violations. 85. Which of the following is subject to tax return preparer penalties? a. Meredith is the director of Federal taxes for a C corporation. b. Sammy is a volunteer who prepares returns at the retirement home under the IRS Tax Counseling for the Elderly program. c. Abbie prepares her mother’s tax returns for $50 a year. A CPA, Abbie would charge a client $750 for completing a similar return. d. Lizzie, the firm’s administrative assistant, makes copies of returns and assembles the mailings that the client must make to the taxing agencies. 86. Megan prepared a Federal income tax return for Joan for compensation. Joan’s return included an aggressive interpretation of the rules concerning overnight business travel. Megan is not liable for a preparer penalty for taking an unreasonable tax return position if: a. The tax reduction attributable to the disputed deduction did not exceed $5,000. b. There was a reasonable basis for Joan’s interpretation of the travel deduction rules. c. There was substantial authority for Joan’s interpretation of the travel deduction rules. d. The IRS found that the travel deduction was frivolous, but Joan had disclosed the position in an attachment to the return. 87. Mikel prepared a Federal income tax return for Mona for compensation. Her return included an aggressive interpretation of the rules concerning the home office deduction of a sole proprietor. Mikel is not liable for a preparer penalty for taking an unreasonable tax return position if: a. The tax reduction attributable to the disputed deduction did not exceed $5,000. b. Mona is assessed her own penalty for an understatement of tax due to disregard of IRS rules. c. The IRS found that the disputed deduction was frivolous, but Mona had disclosed the position in an attachment to the return. d. There was a reasonable basis for Mona’s interpretation of the home office deduction rules, and she had disclosed the position in an attachment to the return. 88. Freddie has been assessed a preparer penalty for willful and reckless conduct. When he completed Peggy’s Federal income tax return, he purposely omitted $100,000 of cash receipts that should have been reported as gross income. Freddie charged Peggy $4,000 to prepare the return. What is Freddie’s preparer penalty? a. $-0- because Peggy incurred her own understatement penalty for the return. b. $2,000 c. $4,000 d. $5,000 89. The privilege of confidentiality applies to a CPA tax preparer concerning the client’s information relative to: a. Financial accounting tax accrual workpapers. b. A tax research memo used to support a tax planning idea. c. Building a defense against a penalty assessed for the use of a tax shelter. d. Building a defense against a charge brought by the SEC. Powered by Cognero

Page 12


Name:

Class:

Date:

Chapter 17: Tax Practice and Ethics 90. The Statements on Standards for Tax Services are issued by the: a. IRS. b. AICPA. c. ABA. d. SEC. Completion 91. The IRS is a subsidiary agency of the Department of the ____________________. 92. About ____________________% of all Forms 1040 are audited each year. 93. The IRS is one of the largest Federal agencies, employing about ____________________ people throughout the year. 94. The difference between how much tax is collected and how much should be collected is called the ________________. 95. The IRS processes about ____________________ million income tax returns every year. 96. The chief executive of the IRS is the _________________________. 97. The ____________________, a presidential appointee, is the “IRS’s attorney.” 98. A letter ruling should be requested when the taxpayer wants to know how the IRS's interpretation of the tax law will be applied to a ____________________ (proposed, completed) transaction. 99. During an audit, the IRS might require that the taxpayer produce the ____________________ that underlie the tax return data. 100. After the completion of an audit, the taxpayer has 90 days to petition the ____________________ Court to dispute the proposed tax due. 101. The IRS can pay a discretionary informant’s award of up to ____________________% of the recovered tax, interest, fines, and penalty amounts. The reward can be as high as ____________________% for information provided under the whistleblower program. 102. A taxpayer’s return might be selected for audit on the basis of the ____________________ score that the IRS computes. 103. In taking a dispute to the Appeals Division, a written protest is required of the taxpayer when the proposed deficiency exceeds $____________________. 104. A taxpayer can take a dispute to the ____________________ Division of the Tax Court when the disputed amounts do not exceed $50,000. 105. The IRS charges ____________________% interest over the Federal short-term rate on taxes that an individual taxpayer underpays. 106. The IRS pays interest on a refund to the taxpayer unless the IRS’s payment is made within ____________________ days of the date that a return is filed. Powered by Cognero

Page 13


Name:

Class:

Date:

Chapter 17: Tax Practice and Ethics 107. With respect to tax misconduct, a ____________________ penalty usually involves only a fine, but a ____________________ penalty also can include jail time. 108. If the taxpayer shows ____________________ for an underpayment of tax, the failure to pay penalty can be reduced or eliminated. 109. The accuracy-related penalties typically relate to ____________________on the part of the taxpayer or other understatements of tax. 110. A negligence penalty can be waived if the taxpayer discloses a tax return position that is contrary to a judicial precedent and there is a ____________________ for the taxpayer’s position. 111. The IRS can waive the penalty for a substantial understatement of a tax liability if the taxpayer shows ____________________ for the position taken on the tax return. 112. A ____________________% penalty may result when the value of an asset contributed to a charity is reported at an amount that exceeds 150% of the correct valuation. 113. A penalty can be assessed on an appraiser who knew that her or his improper appraisal would be used as part of a tax computation in the amount of the lesser of ____________________% of the tax understatement or ____________________ % of the appraisal fee collected. 114. Quon filed an amended return claiming a $100,000 refund. The IRS disallowed the refund, and it can assess a penalty in the amount of ____________________% of the disallowed amount if there was no reasonable cause for making the refund claim. 115. An individual is not subject to an underpayment penalty until more than $____________________ is due and unpaid through estimated taxes. 116. Maria’s AGI last year was $95,000. To avoid a penalty, her estimated tax payments and withholdings for the currentyear must equal the lesser of ____________________ % of prior-year taxes or ____________________ % of the currentyear's taxes. 117. The general statute of limitations regarding Federal tax returns extends for ____________________ years. It is ____________________ years if a substantial understatement of income is found, and ____________________ years with respect to worthless securities. 118. An individual’s amended tax return that computes a refund or amount due generally uses Form ____________________. 119. A CPA, an attorney, or a(n) ____________________ can represent taxpayers before the IRS in an Appeals conference. 120. The Treasury document regulating the professional conduct of tax return preparers is the Regulation known as _________________________. 121. A(n) ___________________ must obtain a Preparer Tax Identification Number (PTIN). 122. For purposes of tax penalties, a VITA volunteer is not classified as a tax return ____________________. Powered by Cognero

Page 14


Name:

Class:

Date:

Chapter 17: Tax Practice and Ethics 123. The tax preparer penalty for taking an unreasonable tax return position is the greater of $____________________ or ____________________ of the tax professional’s income from preparing the return. 124. A(n) $____________________ penalty applies if the tax preparer does not sign the client’s tax return. 125. The tax ____________________ workpapers prepared as part of an independent financial audit are not privileged communications that can be kept confidential from an IRS subpoena. 126. A privilege of ____________________ exists between certain types of tax preparers and the client as to tax advice rendered. 127. A(n) ___________________ (ABA, AICPA, CFP) member must follow the Statements of Standards for Tax Services in conducting a tax practice. Matching Match each of the following tax penalties with the type of tax penalty as specified by the Code. A letter can be used more than once. The correct solution may include more than one letter. a. Taxpayer penalty b. Tax preparer penalty c. Appraiser’s penalty 128. Failure to file a tax return. 129. Fraudulent failure to file a tax return. 130. Failure to sign the return. 131. Negligence. 132. Improper refund claim. 133. Misstatement of withholding allowances. 134. Willful and reckless conduct. 135. Aiding in preparing an improper tax return. 136. Substantial understatement of a tax. 137. Failure to pay a tax. For each of the indicated tax penalties, indicate the appropriate taxpayer defense. A letter may be used more than once. Not all of the letters need be used. a. Ignorance of the tax law b. Reasonable basis c. Reasonable cause d. Complexity of the tax law e. Substantial authority Powered by Cognero

Page 15


Name:

Class:

Date:

Chapter 17: Tax Practice and Ethics f. Disclosure on return 138. Negligence in filing a return. 139. Substantial understatement of tax liability. 140. Failure to pay a tax that is due. 141. Undervaluation of a reported item. 142. Filing an improper refund claim. 143. Failure to deposit withholding tax. 144. Preparer penalty for taking an unreasonable but disclosed tax return position. 145. Preparer penalty for reckless conduct. Subjective Short Answer 146. Maria did not pay her Federal income tax on time. When she eventually filed the return, she reported a balance due. Compute Maria’s failure to file penalty in each of the following cases. Disregard any failure to pay penalty. a. b. c. d. e.

Three months late, $500 additional tax due. Four months late, $2,000 additional tax due. Ten months late, $10,000 additional tax due. Four months late due to fraud by Maria, $10,000 additional tax due. Fifteen months late due to fraud by Maria, $10,000 additional tax due.

147. Quon filed his Federal income tax return on time, but he did not remit the full balance due. Compute the amount of his failure to pay penalty in each of the following cases. The IRS has not yet issued a deficiency notice. a. b. c.

Three months late, $5,000 additional tax due. Nine months late, $5,000 additional tax due. Five years late, $5,000 additional tax due.

148. Compute the failure to pay and failure to file penalties for John, who filed his 2024 income tax return on December 14, 2025, paying the $10,000 amount due. On April 1, 2025, John submitted a six-month extension of time in which to file his return; he paid no tax with the extension request. He has no reasonable cause for failing to file his return by October 15, or for failing to pay the tax that was due on April 15, 2025. John’s failure to comply with the tax laws was not fraudulent. 149. Silvio, a cash basis calendar year taxpayer, filed his income tax return 75 days after the due date. He never extended his return, and with the return, he paid the full amount of taxes that were due. What penalties will Silvio incur, and how much is the penalty if his additional tax is $5,000? Disregard any additional interest he must pay. 150. Lori, a calendar year individual taxpayer, files her 2023 return on February 10, 2025. She had obtained a six-month extension for filing her return. There was additional income tax of $30,000 due with the return.

a.

What are Lori’s penalties for failure to file and to pay?

Powered by Cognero

Page 16


Name:

Class:

Date:

Chapter 17: Tax Practice and Ethics

b.

Would your answer to a. change if Lori, before the due date of the return, had retained a CPA to prepare the return and it was the CPA’s negligence that caused the delay?

151. Clarita underpaid her taxes by $50,000. Of this amount, $35,000 was due to negligence on her part, because her record-keeping system is highly inadequate. Determine the amount of any negligence penalty. 152. Linwell underpaid his taxes by $250,000. Portions of the underpayment were attributable to negligence ($210,000) and to civil fraud ($40,000). Compute the total penalties incurred. 153. Compute the undervaluation penalty for each of the following independent cases involving the executor’s reporting of the value of a closely held business in the decedent’s gross estate. In each case, assume a marginal Federal estate tax rate of 40%.

a. b. c. d.

Reported Value $10,000 60,000 80,000 50,000

Corrected IRS Value $ 20,000 100,000 150,000 300,000

154. DeShawn made a charitable contribution of property that he valued at $80,000. He deducted this amount as an itemized deduction on his tax return. The IRS can show that the actual value of the property is $50,000. DeShawn is in the 32% Federal income tax bracket. Determine the amount due for any penalty. 155. Chung’s AGI last year was $180,000. Her Federal income tax came to $45,000, which she paid through a combination of withholding and estimated payments. This year, her AGI will be $250,000 with a projected tax liability of $60,000, all to be paid through estimates. Ignore the annualized income method. Compute Chung’s quarterly estimated payment schedule for this year. 156. Four Square Services Corporation estimates that its 2025 taxable income will be $1,000,000. Thus, it incurs a $210,000 liability. For the following independent cases, compute Four Square’s minimum quarterly estimated tax payments that will avoid an underpayment penalty. a.

For 2024, taxable income was $800,000, and tax liability was $168,000.

b.

For 2023, taxable income was $2 million, and tax liability was $680,000. For 2024, taxable income was $100,000, and tax liability was $21,000.

157. Loren Ltd., a calendar year taxpayer, had the following transactions, all of which were properly reported on a timely filed return. Presuming the absence of fraud, how much of an omission from gross income must occur for Loren before the six-year statute of limitations applies? Show your computations. Gross receipts Cost of sales Gross profit Capital gain Capital loss Total income

Powered by Cognero

$3,500,000 (1,000,000) $2,500,000 $100,000 (30,000)

70,000 $2,570,000

Page 17


Name:

Class:

Date:

Chapter 17: Tax Practice and Ethics 158. Yin-Li is the preparer of the Form 1120 for Cloves Corporation. On the return, Cloves claimed a deduction that the IRS later disallowed on audit. Compute the tax preparer penalty that could be assessed against Yin-Li in each of the following independent situations.

a. b. c. d. e.

Probability That the Form 8275 Disclosure Tax Reduction That Yin-Li’s Fee to Deduction Would be on the Return of the Resulted From the Complete the Cloves Approved by the Disputed Deduction? Deduction ($) Return ($) Courts (%) No 40,000 75 10,000 No 50,000 30 500 No 60,000 40 3,000 Yes 70,000 30 6,000 Yes 80,000 20 6,000

Essay 159. A tax professional needs to know how the IRS is structured and how it works to carry out its mission. Evaluate this statement. 160. The IRS national office is organized into operating divisions, broadly on the basis of the tax returns that are the responsibility of each division. List the four IRS operating divisions. 161. The Treasury issues private letter rulings, usually in response to a taxpayer request. What is the purpose of the rulings program? Answer from both the point of view of the taxpayer and the government. 162. Specific factors that are used in selecting tax returns for audit and the weights for such factors are not released to the public. Summarize the public information about tax audit selection; how might a tax return’s chances of audit increase above national norms? 163. Describe the potential outcomes to a party who provides information to the IRS under the informant or whistleblower program. 164. When a tax dispute is resolved, interest is paid by or to the government. How are IRS interest amounts determined? To which tax amounts do they apply? 165. In connection with the taxpayer penalty for substantial understatement of tax liability, what defenses (if any) are available? 166. A taxpayer penalty may be waived if there is shown to be reasonable cause for the misstatement on the tax return. The courts have applied this standard strictly. List three or more assertions for a waiver of a taxpayer penalty that courts have found not to constitute reasonable cause. Example: Ignorance of the tax law. 167. The taxpayer or a tax adviser may be subject to penalties if there is a misstatement of the valuation of an item reported on the tax return. Describe how these penalties work. 168. Some taxpayers must make quarterly estimated payments of their income tax. Describe these requirements. In your answer, include both corporate and noncorporate taxpayers. 169. Evaluate this statement: A taxpayer is liable not only for tax penalties as described in the tax Code and Regulations. Sanctions elsewhere in Federal law, such as in the criminal code, also apply in a tax dispute. Powered by Cognero

Page 18


Name:

Class:

Date:

Chapter 17: Tax Practice and Ethics 170. Circular 230 requires that the tax practitioner use the best practices of the tax profession in carrying out a tax engagement. Specify what some of these best practices entail. 171. Attorneys are allowed an attorney-client privilege of confidentiality with clients. Does a CPA tax preparer enjoy the same privilege? If so, on which types of tax and financial work? 172. To whom do the AICPA’s Statements on Standards for Tax Services apply? Are they binding or merely advisory? 173. Maddie, your tax client, has decided to dispute the Revenue Agent’s Report. List and discuss at least four of your next steps as you prepare to represent Maddie concerning her tax issue within the IRS and in the courts.

Powered by Cognero

Page 19


Name:

Class:

Date:

Chapter 17: Tax Practice and Ethics Answer Key 1. True 2. True 3. True 4. True 5. True 6. True 7. False 8. True 9. False 10. True 11. True 12. True 13. True 14. True 15. True 16. False 17. True 18. False 19. False 20. True 21. False 22. True 23. False 24. False 25. False Powered by Cognero

Page 20


Name:

Class:

Date:

Chapter 17: Tax Practice and Ethics 26. True 27. False 28. False 29. False 30. True 31. True 32. False 33. True 34. True 35. True 36. True 37. True 38. False 39. False 40. True 41. True 42. False 43. False 44. False 45. False 46. False 47. True 48. True 49. b 50. a Powered by Cognero

Page 21


Name:

Class:

Date:

Chapter 17: Tax Practice and Ethics 51. b 52. c 53. a 54. b 55. c 56. d 57. d 58. c 59. b 60. c 61. b 62. d 63. e 64. a 65. a 66. b 67. d 68. a 69. d 70. b 71. d 72. b 73. d 74. b 75. d 76. c Powered by Cognero

Page 22


Name:

Class:

Date:

Chapter 17: Tax Practice and Ethics 77. d 78. b 79. b 80. a 81. a 82. d 83. d 84. c 85. c 86. c 87. d 88. d 89. b 90. b 91. Treasury 92. less than one 93. 79,000 94. tax gap 95. 260 96. Commissioner 97. Chief Counsel 98. proposed 99. books and records 100. Tax 101. Powered by Cognero

Page 23


Name:

Class:

Date:

Chapter 17: Tax Practice and Ethics 15, 30 102. DIF 103. 10,000 104. Small Cases 105. 3 106. 45 107. civil, criminal 108. reasonable cause 109. negligence 110. reasonable basis 111. substantial authority 112. 20 113. 10, 125 114. 20 115. 1,000 116. 100, 90 117. three, six, seven 118. 1040X 119. enrolled agent 120. Circular 230 121. paid tax preparer 122. preparer 123. 1,000, one-half 124. 60

Powered by Cognero

Page 24


Name:

Class:

Date:

Chapter 17: Tax Practice and Ethics 125. accrual 126. confidentiality 127. AICPA 128. a 129. a 130. b 131. a 132. a 133. a 134. b 135. b 136. a 137. a 138. c 139. e 140. c 141. c 142. c 143. c 144. b 145. f 146. The failure to file penalty is 5% per month of the tax due with a $485 minimum penalty and a maximum penalty of 25%. In the case of fraud, the penalty rate is tripled. a. $75 (5% × 3 months × $500 tax due), but apply the larger $510 minimum penalty. b. $400 penalty (5% × 4 months × $2,000 tax due), but apply the larger $510 minimum penalty . c.$2,500 penalty (5% × 5 months maximum × $10,000 tax due). d. $6,000 penalty (15% × 4 months × $10,000 tax due). Powered by Cognero

Page 25


Name:

Class:

Date:

Chapter 17: Tax Practice and Ethics e. $22,500 (15% × 15 months × $10,000 tax due), but limited to $7,500 (75% of the tax due).

147. The failure to pay penalty is 0.5% per month of the tax due, with a maximum penalty of 25% of the unpaid tax (= 50 months of penalty charges). a. b. c.

$75 penalty (no minimum amount) (0.5% × 3 months × $5,000 tax due). $225 penalty (0.5% × 9 months × $5,000 tax due). $1,250 penalty (0.5% × 50 months maximum × $5,000 tax due).

148. Failure to pay Underpayment Penalty rate Penalty per month outstanding Months late Penalty

$10,000 × 0.005 $ 50 × 8 $ 400

Failure to file Underpayment Penalty rate Penalty per month outstanding Months late Penalty before reduction Less: Concomitant failure to pay penalty (2 months × $50) Penalty

$10,000 × 0.05 $ 500 × 2 $ 1,000 (100) $ 900

The failure to file penalty applies from the extended due date of the return, two months in this case. The automatic sixmonth extension of time to file does not extend the time to pay the tax. Thus, the failure to pay penalty applies for eight months. 149. The penalties cannot overlap. They apply to every month or portion of month of late filing. Failure to pay penalty (1/2% × $5,000) × 3 months Plus: Failure to file penalty (5% × $5,000) × 3 months Less: Failure to pay penalty for same period Total penalties

$ 75 $750 (75)

675 $750

150. a. The penalties cannot overlap. They apply to every month or portion of month of late filing. Failure to pay penalty Powered by Cognero

$1,500 Page 26


Name:

Class:

Date:

Chapter 17: Tax Practice and Ethics (1/2% × $30,000) × 10 months Plus: Failure to file penalty (5% × $6,000 $30,000) ×4 months Less: Failure to pay (600) penalty for same period

5,400

Total penalties

$6,900

The failure to file penalty cannot exceed a total of 25%. Consequently, this penalty ceases to increase after 5 months (i.e., 5% × 5 months = 25%). The failure to pay penalty also is limited to 25%. At the rate of 1/2 of 1% per month, such penalty can continue to run for as long as 50 months. b. No. A taxpayer's reliance on a CPA generally does not constitute reasonable cause to avoid the failure to file and pay penalties. The CPA may incur preparer penalties as well. 151. $7,000 (20% × $35,000). 152. Penalty attributable to civil fraud (75% × $40,000) Penalty attributable to negligence (20% × $210,000) Total penalties

$30,000 42,000 $72,000

If the underpayment is partially attributable to negligence and partially attributable to fraud, the fraud penalty is applied first. The negligence penalty then applies to the remaining deficiency. 153. a. $0. Additional tax ($4,000) is less than $5,000. b.

$3,200. Valuation claimed ($60,000) is 65% or less of the correct value ($100,000). Penalty = 20% × tax understatement (40% × $40,000).

c.

$5,600 penalty assessed ($28,000 additional tax × 20% penalty rate).

d.

$40,000 penalty assessed [$100,000 additional tax × 40% penalty rate for gross undervaluation (reported value is 40% or less than the correct value)].

Powered by Cognero

Page 27


Name:

Class:

Date:

Chapter 17: Tax Practice and Ethics

154. The § 6662 penalty for an individual applies when the tax understatement exceeds $5,000 and the valuation used is 150% or more of the correct value. DeShawn’s underpayment of tax is $9,600 (32% tax bracket × $30,000 overvaluation). Thus, the penalty for overvaluation is 20% of this amount, or $1,920. 155. Current-Year Method First quarter payment [($60,000 tax ÷ 4 payments) × 90% required)] Second quarter payment Third quarter payment Fourth quarter payment Total estimates paid

$13,500 13,500 13,500 13,500 $54,000

Prior-Year Method First quarter payment [($45,000 ÷ 4) × 110% required] Second quarter payment Third quarter payment Fourth quarter payment Total estimates paid

$12,375 12,375 12,375 12,375 $49,500

Thus, Chung will use the payment schedule of the prior-year method. The remaining tax ($60,000 – $49,500 = $10,500) is due with the return, but no underpayment penalty is assessed. 156. a. Prior-Year Method First quarter payment ($168,000/4 payments) Second quarter payment Third quarter payment Fourth quarter payment Total payments

$ 42,000 42,000 42,000 42,000 $168,000

Use the prior-year exception. The remaining liability of $42,000 ($210,000 – $168,000 paid in installments) is due without penalty with the Form 1120. A Form 2220 should be attached to prove that no underpayment penalty is due. b. First quarter payment (25% × $21,000) Second quarter payment ($52,500 × 2 = $105,000 due – $5,250 paid first quarter) Third quarter payment Fourth quarter payment Total payments

$ 5,250 99,750 52,500 52,500 $210,000

$52,500 (25% × $210,000) is the basic payment. Four Square is a large Powered by Cognero

Page 28


Name:

Class:

Date:

Chapter 17: Tax Practice and Ethics corporation, so it can use the prior-year exception but only on its first quarter payment. Any underpayments from the first quarter are due with the second installment.

157. A taxpayer must omit an amount of gross income that exceeds 25% of the gross income stated on the tax return before the six-year statute of limitations applies. This rule affects gross income items only (i.e., not deductions and exclusions such as cost of sales). For this purpose, capital gains and losses are not netted together. Here, the three-year statute applies until the gross income omission exceeds $900,000 [($3,500,000 + $100,000) × 25%]. 158. The potential § 6694 penalty does not depend on the amount of tax reduction that resulted from the disputed position. The penalty is the greater of $1,000 or one-half of the tax preparer’s fee for working on the return. Form 8275 disclosure is required to avoid the penalty if there is a reasonable basis for the disputed tax return position. The penalty also can be waived by showing reasonable cause for the position and that the preparer acted in good faith.

Form 8275 Disclosure on the Return of the Disputed Deduction?

a.

b.

c.

No

No

No

Probability That the Yin-Li’s Fee Tax Reduction That Deduction to Complete Resulted From the Would be the Cloves Deduction ($) Approved by Return ($) the Courts (%) 40,000

50,000

60,000

75

30

40

Amount of Preparer Penalty ($)

10,000

0. Substantial authority for the position.

500

1,000 minimum penalty. No substantial authority, no disclosure.

3,000

1,500. No substantial authority, no disclosure.

d.

Yes

70,000

30

6,000

0. No substantial authority, but disclosure was made.

e.

Yes

80,000

20

6,000

3,000. No reasonable basis.

159. The tax practitioner needs to study the workings of the IRS to:

Powered by Cognero

Page 29


Name:

Class:

Date:

Chapter 17: Tax Practice and Ethics ∙

Be able to apply and use to the taxpayer’s advantage the administration process and the opportunities for appeal provided by the law.

Understand the negative sanctions, such as penalties and interest, that may be brought to bear when the taxpayer is unsuccessful in taking a tax return position.

Apply the ethical and professional constraints that relate to the advice that a tax practitioner gives to a client.

160. ∙ Wage and investment. ∙

Small business and self-employed.

Large business and international.

Tax-exempt/governmental entities.

161. From the taxpayer’s point of view, the IRS advance rulings program reduces the uncertainty of potential tax consequences resulting from a proposed course of action and, therefore, facilitates tax planning. In addition, the issuance of published rulings permits taxpayers better to understand the position of the IRS on a given issue (e.g., in the event of subsequent audit of a completed transaction that is currently being proposed). From the standpoint of the government, the IRS ruling policy is an attempt to promote a uniform application of the tax laws. In addition, rulings may reduce the volume of litigation or number of disputes with revenue agents that otherwise would result. 162. These circumstances likely increase one’s chances to be audited this year. ∙

Type of tax return: high-income, self-employed, cash-oriented.

Information returns sent to the IRS do not match contents of the tax return.

Notification provided by an informant or whistleblower.

Identification by IRS audit staff from other information sources (e.g., newspapers, web pages, other Federal government agencies.)

Taxpayers audited in a past year and the audit led to the assessment of a substantial deficiency

Itemized deductions that are in excess of norms established for various income levels

163. An informant may qualify for a discretionary award of up to 15% of the tax, penalty, and interest that the IRS collects and is attributable to the information provided. A whistleblower’s award may be as much as 30% of the recovered amounts. This higher award is available if the gross income of the informed-upon taxpayer exceeds $200,000 and more than $2 million of tax, penalty, fines, and interest is under dispute. 164. These observations can be made about the use of interest in the IRS’s management of tax liabilities.

Powered by Cognero

Page 30


Name:

Class:

Date:

Chapter 17: Tax Practice and Ethics ∙

Interest is paid by the IRS or by the taxpayer, whichever owes the disputed tax.

Interest rates are set quarterly by the Treasury to approximate the rates found in current financial markets.

One percentage point is subtracted from the norm in computing the rate paid for corporate overpayments.

Interest is compounded daily.

Interest is not payable by the IRS if the refund due is outstanding for 45 days or less.

Refund claims must be made in processible form (i.e., it must be on a form that is readable by the IRS and containing sufficient information to substantiate the claim).

165. The defenses to the imposition of the penalty include the following. ∙

The taxpayer has substantial authority for such treatment.

The relevant facts affecting the treatment are adequately disclosed in the return on Form 8275, and the taxpayer has a reasonable basis for taking the disputed position.

166. Courts have found the following taxpayer assertions not to constitute reasonable cause. ∙

Delegation of the filing responsibility to a third party with little or no review of the return or of the act of filing.

Lack of information as of the due date of the return.

Illness that did not incapacitate the taxpayer.

Refusal of the taxpayer’s spouse to cooperate on a joint return.

Misunderstanding of the tax law.

167. Several penalties might arise with respect to valuation questions on a tax return. ∙

An overvaluation penalty might be imposed when the taxpayer, for example, overstates the amount of a deduction. The basic penalty is 20% of the tax reduction that resulted from the overvaluation. Defenses to the penalty include reasonable cause and good faith efforts.

An undervaluation penalty might arise when the taxpayer, for example, understates the value of an asset on an estate or gift tax return. The basic penalty is 20% of the tax reduction that resulted from the undervaluation. Defenses to the penalty include Powered by Cognero

Page 31


Name:

Class:

Date:

Chapter 17: Tax Practice and Ethics reasonable cause and good faith efforts. ∙

A penalty can be assessed on an appraiser when it was more likely than not that the appraised value used on a tax return was improper. The penalty is the lesser of 10% of the understated tax attributable to the appraisal, but at least $1,000, or 125 percent of the gross income received by the appraiser from the engagement. This penalty is assessed in addition to the taxpayer valuation penalties.

168. C corporations must make estimates when their tax liability is a least $500. For all other taxpayers, this amount is $1,000. For calendar year taxpayers, the payments are due on the fifteenth day of April, June, September, and the succeeding January. A calendar year C corporation makes its fourth quarter payment by December 15 of the tax year. A taxpayer other than a C corporation must remit at least 90% of the current-year tax or 100% of the prior year tax. The 100% requirement becomes 110% if an individual’s prior year AGI exceeds $150,000. For the prior-year method, a return must have been submitted for a full 12month tax year. A C corporation must remit at least 100% of the current-year tax or 100% of the prior year tax. For the prior year method, a nonzero tax amount must have been shown on a tax return submitted for a full 12-month tax year. Large corporations can use the prior-year method only on the first quarter payment.

The penalties are assessed on a quarterly basis. Annualized income computations may be used by the taxpayer to compute the quarterly tax amount due. 169. A series of criminal penalties are assessed from taxpayers who repeatedly and willfully act to defeat a Federal tax. Criminal penalties bring about fines and potential imprisonment for the offending taxpayer. Criminal tax cases are rare, and they are carried out through the usual criminal justice system in which the taxpayer must be found guilty beyond the shadow of any reasonable doubt. To avoid a criminal tax penalty, the taxpayer needs to create a degree of doubt as to their guilt, with the court or jury. Thus, the taxpayer might assert that they were confused or ignorant of how the tax law applied, or that they relied on the erroneous advice of a competent tax adviser. Another defense against a criminal tax penalty is the inability to plan and execute an evasion of taxes, due for example, to a mental disease or limitation. Other violations of the Federal criminal code might arise in the context of filing tax returns, including: ∙

Making a false claim against the Federal government.

Participating in a conspiracy to evade Federal taxes.

Making a false statement to the Federal government or filing a false document (i.e., perjury).

170. Tax advisers are to provide clients with the highest quality representation as to Federal tax issues. In addition to other ethical standards, the tax adviser must: Powered by Cognero

Page 32


Name:

Class:

Date:

Chapter 17: Tax Practice and Ethics ∙

Communicate clearly with the client as to the terms of the engagement.

Identify pertinent assumptions, gather relevant facts, and apply the tax law properly to arrive at supportable conclusions and recommendations for the client.

Use tax software effectively.

Adopt and execute office procedures with professional quality preparation and review.

Keep client information confidential and secure.

Use adequate backup procedures for internal and client electronic data.

Act fairly and with integrity in practice before the IRS.

Inform the client as to risks and penalties that might arise as a result of a recommended course of action.

171. A limited confidentiality privilege is granted to nonattorneys with respect to tax-related work. The privilege applies only with respect to contacts with the IRS, and not with other Federal agencies such as the SEC. The privilege does not apply with respect to either criminal charges or transactions involving tax shelters. The tax preparer’s privilege corresponds with what would be available between an attorney and their client in the relevant U.S. state. The privilege relates to the preparation of a Federal tax return but business and other tax advice are not protected. The privilege does not protect the tax professional’s tax accrual workpapers prepared for a client, since these relate to the preparation of financial statements. 172. The Statements are enforceable standards of practice for AICPA members who are involved in a Federal or state/local tax practice. Others are subject to a state’s ethical rules applicable to CPAs and to those contained in Circular 230 as well as the penalty provisions included in the Code. 173. Carrying out a level of due diligence in preparing for such a proceeding is part of the tax professional’s responsibility in representing the client. ∙

Make certain that both sides agree on the issues to be resolved in the audit. The goal here is to bring closure to the agent’s list of open issues.

Identify all of the facts underlying the issues in dispute, including those favorable to the IRS. Gather evidence to support the taxpayer’s position, and evaluate the evidence supporting the other side.

Identify current tax law authorities as they bear on the facts and open issues. Remember that the IRS agent is bound only by Supreme Court cases and IRS pronouncements. Determine here the degree of discretion that the IRS is likely to have in disposing of the case. Powered by Cognero

Page 33


Name:

Class:

Date:

Chapter 17: Tax Practice and Ethics ∙

Prepare a list of points ranging from those bearing little weight to the core principles upon which the taxpayer’s case may rest to those supporting and contradicting that position. Short research memos will be useful as well in the discussion with the agent. Pro taxpayer points should be mentioned during the discussion and “entered into the record.”

Prepare tax and interest computations as to the effects of points that are in dispute so that the effects of closing or compromising an issue always can be determined.

Determine a litigation point (i.e., at which the taxpayer’s position will be withdrawn from further audit negotiation and taken to the courts). This position should reflect the dollars of tax, interest, and penalty involved, the chances of prevailing in various trial-level courts, and other strategies discussed with the taxpayer. One must have an end game strategy for the audit, and in rare cases litigation may present a possible resolution.

Powered by Cognero

Page 34


Name:

Class:

Date:

Chapter 18: The Federal Gift and Estate Taxes True / False 1. A lifetime transfer that is supported by full and adequate consideration is not a gift. a. True b. False 2. An individual generally tries to reduce the present value of any Federal transfer tax liability. a. True b. False 3. The Federal transfer tax system includes three separate taxes. a. True b. False 4. Some states impose inheritance taxes, but the Federal tax system does not. a. True b. False 5. Manuel, a citizen and resident of Argentina, makes a gift of a ranch located in Colorado to his children. Manuel is subject to the U.S. gift tax. a. True b. False 6. Kim, a resident and citizen of Korea, dies during an operation at the Mayo Clinic in Rochester (MN). Because Kim died in the United States, she will be subject to the Federal estate tax. a. True b. False 7. For Federal estate tax purposes, the gross estate does not include property that will pass to a surviving spouse. a. True b. False 8. For Federal estate and gift tax purposes, the exemption equivalent is the same thing as the exclusion amount. a. True b. False 9. For both the Federal gift and estate taxes, a deduction is allowed for certain transfers to a spouse. a. True b. False 10. The amount of the exemption equivalent is the same for both transfers by gift and transfers by death. a. True b. False 11. If the value of the gross estate is lower on the alternate valuation date than on the date of death, the date of death valuation cannot be used. Powered by Cognero

Page 1


Name:

Class:

Date:

Chapter 18: The Federal Gift and Estate Taxes a. True b. False 12. Under the alternate valuation date election, each asset in the gross estate is valued at the lesser of the date of death value or six months thereafter. a. True b. False 13. A father wants to give a parcel of land to his two children. If he wants the survivor to have sole ownership, he should list ownership of the property as joint tenants. a. True b. False 14. Sandy pays a local college for her boyfriend’s tuition. The payment is subject to the Federal gift tax. a. True b. False 15. Barry pays State University for his daughter’s room and board. Barry has made a transfer that is subject to the Federal gift tax. a. True b. False 16. Transfers to political organizations are exempt from the application of the Federal gift tax. a. True b. False 17. Pauline sells antique furniture to her daughter, Nicole, for $10,000. If the furniture is really worth $100,000, Pauline has made a gift to Nicole of $100,000. a. True b. False 18. Felipe pays the surgeon and the hospital for his aunt’s heart bypass operation. The transfer is not subject to the gift tax. a. True b. False 19. Sam purchases a U.S. savings bond that he registers as follows: “Sam, payable to Anya upon Sam’s death.” A gift occurs when Sam purchases the bond. a. True b. False 20. A timely issued disclaimer by an heir transfers the property to someone else without a Federal gift tax result. a. True b. False 21. In 2024, grandparents contribute jointly owned funds to a § 529 qualified tuition plan on behalf of their granddaughter. The maximum annual exclusion allowed to them is $180,000 ($36,000 × 5 years). Powered by Cognero

Page 2


Name:

Class:

Date:

Chapter 18: The Federal Gift and Estate Taxes a. True b. False 22. Although qualified tuition plans under § 529 are treated favorably for gift tax purposes, such plans are included in the gross estate upon the grantor’s death. a. True b. False 23. To make the election to split gifts, spouses must file a Form 709 (Federal gift tax return). a. True b. False 24. A married couple make a gift of their vacation home, held as tenants in common, to their adult children. The giftsplitting election must be made. a. True b. False 25. Under certain circumstances, the gift-splitting election can be made even though the electing spouses no longer are married to each other. a. True b. False 26. Iris dies intestate (i.e., without a will). All of her property passes to her heirs in accordance with the order of distribution prescribed under the Internal Revenue Code. a. True b. False 27. A surviving spouse’s share of the community property is not included in the deceased spouse’s gross estate. a. True b. False 28. Using his separate funds, Francisco purchases an annuity that pays him a specified amount until death. Upon Francisco’s death, a reduced amount is to be paid to Marcia for her life. Marcia predeceases Francisco. Nothing concerning the annuity contract is included in Marcia’s gross estate. a. True b. False 29. A tenancy by the entirety is restricted in most states to having more than two joint owners. a. True b. False 30. In year 1 and with $100,000, Ronald establishes a joint savings account with his cousin, Allison. In year 2, Allison withdraws the $100,000 and leaves the country. Ronald made a gift to Allison in year 2. a. True b. False Powered by Cognero

Page 3


Name:

Class:

Date:

Chapter 18: The Federal Gift and Estate Taxes 31. Ming and Quan are married. Using his funds, Ming purchases real estate that he lists as: “Ming and Quan, tenants by the entirety with right of survivorship.” If Quan dies first, none of the real estate is included in her gross estate. a. True b. False 32. Harry and Brenda are married. Using her funds, Brenda purchases real estate that she lists as: “Harry and Brenda, joint tenants with right of survivorship.” If Brenda dies first, all of the value of the real estate is included in her gross estate. a. True b. False 33. Lyle and Kelly are brother and sister. Using his funds, Lyle purchases land, listing title as: “Lyle and Kelly, joint tenants with right of survivorship.” If Kelly dies first, none of the land is included in her gross estate. a. True b. False 34. Ray purchases U.S. savings bonds that he lists as “Ray and Donna” as co-owners. Donna is Ray’s daughter. Donna predeceases Ray. No gift or estate tax consequences result from this situation. a. True b. False 35. Zan owns an insurance policy on the life of Shing, with Belle as the designated beneficiary. Upon Zan’s death, nothing regarding this policy is included in her gross estate. a. True b. False 36. Georgia owns an insurance policy on the life of Jake with Scarlet as the designated beneficiary. Upon Scarlet’s death, no Federal transfer tax consequences result. a. True b. False 37. At the time of his death, Raj owed Federal income taxes on income earned in a prior year. Raj’s estate can claim an estate tax deduction for the income tax it pays. a. True b. False 38. All charitable organizations that qualify for estate tax purposes also qualify for income tax purposes. a. True b. False 39. In his will, Hernando provides for $50,000 to go to the Madrid, Spain, school system. Because it is a foreign charity, the bequest will not qualify as a charitable deduction for estate tax purposes. a. True b. False 40. Manfredo makes a donation of $50,000 to the church in Mexico City where he was baptized. The gift does not qualify as a charitable contribution for Federal income tax purposes. a. True Powered by Cognero

Page 4


Name:

Class:

Date:

Chapter 18: The Federal Gift and Estate Taxes b. False 41. The essential purpose of the marital deduction is to defer any estate tax liability until the second spouse dies. a. True b. False 42. At the time of his death, Raul owned a residence with his wife, Manuela, as joint tenants. Manuela purchased the residence 10 years ago at a cost of $300,000; it now has a fair market value of $1.4 million. Raul’s estate will be allowed no marital deduction as to the property. a. True b. False 43. Sally’s will passes real estate to Mario (her surviving husband). The real estate is worth $8,000,000 but is subject to a mortgage of $2,000,000. The transfer provides Sally’s estate with a marital deduction of $6,000,000. a. True b. False 44. Lila is the owner and beneficiary of a policy on the life of her wife, Anna. Upon Anna’s death, the insurance proceeds paid to Lila do not qualify for the marital deduction. a. True b. False 45. To avoid the terminable interest limitation on the marital deduction, a QTIP election should be made. a. True b. False 46. In the case of a transfer by gift, a QTIP election causes the property to be subject to the estate tax upon the death of the donee's (second to die) spouse. a. True b. False 47. Sidney dies and leaves property to his sister Giselle. Thirteen months later, Giselle dies. Giselle’s estate can claim a full credit for any Federal estate taxes paid by Sidney’s estate as to amounts passing to Giselle. a. True b. False 48. In 2013, Katelyn inherited considerable property when her father died. When Katelyn dies in 2025, her estate may be able to claim a credit as to some of the estate taxes paid by her father’s estate. a. True b. False 49. At the time of Dylan’s death, he was a resident of the United States. He owns land located in a foreign country, which is subject to that country’s estate tax. This same land also can be subject to the Federal estate tax. a. True b. False 50. Peggy gives $200,000 to her grandson. This is an example of a direct skip for purposes of the generation-skipping Powered by Cognero

Page 5


Name:

Class:

Date:

Chapter 18: The Federal Gift and Estate Taxes transfer tax (GSTT). a. True b. False Multiple Choice 51. Which, if any, of the following is not a characteristic of the Federal gift tax? a. A charitable deduction is available. b. The alternate valuation date of § 2032 can be elected. c. A disclaimer procedure may avoid the tax. d. A marital deduction is available. 52. Which, if any, of the following is a characteristic of the Federal estate tax? a. A foreign tax credit is available. b. A credit for tax on prior transfers may be available. c. A charitable deduction is available. d. All of these are correct. 53. Which of the following is not a characteristic of both the Federal gift tax and the Federal estate tax? a. A deduction for state death taxes may be available. b. A charitable deduction is available. c. A marital deduction is available. d. An exclusion amount is available in computing the tax. 54. The Federal transfer taxes generally apply at a rate of: a. 10%. b. 40%. c. 50%. d. 65%. 55. The Federal transfer taxes are applied in a manner that is: a. Unified among the taxes. b. Cumulative over the individual's lifetime. c. Both a. and b. d. Neither a. nor b. 56. Which, if any, of the following items is subject to indexation (adjusted to reflect inflation)? a. The election to split gifts under § 2513. b. The limitation placed on the amount allowed as a charitable contribution for estate tax purposes (§ 2055). c. Annual gift tax exclusion. d. Unified transfer tax rates. 57. Which, if any, of the following statements correctly reflects the rules applicable to the alternate valuation date? a. The election is made by the executor. Powered by Cognero

Page 6


Name:

Class:

Date:

Chapter 18: The Federal Gift and Estate Taxes b. It can be elected even though no estate tax return (i.e., Form 706) need be filed. c. It can be elected only if it reduces the amount of the gross estate or reduces the estate tax liability. d. Its election does not affect the income tax basis of property included in the gross estate. 58. In which, if any, of the following independent situations can the alternate valuation date be elected? Value of Gross Estate Estate Tax Result Date of Death Alternate Date Date of Death Alternate Date a. $6,000,000 $6,100,000 $400,000 $390,000 b.

c.

d.

$5,900,000

$5,800,000

$400,000

$405,000

$6,100,000

$6,000,000

$390,000

$380,000

$6,200,000

$6,300,000

$500,000

$490,000

59. At the time of his death, Tom owned the following common stock.

Citron Corporation Grey Corporation

Date of Death Value $1,500,000 1,300,000

Value Six Months Later $1,100,000 1,400,000

If the alternate valuation date is properly elected, the value of Tom’s estate as to these stocks is: a. $2,300,000. b. $2,400,000. c. $2,500,000. d. $2,700,000. 60. The Federal unified transfer tax exclusion amount: a. Is indexed for inflation. b. Applies only to the estate tax. c. Is a different amount for the estate and gift taxes. d. Is doubled on a joint gift tax return. 61. The alternate valuation date: a. Applies for Federal estate tax purposes. b. Applies for Federal gift tax purposes. c. Both a. and b. d. Neither a. nor b. 62. Ling and Jiang are unrelated and equal joint tenants in a plot of land. Ling died this year. Ling’s share of the land goes Powered by Cognero

Page 7


Name:

Class:

Date:

Chapter 18: The Federal Gift and Estate Taxes to: a. The party named in Ling’s will. b. Ling’s surviving spouse. c. Jiang under community property principles. d. Jiang under a right of survivorship. 63. In which of the following independent situations has Jean made a gift? a. She gives her 19-year old son $20,000 to use for his college expenses. b. She buys her nondependent grandfather a new $120,000 RV for his birthday. c. She sends $54,000 to Collins University to cover her nephew’s tuition. The nephew does not qualify as Jean’s dependent. d. She contributes $10,000 to her U.S. Senator’s reelection campaign. 64. In which of the following independent situations has Trent made a gift? a. He established an irrevocable trust, income payable to himself for life and, upon his death, remainder to his children. b. He dies owning a U.S. savings bond with ownership listed as: “Trent, payable to Sue on Trent’s death.” Sue redeems the bond. c. He sends $25,000 to Alice’s oral surgeon in payment of her dental implants. Alice is Trent’s sister and does not qualify as his dependent. d. He pays Eva $800,000 in a property settlement of her marital rights. One month later, Trent and Eva are divorced. 65. Stacey inherits unimproved land (fair market value of $6 million) from her father on June 1, 2024. She disclaims her interest in the property as follows: one-third on December 1, 2024; one-third on January 3, 2025; and the remaining onethird on May 31, 2025. In all cases, the disclaimers pass the interest to her son (the next heir under state law). The Federal gift tax applies to Stacey for: a. All of the disclaimers. b. The disclaimer made in 2024. c. The May 31, 2025 disclaimer. d. All of these disclaimers made in 2025. e. None of these disclaimers. 66. The Federal gift tax does not include a: a. Deduction for state gift taxes paid. b. Charitable deduction. c. Gift-splitting election. d. Marital deduction. 67. Which of the following is a correct statement regarding the filing of a gift tax return (Form 709)? a. A donor must file a Form 709 in the same year in which the gift was made. b. The due date of a Form 709 is the same as the due date of the donor’s Form 1040. c. A Form 709 may have to be filed even though the value of the gift was less than the amount of the annual exclusion. d. Genji gives her husband a new Mercedes convertible for his birthday. She must file a Form 709 to report the Powered by Cognero

Page 8


Name:

Class:

Date:

Chapter 18: The Federal Gift and Estate Taxes gift even though no gift tax results. 68. The Federal gift-splitting election: a. Allows the annual exclusion of both spouses to reduce the gift tax due. b. Allows the exemption equivalent of both spouses to reduce the gift tax due. c. Is made on both spouses’ Forms 709. d. All of these are correct. 69. Andrea dies on April 30. Which, if any, of the following items is included in her gross estate? a. Rents for the month of May (received on May 2) on an apartment building she owned. b. Rents for the month of March (received on May 2) on an apartment building she owned. c. Insurance recovery from a fire that occurred on November 1 and destroyed Andrea’s residence. d. A loan made by Andrea to her daughter and forgiven by Andrea in a prior tax year. 70. At the time of his death, Norton was involved in the following: ∙

Owned land in joint tenancy with Emily. The land is worth $600,000 and was purchased by Norton 15 years ago for $150,000.

Owned land in a tenancy by the entirety with his wife Zhi. The land is worth $800,000 and was purchased by Norton five years ago for $450,000.

Owned land in an equal tenancy in common with Noah. The land is worth $400,000 and was purchased by Norton four years ago for $300,000.

Owned City of Dayton bonds worth $500,000.

What amount is included in Norton’s gross estate? a. $900,000 b. $1,100,000 c. $1,700,000 d. $2,100,000 71. At the time of her death on October 4, Kaitlyn: ∙

Was the sole life beneficiary of a trust (assets worth $2 million) created 10 years ago by Paul (Kaitlyn’s husband). The transfer was by gift of securities then worth $500,000. Paul and Kaitlyn’s children are the remainder beneficiaries. There was no QTIP election.

Owned stock in Mauve Corporation (basis of $800,000 and fair market value of $1 million). On September 7, a dividend of $48,000 was declared on the stock payable to all shareholders of record on October 3. The $48,000 was received by Kaitlyn’s executor on October 19.

Made a taxable gift of $400,000 in a prior tax year.

As to these transactions, Kaitlyn’s gross estate includes: Powered by Cognero

Page 9


Name:

Class:

Date:

Chapter 18: The Federal Gift and Estate Taxes a. $1,048,000. b. $1,448,000. c. $3,000,000. d. $3,048,000. 72. At the time of his death, Jason was a participant in Silver Corporation’s qualified pension plan and group term life insurance. The balance of the survivorship feature in his pension plan is that: Contributions by Silver After-tax contributions by Jason Accumulated plan earnings

$800,000 400,000 300,000

The term insurance has a maturity value of $100,000. All amounts are paid to Pam, Jason’s daughter. One result of these transactions is: a. Pam must pay income tax on $300,000. b. Pam must pay income tax on $1,100,000. c. Jason’s gross estate must include $1,200,000. d. Jason’s gross estate must include $1,500,000. 73. At the time of her death, Juliana: ∙

Owned an insurance policy on the life of her father with a replacement cost of $250,000 and maturity value of $800,000. The designated beneficiary of the policy is Juliana’s estate.

Was an equal tenant in common with her brother in a tract of land worth $800,000. The land was inherited from their grandmother 10 years ago when it had a value of $200,000.

Was a joint tenant with her two sisters in stock worth $1,500,000. The stock was inherited from their grandmother 10 years earlier when it had a value of $500,000.

As to these transactions, Juliana’s gross estate must include: a. $250,000. b. $1,150,000. c. $1,400,000. d. $2,150,000. 74. Mandy and Hal (mother and son) purchased land for $600,000 as joint tenants with right of survivorship. Of the $600,000 purchase price, Mandy provided $300,000 and Hal $300,000 (of which $200,000 had been received as a gift from Mandy). Hal dies first when the land is worth $3,000,000. As to the land, Hal’s gross estate must include: a. $500,000. b. $1,500,000. c. $2,500,000. d. $3,000,000. 75. In 2021, Thalia purchased land for $900,000 and lists title in the names of her daughters as follows: “April and Theresa, joint tenants with right of survivorship.” In 2023, April and Theresa purchase an apartment building for $1 million as equal tenants in common; April furnished $400,000 and Theresa furnished $600,000 of the cost. April died Powered by Cognero

Page 10


Name:

Class:

Date:

Chapter 18: The Federal Gift and Estate Taxes before Theresa in 2025 when the land is worth $1.5 million and the apartment building is worth $2 million. One of the results of these transactions is: a. April made a gift to Theresa of $100,000 in 2023. b. None of the land is included in April’s gross estate. c. April’s gross estate includes $800,000 (40% × $2 million) as to the apartment building. d. April’s gross estate includes $1,750,000 as to these properties. 76. Matt and Patricia are married and live in Oregon. In 2017 and using her funds, Patricia purchased a residence for $400,000, listing title to the property as “Matt and Patricia, joint tenants with right of survivorship.” In 2025, Matt dies before Patricia when the residence is worth $2 million. A correct statement as to these transactions is: a. In 2025, Matt’s gross estate includes $1 million and a marital deduction of $1 million is allowed for estate tax purposes. b. In 2017, Patricia made a gift to Matt but no marital deduction is available for gift tax purposes. c. In 2017, Patricia did not make a taxable gift to Matt. d. In 2025, Matt’s estate includes nothing as to the property. 77. Homer and Laura were married. At the time of Homer’s death, they owned the following: land as tenants by the entirety worth $2,000,000 (purchased by Homer) and stock as equal tenants in common worth $3,000,000 (purchased by Laura). Homer owned an insurance policy on his life (maturity value of $1,000,000) with Laura as the designated beneficiary. Homer’s will passes all his property to Laura. How much marital deduction is allowed for Homer’s estate? a. $2,000,000 b. $2,500,000 c. $3,500,000 d. $4,500,000 78. In 2014, Drew creates an irrevocable trust with $1,000,000 of securities. Under the terms of the trust, Paula (Drew’s wife) is granted a life estate with remainder to their children. Drew makes a QTIP election as to the trust. Drew dies in 2016 when the trust is worth $1,500,000, and Paula dies in 2025 when the trust is worth $5,000,000. Which, if any, of the following is a correct statement? a. The trust is included in Drew’s gross estate when he dies in 2016. b. None of the trust is included in Paula’s gross estate when she dies in 2025. c. Drew does not get a marital deduction in 2014. d. All of the value of the trust ($5,000,000) is included in Paula’s gross estate when she dies in 2025. 79. Pursuant to Corey’s will, Emma (Corey’s sister) inherits his property. Emma dies in a later tax year. The estate tax attributable to the inclusion of the property in Corey’s gross estate was $300,000. The estate tax attributable to the inclusion of the property in Emma’s gross estate is $400,000. Emma’s credit for the tax on prior transfers is: a. $0 if Emma died nine and one-half years after Corey. b. $300,000 if Emma died three years after Corey. c. $400,000 if Emma died one year after Corey. d. $180,000 if Emma died five and one-half years after Corey. 80. Concerning the Federal tax on generation-skipping transfers: a. The tax applies in addition to any applicable gift or estate tax. b. The tax applies in lieu of any applicable gift or estate tax. c. The tax is applied at a flat 33 percent tax rate. Powered by Cognero

Page 11


Name:

Class:

Date:

Chapter 18: The Federal Gift and Estate Taxes d. The annual gift tax exclusion cannot be used to reduce the tax. 81. Concerning the Federal tax on generation-skipping transfers: a. The charitable deduction is allowed to reduce the tax. b. The marital deduction is allowed to reduce the tax. c. A credit is allowed for any state-level GST tax paid. d. All of these statements are true. Matching Classify each of the following statements: a. No taxable transfer occurs. b. Gift tax applies. c. Estate tax applies. 82. Hector transfers funds to his aunt so she can obtain a hip operation. 83. Under a prenuptial agreement, Violeta transfers stock to Pedro. One month later, Violeta and Pedro are married. 84. In full settlement of his marital rights, Henry transfers property to his spouse, Robert. Three months later, Henry and Robert are divorced. 85. Kumar pays for the tuition for his niece to attend Edgar University. 86. Under her father’s will, Faith is to receive 10,000 shares of GE common stock. Ten months after her father’s death, Faith disclaims 10,000 shares. 87. Under her father’s will, Faith is to receive 10,000 shares of GE common stock. Eight months after her father’s death, Faith disclaims the 10,000 shares. 88. Anish takes out an insurance policy on his life and designates his estate as the beneficiary. Anish dies four years later. 89. Van takes out an insurance policy on Leah’s life and designates himself as the beneficiary. Leah is Van’s wife. Two years later, Leah dies, and Van collects the insurance proceeds. Match each statement with the correct choice. Some choices may be used more than once or not at all. a. In the current year, Luna, a widow, dies. Two years ago she inherited a large amount of wealth from her brother. b. Death does not defeat an owner’s interest in property. c. Exists only if a married couple is involved. d. A type of state tax on transfers by death. e. Must decrease the amount of the gross estate. f. Annual exclusion not allowed. g. Cumulative in effect. h. Right of survivorship present as to type of ownership. Powered by Cognero

Page 12


Name:

Class:

Date:

Chapter 18: The Federal Gift and Estate Taxes i. Overrides the terminable interest rule of the marital deduction. j. Exemption equivalent. k. Bypass amount. l. No correct match provided. 90. Election to split gifts 91. Exclusion amount 92. Federal gift tax 93. Inheritance tax 94. QTIP election 95. Tenancy by the entirety 96. Joint tenancy 97. Tenancy in common 98. Community property 99. Credit for tax on prior transfers 100. Alternate valuation date 101. Future interest Classify each of the following statements. a. No taxable transfer occurs. b. Gift tax applies. c. Estate tax applies. 102. Homer purchases a U.S. savings bond listing title as: “Homer, payable to Bianca upon Homer’s death.” Bianca is Homer’s sister. 103. Homer purchases a U.S. savings bond listing title as: “Homer, payable to Bianca upon Homer’s death.” Bianca is Homer’s sister. Homer dies four years later, and Bianca cashes in the bond and keeps the proceeds. 104. Howard establishes a trust, life estate to his children and remainder to the grandchildren. Under its terms, the trust can be revoked by Howard. 105. Howard establishes a trust, life estate to his children, remainder to the grandchildren. Under its terms, the trust can be revoked by Howard. Howard later relinquishes the right to revoke the trust. 106. Meg gives her 18-year-old son money for his college tuition and living expenses (e.g., room and board). 107. Javiere pays the medical providers (e.g., physicians, hospital) for his aunt’s knee replacement operation. Powered by Cognero

Page 13


Name:

Class:

Date:

Chapter 18: The Federal Gift and Estate Taxes 108. Cash donation to the reelection campaign of a candidate for the U.S. House. 109. Maggie purchased an insurance policy on Jim’s life and designated Susan as the beneficiary. 110. Maggie purchased an insurance policy on Jim’s life and designated Susan as the beneficiary. Four years later, Jim dies, and Susan collects the insurance proceeds. 111. Using his own funds, Horace establishes a savings account designating ownership as follows: “Horace and Nadine as joint tenants with right of survivorship.” Horace later withdraws all of the funds. 112. Using his own funds, Horace establishes a savings account designating ownership as follows: “Horace and Nadine as joint tenants with right of survivorship.” Nadine predeceases Horace. 113. Using his own funds, Horace establishes a savings account designating ownership as follows: “Horace and Nadine as joint tenants with right of survivorship.” Horace predeceases Nadine. Classify each of the following independent statements:. a. Some or all of the asset is included in the decedent’s gross estate. b. None of the asset is included in the decedent’s gross estate. 114. Interest on municipal bonds accrued after death. 115. Cash dividends on stock owned by the decedent (declaration and record dates preceded death but payment date was after death). 116. State income tax refund received after death on a tax return filed before death. 117. Note receivable issued by a grandson and forgiven by the decedent in her will. 118. Ten cemetery lots purchased by decedent prior to death for use by himself and his family. 119. Bank account held as joint tenant with mother. Mother provided all of the funds. Mother survives. 120. Land held as tenants by the entirety with surviving spouse. Decedent provided none of the funds. 121. Proceeds of an insurance policy on decedent’s life. Decedent’s son purchased the policy and is its owner and beneficiary. 122. Decedent owned a policy on the life of his spouse with himself as the designated beneficiary. The spouse survives. Classify each statement appropriately. a. Deductible from the gross estate in arriving at the taxable estate. b. Not deductible from the gross estate in arriving at the taxable estate. 123. Mortgage on land included in gross estate and willed to decedent’s children. 124. State death tax imposed on the estate. 125. Selling expenses incurred to sell estate assets in order to pay administration expenses. 126. Post-death property taxes paid to the county on realty included in the gross estate. Powered by Cognero

Page 14


Name:

Class:

Date:

Chapter 18: The Federal Gift and Estate Taxes 127. Casualty loss to property before the death of the owner. 128. Casualty loss to property after the death of the owner. 129. Payment of unpaid gift taxes. 130. State income taxes accrued prior to death. Subjective Short Answer 131. Calvin’s will passes $800,000 of cash to his widowed sister, Fang. The estate tax attributable to the cash is $110,000. Fang dies five years later, and the estate tax generated by the $800,000 is $100,000. How much of a credit for tax on prior transfers will Fang’s estate be allowed? 132. Ben and Lynn are married and have four preteen grandchildren. They want to contribute to a § 529 plan on behalf of their education. For 2024, what is the maximum amount they can transfer to the plan without making a taxable gift? 133. At the time of her death on June 6, Mary owned the following assets. ∙

Taupe Corporation stock (cost $400,000, FMV $800,000). On May 4, Taupe declared a cash dividend, payable on June 15, to shareholders as of the record date of June 4. Mary’s executor received the $40,000 dividend on the scheduled payment date.

City of Boise bonds (cost $800,000, FMV $780,000). Interest accrued to June 6 was $42,000. The executor eventually collected $50,000 (included postdeath accrual of $8,000) on July 20.

As to these transactions, how much is included in Mary’s gross estate? 134. At the time of her death, Sophia was a participant in her employer’s qualified pension plan. Her accrued balance in the plan is as follows. Employer’s contribution Sophia’s contribution (pre-tax) Income earned by plan over the years

$1,300,000 800,000 900,000

Sophia also was covered by her employer’s group term life insurance program. Her policy (maturity value of $100,000) is made payable to Aiden (Sophia’s husband). Aiden also is the designated beneficiary of the pension plan. a.

Regarding these assets, how much is included in Sophia’s gross estate?

b.

In Sophia’s taxable estate?

c.

How much gross income must Aiden recognize, when collecting on these items?

135. On the date of her death, Ava owned the following. ∙ An insurance policy (face amount of $500,000) on the life of Benjamin (Ava’s current Powered by Cognero

Page 15


Name:

Class:

Date:

Chapter 18: The Federal Gift and Estate Taxes husband) with herself as the designated beneficiary. The policy has a cash surrender value of $50,000. ∙

A life estate in a trust created by Alexander (Ava’s deceased prior husband). The trust (current value of $2,900,000) was worth $1,000,000 when created 10 years ago. A QTIP election was made by the executor of Alexander’s estate.

Federal income tax refund of $80,000 on a prior year’s tax return and paid to the executor of Ava’s estate. As to these items, how much is included in Ava’s gross estate? 136. Daniel and Mia acquired realty for $2 million with Daniel furnishing $1.5 million of the purchase price and Mia providing the balance. Title to the property is listed as: “Daniel and Mia, joint tenants with right of survivorship.” This year Mia dies before Daniel when the realty is worth $4 million. How much is included in Mia's gross estate under the following circumstances?

a.

Daniel and Mia are brother and sister.

b.

Daniel and Mia are a married couple.

137. In 2017, Noah and Kelly acquired real estate for $2,000,000 with Noah furnishing $400,000 of the purchase price and Kelly providing the balance. Title to the property is listed as: “Noah and Kelly, equal tenants in common.” Noah dies before Kelly in 2025 when the real estate is worth $4,000,000. a.

Were there any tax consequences in 2017? Explain.

b.

As to the real estate, how much is included in Noah’s gross estate?

c.

As to choices a. and b., would it make any difference whether Noah and Kelly are brother and sister or a married couple?

138. Jesse owns an insurance policy on the life of his father, Logan. Upon Logan’s death, the policy proceeds of $2,000,000 are paid to the designated beneficiary, Grace. What are the transfer tax consequences resulting from Logan’s death based on the following independent assumptions? a.

Grace is Jesse’s daughter.

b.

Grace is Jesse’s spouse.

c.

What are the tax consequences if Jesse dies before both Grace and Logan)?

139. At the time of her death, Amber owns property worth $5,000,000. Other information regarding her affairs is as follows. Powered by Cognero

Page 16


Name:

Class:

Date:

Chapter 18: The Federal Gift and Estate Taxes Unpaid pledge to the building fund of her church College graduation gift she had promised her grandson Local property taxes owed (accrued prior to death) Mortgage owed on personal residence

$50,000 20,000 100,000 800,000

All of these items were paid by her estate, and none were deducted on other tax returns. What is Amber’s taxable estate? 140. At the time of Clint’s death, part of his estate consists of the following. ∙

Roth IRA (value of $1,000,000) with Jennifer as the designated beneficiary.

Land (worth $3,000,000) held in joint tenancy with Jennifer. Jennifer is Clint’s wife and originally furnished the purchase price.

Building (worth $3,000,000) held as equal tenants in common with Jennifer and Dana. Dana is Clint’s mother, and she originally purchased the property.

Under Clint’s will, all of his property passes to his wife, Jennifer. How much marital deduction is Clint’s estate allowed? Clint and Jennifer live in Tennessee. Essay 141. Chung is his mother’s sole heir and is the designated executor of her estate. Although the alternate valuation date would yield a smaller gross estate and less estate tax liability, the election is not made. Instead, Chung files a Form 706 for his mother’s estate using higher date of death values. Why? 142. Lily pays for her grandson’s college expenses. Under what conditions might such payments not constitute a gift? 143. What is the justification for the terminable interest rule that is applicable to the marital deduction? Give an example. 144. Walt dies without a will in the current year with a gross estate valued at $24,000,000. Under applicable state law, Walt’s property passes to Kelly and to Belle in that order. Kelly has an estimated net worth of $18,000,000 while Belle's is zero. From a tax planning standpoint, what course of action might be advisable?

Powered by Cognero

Page 17


Name:

Class:

Date:

Chapter 18: The Federal Gift and Estate Taxes Answer Key 1. True 2. True 3. True 4. True 5. True 6. False 7. False 8. True 9. True 10. True 11. False 12. False 13. True 14. False 15. False 16. True 17. False 18. True 19. False 20. True 21. True 22. False 23. True 24. False 25. True Powered by Cognero

Page 18


Name:

Class:

Date:

Chapter 18: The Federal Gift and Estate Taxes 26. False 27. True 28. True 29. False 30. True 31. False 32. False 33. True 34. True 35. False 36. True 37. True 38. False 39. False 40. True 41. True 42. False 43. True 44. True 45. True 46. True 47. True 48. False 49. True 50. True Powered by Cognero

Page 19


Name:

Class:

Date:

Chapter 18: The Federal Gift and Estate Taxes 51. b 52. d 53. a 54. b 55. c 56. c 57. a 58. c 59. c 60. a 61. a 62. d 63. b 64. a 65. c 66. a 67. c 68. d 69. b 70. c 71. a 72. b 73. b 74. a 75. d 76. a Powered by Cognero

Page 20


Name:

Class:

Date:

Chapter 18: The Federal Gift and Estate Taxes 77. c 78. d 79. d 80. a 81. d 82. b 83. b 84. a 85. a 86. b 87. a 88. c 89. a 90. c 91. j 92. g 93. d 94. i 95. c 96. h 97. b 98. c 99. a 100. e 101. f Powered by Cognero

Page 21


Name:

Class:

Date:

Chapter 18: The Federal Gift and Estate Taxes 102. a 103. c 104. a 105. b 106. a 107. a 108. a 109. a 110. b 111. a 112. a 113. c 114. b 115. a 116. a 117. a 118. b 119. b 120. a 121. b 122. a 123. a 124. a 125. a 126. b 127. b Powered by Cognero

Page 22


Name:

Class:

Date:

Chapter 18: The Federal Gift and Estate Taxes 128. a 129. a 130. a 131. $60,000 (60% × $100,000). 132. $720,000. [2 (number of donors) × 4 (number of donees) × $18,000 (annual exclusion) × 5 (number of years)] 133. $1,662,000 [$800,000 (FMV of Taupe stock) + $40,000 (dividend on Taupe stock) + $780,000 (FMV of Boise bonds) + $42,000 (accrued interest)] 134. a. $3,100,000. ($1,300,000 + $800,000 + $900,000 + $100,000). b. $-0-. Because all of these items pass to the decedent’s surviving spouse, they qualify for the unlimited marital deduction. c. $3,000,000. ($1,300,000 + 800,000 +900,000). (The life insurance proceeds ($100,000) are exempt from income tax). Since Sophia's contribution to the pension plan was made with pre-tax earnings, it will be subject to income tax when distributed.

135. $3,030,000. [$50,000 (life insurance policy not yet matured) + $2,900,000 (QTIP trust) + $80,000 (tax refund)]. 136. a. $1,000,000. (25% × $4,000,000). b.

$2,000,000. (50% × $4,000,000). Automatic 50-50 rule.

137. a. When the tenancy was created, Kelly made a gift to Noah of $600,000 [$400,000 (Noah’s contribution) – $1,000,000 (value of Noah’s one-half interest in the property)]. b.

$2,000,000. (50% × $4,000,000).

c.

The only difference would be in choice a. If Noah and Kelly are a married couple, the $600,000 gift would be offset by the marital deduction.

138. a. Jesse has made a gift to Grace of $2,000,000. Powered by Cognero

Page 23


Name:

Class:

Date:

Chapter 18: The Federal Gift and Estate Taxes b.

A $2,000,000 gift still occurs, but it is offset by the marital deduction.

c.

Because the policy has not matured, § 2042 does not apply. However, the policy is an asset owned by Jesse and will be included in his gross estate under § 2033 at its replacement cost.

139. $4,050,000. [$5,000,000 (gross estate) – $50,000 (church pledge) – $100,000 (local property taxes) – $800,000 (mortgage)]. The graduation gift is not supported by consideration and does not qualify as an obligation of the estate , so no deduction is allowed. A special rule allows a deduction as to unpaid church pledges. 140. $3,500,000 [$1,000,000 + (50% × $3,000,000) + (1/3 × $3,000,000)]. 141. Presuming Chung is acting rationally, he is probably sacrificing an estate tax saving to achieve the additional income tax basis that comes with the use of higher date of death valuation. Such additional basis could provide Chung with an immediate income tax loss (or less gain) should he sell some of his mother’s assets. For assets retained, it generates greater depreciation deductions. Upon ultimate disposition of the assets, less gain (or more loss) results. Manipulating the gain in this manner could be particularly attractive if such gain would be ordinary (rather than capital). 142. Lily's grandson might be her dependent and providing for the support of a dependent is not a gift. Even if he is not a dependent, under applicable state law, Lily could have an obligation to provide support for the education of the grandchild, and satisfying an obligation of support is not a gift. Lastly, payments for tuition made directly to the educational institution are not subject to the gift tax. (This provision applies to the payment of tuition only and not room and board, books, supplies and other educational expenses). 143. Example. Spouse 1 dies and leaves their property in trust, life estate to Spouse 2 and remainder to their children. Spouse 2 dies five years later. Without the terminable interest rule, most of the trust would not be subject to an estate tax. Spouse 1 would be allowed a marital deduction and Spouse 2’s interest is valued at zero at their death. It seems unfair to allow a deduction without creating a gross estate inclusion at a later date; the marital deduction should be seen as a deferral, not as a forgiveness, device. With a QTIP election Spouse 1’s estate receives the marital deduction and the trust is included in Spouse 2’s gross estate. 144. This might be a good situation to make use of a disclaimer. Thus, Kelly could disclaim some of the $24,000,000 inheritance and, in effect, pass it to Belle free of any transfer tax to Kelly (i.e., estate or gift tax). This presumes that Kelly otherwise would transfer some of her wealth to Belle in the future in a taxable transfer.

Powered by Cognero

Page 24


Name:

Class:

Date:

Chapter 19: Family Tax Planning True / False 1. One of the objectives of family tax planning is to minimize income taxes on transfers of property within the family unit. a. True b. False 2. If a decedent’s household goods are sold through a public auction, the price received should be the valuation used for Federal estate tax purposes. a. True b. False 3. At the time of his death, Rex owned an RV. For valuation purposes, the RV should be included in his gross estate at the price a dealer in RVs would pay for the property. a. True b. False 4. Doug inherited his mother’s bedroom furniture worth $3,000. For sentimental reasons, Martha, the daughter, pays Doug $3,500 for the furniture. The furniture should be included in the mother’s gross estate at $3,500. a. True b. False 5. At the time of his death, Fred held some notes receivable for loans he made to his two daughters, the payment of which he forgives in his will. The amount to be included in Fred’s gross estate as to these notes is not affected by his forgiveness. a. True b. False 6. Valuation tables issued by the IRS are used when a trust is created and beneficiaries are named. a. True b. False 7. Edgar creates a trust, life estate to Connie (age 46), remainder to Arturo (age 18). In determining the value of the life estate, use the multiple given in the IRS valuation table for a person age 18. a. True b. False 8. Natalie creates a trust, income payable to Ava (age 12) for 10 years, remainder to Samantha (age 32). In determining the value of these two gifts (i.e., income interest for a term certain and a remainder interest), use of more than one IRS valuation table will be necessary. a. True b. False 9. A farm has a best-use valuation of $2.9 million and a current use valuation of $2 million. If § 2032A is elected, the farm can be valued in the deceased owner’s gross estate at $2 million. a. True b. False Powered by Cognero

Page 1


Name:

Class:

Date:

Chapter 19: Family Tax Planning 10. A farm has a best-use valuation of $3 million and a current use valuation of $1.2 million. If death occurs in 2024 and § 2032A is elected, the farm can be valued in the deceased owner’s gross estate at $1.2 million. a. True b. False 11. A qualifying heir can make the § 2032A special valuation election only if he is not sure that he will keep the property for the full 10 years. a. True b. False 12. A recapture of special use valuation will occur if a qualified heir ceases to use the property for farming purposes but does not otherwise dispose of it. a. True b. False 13. The election of special use valuation estate tax savings has Federal income tax implications. a. True b. False 14. Any recapture of special use valuation estate tax savings is imposed on the executor of the estate. a. True b. False 15. One way to dispute the existence of large goodwill in valuing an enterprise is to argue that the decedent was not a key person in the operation of the business. a. True b. False 16. Application of the blockage rule has not been limited to just the valuation of stocks and securities; it has been applied to other assets. a. True b. False 17. Buy-sell agreements can be used to help solve the estate tax valuation and liquidity problems involved in the transfer by death of an interest in a small business. a. True b. False 18. Funding an entity type of buy-sell agreement with life insurance requires fewer policies than in the case of the crosspurchase type. a. True b. False 19. As to property received as a gift, a donee’s income tax basis for gain or loss may not be the same. a. True b. False Powered by Cognero

Page 2


Name:

Class:

Date:

Chapter 19: Family Tax Planning 20. Eight months after Noah died, Monique sold stock that she inherited from him. If the stock was sold for less than its value on the date of Noah’s death, Monique recognizes a short-term capital loss. a. True b. False 21. Richard and Marie are joint tenants in a tract of land. Upon Richard’s prior death, Marie’s income tax basis in the land does not change. a. True b. False 22. Rick and Gail are equal tenants in common in real estate. Upon Gail’s prior death, Rick’s basis in the real estate does not change. a. True b. False 23. The election by an estate of special use valuation as to real estate or the alternate valuation date will have no effect on the income tax basis of the property received by the heirs. a. True b. False 24. Brad and Heather are husband and wife and live in New Mexico. Under Brad’s will, his share of the community property passes to the children. Upon Brad’s prior death, there will be a change in her income tax basis in her half of the community property. a. True b. False 25. Mateo makes a gift of property (basis of $100,000; fair market value of $500,000) to his aunt, Mary. Ten months later and when the property is worth $550,000, Mary dies. Under Mary’s will, the property passes to Isabella (Mateo’s wife). Isabella’s basis in the property is $550,000. a. True b. False 26. If a traditional IRA is subject to both estate and income taxes, a withdrawal by the heir constitutes income in respect of a decedent (IRD). a. True b. False 27. The special use valuation method of § 2032A is available for valuing transfers by gift. a. True b. False 28. Passing installment notes by death will not avoid any income tax on the deferred gain. a. True b. False 29. If depreciable property is transferred by gift, any depreciation recapture potential carries over to the donee. a. True Powered by Cognero

Page 3


Name:

Class:

Date:

Chapter 19: Family Tax Planning b. False 30. If depreciable property is passed by death, any depreciation recapture potential carries over to the heir. a. True b. False 31. By maximizing the marital deduction, any estate tax is postponed until the death of the surviving spouse, an advantage in present value terms. a. True b. False 32. Under proper circumstances, a disclaimer by an heir can increase the charitable deduction allowed to a decedent. a. True b. False 33. Because of the estate tax deduction, a bequest to charity at death is preferable to a lifetime transfer. a. True b. False 34. Taj leaves one-half of his estate to his wife, Ramona, and the remainder to a qualified charity. Taj’s estate taxes are not reduced if Ramona disclaims her interest in favor of the charity. a. True b. False 35. A disclaimer by a surviving spouse will generate additional estate tax because it reduces the amount of marital deduction allowed. a. True b. False 36. For deaths in 2024, it is possible for a surviving spouse to receive an additional bypass amount of as much as $13.61 million. a. True b. False 37. For the IRS to grant a discretionary extension of time to pay estate taxes, the executor must show that the estate would otherwise undergo undue hardship. a. True b. False 38. In satisfying the more-than-35% test in qualifying for a deferral of estate taxes related to a closely held business, interests in multiple businesses are aggregated when the decedent’s gross estate includes 20% or more of the value of each such business. a. True b. False 39. A decedent owned 25% of the voting stock of Siskin Corporation, which has 53 shareholders. The decedent’s estate cannot elect the deferral of estate tax payments. Powered by Cognero

Page 4


Name:

Class:

Date:

Chapter 19: Family Tax Planning a. True b. False Multiple Choice 40. Which, if any, of the following statements reflects the correct Federal transfer tax valuation rules? a. Amounts listed in the classified section of the newspaper are not representative of the value of tangible personalty. b. The value of a note receivable is its face amount. c. Sentimental value should not be considered. d. The geographical location of the property is not relevant. 41. At the time of Elijah’s death, he owned stock in Grey Corporation. The stock is traded on a local exchange with the most recent selling prices as follows.

Four trading days prior to Elijah’s death Six trading days after Elijah’s death

Per Share Price $120 100

Presuming no alternate valuation date election, Elijah’s gross estate should include a per share value of: a. $108. b. $110. c. $112. d. $120. 42. Which of the following independent statements correctly reflects the valuation rules applicable to estate and gift taxes? a. In valuing a trust interest transferred to an individual with a very short life expectancy, use the tables issued by the IRS. b. In valuing an unmatured life insurance policy on which further premiums need to be paid, use the policy’s replacement value. c. In valuing a note receivable, the issuer’s bankruptcy should not be taken into account. d. In valuing a used car, use the trade-in value offered by a dealership as a down payment on a new model. 43. Which, if any, are characteristics of the valuation tables issued by the IRS? a. The IRS must issue new updated tables once a year. b. To determine the factor for a remainder interest, subtract the life estate factor provided in the table from one. c. The same table that provides the factor for a life estate can be used to determine the value of an income interest for a term certain. d. To use the tables, the Federal mid-term interest rate must be known. 44. Which, if any, of the following statements correctly reflects the operational rules under “special use” valuation? a. The election is available for gift tax situations. b. The election permits the valuation of qualifying property at its “most suitable” use value. c. In meeting the 50% test, the qualifying property is considered at its “special” use value. Powered by Cognero

Page 5


Name:

Class:

Date:

Chapter 19: Family Tax Planning d. If the election is made, a sale of the qualifying property within the next 10 years will cause recapture. 45. In 2024, Donna’s father dies and leaves her the family farm. The farm has a current use value of $4,000,000 and a best use value of $5,000,000. If the special use valuation election is made, the farm should be included in the father’s gross estate at a value of: a. $1,390,000. b. $2,610,000 ($4,000,000 – $1,390,000). c. $3,610,000 ($5,000,000 – $1,390,000). d. $4,000,000. 46. A special-use valuation election offers the estate: a. An advantage in terms of the time value of money. b. A disadvantage in terms of the time value of money. c. An extension of the due date of the Form 706. d. A means to deduct a realized loss on real estate. 47. Concerning the factor(s) utilized in valuing the stock in a closely held corporation, which of the following can be considered? a. The company’s dividend-paying capacity. b. The nature of the business. c. The history of the company since its inception. d. All of these factors can be utilized. 48. At the time of his death, Jye was a shareholder in Grebe Corporation. In valuing the Grebe stock included in Jye’s gross estate, the IRS contends that the corporation possessed considerable goodwill. In disputing this contention, which of the following point(s) is/are relevant? a. Average net profit figures include large gains from unrelated investments. b. The rate of return used by the IRS for the type of business involved is too high. c. Jye was not an employee of Grebe but was merely a passive investor. d. To provide financing, Grebe has been obtaining its working capital from banks at a market rate of interest. 49. At the time of Addison’s death, he owned 70% of the stock in Robin Corporation, a closely held family business. Over the past five years, Robin has averaged annual profits of $400,000 in an industry where the typical after-tax rate of return is 9%. If the book value of the corporation’s assets is $1,000,000 and goodwill exists, what might be a realistic value of the stock in Addison’s gross estate? a. $310,000 b. $1,550,000 c. $1,785,000 d. $2,550,000 50. With respect to a stock interest in a closely held corporation, which, if any, of the following works to increase the gross estate value of the interest? a. The stock is not marketable. b. A majority interest is involved. c. The profits of the business are less than the industry average. Powered by Cognero

Page 6


Name:

Class:

Date:

Chapter 19: Family Tax Planning d. The blockage rule applies. 51. Which, if any, of the following statements properly characterizes buy-sell agreements? a. If properly structured, the agreements can control valuation for estate tax purposes. b. If the number of owners is large, utilize a cross-purchase type. c. Agreements cannot be used to control disposition of partnership interests. d. Arrangements work best when the interest to be transferred involves publicly traded securities. 52. Concerning the estate tax valuation of a closely held business: a. Good tax planning argues against any lifetime gifts of ownership interests to other parties. b. Good tax planning strives for a measure of certainty in the valuation if an IRS audit occurs. c. IRS appraisers will determine the estate tax valuation. d. Appraisers for the decedent will determine the estate tax valuation. 53. Mia and Sandra (sisters) purchase realty for $600,000 listing title as “Mia and Sandra, joint tenants with right of survivorship.” Each furnished one-half of the purchase price. Mia predeceases Sandra when the realty is worth $6,000,000. Sandra’s income tax basis in the property is: a. $300,000. b. $3,000,000. c. $3,300,000. d. $6,000,000. 54. Liam and Isla are husband and wife and have always lived in a community property state. At the time of Isla’s prior death, part of their community property includes:

Stock in Scarlet Corporation Apartment building

Adjusted Fair Market Basis Value $2,000,000 $1,000,000 3,000,000 6,000,000

Under Isla’s will, all of her property passes to Liam. After Isla’s death, Liam’s income tax basis in these properties is: a. $2,500,000. b. $3,500,000. c. $7,000,000. d. $8,000,000. 55. Assets that generate income in respect of a decedent (IRD): a. Are excluded from the gross estate. b. Do not generate gross income when the income is collected. c. Both a. and b. are correct. d. Neither a. nor b. is correct. 56. An individual’s lifetime gifts tend to: a. Increase the present value of gift and estate tax liabilities. b. Decrease the present value of gift and estate tax liabilities. c. Increase the chances of an IRS audit. Powered by Cognero

Page 7


Name:

Class:

Date:

Chapter 19: Family Tax Planning d. Increase the frequency of gifts between spouses. 57. An individual’s lifetime gifts tend to: a. Remove asset appreciation from the donor’s gross estate. b. Include life insurance policies payable upon the donor’s death. c. Both a. and b. d. Neither a. nor b. 58. Curt owned the following assets before he gave them to his daughter Carla this year. No Federal gift tax results.

Land Securities

Basis $200,000 800,000

Fair Market Value $400,000 600,000

If Carla immediately sells these assets for $1 million ($400,000 + $600,000), she recognizes: a. No gain or loss. b. A $200,000 LTCG and no loss. c. A $200,000 STCG and $200,000 STCL. d. A $200,000 STCG and no loss. 59. Good family tax planning often means that: a. Gifts of appreciated property will be made from low-bracket children to high-bracket parents. b. Gifts of income-producing property, like corporate bonds, will be made from low-bracket parents to highbracket children. c. Both a. and b. are correct. d. Neither a. nor b. is correct. 60. Good family tax planning often means that: a. Gifts of appreciated property will be made from high-bracket parents to low-bracket children. b. Gifts of income-producing property, like corporate bonds, will be made from low-bracket parents to highbracket children. c. Both a. and b. are correct. d. Neither a. nor b. is correct. 61. Eric, age 80, has accumulated about $16,000,000 in net assets. Among his assets are the following marketable securities held as investments. Cardinal Corporation stock Crane Corporation stock Hawk Corporation stock

Basis $200,000 300,000 50,000

FMV $250,000 250,000 250,000

Eric would like to donate (either by lifetime or testamentary transfer) $250,000 in value to his church. In addition, to consummate a land deal, he needs $250,000 in cash. Looking solely at tax considerations and using only the assets described above, Eric’s best choice is to: a. Donate the Crane stock by gift to the church and sell the Hawk stock now. b. Donate the Hawk stock by death to the church and sell the Cardinal stock now. Powered by Cognero

Page 8


Name:

Class:

Date:

Chapter 19: Family Tax Planning c. Donate the Hawk stock by gift to the church and sell the Crane stock now. d. Donate the Cardinal stock by gift to the church and sell the Hawk stock now. 62. Lisa has been widowed three times. Her first husband died in 2013, leaving an unused exclusion amount of $3.5 million. The second husband died in 2016, leaving an unused exclusion amount of $5.4 million. Lisa’s third husband died in 2021 with an unused exclusion amount of $4 million. Lisa’s DSUE amount upon her death this year is: a. $3.5 million. b. $4 million. c. $5.4 million. d. $12.9 million. 63. Charlotte dies with a gross estate of $30 million. Her estate plan probably should: a. Transfer all of her assets to her husband Trevor, who was one year older than Charlotte. b. Make asset transfers to the children of Charlotte and Trevor equal to her bypass amount and the remainder to Trevor. c. Transfer all of Charlotte’s assets by will to Local Symphony, Charlotte’s favorite charity. d. Transfer Charlotte’s bypass amount to Local Symphony and the rest to the children. 64. For most individuals, an estate plan should prefer: a. Disclaimers of all inheritances by the surviving spouse. b. Disclaimers of all inheritances by the surviving children. c. Lifetime gifts to charity rather than such gifts only at death. d. Gifts to charity through the will rather than during lifetime. 65. Which, if any, of the following items characterizes the election to defer Federal estate tax payments relative to an interest in a closely held business? a. No estate tax due need be paid for the first 5 years. b. No interest needs to be paid for the first 5 years. c. In satisfying the more-than-35% test for qualification, all interests in closely held businesses can be aggregated. d. The 2% rate of interest applies to the total amount of estate tax value. 66. For purposes of the election to defer Federal estate tax payments relative to an interest in a closely held business, an interest in a closely held business does not include: a. A 16% interest in a partnership that has 36 partners. b. A 22% interest in a partnership that has 50 partners. c. A 10% interest in a partnership that has 48 partners. d. A sole proprietorship. Matching Match each statement with the correct choice. Some choices may be used more than once or not used at all. a. Use IRS valuation tables b. Blockage rule c. Special use value Powered by Cognero

Page 9


Name:

Class:

Date:

Chapter 19: Family Tax Planning d. Best or most suitable use value e. Cost of going public f. Replacement cost of a comparable contract g. Portion of a deceased spouse’s share of community property that passes to a surviving spouse h. Cross-purchase buy-sell agreement i. Entity buy-sell agreement j. Bypass amount k. No correct choice is given 67. Valuation of a commercial annuity contract. 68. Valuation of a life insurance policy that is not paid up. 69. Marital deduction allowed. 70. Discount for lack of marketability as to stock. 71. Current use valuation as to certain realty. 72. Discount attributable to a large number of shares. 73. Valuation of life estate interest created by transfer in trust. 74. Exemption equivalent. 75. Surviving owners agree to purchase withdrawing owner’s interest. 76. Corporation agrees to redeem withdrawing shareholder’s stock. Match each statement with the correct choice. Some choices may be used more than once or not used at all. a. Step-up in basis b. Step-down in basis c. Gift of installment notes receivable d. Gift of property that has potential for recapture of depreciation e. Donor’s basis on date of gift (appreciated property given, no gift tax due) f. The amount of the deceased spouse’s taxable estate does not change g. Fair market value on date of gift h. Expected surviving spouse is in good health and 10 years younger i. No income tax consequences to the beneficiary j. Income in respect of a decedent (IRD) k. No correct choice is given 77. Maximize marital deduction. 78. Decedent owned Roth IRA that has appreciated. 79. Donee’s basis for loss. Powered by Cognero

Page 10


Name:

Class:

Date:

Chapter 19: Family Tax Planning 80. Donee’s basis for gain. 81. A gift will cause income tax consequences to the recipient. 82. Decedent owned stock that had depreciated in value. 83. Decedent owned traditional IRA that has appreciated. 84. Surviving spouse disclaims inheritance in favor of bypass amount. 85. A gift will not cause income tax consequences to the donor. 86. Measure of income tax deduction on a gift of depreciated property to charity. Match each statement with the correct choice. Some choices may be used more than once or not used at all. a. Income in respect of a decedent (IRD) b. Discretionary extension of time to pay estate tax c. Extension of time to pay Federal estate tax involving interest in closely held business d. Special use valuation as to certain realty e. Depreciation recapture potential eliminated f. Alternate valuation date g. Election to split gifts h. No correct choice is given 87. Election by estate will reduce the amount of the gross estate. 88. Eliminates preferred stock from donor's gross estate. 89. Transfer by death of depreciable property. 90. Reasonable cause will justify deferral. 91. Doubles the number of annual exlusions available. 92. No step-up in basis at death. 93. Can postpone the payment of any estate tax for 15 years from due date of the return. Subjective Short Answer 94. Ling makes a gift of stock in Snipe Corporation, which is not closely held but is traded in an over-the-counter market. The transactions involving this stock that occurred closest to the date of gift took place five trading days before (mean selling price of $130) and seven days after (mean selling price of $140). Determine the fair market value of the Snipe stock on the date of the gift. 95. Presuming that the special-use valuation election is properly made for a decedent who died in 2024, what value is included in the gross estate in each of the following independent situations? Powered by Cognero

Page 11


Name:

Class:

Date:

Chapter 19: Family Tax Planning Special Use Value $4,000,000 1,200,000

Situation A Situation B

Best Use Value $4,900,000 3,000,000

96. At the time of her death, Latonya owned 60% of the stock in Drab Corporation with the balance of the stock held by her family members. Drab Corporation’s total profits for the past five years are $3,200,000, and the book value of its stock is $2,400,000. If 8.5% is an appropriate after-tax rate of return and goodwill exists, what is a reasonable value for the stock to be included in Latonya’s gross estate? 97. Andrew makes a gift of securities (basis of $1,100,000; fair market value of $800,000) to his son, Carl. What is Carl’s income tax basis in the securities? 98. Fred and Taniel always have lived in a community property state. At the time of Fred's death, the couple held stock that cost them $600,000 but was valued as follows.

Date of Death $5,300,000

Six Months Later $5,200,000

Under Fred’s will, his half of the stock passes to their daughter, Brandi. What income tax basis will Taniel and Brandi have in the stock, if Fred’s estate:

a.

Elects the alternate valuation date?

b.

Does not elect the alternate valuation date?

99. Bob and Paige are married and live in a common law state. Bob owns some real estate (fair market value of $650,000) that the couple would like to give to their five adult married children. The spouses of their children (e.g., son-in-law, daughter-in-law) are to be included in the gifts. Bob and Paige do not want to use any of their unified transfer tax credit. Assuming an annual exclusion for the Federal gift tax of $18,000, suggest a viable way to structure the transfer. 100. In each of the following independent situations, describe the effect of the disclaimer procedure on Ron’s taxable estate. In this regard, advise as to whether a disclaimer should be made, how much should be disclaimed, and by whom, considering only the tax consequences. All events occur in 2024. a.

Ron’s will leaves $15,000,000 to his adult son Phil and the remainder ($5,000,000) to Brittany (Ron’s surviving spouse).

b.

Ron’s will leaves $15,000,000 to Brittany (Ron’s surviving spouse) and the remainder ($5,000,000) to his adult daughter Paula.

Essay 101. Rashan’s estate includes a number of notes receivable signed by his daughter, Tammy. These notes were issued by Tammy on different occasions when she obtained funds from Rashan. The total face amount of these notes is $150,000. Powered by Cognero

Page 12


Name:

Class:

Date:

Chapter 19: Family Tax Planning The notes are forgiven by Rashan’s will. How much, if any, regarding these notes should be included in Rashan’s gross estate? 102. Barney creates a trust, income payable to Madison for five years, remainder to Emma. Emma is Barney’s daughter (and a single parent), and Madison is his 19-year old granddaughter. What might be the justification for this type of trust? 103. Using investments worth $3 million, Keith establishes a trust, life estate to Melinda, remainder to Dana. a.

Has Keith made one gift or two gifts?

b.

How does your answer affect the Federal transfer tax liabilities?

104. If the special use valuation method is elected and certain continuing requirements are not satisfied, recapture occurs. What is the amount of recapture and upon whom is it imposed? 105. In arriving at the value of stock in a closely held business, the IRS frequently imputes goodwill. Comment on how the following independent factors would affect the determination of goodwill. a.

Past profits include a large nonoperating gain.

b.

Shareholder-employees have not been receiving adequate compensation for their services.

c.

The shareholders have been financing corporate operations with interest-free loans.

106. The quantity of stock that a decedent owns in a closely held corporation can have an effect on its valuation. Explain this statement. Hint: Use the terms discount and premium in your answer. 107. At the time of his death, Al owned a majority interest in Macaw Corporation, a closely held holding company. Macaw’s major asset is stock in ABC Grocers, a regional supermarket chain. The ABC stock has a per share basis to Macaw of $50 and is currently worth $2,000. In what manner, if any, could these facts have a bearing on the value of the Macaw stock included in Al’s gross estate? 108. A well-known artist dies and among her assets is a large number of her unsold paintings. What approach might be taken to lower the value of her estate? 109. When will the price specified in a buy-sell agreement control the value used for estate tax purposes? 110. Robert and Manuel are married. Ten years ago, they purchased stock for $500,000. Manuel dies before Robert, when the property is worth $1,000,000. When the property passes to Robert, what is his income tax basis under the following ownership assumptions?

a.

A joint tenancy is involved.

b.

Community property is involved.

c.

Why are the results different?

111. One of the objectives of establishing a program of lifetime giving is to transfer assets that are likely to appreciate in Powered by Cognero

Page 13


Name:

Class:

Date:

Chapter 19: Family Tax Planning value. Examples include life insurance policies, valuable collections (e.g., art works), vacation homes, and undeveloped real estate. However, such gifts could cause economic problems for certain donees. Explain why. 112. Giselle, a widow, has an extensive investment portfolio that has appreciated in value. Starting this year, she initiates a series of annual gifts of securities to her grandchildren who are attending college. Evaluate Giselle’s policy in terms of the resulting: a.

Federal gift and estate taxes.

b.

Federal income tax.

113. In terms of future Federal income and estate tax savings, there are numerous advantages to making lifetime transfers. In this connection, comment on each of the following. a.

Avoiding a transfer tax on future appreciation.

b.

Using gift tax annual exclusions.

c.

Payment of expenses prior to death.

114. In making a choice as to which assets to use in carrying out a program of lifetime giving, comment on each of the following assets. a.

Installment notes receivable.

b.

Property subject to recapture of depreciation.

115. In making a choice as to which assets to use in carrying out a program of lifetime giving, comment on the use of each of the following assets. a.

Securities that have declined in value.

b.

Land that has appreciated in value.

c.

Life insurance policies.

116. How can a disclaimer by an heir increase the charitable deduction allowed a decedent? Is such a disclaimer always wise? Explain. 117. In each of the following independent situations, describe the effect of the disclaimer procedure on Ray’s taxable estate, considering only the Federal tax consequences. In this regard, advise as to whether a disclaimer should be made, by whom, and for what amount. Ray died in 2024.

a.

Ray’s will leaves $15,000,000 to his adult son Tyler and the remainder ($2,000,000) to April (Ray’s surviving spouse).

Powered by Cognero

Page 14


Name:

Class:

Date:

Chapter 19: Family Tax Planning b.

Ray’s will leaves $14,000,000 to April (Ray’s surviving spouse) and the remainder ($3,000,000) to his adult son Tyler.

c.

Ray’s will leaves $14,000,000 to April (Ray’s surviving spouse) and the remainder ($500,000) to the local zoo, a qualified charity.

118. What are the advantages of the election to extend the payment of Federal estate taxes when the estate consists largely of an interest in a closely held business?

Powered by Cognero

Page 15


Name:

Class:

Date:

Chapter 19: Family Tax Planning Answer Key 1. True 2. True 3. False 4. False 5. True 6. True 7. False 8. False 9. True 10. False 11. False 12. True 13. True 14. False 15. False 16. True 17. True 18. True 19. True 20. False 21. False 22. True 23. False 24. True 25. False Powered by Cognero

Page 16


Name:

Class:

Date:

Chapter 19: Family Tax Planning 26. True 27. False 28. True 29. True 30. False 31. True 32. True 33. False 34. True 35. False 36. True 37. False 38. True 39. False 40. c 41. c 42. b 43. d 44. d 45. d 46. a 47. d 48. a 49. c 50. b Powered by Cognero

Page 17


Name:

Class:

Date:

Chapter 19: Family Tax Planning 51. a 52. b 53. c 54. c 55. d 56. b 57. c 58. b 59. d 60. a 61. c 62. b 63. b 64. c 65. a 66. c 67. f 68. k 69. g 70. e 71. c 72. b 73. a 74. j 75. h 76. i Powered by Cognero

Page 18


Name:

Class:

Date:

Chapter 19: Family Tax Planning 77. h 78. i 79. g 80. e 81. c 82. b 83. j 84. f 85. d 86. g 87. d 88. h 89. e 90. b 91. g 92. a 93. c 94. $134.17.

95. Situation A is $4,000,000 because the § 2032A reduction cannot go below special use value. Situation B is $1,610,000 ($3,000,000 – $1,390,000). 96. $2,748,000. Average profit ($3,200,000 ÷ 5 years) 8.5% (rate of return) × $2,400,000 (book value) Excess earnings over 8.5% Powered by Cognero

$ 640,000 (204,000) $ 436,000 Page 19


Name:

Class:

Date:

Chapter 19: Family Tax Planning Goodwill (5 × $436,000) Add book value Value of stock Latonya’s interest Value of stock in gross estate

$2,180,000 2,400,000 $4,580,000 × 60% $2,748,000

97. $1,100,000 basis of gain and $800,000 basis for loss. 98. a. $2,600,000 (1/2 of $5,200,000) for each. b.

$2,650,000 (1/2 of $5,300,000) for each.

99. With 10 donees and two donors, $360,000 per year (10 × 2 × $18,000) can be given without exceeding the annual exclusion. Thus, a 50% undivided interest in the real estate can be transferred each year. If the two gifts occur within close proximity to each other (e.g., December and January), the entire value of the property can be transferred gift tax free. 100. a. Phil might consider disclaiming $1,390,000, which would bring his bequest down to $13,610,000 (the bypass amount for the year). The extra $1,390,000 that goes to Brittany then is allowable as a deduction and reduces the family's aggregate estate tax. b.

Brittany should disclaim $8,610,000 of her bequest. As a result, this passes $13,610,000 to Paula, which is the bypass amount for the year, and no increase in the estate tax liability is incurred, while passing additional assets to a younger beneficiary.

101. Unless the parties can prove otherwise, the notes are to be included in the holder’s gross estate at their face amount. The fact that the decedent forgives the note in his will has no effect on valuation. To establish a lower valuation, Rashan’s executor might show a lower-than-market (or zero) interest rate and a distant maturity date. To show that the notes are worthless (and, therefore, have no gross estate value) or partially worthless, look at Tammy’s financial condition and the absence (or questionable value) of any assets pledged or collateral. 102. Probably to help pay for Madison’s college education and then give a measure of security for Emma during her lifetime. 103. a. Keith has made two gifts: a life estate to Melinda and a remainder interest to Dana. b.

The life estate probably qualifies for the annual exclusion (as a present interest), while the remainder interest does not (future interest limitation).

Powered by Cognero

Page 20


Name:

Class:

Date:

Chapter 19: Family Tax Planning 104. The amount recaptured is the estate tax saved (by the estate of the decedent) from using special use valuation (as opposed to best or most suitable use). Full recapture occurs unless 10 years has passed since the owner’s death. The recapture is imposed on the qualified heir. No penalties or interest are imposed then, though.

105. a. Goodwill should be based on an average of operating profits. Thus, nonoperating gains and losses should not be taken into account. b.

Operating profits should be based on what should have been the salary expense - not what was paid. As a result, reported profits have been overstated.

c.

Operating profits should reflect the market rate of interest that would be charged by third party lenders. To this extent, therefore, past profits are overstated.

106. If the interest held is small, particularly as to voting power, the inability to control corporate policy could warrant a discount for a minority interest. On the other hand, if the interest is large enough to carry control, its value is increased. If too large, however, the blockage rule could apply. This lowers value based on the premise that disposing of a large amount depresses the price for a particular item. 107. Before the estate can enjoy the value of the stock in ABC Grocers, Macaw must be liquidated. Because there is considerable gain to be recognized in the liquidation (i.e., low basis/high value assets), a discount for the income tax that would result could be allowed. 108. A blockage discount might be warranted if the estate is composed of a large number of art works, the bulk sale of which would depress value. This was the case with David Smith (sculptures) and Georgia O’Keeffe (paintings). 109. The price set forth controls when the following conditions are satisfied. ∙

All of the terms of the agreement are executed by the parties as written. The price is the result of a bona fide business agreement.

The agreement is not a device to transfer property to family members at less than fair market value.

The agreement is comparable to other agreements entered into by persons dealing at arm’s length.

110. a. $750,000 [$250,000 (Robert’s cost basis) + $500,000 (amount included in Manuel’s gross estate)]. b.

$1,000,000 [$500,000 (Robert’s share) + $500,000 (Manuel’s share)].

Powered by Cognero

Page 21


Name:

Class:

Date:

Chapter 19: Family Tax Planning c.

Unlike a joint tenancy (or tenancy by the entirety), the basis of a surviving spouse’s share of community property will conform to that of the deceased spouse’s share.

111. These “dry assets” require maintenance and upkeep. Thus, premiums may have to be paid on insurance policies to keep them from lapsing. Collections require maintenance and upkeep costs plus provision for security. Real estate is subject to state and local ad valorem property taxes. Consequently, a donor must ensure that donees of such non-incomeproducing gifts have the financial resources to properly preserve the property. 112. a. Although the number of annual exclusions is being maximized, gift taxes or some of the amount could be avoided by making direct transfers to the educational institution—see text section 18-2a. However, this provision covers only tuition costs and would require Giselle to use cash (rather than securities). By making these gifts, Giselle is reducing her wealth, thereby decreasing future estate taxes. b.

Giselle’s major objective probably is to shift income tax consequences to lower bracket donees. If the grandchildren have low (or no) taxable income, capital gains are taxed at a 0% rate (rather than Giselle’s 15% or 20% presumed rate).

113. a. By gifting property that will appreciate in value, a transfer tax is saved on any such increase in value that does materialize. In this regard, assets such as life insurance policies, works of art, and real estate are ideal gift properties. b.

The annual exclusions used on lifetime gifts escape all transfer taxes—both gift and estate. Although the amount allowed (currently $18,000 per donee per year) does not seem material, it can easily become substantial.

c.

Payment of some expenses prior to death may generate income tax benefits and save on estate taxes. Major examples of such expenses are medical and charitable items. The estate tax saving comes about because the funds used to pay the expenses no longer are included in the gross estate.

114. a. A gift of installment notes receivable is treated as a taxable disposition of the notes. The donor recognizes gain measured by the difference between the fair market value and the basis of the notes. b.

A gift of depreciable property does not trigger recapture to the donor. Instead, the recapture potential carries over to the donee.

Powered by Cognero

Page 22


Name:

Class:

Date:

Chapter 19: Family Tax Planning 115. a. Investments that have declined in value should be sold rather than given as gifts. Thus, any loss can be realized and recognized. If the property is given, the potential loss disappears. The donee’s basis for loss is the fair market value of the property on the date of the gift. However, the basis for gain carries over. If the property is retained until death, it receives a step-down in income tax basis for both gain and loss purposes. b.

Land that has appreciated in value should be held until death to obtain the step-up in basis under § 1014.

c.

Transfer taxes are saved by making gifts of life insurance policies. The value of an unmatured policy generally is considerably less than its maturity value.

116. A person can structure a will so that property passes to a qualified charity if the primary beneficiary refuses to accept it. Often, it is preferable for the beneficiary to accept the property and then donate it to charity. This causes the loss of the § 2055 estate tax deduction but allows the beneficiary a § 170 income tax deduction. If the disclaimer merely substitutes one type of estate tax deduction for another (i.e., marital for charitable), then it need not be made. 117. a. Tyler should disclaim $1,390,000. Since the disclaimed amount goes to April, it increases the marital deduction. The son’s $13,610,000 bequest is fully offset by the bypass amount, and the tax on the $1,390,000 transfer to Tyler is eliminated. b.

c.

April should disclaim $10,610,000, with the bypass amount of $13,610,000 then going to the son. No Federal estate tax is incurred, and the cash is transferred to a younger beneficiary. A disclaimer by April merely would substitute a charitable deduction for a marital deduction, so a disclaimer need not be made.

118. No estate taxes on the business assets are paid for the first five years (Only interest is paid for the first five years). After that, the payments can be stretched over the next 10 years. Furthermore, the interest charge on a large portion of the deferred taxes (up to $1,850,000 for deaths in 2024) is an attractive 2%. Thus, § 6166 enables an estate to pay its taxes over an extended period of time, presumably out of post death profits, and this deferral may avoid a forced sale of the business.

Powered by Cognero

Page 23


Name:

Class:

Date:

Chapter 20: Income Taxation of Trusts and Estates True / False 1. Tax planning motivations usually predominate over other objectives in deciding whether to create a trust. a. True b. False 2. A trust might be used when a married couple is divorcing. a. True b. False 3. Like a corporation, the fiduciary reports and pays its own Federal income tax liability. a. True b. False 4. An estate’s remainder beneficiary generally must wait until the entity is terminated by the executor to receive any distributions. a. True b. False 5. With respect to a trust, the terms creator, donor, and grantor are synonyms. a. True b. False 6. Corpus, principal, and assets of the trust are synonyms. a. True b. False 7. If provided for in the controlling agreement, a trust might terminate when the income beneficiary reaches age 35. a. True b. False 8. Under IRS regulations, the decedent’s estate must terminate within four years of the date of death to minimize incomeshifting techniques. a. True b. False 9. Trusts can select any Federal income tax year-end. a. True b. False 10. A complex trust pays tax on the income that it accumulates (i.e., that it does not distribute). a. True b. False 11. The first step in computing an estate’s taxable income is the determination of its fiduciary accounting income for the year. Powered by Cognero

Page 1


Name:

Class:

Date:

Chapter 20: Income Taxation of Trusts and Estates a. True b. False 12. Generally, capital gains are allocated to fiduciary income, because they arise from current-year transactions as directed by the trustee. a. True b. False 13. Usually, a beneficiary takes a carryover basis when a trust distributes a noncash asset. a. True b. False 14. A decedent’s income in respect of a decedent is subject to the Federal income tax, but it is excluded from the estate tax. a. True b. False 15. An example of income in respect of a decedent is the taxpayer’s last paycheck uncollected at death. a. True b. False 16. When a trust operates a trade or business, it can claim a deduction for wages paid to employees. a. True b. False 17. Estates and trusts can claim Federal income tax deductions for costs incurred in maintaining investments in U.S. state and local bonds. a. True b. False 18. The Stratford Estate incurs a $25,000 casualty loss in disposing of the decedent's real property. The deduction is claimed against the Federal estate tax unless by election it is claimed on the estate’s income tax return. a. True b. False 19. The Stratford Estate incurs a $25,000 legal fee in disposing of the decedent's real property. The executor can decide to claim a $5,000 deduction against the Federal estate tax and a $20,000 deduction on the estate’s income tax return. a. True b. False 20. Judy can claim one-third of the Sweet Estate’s cost recovery deductions, because she received one-third of the fiduciary’s distributable net income (DNI). a. True b. False 21. The Malik Estate operates a manufacturing business. Malik made no income distributions this year. It can claim Powered by Cognero

Page 2


Name:

Class:

Date:

Chapter 20: Income Taxation of Trusts and Estates the full deduction for the MACRS depreciation of the assets of the business. a. True b. False 22. The Whitmer Trust operates a manufacturing business and distributes the profits to its income beneficiaries. Whitmer also passes through to the income beneficiaries the data needed to compute their deduction for qualified business income. a. True b. False 23. The Whitmer Trust operates a manufacturing business. When Whitmer incurs a net operating loss, the current-year deduction passes through to the income beneficiaries. a. True b. False 24. The Griffin Trust makes a gift of long-term capital gain property to a qualifying charity. Griffin’s entity-level deduction cannot exceed 30% of distributable net income. a. True b. False 25. The Julius Trust made a gift to the United Charity on August 1, year 2, from its year 1 business profits. The trust’s charitable contribution deduction can be claimed in year 2. a. True b. False 26. Of the income received by the Atom Trust this year, 60% constituted municipal bond interest. Atom’s trustee also made a $100,000 gift to the United Fund, a qualifying charity. The charitable deduction associated with this gift is limited to $60,000. a. True b. False 27. A fiduciary’s distribution deduction shifts the tax burden for the distributed amount of current-year income from the entity to the beneficiary. a. True b. False 28. Harry, the sole income beneficiary, received a $40,000 distribution from the Lucy Trust in a year when the trust’s distributable net income was $30,000. Harry’s AGI increases by $40,000. a. True b. False 29. Harry, the sole income beneficiary, received a $40,000 distribution from the Lucy Trust in a year when the trust’s distributable net income was $50,000. Harry’s AGI increases by $40,000. a. True b. False 30. Harry, the sole income beneficiary, received a $40,000 distribution from the Lucy Trust in a year when the trust’s Powered by Cognero

Page 3


Name:

Class:

Date:

Chapter 20: Income Taxation of Trusts and Estates distributable net income was $50,000. Harry’s AGI can increase by as much as $50,000. a. True b. False 31. One-fourth of the Cruger Estate’s distributable net income consists of net long-term capital gains. Thus, when income beneficiary Susie receives a $40,000 income distribution from the estate, $10,000 of it qualifies for the beneficial Federal income tax rate on long-term capital gains. a. True b. False 32. In computing distributable net income (DNI) for a trust, one removes any net capital gain or loss that is allocable to income. a. True b. False 33. The Gable Trust reports $20,000 business income and $10,000 exempt interest income, and it paid a $3,000 fiduciary fee. Gable’s distributable net income (DNI) includes $9,000 for the interest income. a. True b. False 34. The Crown Trust distributed one-half of its accounting income to Lee this year. Lee also is allocated one-half of Crown’s credit for building low-income housing. a. True b. False 35. The Gable Trust reports $20,000 business income and $10,000 exempt interest income, and it paid a $3,000 fiduciary fee. Gable’s distributable net income includes $10,000 of net tax-exempt interest income. a. True b. False 36. First-tier distributions allowed by the will or trust document are made at the discretion of the executor or trustee. a. True b. False 37. When DNI includes exempt interest income, the beneficiary includes less than the full amount of DNI in current-year gross income. a. True b. False 38. Winston is classified as a grantor trust because Hernan, the donor, can revoke the trust. Consequently, Winston need not file an annual Form 1041, and Hernan reports the trust items on his own Form 1040. a. True b. False 39. Reyes contributed assets to a trust and designated daughter Maria as the income beneficiary and grandson Julio as the remainder beneficiary. This year, fiduciary accounting income was $50,000. The trustee paid $5,000 of this amount as premiums for a life insurance policy on Anita, Reyes’s wife. Reyes pays Federal income tax on $5,000 for the year. Powered by Cognero

Page 4


Name:

Class:

Date:

Chapter 20: Income Taxation of Trusts and Estates a. True b. False 40. Reyes contributed assets to a trust and designated daughter Maria as the income beneficiary and grandson Julio as the remainder beneficiary. This year, fiduciary accounting income was $50,000. The trustee paid $5,000 of this amount as Julio’s high school tuition. Reyes pays Federal income tax on $5,000 for the year. a. True b. False 41. Reyes contributed assets to a trust and designated daughter Maria as the income beneficiary and grandson Julio as the remainder beneficiary. This year, fiduciary accounting income was $50,000. The trustee accumulated $5,000 of this amount and added it to trust corpus. Reyes pays Federal income tax on $5,000 for the year. a. True b. False 42. The unextended due date for a calendar year trust to file its Form 1041 is March 15. a. True b. False 43. Tax planning usually dictates that high-income and high-wealth individuals be specified as second-tier beneficiaries of a trust arrangement. a. True b. False 44. When a trust distributes an in-kind asset with a realized loss, this loss most likely cannot be immediately deducted by the first-tier beneficiary. a. True b. False 45. The Philmore Estate is subject to a 40% Federal estate tax rate and a 25% Federal and state income tax rate. Generally, an administrative expense should be claimed on the estate tax return. a. True b. False 46. Generally, an administrative expense attributable to municipal bond interest should be claimed on the estate’s Form 706. a. True b. False Multiple Choice 47. The tax rules regarding the income taxation of trusts and estates are included in which Subchapter of the Internal Revenue Code? a. C b. J c. K Powered by Cognero

Page 5


Name:

Class:

Date:

Chapter 20: Income Taxation of Trusts and Estates d. S 48. Which of the following is a typical duty of an executor of an estate? a. Pay funeral expenses. b. Pay off the decedent’s financial liabilities. c. Distribute the net assets of the probate estate. d. Manage the decedent’s assets until they are liquidated or distributed. e. All of these choices are correct. 49. Which of the following is a typical duty of a trustee? a. Modify the language of the trust instrument so as to lower the entity’s Federal income tax. b. Make decisions as to how to invest the trust corpus portfolio. c. Allocate items between income and corpus using Subchapter J rules. d. All of these choices are correct. 50. Which of the following is a typical duty of a trustee? a. File the entity’s state and Federal income tax returns. b. Invest the assets that comprise the corpus of the entity. c. Distribute entity accounting income to the beneficiaries in accordance with the provisions of the trust instrument. d. All of these choices are correct. 51. The Code defines a simple trust as which of the following? a. One that is allowed to file Form 1041-EZ. b. One that has only one income beneficiary. c. One whose grantor was an individual who still is alive. d. One that must distribute its accounting income every year. 52. Which of the following taxpayers can be subject to an entity-level Federal income tax? a. Complex trust. b. Partnership. c. Limited liability company. d. All of these taxpayers are pass-through entities, and they never are subject to an entity-level Federal income tax. 53. Which of the following taxpayers use a Schedule K and K-1 to pass through income, loss, and credit amounts to the owners or beneficiaries? a. Complex trust. b. Partnership. c. S corporation. d. All of these choices are correct. 54. The Chen Trust is required to distribute its accounting income every year, one-half to Missy Chen, and one-half to the local church’s homeless shelter. What is the Chen Trust’s personal exemption? a. $0 Powered by Cognero

Page 6


Name:

Class:

Date:

Chapter 20: Income Taxation of Trusts and Estates b. $100 c. $300 d. $600 55. The Prakash Estate has equal income beneficiaries Sam and Janet. As allowed by the terms of the will, the estate makes no income distributions during the current tax year. The estate’s personal exemption is: a. $0. b. $100. c. $300. d. $600. 56. The Prakash Trust is required to pay its entire annual accounting income to beneficiaries Sam and Janet. The trust’s personal exemption is: a. $0. b. $100. c. $300. d. $600. 57. The Prakash Trust is required to pay its entire annual accounting income to the Daytona Museum, a qualifying charity. The trust’s personal exemption is: a. $0. b. $100. c. $300. d. $600. 58. The trustee of the Epsilon Trust distributed an asset to Telly, a qualifying income beneficiary. The asset’s basis to the trust was $10,000, and its fair market value on the distribution date was $25,000. Which of the following statements is true? a. Assuming that the trustee made an election under § 643(e), the trust is allowed a $10,000 distribution deduction for this transaction. b. Assuming that the trustee made an election under § 643(e), Telly recognizes $10,000 gross income on the distribution. c. Lacking any election by the trustee, the trust recognizes $15,000 gross income on the distribution. d. Lacking any election by the trustee, Telly’s basis in the asset is $10,000. 59. Three months after Brianna Timkin died, her executor received the final $40,000 installment from a sale of land that Brianna completed several years ago. Which of the following statements is true? a. The $40,000 is both included in Brianna’s gross estate and subject to tax on her estate’s income tax return. b. The $40,000 is subject to neither income nor estate tax, because it was received after Brianna’s death. c. The $40,000 is only subject to tax on her estate’s income tax return. d. The $40,000 is included only in Brianna’s gross estate. 60. Income beneficiary Turk received $30,000 from the Urgent Trust. Trust accounting income for the year was $100,000. The trust generated $20,000 in cost recovery deductions. How much can Turk deduct with respect to the cost recovery deductions that Urgent generated? a. $0 Powered by Cognero

Page 7


Name:

Class:

Date:

Chapter 20: Income Taxation of Trusts and Estates b. $6,000 c. $14,000 d. $20,000 61. Three weeks after Abed died, his brother Tony properly received Abed’s last paycheck from his employer. The gross amount of the check was $4,000, and a $300 deduction for state income taxes was subtracted in computing the net amount of the payment. Which of the following statements about the $300 is true? a. It is deductible on neither Tony’s income tax return nor on Abed’s estate tax return. b. It is deductible on both Tony’s income tax return and on Abed’s estate tax return. c. It is deductible only in computing Abed’s taxable estate. d. It is deductible only on the income tax return of Abed’s estate. 62. The Chen Trust incurred the following items during the year. What is Chen’s deduction for the tax preparation fees? Taxable interest received Tax-exempt interest received Tax preparation fees paid

$40,000 60,000 5,000

a. $0 b. $2,000 c. $3,000 d. $5,000 63. Which of the following statements, if any, relates to the tax treatment of both estates and trusts? a. The termination date of the entity is specified in the controlling document. b. The entity must use the same tax year as its creator (i.e., grantor, decedent). c. The entity is required to distribute all of its income currently to its beneficiaries. d. The entity can have both income and remainder beneficiaries. 64. Ellie, a calendar year individual, wants to make a gift to a charity that is deductible on her year 1 Form 1040. The gift must be made by Ellie: a. During year 1. b. During years 1 and 2. c. On or before April 15, year 2. d. On or before September 30, year 2. 65. The distributable net income (DNI) of a fiduciary taxpayer: a. Constitutes the maximum amount for the fiduciary’s distribution deduction. b. Specifies the character of the distributions in the hands of the year’s income beneficiaries. c. Marks the maximum amount of gross income that income beneficiaries must report when receiving distributions. d. All of these choices are correct. 66. The Ellie Trust, a calendar year entity, wants to make a gift to a charity that is deductible on its year 1 Form 1041. The gift must be made by Ellie: Powered by Cognero

Page 8


Name:

Class:

Date:

Chapter 20: Income Taxation of Trusts and Estates a. During year 1. b. During year 1 or 2. c. On or before April 15, year 2. d. On or before September 30, year 2. 67. Ellie Inc., a calendar year C corporation, wants to make a gift to a charity that is deductible on its year 1 Form 1120. The gift must be made by Ellie: a. During year 1. b. During year 1 or 2. c. On or before April 15, year 2. d. On or before September 30, year 2. 68. Which of the following is the annual maximum amount to be included as gross income by all of the income beneficiaries of the trust or estate? a. Entity taxable income. b. Entity adjusted gross income. c. Distributable net income. d. Fiduciary accounting income. 69. This year, the Nano Trust reported $50,000 entity accounting income and $40,000 distributable net income (DNI). Nano distributed $30,000 cash to Horatio, its sole income beneficiary. Nano is a complex trust. Nano’s distribution deduction is: a. $50,000. b. $40,000. c. $30,000. d. $0. Because the distributions of a complex trust are discretionary, no deduction is allowed. 70. This year, the Nano Trust reported $50,000 entity accounting income and $40,000 distributable net income (DNI). Nano distributed $60,000 cash to Horatio, its sole income beneficiary. Nano is a simple trust. Nano’s distribution deduction is: a. $60,000. b. $50,000. c. $40,000. d. $0. 71. This year, the Huang Trust is a complex trust. This year, it distributed all of its accounting income and $5,000 from corpus to its sole income beneficiary Kun. Huang’s taxable income for the year is: a. $0. b. ($100). c. ($300). d. ($5,000). 72. The Doyle Trust reports distributable net income of $100,000 for the year and no income from tax-exempt sources. Under the terms of the trust instrument, the trustee must distribute $30,000 to Roger and $30,000 to Sally. After payment of these amounts, the trustee can make additional distributions at its discretion. Exercising this authority, the trustee distributes an additional $25,000 to Roger and $25,000 to Sally. How much income from the trust must Sally recognize? Powered by Cognero

Page 9


Name:

Class:

Date:

Chapter 20: Income Taxation of Trusts and Estates a. $25,000. b. $30,000. c. $50,000. d. $55,000. 73. The Doyle Trust reports distributable net income of $100,000 for the year and no income from tax-exempt sources. Under the terms of the trust instrument, the trustee must distribute $20,000 to Roger and $20,000 to Sally. After paying these amounts, the trustee can make additional distributions at its discretion. Exercising this authority, the trustee distributes an additional $25,000 to Roger and $50,000 to Sally. How much gross income from the trust must Sally recognize? a. $70,000 b. $60,000 c. $40,000 d. $20,000 74. The Doyle Trust reports distributable net income of $100,000 for the year and no income from tax-exempt sources. Under the terms of the trust instrument, the trustee must distribute $20,000 to Roger and $20,000 to Sally. After paying these amounts, the trustee can make additional distributions at its discretion. Exercising this authority, the trustee distributes an additional $25,000 to Roger and $50,000 to Sally. How much gross income from the trust must Roger recognize? a. $50,000 b. $45,000 c. $40,000 d. $20,000 75. The Uldis Trust reports distributable net income of $100,000 for the year and no income from tax-exempt sources. Under the terms of the trust instrument, the trustee must distribute $75,000 to Roger and $75,000 to Sally. After paying these amounts, the trustee can make additional distributions at its discretion. Exercising this authority, the Uldis trustee distributes an additional $10,000 to Roger and $30,000 to Sally. How much gross income from the trust must Sally recognize? a. $30,000 b. $50,000 c. $75,000 d. $100,000 76. For the year, the Uldis Trust has distributable net income (DNI) of $100,000 and no income from tax-exempt sources. Under the terms of the trust instrument, the trustee must distribute $75,000 to Roger and $75,000 to Sally. After paying these amounts, the trustee can make additional distributions at its discretion. Exercising this authority, the Uldis trustee distributes an additional $10,000 to Roger and $30,000 to Sally. How much gross income from the trust must Roger recognize? a. $10,000 b. $50,000 c. $75,000 d. $85,000 77. During the current year, the Santo Trust received $30,000 of taxable interest income, paid trustee’s commissions of $3,000, and had no other income or expenses. The Santo trust instrument requires that $20,000 be paid annually to Powered by Cognero

Page 10


Name:

Class:

Date:

Chapter 20: Income Taxation of Trusts and Estates Marilyn and $40,000 be paid annually to Domingo. How much gross income must Marilyn and Domingo recognize? a. $20,000 by Marilyn and $40,000 by Domingo. b. $15,000 by Marilyn and $15,000 by Domingo. c. $13,500 by Marilyn and $13,500 by Domingo. d. $9,000 by Marilyn and $18,000 by Domingo. 78. For the year, the Brighton Trust has distributable net income of $100,000 and no income from tax-exempt sources. Under the terms of the trust instrument, the trustee is required to distribute $25,000 to Roger and $50,000 to Sally. After payment of these amounts, the trustee can make additional distributions at its discretion. Exercising this authority, the Brighton trustee distributes an additional $20,000 to Roger and $30,000 to Sally. How much income from the trust must Sally recognize? a. $80,000 b. $65,000 c. $50,000 d. $30,000 79. This year, the Zhao Estate generated distributable net income (DNI) of $100,000, one-fourth of which was tax-exempt interest, and the balance of which was long-term capital gain. Kyle Zhao, the sole income beneficiary of the estate, received a distribution of the entire $125,000 accounting income of the entity. How does Kyle report the distribution? a. $75,000 long-term capital gain, $25,000 exempt interest. b. $50,000 long-term capital gain, $50,000 exempt interest. c. $75,000 long-term capital gain, $25,000 ordinary income. d. $93,750 long-term capital gain, $31,250 exempt interest. 80. This year, the Suarez Trust generated distributable net income (DNI) of $150,000, two-thirds of which was portfolio income, and the balance of which was exempt interest. Under the terms of the trust, Clara Suarez is to receive an annual income distribution of $30,000. At the discretion of the trustee, additional distributions can be made to Clara or to Clark Suarez III. This year, the trustee’s distributions to Clara totaled $60,000. Clark received $90,000. How much of the trust’s DNI is assigned to Clara? a. $75,000 b. $60,000 c. $45,000 d. $30,000 81. This year, the Suarez Trust generated distributable net income (DNI) of $150,000, two-thirds of which was portfolio income, and the balance of which was exempt interest. Under the terms of the trust, Clara Suarez is to receive an annual income distribution of $30,000. At the discretion of the trustee, additional distributions can be made to Clara, or to Clark Suarez III. This year, the trustee’s distributions to Clara totaled $60,000. Clark received $90,000. How much of the trust’s DNI is assigned to Clark? a. $0, only first-tier distributions are subject to Federal income tax. b. $60,000 c. $75,000 d. $90,000 82. The Willis Trust instrument provides that Tamara, the sole income beneficiary, is to receive $40,000 annually. If trust accounting income is not sufficient to pay this amount, the Willis trustee is empowered to invade corpus to the extent necessary. During the current year, the trust reports distributable net income (DNI) of $100,000, including $30,000 of net Powered by Cognero

Page 11


Name:

Class:

Date:

Chapter 20: Income Taxation of Trusts and Estates tax-exempt interest. In accordance with the trust instrument, $40,000 is paid to Tamara. What is Tamara’s gross income from the Willis Trust for the current year? a. $100,000 b. $70,000 c. $40,000 d. $28,000 83. Which of the following restrictions applies concerning distributions to trust beneficiaries? a. Special allocations are not allowed under Subchapter J. b. Special allocations are allowed but only in the trust’s termination year. c. Special allocations are allowed but only for portfolio income items. d. Special allocations of income types are allowed assuming that they carry a substantial economic effect. 84. The Jiang Trust operates a manufacturing business. Yan created the trust several years ago by contributing a successful sole proprietorship. The trust distributed one-half of its DNI this year to Yan. With proper trust agreement language in place, Yan can retain all of the trust’s depreciation deductions if: a. Yan is in a higher income tax bracket than is the trust. b. Jiang is in a higher income tax bracket than is Yan. c. The depreciable assets were contributed by Yan when the trust was created. d. One-half of the deductions belongs to the trust. No special allocations are allowed by Subchapter J. 85. The Jiang Trust manages investment assets that were contributed by Kong several years ago. The trust distributed onehalf of its DNI this year to Kong. With proper trust agreement language in place, Kong can retain all of the trust’s taxexempt interest income if: a. One-half of the exempt income belongs to the trust. No special allocations are allowed by Subchapter J. b. Jiang is in a higher income tax bracket than Kong. c. Kong is in a higher income tax bracket than the trust. d. The tax-exempt bonds were contributed by Kong when the trust was created. 86. The Code defines a grantor trust as which of the following? a. One that is required to file Form 1041-G. b. One that the grantor can revoke or otherwise amend. c. One in which a member of the grantor’s family is the sole trustee. d. One that makes annual payments to designated charities. 87. The Yellow Trust incurred $10,000 of portfolio income. Its corporate trustee paid fiduciary fees of $1,000 therefrom, and also paid $1,000 in premiums for a life insurance policy on Marcia, the grantor of the trust. How much gross income does Marcia include with respect to these trust activities? a. $10,000 b. $9,000 c. $1,000 d. $800 88. The Zhong Trust is a calendar year taxpayer. Its Form 1041 is due on which date in the following year? a. March 15. Powered by Cognero

Page 12


Name:

Class:

Date:

Chapter 20: Income Taxation of Trusts and Estates b. April 15. c. June 15. d. September 15 if extended. 89. The present value of a trust’s Federal income tax liability is reduced when: a. The entity is classified as a simple trust. b. The trust invests in high-yield stocks. c. The trust invests in growth stocks. d. The trust collects income in respect of a decedent. 90. A trust holds assets that will generate net investment income (NII) and trigger the additional Medicare tax. To reduce the entity's NII tax, the trustee should: a. Distribute all of the NII to trust beneficiaries. b. Retain all of the NII every tax year. c. Increase the turnover rate of the investments that the entity holds. d. Sell the NII assets and invest in high-yield bonds instead. 91. The Cai Trust is a calendar year taxpayer. Its Form 1041 is due on which date in the following year? a. April 15. b. June 15. c. September 15 if extended. d. December 15 if extended. 92. The Yellow Trust incurred $10,000 of portfolio income. Its corporate trustee paid fiduciary fees of $1,000 therefrom. Yellow’s accounting income is distributed as follows: ∙ $5,000 to income beneficiary Larry. ∙ $4,000 to pay part of the high school tuition bills for Carrie, the daughter of Yellow’s grantor Marcia. How much gross income does Marcia include with respect to these trust activities? a. $0 b. $4,000 c. $9,000 d. $10,000 93. Jose is subject to the top marginal Federal income tax rates. Carlita is considering establishing a trust in which Jose would be an income beneficiary. Considering only income tax consequences, Jose should be designated as: a. A first-tier beneficiary. b. A second-tier beneficiary. c. Only a remainder beneficiary. d. Both a first- and a second-tier beneficiary. 94. The Drabb Trust owns a plot of business-related land, basis of $50,000, fair market value of $35,000. Drabb is subject to a 35% marginal income tax rate. Its sole beneficiary, Eddie, is subject to a 12% marginal income tax rate. Drabb’s current-year distributable net income is $95,000. What is the most preferable action for Drabb's trustee to take considering only the related Federal income tax consequences? Powered by Cognero

Page 13


Name:

Class:

Date:

Chapter 20: Income Taxation of Trusts and Estates a. Distribute the land to Eddie and make a § 643(e) election. b. Distribute the land to Eddie and make no § 643(e) election. c. Sell the land to a third party. d. Neither sell nor distribute the land. Completion 95. The grantor of a trust generally designates both ____________________ and ____________________ beneficiaries under the controlling agreement. 96. In computing the Federal taxable income of a trust, the ____________________ (first, last) step is to determine its fiduciary accounting income. 97. In computing the Federal taxable income of a trust, a modified ____________________ approach is used. 98. Under the Federal income tax rules for trusts and estates, a(n) ____________________ generally must use a calendar tax year, but a(n) ____________________ can select any tax year-end. 99. Every simple trust is allowed a $____________________ personal exemption. 100. The trust usually makes a distribution to the income beneficiaries in the amount of its fiduciary ____________________ income. 101. Generally, an estate’s taxable income is computed in a manner similar to that used for a(n) ____________________. 102. Entity accounting income is controlled by the terms of the ____________________ for an estate or the ____________________ for a trust. 103. The trust instrument indicates whether cost recovery is ____________________ to fiduciary accounting income, thereby reducing the amount of the distribution to the income beneficiary. 104. The interest income of a trust usually is allocable to ____________________ (income, remainder) beneficiaries. 105. The rental income of a trust usually is allocable to ____________________ (income, remainder) beneficiaries. 106. When a beneficiary receives a distribution of an asset other than cash from a trust, generally a(n) ____________________ basis is assigned to the asset. 107. When a beneficiary receives a distribution from a trust of an asset other than cash, the realized loss generally is ____________________ (recognized, not recognized) by the trust. 108. A fiduciary’s cost recovery deductions are assigned corresponding to the disposition of entity ____________________ income for the year. 109. A gift to charity from its 2025 income is deductible on an estate’s Form 1041 if it is made by the end of the ____________________ (2025, 2026) tax year. 110. The deduction for the Sharma Trust’s $100,000 gift to charity is ____________________ when one-third of Sharma’s accounting income for the tax year constitutes exempt interest income. Powered by Cognero

Page 14


Name:

Class:

Date:

Chapter 20: Income Taxation of Trusts and Estates 111. Distributable net income (DNI) is the ____________________ (maximum, minimum) amount that can be included in the beneficiaries’ gross incomes from the fiduciary for the year. 112. A fiduciary assigns its tax credits to beneficiaries corresponding to the disposition of its ____________________ income for the year. 113. A ____________________ (first-, second-, third-) tier distribution is one that the trust agreement requires to be made by the trustee to the income beneficiary. 114. Income beneficiary Molly wants to receive all of the municipal bond interest income of the Brenner Trust. A special allocation of this sort must be supported by a nontax ____________________. 115. Income is taxed to the creator of a(n) ____________________ trust instead of to the entity. 116. The entity is a ___________________ trust if the grantor retains the power to revoke the trust. 117. A Form 1041 must be filed by a trust that has $____________________or more gross income for the tax year. 118. The Form 1041 of a calendar year trust is due on ____________________ 15 (not including extensions) of the following year. 119. Beneficiary information concerning a trust’s income and distributions are conveyed on Schedule ____________________ of the Form 1041. 120. A fiduciary tax year end should be chosen so as to avoid the ________ of taxable income for the beneficiaries. Matching For each of the following independent statements, choose the best answer. a. Tax attribute of complex trusts only b. Tax attribute of estates only c. Tax attribute of estates and complex trusts d. Tax attribute of neither estates nor complex trusts 121. The entity has a legal identity separate from its beneficiaries. 122. The entity’s AMT preferences and adjustments pass through to the income beneficiaries. 123. The entity must file an income tax return if its gross income for the year is $600 or more. 124. The entity can choose between the cash and accrual methods of reporting its income and deductions. 125. The entity is entitled to a personal exemption of $600. 126. The entity is controlled by Federal-level probate laws. 127. Undistributed income is subject to the additional tax on net investment income. 128. The entity’s management generally is directed by the controlling document as to the date of the entity’s termination. Powered by Cognero

Page 15


Name:

Class:

Date:

Chapter 20: Income Taxation of Trusts and Estates 129. The entity is an information-reporting, not a tax-paying, taxpayer. 130. Distributable net income is used to account for the entity's distributions to its beneficiaries. 131. A decedent created the entity. 132. The entity was created by either a decedent or a living person. 133. For a calendar year entity, the Form 1041 has an unextended due date of April 15. 134. The entity typically can choose any fiscal tax year. For each of the following items, insert the best term or phrase. An answer choice may be used more than once, but only one choice is the best for each descriptive phrase. a. Complex b. Decedent c. Executor d. Grantor e. Administrator f. Reversionary g. Simple h. Sprinkling i. Trustee 135. An estate always is created upon the death of this party. 136. A trust whose income can be distributed to beneficiaries and in amounts at the trustee’s discretion. 137. A trust that is required to distribute annual accounting income. 138. The person who transfers assets to a trust. 139. A trust whose remainder beneficiary is its grantor. 140. The fiduciary in charge of a trust. 141. A synonym for executor. 142. The fiduciary in charge of an estate. 143. A trust that can accumulate rather than distribute its accounting income. 144. A trust whose income is taxed to the donor, not the beneficiaries. Subjective Short Answer 145. The Raja Trust operates a welding business. Its current-year cost recovery deductions properly amount to $75,000. Raja’s accounting income was $100,000 of which $40,000 was distributed to first-tier beneficiary Chuck, $25,000 was distributed to second-tier beneficiary Ruby, and $35,000 was accumulated by the trustee. Ruby also received a $25,000 Powered by Cognero

Page 16


Name:

Class:

Date:

Chapter 20: Income Taxation of Trusts and Estates discretionary corpus distribution. Raja’s DNI was $80,000. Identify the treatment of Raja’s cost recovery deductions. 146. Corelle is a simple trust that correctly uses the calendar year for tax purposes. Its income beneficiaries (Kathie, Lynn, Mark, and Norelle) are entitled to the trust’s annual accounting income in shares of one-fourth each. For the current calendar year, the trust incurs ordinary business income of $40,000, a long-term capital gain of $20,000 (allocable to corpus), and legal and accounting fees of $4,000 (allocable to corpus). Use the format of Exhibit 20.5 in the text to address the following items: a.

How much income is each beneficiary entitled to receive?

b.

What is the trust’s DNI?

c.

What is the trust’s taxable income?

d.

How much is taxed to each of the beneficiaries?

147. Corelle is a simple trust that correctly uses the calendar year for tax purposes. Its income beneficiaries (Kathie, Lynn, Mark, and Norelle) are entitled to the trust’s annual accounting income in shares of one-fourth each. For the current calendar year, the trust incurs ordinary business income of $40,000, a long-term capital gain of $20,000 (allocable to income), and a trustee commission expense of $4,000 (allocable to corpus). Use the format of Exhibit 20.5 in the text to address the following items: a.

How much income is each beneficiary entitled to receive?

b.

What is the trust’s DNI?

c.

What is the trust’s taxable income?

d.

How much is taxed to each of the beneficiaries?

148. The Gomez Trust is required to distribute $80,000 annually split equally between its two income beneficiaries, Lara and Byron. If trust income is not sufficient to pay these amounts, the trustee can invade corpus to the extent necessary. During the current year, the trust reports DNI of $60,000. Byron receives an additional $30,000 discretionary corpus distribution. a.

How much of the $40,000 distributed to Lara is included in her gross income?

b.

How much of the $70,000 distributed to Byron is included in his gross income?

c.

How much of these distributions are first-tier distributions or second-tier distributions?

149. The Willa estate reports $100,000 DNI composed of $50,000 dividends, $20,000 taxable interest, $10,000 passive income, and $20,000 tax-exempt interest. Willa’s two noncharitable income beneficiaries, Shanna and Tom, receive distributions of $75,000 each. How much of each class of income is deemed to have been distributed to Shanna? To Tom? Use the following template to structure your answer.

Powered by Cognero

Page 17


Name:

Class:

Date:

Chapter 20: Income Taxation of Trusts and Estates DNI, Income Type Beneficiary

Amount Received Dividends

Taxable Interest

Passive

Exempt Interest

Corpus, Nontaxable

Shanna Tom Totals in DNI

150. The trustee of the Miguel Trust can distribute any amount of accounting income and corpus to the trust’s income beneficiaries, Paula and George. This year, the trust incurred the following items. Taxable interest income Tax-exempt interest income Long-term capital gains—allocable to corpus Legal and accounting’s fees—allocable to corpus

$40,000 20,000 10,000 6,000

The trustee distributed $40,000 to Paula and $40,000 to George. a.

What is Miguel’s trust accounting income?

b.

What is Miguel’s DNI?

c.

What is Miguel’s taxable income?

d.

How much gross income is recognized by each of the beneficiaries?

Essay 151. List at least three nontax reasons that you might suggest to your client Chan that she should consider shifting some income and assets to a trust. 152. Identify the parties involved when an estate is created and their key duties. Then do the same for a trust. 153. Tax professionals use the terms simple trust and complex trust when dealing with fiduciary arrangements. How does one know whether a trust is simple or complex? When is this determination made? 154. Consider the term fiduciary accounting income as it is used with respect to the Federal income taxation of trusts and estates. How is this amount computed? Where is it used in computing the parties’ taxable incomes? 155. The Ming Trust distributes an asset to its income beneficiary that shows a realized gain. What are the Federal income tax consequences of the distribution? What if the asset shows a realized loss? 156. The Gibson Estate is responsible for collecting outstanding income amounts and paying the remaining obligations of Juanita Gibson, the deceased. How does Federal income tax law treat these items? Hint: Define and use the term income in respect of a decedent in your answer. 157. The Circle Trust reports some exempt interest income for the year. How does this investment income affect Circle’s Powered by Cognero

Page 18


Name:

Class:

Date:

Chapter 20: Income Taxation of Trusts and Estates deduction of its fiduciary fees? Charitable contributions? 158. The DaSinzi Estate has two equal income beneficiaries (Rollo and Luisa) and one remainder beneficiary (Coco). The estate operates a business and generates cost recovery deductions. Which taxpayer(s) (e.g., the deceased, the estate, Rollo, Luisa, or Coco) if any, can deduct these items? 159. Consider the term distributable net income as it is used with respect to the Federal income taxation of trusts and estates. How is this amount computed? How is it used in computing the parties’ taxable incomes? 160. Does the distributable net income of an estate or trust always equal its deduction for distributions to beneficiaries? Are the two amounts ever equal in amount? Answer for estates, simple trusts, and complex trusts. 161. The Yeoman Trust has generated several Federal income tax credits for this tax year. Which taxpayer(s), if any, can use these credits in computing a Federal income tax liability? (e.g., the grantor, the trust, its income beneficiaries, its remainder beneficiaries) Answer for estates, simple trusts, and complex trusts. 162. List and discuss the three major functions of distributable net income (DNI) as that amount is used under Federal income tax law. 163. Your client Pryce is one of the income beneficiaries of the Santiago Trust. Pryce says to you, “I want all of the exempt interest income from Santiago to be allocated to me, because I am the income beneficiary who is subject to the highest marginal Federal income tax rate.” How do you respond to Pryce’s request? 164. Olsen is a grantor trust. What are the tax consequences of this tax status? What are the tax return filing requirements for Olsen and for Peggy, the donor of the trust’s assets? 165. Marcus is a grantor trust. Your tax partner explains that this probably happened because the donor of the trust assets retained excessive powers over the operation of the trust or the use of its assets and income. To what powers is your tax partner referring? 166. You are responsible for the Federal income tax filings of the Tyrone Trust. Summarize the relevant due dates and filing requirements for Tyrone. 167. Your client Ming is a complex trust. Make a list of five or more tax planning opportunities that you might suggest to Ming.

Powered by Cognero

Page 19


Name:

Class:

Date:

Chapter 20: Income Taxation of Trusts and Estates Answer Key 1. False 2. True 3. True 4. True 5. True 6. True 7. True 8. False 9. False 10. True 11. True 12. False 13. True 14. False 15. True 16. True 17. False 18. True 19. True 20. False 21. True 22. True 23. False 24. False 25. True Powered by Cognero

Page 20


Name:

Class:

Date:

Chapter 20: Income Taxation of Trusts and Estates 26. False 27. True 28. False 29. True 30. False 31. True 32. False 33. True 34. True 35. False 36. False 37. True 38. False 39. True 40. True 41. False 42. False 43. True 44. True 45. True 46. True 47. b 48. e 49. b 50. d Powered by Cognero

Page 21


Name:

Class:

Date:

Chapter 20: Income Taxation of Trusts and Estates 51. d 52. a 53. d 54. c 55. d 56. c 57. c 58. d 59. a 60. b 61. b 62. b 63. d 64. a 65. d 66. b 67. c 68. c 69. c 70. c 71. b 72. c 73. b 74. c 75. b 76. b Powered by Cognero

Page 22


Name:

Class:

Date:

Chapter 20: Income Taxation of Trusts and Estates 77. d 78. b 79. a 80. b 81. d 82. d 83. d 84. c 85. d 86. b 87. c 88. b 89. c 90. a 91. a 92. b 93. b 94. c 95. income, remainder remainder, income 96. first 97. flowthrough pass-through 98. trust, estate 99. 300 100. accounting Powered by Cognero

Page 23


Name:

Class:

Date:

Chapter 20: Income Taxation of Trusts and Estates 101. individual 102. will, trust agreement 103. allocated 104. income 105. income 106. carryover 107. not recognized 108. accounting 109. 2026, applying the one-year grace period. 110. $66,667 111. maximum 112. fiduciary accounting 113. first1st 114. economic effect 115. grantor 116. grantor 117. 600 118. April 119. K-1 120. bunching 121. c 122. d 123. c 124. c 125. b Powered by Cognero

Page 24


Name:

Class:

Date:

Chapter 20: Income Taxation of Trusts and Estates 126. d 127. c 128. a 129. d 130. c 131. b 132. a 133. c 134. b 135. b 136. h 137. g 138. d 139. f 140. i 141. e 142. c 143. a 144. d 145. The Raja Trust’s cost recovery deductions follow the disposition of its current-year accounting income (not DNI or total distributions). Thus, Chuck can deduct $30,000 in this regard [$75,000 × ($40,000 ÷ $100,000)]. Similarly, Ruby can deduct $18,750 [$75,000 × ($25,000 ÷ $100,000)], and Raja Trust can deduct $26,250 [$75,000 × ($35,000 ÷ $100,000)]. Irrelevant to this computation are (1) the first-tier or second-tier status of the beneficiaries’ distributions, (2) Ruby’s corpus distribution, and (3) the trust’s allocation of cost recovery to income or to corpus. 146. a. $10,000 (1/4 of $40,000 accounting income). b. $36,000. c. $19,700. Powered by Cognero

Page 25


Name:

Class:

Date:

Chapter 20: Income Taxation of Trusts and Estates d. $9,000 (1/4 of $36,000).

Item

Totals

Ordinary income Net long-term capital gain Fiduciary fees Personal exemption

$40,000 20,000 4,000 (300)

Accounting Income/Taxable Income Before the Distributions Deduction

Accounting Income $40,000

Distributable Net Income/ Taxable Income Distribution Deduction $40,000 20,000 (4,000) (300)

$40,000

$55,700

STEP 1

STEP 2

$55,700

300

Exemption Corpus Capital Gain/Loss Net Exempt Income Distributable Net Income

(20,000) $36,000 (36,000)*

Distribution Deduction

STEP 3 $19,700**

Entity Taxable Income

STEP 4

*Lesser of DNI ($36,000) or actual distribution ($40,000). **PROOF: The trust is taxed on “its” $20,000 capital gain less the personal exemption of $300. 147. a. $15,000 (1/4 of $60,000 accounting income). b.

$56,000.

c.

($300).

d.

$14,000 (1/4 of $56,000 DNI). Distributable Net

Item

Totals

Ordinary income Net long-term capital gain Fiduciary fees Personal exemption

$40,000 20,000 4,000 (300)

Accounting Income/Taxable Income Before the Distributions Deduction Exemption Powered by Cognero

Accounting Income $40,000 20,000

Taxable Income $40,000 20,000 (4,000) (300)

$60,000

$55,700

STEP 1

STEP 2

Income/ Distribution Deduction

$55,700 300 Page 26


Name:

Class:

Date:

Chapter 20: Income Taxation of Trusts and Estates Corpus Capital Gain/Loss Net Exempt Income Distributable Net Income

$56,000 (56,000)*

Distribution Deduction

STEP 3 ($ 300)

Entity Taxable Income

STEP 4

*Lesser of DNI ($56,000) or actual distribution ($60,000). 148. a. $30,000, one-half of DNI. b.

$30,000.

c.

First-tier of $40,000 to both beneficiaries. First-tier distributions are distributions that are composed of trust accounting income that is required to be distributed currently. The required distributions of $80,000 ($40,000 to each Lara and Byron) are first-tier distributions. The additional $30,000 discretionary distribution paid to Byron is a second-tier distribution.

149. DNI, Income Type Beneficiary

Shanna Tom Totals in DNI

Amount Received

$75,000 $75,000

Dividends

Taxable Interest

$25,000 $25,000 $50,000

$10,000 $10,000 $20,000

Passive

Exempt Interest

$5,000 $5,000 $10,000

$10,000 $10,000 $20,000

Corpus, Nontaxable

$25,000 $25,000 $50,000

150. a. $60,000. b.

$54,000.

c.

$9,900.

d.

$18,000 taxable interest income to both. Distributable Net

Item

Totals

Taxable interest income Exempt interest income Net long-term capital gain Fiduciary fees

$40,000 20,000 10,000 6,000

Powered by Cognero

Accounting Income $40,000 20,000

Taxable Income

Income/ Distribution Deduction

$40,000 10,000 (4,000)* Page 27


Name:

Class:

Date:

Chapter 20: Income Taxation of Trusts and Estates Personal exemption

(100) $60,000

Accounting Income/Taxable Income Before the Distributions Deduction

STEP 1

$45,900

STEP 2

$45,900

100

Exemption Corpus Capital Gain/Loss Net Exempt Income Distributable Net Income

(10,000) 18,000** $54,000 (36,000)

Distribution Deduction

STEP 3 Entity Taxable Income

$ 9,900 STEP 4

* 40/60 × $6,000 ** $20,000 – $2,000 (disallowed fees) Lesser of total distributions ($80,000) or deductible portion of DNI ($54,000 – $18,000 = $36,000). Corpus LTCG $10,000 – personal exemption $100. Beneficiary

Amount Received

Paula George Totals in DNI

$40,000 $40,000

Income Type [Step 5] Taxable Interest Exempt Interest $18,000∗ $9,000 $18,000 $9,000 $36,000 $18,000

∗ ($40,000 distribution ÷ $80,000 total second-tier distributions) × $36,000 taxable interest in DNI (assigning the

fiduciary fees proportionately to the two types of income). 151. Trusts often are used to shift management responsibilities for assets (e.g., in saving for college or retirement or to protect against medical difficulties). Fiduciary entities can be used by divorcing couples and by those executing an estate plan. Trusts also are attractive vehicles for holding special-use assets, such as life insurance policies and investment portfolios. 152. An estate of a decedent is created either by law or by will. The executor manages the decedent’s assets and satisfies any obligations. Distributions are made to income beneficiaries. When the estate terminates, the corpus is distributed to the remainder beneficiaries. A trust is created when the grantor transfers assets to the trustee who takes title to the assets so assigned. The trustee manages the corpus, making distributions to the income beneficiaries. When the trust terminates, the corpus is distributed to the remainder beneficiaries. 153. Under the rules of Subchapter J, a simple trust (1) must distribute its entire accounting income every year, (2) have no charitable beneficiaries, and (3) distribute no corpus. A complex trust is one that is not a simple trust. The classification as simple or complex is made every year. Thus, every trust is classified as a complex trust in the year in which it terminates (because it distributes all of its corpus during that year). 154. Powered by Cognero

Page 28


Name:

Class:

Date:

Chapter 20: Income Taxation of Trusts and Estates Entity accounting income is computed using state fiduciary laws and GAAP to comply with the specifications of the will or trust agreement. Accounting income is the first computation needed in deriving DNI and the entity's taxable income. The Uniform Fiduciary Income and Principal Act provides common rules for the states to adopt in characterizing entity accounting income. Generally, operating and investment income is allocated to income beneficiaries, capital gain/loss and cost recovery are allocated to corpus, and fiduciary fees are split equally between the two. 155. By default (i.e., no election is made), an in-kind distribution is not a taxable event for the fiduciary. The beneficiary takes a carryover basis in the asset received. By election, however, the fiduciary can recognize the realized gain or loss on the date of the distribution. But if the related party rules apply, the recognition of any loss may be deferred (Realized losses on related party transactions are not recognized). 156. Income in respect of a decedent (IRD) exists when a fiduciary or beneficiary receives an item that was earned but not recognized as gross income during the lifetime of the decedent. In general, IRD is treated in the following fashion: ∙

IRD is included in the gross estate for estate tax purposes and in the gross income for income tax purposes of the ultimate recipient.

There is no step-up or step-down of the basis of an IRD item.

Expenses related to generating the IRD are deductible for income tax purposes by the recipient and by the gross estate for estate tax purposes.

157. A proportionate amount of the deduction for the fees and charitable contributions is disallowed in computing entity taxable income. The disallowed portion of the deduction is the amount paid or accrued multiplied by the following fraction. exempt items of entity accounting income total accounting income An exception can apply to the contribution deduction. If the will or trust agreement specifies that the charitable gift must be made from a taxable income source, the full deduction is allowed. 158. If the fiduciary entity is operating a trade or business or holds investment property that generates cost recovery deductions, those deductions are allocated proportionately among the recipients of entity accounting income. If any accounting income is retained by the entity, it therefore can claim an allocable portion of the cost recovery deductions. 159. Distributable net income for any taxable year means the taxable income before the distribution deduction of the estate or trust adjusted for the following. + – + +

the personal exemption corpus capital gains corpus capital losses net tax-exempt interest

Reg. § 1.643(a)-0 states that the term distributable net income has no application except in the taxation of estates and trusts and their beneficiaries. DNI limits the deductions allowable to estates and trusts for amounts paid, credited, or required to be distributed to beneficiaries, and it is used to determine how much of an amount paid, credited, or required to be distributed to a beneficiary is includible in their gross income. DNI also is used to determine the character of Powered by Cognero

Page 29


Name:

Class:

Date:

Chapter 20: Income Taxation of Trusts and Estates distributions to the beneficiaries. 160. The distribution deduction for an estate or complex trust is the lesser of the deductible portion of DNI (i.e., after removing any net tax-exempt income) or the taxable amount that actually was distributed to beneficiaries during the tax year. For a simple trust, a full distribution of DNI is assumed even if actual asset distributions have not yet been made. 161. A simple trust does not claim any tax credits; its taxable income always is negative. An estate or complex trust claims a tax credit in proportion to the entity accounting income that it has retained. The entity’s beneficiaries similarly claim tax credits in proportion to the entity accounting income that they were assigned for the tax year. 162. ∙ DNI determines the maximum amount that can be taxed to the beneficiaries for the tax year. ∙

The makeup of DNI carries through to the beneficiaries (e.g., if DNI is constituted of 25% dividend income, each beneficiary’s distribution also is accounted for as 25% dividend income). DNI is used to compute the distribution deduction for the entity and is the maximum amount the entity can use as a distribution deduction for the year.

163. Special allocations of DNI are allowed only rarely under Subchapter J. A special allocation must be required in the trust instrument and supported by an economic effect that is independent of cash-flow and income tax consequences. This rule is similar to that used in partnership taxation under § 704(b)(2). Lacking such economic effects, all income beneficiaries share proportionately the character of the DNI. 164. A grantor trust essentially is ignored under the tax law. The entity still files a Form 1041, but for the most part it is “blank.” The Form 1041 reports the names and identification numbers of the grantor and beneficiaries of the trust; all income, deduction, credit, and AMT-related items are assigned to the grantor. Thus, grantor trust rules restrict the ability of the trust creator to shift income and other tax consequences to the income beneficiaries (The income of the trust may be taxed to the grantor under certain circumstances). 165. One or more of the following conditions can trigger grantor trust status. ∙ The trust is reversionary as to the grantor. ∙ The grantor can revoke the trust. ∙ The grantor retains the entity’s income or enjoyment of the corpus. ∙ The grantor can borrow from trust corpus for less than full and adequate consideration. 166. A Form 1041 is required if the estate or trust recognizes at least $600 gross income or if a trust has a positive amount of taxable income. The return and any amount of tax required are due before extensions no later than the fifteenth day of the fourth month following the end of the tax year; for a calendar taxpayer, this is April 15 of the following year The extended due date would be September 30. Estimated tax payments must be made by a fiduciary entity. Estates need to make estimated payments only beginning with the tax years that end two or more years after the date of the decedent’s death. 167. When fiduciary entities are used to shift income among taxpayers, the following planning objectives should be kept in mind. ∙

Sprinkling trusts allow flexibility for the trustee to direct income in a taxwise manner. Powered by Cognero

Page 30


Name:

Class:

Date:

Chapter 20: Income Taxation of Trusts and Estates ∙ ∙ ∙ ∙ ∙ ∙

High-income and high-wealth taxpayers usually should be given second-tier beneficiary status. Income shifting goals are limited by the fiduciary’s high marginal tax rates, and by the kiddie tax. Trust vehicles should be restricted to cases for which professional management will produce asset returns sufficient to justify the associated administrative costs. Fiduciaries should manage their exposure to the additional income taxes on net investment income and capital gains. Entity termination dates should be chosen to maximize fiduciary deductions but not to bunch beneficiary income recognition. Fiduciary expenses generally should be claimed by the taxpayer who is subject to the highest marginal income tax rate.

Powered by Cognero

Page 31


Turn static files into dynamic content formats.

Create a flipbook
Issuu converts static files into: digital portfolios, online yearbooks, online catalogs, digital photo albums and more. Sign up and create your flipbook.